Anda di halaman 1dari 235

UP College of Law

Part I Local Governments: Creation, Merger, Abolition and Powers

CONSTITUTIONAL BASIS
Art X, 1987 Constitution
Section 1. The territorial and political subdivisions of the Republic of the Philippines are the provinces, cities, municipalities, and barangays. There shall be autonomous regions in Muslim Mindanao and the Cordilleras as hereinafter provided. Section 2. The territorial and political subdivisions shall enjoy local autonomy. Section 3. The Congress shall enact a local government code which shall provide for a more responsive and accountable local government structure instituted through a system of decentralization with effective mechanisms of recall, initiative, and referendum, allocate among the different local government units their powers, responsibilities, and resources, and provide for the qualifications, election, appointment and removal, term, salaries, powers and functions and duties of local officials, and all other matters relating to the organization and operation of local units. Section 4. The President of the Philippines shall exercise general supervision over local governments. Provinces with respect to component cities and municipalities, and cities and municipalities with respect to component barangays shall ensure that the acts of their component units are within the scope of their prescribed powers and functions. Section 5. Each local government unit shall have the power to create its own sources of revenues and to levy taxes, fees, and charges subject to such guidelines and limitations as the Congress may provide, consistent with the basic policy of local autonomy. Such taxes, fees, and charges shall accrue exclusively to the local governments. by law enacted by Congress in the case of a province, city, municipality, or any other political subdivision, or by ordinance passed by the sangguniang panlalawigan or sangguniang panlungsod concerned in the case of a barangay located within its territorial jurisdiction, subject to such limitations and requirements prescribed in this Code. Comment: Congress has the power to create or abolish a province, city, municipality, or any other political subdivision. The sangguniang panlalawigan or sangguniang panlungsod has the power to create and abolish barangays located within its territorial jurisdiction Sec. 7. Creation and Conversion. As a general rule, the creation of a local government unit or its conversion from one level to another level shall be based on verifiable indicators of viability and projected capacity to provide services, to wit: (a) Income. It must be sufficient, based on acceptable standards, to provide for all essential government facilities and services and special functions commensurate with the size of its population, as expected of the local government unit concerned; (b) Population. It shall be determined as the total number of inhabitants within the territorial jurisdiction of the local government unit concerned; and (c) Land Area. It must be contiguous, unless it comprises two (2) or more islands or is separated by a local government unit independent of the others; properly identified by metes and bounds with technical descriptions and sufficient to provide for such basic services and facilities to meet the requirements of its populace. Compliance with the foregoing indicators shall be attested to by the Department of Finance (DOF), the National Statistics Office (NSO), and the Lands Management Bureau (LMB) of the Department of Environment and Natural Resources (DENR).

Chapter 2, RA 7160 (Sec 6 10)


Sec. 6. Authority to Create Local Government Units. A local government unit may be created, divided, merged, abolished, or its boundaries substantially altered either

1 D2010

Local Government
Mariano v. Comelec (242 SCRA 211) Facts: Petitioners assail the Constitutionality of RA 7859 on the ground that it did not properly identify the land area or territorial jurisdiction of Makati by metes and bound. Held: The petitioners in the case have not demonstrated that the delineation of the land area of the proposed area of Makati (without metes and bounds) will cause confusion as to its boundaries. Congress has refrained from using metes and bounds description of land areas of other local government units. Municipality of Jimnez v. Baz Jr. (265 SCRA 182) Facts: EO 258 creating the Mun. of Sinacaban contained a technical description of its land area. Based on said EO 258, Sinacaban claimed portions of several barrios previously thought to be part of the Mun. of Jimenez. The latter Municipality asserted jurisdiction in the claimed portion on the basis of an agreement entered into in 1950 with Sinacaban, approved by the Provincial Board. Held: The technical description, containing the metes and bounds of a municipalitys territory staed in EO 258 creating the Municipality of Sinacaban, Misamis Oriental is binding. Sec. 8. Division and Merger. Division and merger of existing local government units shall comply with the same requirements herein prescribed for their creation: Provided, however, That such division shall not reduce the income, population, or land area of the local government unit or units concerned to less than the minimum requirements prescribed in this Code: Provided, further, That the income classification of the original local government unit or units shall not fall below its current classification prior to such division. The income classification of local government units shall be updated within six (6) months from the effectivity of this Code to reflect the changes in their financial position resulting from the increased revenues as provided herein. Sec. 9. Abolition of Local Government Units. A local government unit may be abolished when its income, population, or land area has been irreversibly reduced to less than the minimum standards prescribed for its creation under Book III of this Code, as certified by the national agencies mentioned in Section 7 hereof to Congress or to the sangguniang concerned, as the case may be. The law or ordinance abolishing a local government unit shall specify the province, city, municipality, or barangay with which the local government unit sought to be abolished will be incorporated or merged. Comment: The Code now lays down three important indicators of viability in the creation of local government units namely: income, population and land area. Sec. 10. Plebiscite Requirement. No creation, division, merger, abolition, or substantial alteration of boundaries of local government units shall take effect unless approved by a majority of the votes cast in a plebiscite called for the purpose in the political unit or units directly affected. Said plebiscite shall be conducted by the Commission on Elections (Comelec) within one hundred twenty (120) days from the date of effectivity of the law or ordinance effecting such action, unless said law or ordinance fixes another date. Abbas v. COMELEC (179 SCRA 278) Held: There is no conflict between the power of the President to merge administrative region with the Constitutional provision, requiring plebiscite in the merger of local government units because the requirement of the plebiscite in a merger expressly applies only to provinces, cities, municipalities or barangays, not administrative regions. City of Pasig v. COMELEC 314 SCRA 179 Facts: Karangalan Village petitioned that they be segregated from Brgy. Manggahan and Dela Paz, Pasig. Plebiscite was scheduled but the Mun. of Cainta moved to suspend due to a pending case in RTC-Antipolo, Rizal for settlement of boundary dispute. Held: A case involving a boundary dispute between local government units presents a prejudicial question which must first be decided before plebiscites for creation of proposed barangay maybe held.

D2010 2

UP College of Law

ARTICLES AND REFERENCES


Public Corporations, Chapters 1-6 (Martin)
CHAPTER ONE: General Principles A corporation is an artificial being created by operation of law, having the right of succession and the powers, attributes, and properties expressly authorized by law or incident to its existence. It exists only in contemplation of law. A corporation is a legal institution devised to confer upon the individuals of which it is composed powers, privileges, and immunities which they would not otherwise possess, the most important of which are continuous legal identity or unity, and perpetual or indefinite succession under the corporate name, notwithstanding successive changes by death or otherwise, in the corporators or members. Classes of corporations 1. Public formed or organized for the government of a portion of a State for the accomplishment of parts of its own public works. These are created by State either by special or general act. 2. Privateformed for some private purpose, benefit, aim or end; organized wholly for the profit and advantage of their own members and cannot constitutionally be granted governmental powers. These are created by the will of the incorporators with the recognizance of the State. 2. incorporators intend to obtain profits from its undertaking, it is quasi-public. Classes of public corporations 1. Quasi-Corporationspublic corporations created as agencies of the State for a narrow and limited purpose. They differ from other public corporations in that they do not possess the powers and liabilities of self-governing corporations. Instead, their powers generally relate to matters of State, and not municipal concerns. Thus, although they are public in nature, they cannot be strictly considered municipal corporations because of their limited number of corporate powers and low grade of corporate existence. The main purpose of their creation is to aid the State in, or to take charge of, some public or state work other than community government for the general welfare. 2. Municipal corporations

CHAPTER TWO: Nature, elements, kinds and history of municipal Corporations A municipal corporation is a body politic constituted by the incorporation of the inhabitants of a city or town for the purpose of its local government. It is established by law partly as an agency of the State to assist in the civil government of a country, but chiefly to regulate and administer the local or internal affairs of the city/town/district which is incorporated. Courts regard these as subordinate branches of government of the State exercising delegated powers (municipal administration as an instrumentality of state administration). Municipal corporations are granted charters for the better government of particular areas or districts. Municipal corporation vs public corporationAll municipal corporations are public corporations, but not all public corporations are municipal corporations. Municipal corporations refer to incorporated villages, towns, and cities with powers of local administration. Elements: 1. 2. A legal creation or incorporation; A corporate name by which the legal entity is known and in which all corporate acts are done;

2.

Public vs Private Consider as criterion the relation of the corporation to the State. If it is created by the State as its own agency or instrumentality to help it carry out its governmental functions, then it is public. Otherwise, it is private. 3. Quasi-publicprivate corporations that render public service or supply public wants; while purposely organized for the gain or benefit of its members, the law requires them to discharge functions for the public benefit (i.e. utility companies).

Private vs Quasi public Consider as criterion the service the corporation renders to the public. If a corporation performs a public sovereign power, even though its

3 D2010

Local Government
3. Inhabitants constituting the population invested with the political and corporate powers which are exercised through duly constituted officers and agents; Place or territory within which the local civil government and corporate functions are exercised. 2. To act as an agency of the inhabitants in the regulation and operation of municipal franchises and public utilities, and the promotion and management of their local affairs (i.e. maintenance of water system, ferries, wharves). In this character, they act as separate entities for their own purposes and not as subdivisions of the State (Lidasan vs COMELEC).

4.

Kinds: 1. Municipal Corporation Properincorporated cities/towns/villages invested with the power of local legislation. These exist and are governed by their respective charters. Its distinctive purpose and distinguishing feature is the power of local government. Quasi-municipal corporationsame as a quasicorporation, i.e. it operates directly as an agency of the State to help in the administration of public functions. Dual nature : Two-fold character: 1. PublicMunicipal corporation acts as an agent of the State for the government of the territory and inhabitants within the municipal limits. It exercises by delegation a part of the sovereignty of the State. PrivateMunicipal corporation acts as a business corporation with functions not strictly governmental or political. It sees to the administration of local affairs which are beyond the sphere of the public purposes for which its governmental powers are conferred (Villa vs City of Manila, Surigao Electric Co. Inc. vs Municipality of Surigao).

2.

2.

Municipal corporation proper vs quasi municipal corporation When invested with the power of local government, the municipal corporation as an agency of the state becomes a municipal corporation proper. Consider as criterion the voluntary or involuntary nature of the corporation; the existence or non-existence of a charter; and whether the purpose of the corporation is solely a governmental agency or one for selfgovernment. Municipal corporation proper vs quasi corporation Both are agents of the state for limited and narrow purposes but have different powers and liabilities. Municipal corporations proper are created either by the direct solicitation or by the free consent of the persons composing them while quasi corporations (also called involuntary corporations) are only local organizations which, for purposes of civil administration, are invested with some characteristics of corporate existence. They are local subdivisions of the state, created by the sovereign legislative power of its own sovereign will and without any particular solicitation, consent or concurrent action from the inhabitants (West Chicago Park Commissioners vs Chicago). Purposes: 1. To serve as an agency or instrumentality of the state in carrying out the functions of government which the state cannot conveniently exercise (i.e. tax collection, assessment of property, enforcement of police regulations)

Origin and history England Municipal corporations can be traced back to the farmer commonwealths of the Teutons, and each wick, ham, stead, or tun took its name form the winsmen who lived there. United States Early American municipal corporations were similar to English municipalities of the 18th century. Prior to the revolution, local governments in the nature of municipalities existed in the American colonies by prescription, sovereign recognition or by virtue of special charters from the sovereign. Philippines under the Spanish Regime The basic unit of local administration was the pueblo, composed of numerous barrios, and governed by the cabeza de barangay, an honorific title continued by the Spaniards. Towns were organized, under the government of a gobernadorcillo, also called capitan, who represented the provincial governor as the arbiter of local questions, collected taxes, assisted the parish priest and entertained visiting officials. He was assisted by tenientes (deputies), alguaciles (subordinate employees) and chiefs of police. Elections for these municipal offices were held annually. Barangays were administered by cabezas de barangay whose principal duties were to act as agents for the

D2010 4

UP College of Law
collection of taxes. They paid no tribute on their own account and were members of the principalia, the voting and privileged class. The position was originally hereditary and breaks were filled by appointments, but eventually, the position became elective and service compulsory. The Maura Law constituted a municipal council of 1 captain and 4 lieutenants, in charge of the active work of governing the municipality, such as administration of public works and the details of taxation. These positions were honorary, and each of the members was required to have special qualifications. The Governor General, provincial council and provincial governor retained disciplinary jurisdiction over the council and its individual members. Philippines under the American Regime General Order No. 43, series of 1899 first provided for the government of municipalities. This was superseded by General Order No. 40, series of 1900, promulgated by the Military Governor, supposedly to give the Filipino people the right to elect their municipal officers, only slightly restricted by certain conditions. President McKinleys instructions to the 2nd Philippine Commission also directed the body to devote their attention to the establishment of municipal governments, giving the natives of the islands the opportunity to manage their own local affairs to the fullest extent of which they are capable, and subject to the least degree of supervision and control which a careful study of their capacities and observation of the workings of native controls show to be consistent with the maintenance of law, order and loyalty. The Philippine Commission passed Act No. 82 on January 31, 1901, providing for the organization and government of municipalities and Act No. 83 on February 5, 1901, for the organization of provinces. These were later modified in the Administrative Code. CHAPTER THREE: Creation of Municipal Corporations The power to create municipal corporations is essentially legislative, exclusive and practically unlimited. Aside from the legislature, the President under his martial law powers can exercise legislative powers and create municipal corporations. In the absence of a Constitutional provision permitting it, this power cannot be delegated by the legislature to any inferior or subordinate tribunal or board. However, it can pass a general law for the incorporation of municipal corporations, giving the conditions on which they may be created, and determining whether such conditions exist. Certain conditions are needed in order to obtain a municipal corporations incorporation as a city or a town. Such conditions may refer to extent of area, nature of the territory, the character of the land and the uses to which it may be devoted, the number of inhabitants, and the density and location of the area to be incorporated. Creation of municipal corporations In the Philippines, there is a general municipal law (Chapter 57 of the Administrative Code) under which the municipalities in regularly organized provinces were organized and a special municipal law (Chapter 64 of the Administrative Code) under which municipalities in the specially organized provinces were organized. The legislature may, by special act organize chartered cities in the Philippines. Under the Local Government Code, a barangay may be created, named, and its boundaries defined by an ordinance of the Sangguniang Panlalawigan or Sangguniang Panlungsod, subject to the approval of the majority in a plebiscite called for the purpose by the COMELEC to be held in the units affected within such a period of time as may be determined by the ordinance creating the said barangay. Under Sec 68 of the Revised Administrative Code, the President may by executive order define the boundaries of any province, sub province, municipality or any political subdivision, and increase or decrease the territory comprised therein, separate any political division other than a province into such portion as may be required, merge any of such subdivisions or portions with another, name any new subdivision so created and change the seat of government within any subdivision, to such place as the public welfare may require, provided that the authorization of Congress shall first be obtained whenever the boundary of any province or sub-province is to be defined or any province is to be divided into one or more sub provinces (**Note that this provision was later declared unconstitutional and repealed by RA No. 5185). Municipality of Cardona vs Binangonan (36 Phil. 547) HELD: Sec 1 of Act No. 1748 which conferred upon the Governor General the power to enlarge, constrict or otherwise change the boundary of a province, subprovince, municipality or township does not constitute an abdication of legislative functions with regard to this

5 D2010

Local Government
particular subject matter. It is simply transference of certain details with respect to provinces, municipalities and townships, many of them newly created, and all of them subject to rapid change both in development and population. The proper regulation of this may require prompt action of such detailed character as not to permit the legislative body to undertake efficiently. Pelaez vs Gimenez (December 24, 1965) HELD: Sec. 68 of the Revised Administrative Code does not meet the requirements for a valid delegation of power to fix the details in the enforcement of a law. It does not enunciate any policy to be carried out or implemented by the President. Neither does it give a standard sufficiently precise. Although the last clause of the first sentence of Sec. 68 gives the President the power to change the seat of the government within any subdivision to such place as the public welfare may require, the phrase as the public welfare may require qualifies not the clauses preceding the one just quoted, but only the place to which the seat of government may be transferred. Calalang vs Williams (70 Phil 726) and People vs Rosenthal (68 Phil 328) upheld public interest and welfare as sufficient standards for a valid delegation of authority to execute a law. However, this doctrine must be construed in the relation to the specific facts and issues involved in these cases, outside of which they do not constitute precedents and have no bonding effect. If the validity of the delegation of power in Sec 68 were upheld, there would no longer be any legal impediment to statutory grant of authority to the President to do anything, which, in his opinion, may be required by public welfare or interest. Such grant of authority would virtually be an abdication of power of Congress in favor of the Executive. The power of control under Sec 10(a) Art X of the Constitution (at the time) implies the right of the President to interfere in the exercise of the discretion vested by law in the officers of the executive departments, bureaus or offices, as well as to act in lieu of such offices. With respect to local governments, the fundamental law permits the President to wield no more authority than of checking whether said local governments or their officers perform their duties as provided by statute. Essential requisites for the incorporation of Municipal Corporations 1. Territory In the United States, it is within the power of the legislature, subject only to constitutional restrictions to determine the nature and extent of the territory to be incorporated and to prescribe the minimum or maximum area which may be incorporated. Aside from whatever restriction the constitution may impose, the area incorporated must be contiguous. 2. Population

The Barrio Charter (RA 2370) specifies 500 as the minimum number of inhabitants required for the creation of a barrio, and inasmuch as municipalities are composed of barrios, there should be at least 500 inhabitants to comply with the requirement as to population. 3. Charter

The charter usually invests the people of the place with the power of local government, determines the form of government of the municipal entity and defines its rights, powers and obligations. It is conferred for political purposes. The rule requiring assent of the inhabitants for incorporation of a municipal corporation to be effective is not applicable in the Philippines. However, under the new Constitution, the creation of a political division should be subject to the approval of majority of the votes cast in the units affected. Municipal corporations may also exist through prescription. Its existence will be presumed in where it is shown that the community has claimed and exercised corporate functions, with the knowledge and acquiescence of the legislature, and without interruption or objection for a period long enough to afford title by prescription. A corporation may exist in fact although not in point of law because of certain defects in some essential features of its organization. It may be considered a de facto municipal corporation, if the following requisites are complied with: 1) a valid law authorizing incorporation; 2) an attempt in good faith to organize under it; 3) a colorable compliance with the law, and; 4) an assumption of corporate powers. Municipality of Malabang vs Benito (March 28, 1968) HELD: In cases where a de facto municipal corporation was recognized as such despite the fact that the statute creating it was later invalidated, the decisions could fairly be made to rest on the consideration that there was

D2010 6

UP College of Law
some other valid law giving corporate vitality to the organization. In the present case, the mere fact that the municipality of Balabagan was organized at a time when the statute had not been invalidated cannot make it a de facto corporation, because independently of Sec 68 of the Administrative Code, there is no other valid statute to give color of authority to its creation. An unconstitutional act is not a law; it is, in legal contemplation, as inoperative as though it had never been passed. Attack against the validity of incorporation The validity of incorporation and the corporate existence of a municipal corporation may not be attacked collaterally. It may be challenged only by the State in a direct proceeding such as quo warranto. But this rule applies only where the municipal corporation is at least a public corporation. Where it is neither a corporation de facto or de jure but a nullity, its existence may be questioned collaterally or directly in any action or proceeding by anyone whose rights or interests are affected thereby, including the citizens of territory incorporated unless they are estopped from doing so (Municipality of Malabang vs Benito). The principle of estoppel applies to an individual who wants to attack the validity of incorporation of a municipal corporation. Where an individual dealt with it and acquiesced in the exercise of its corporate functions, or where he has entered into a contract with the said corporation, he may be estopped to deny its corporate existence. A person or private corporation may likewise be precluded by laches from attacking the validity of the incorporation of a municipality. Beginning of corporate existence The legal existence of a municipal corporation is to be determined by the law creating it. However, where the law is silent as to the beginning of its corporate existence, the rule is that this shall commence from the moment the law or charter creating it becomes effective. Under the Local Government Code, when a new local government unit is created, its corporate existence shall commence upon the qualification of its executive and a majority of the member s of its legislative body unless some other time is fixed therefor by law (Sec 6). A distinction should be made between the creation of the corporation as a legal entity and the organization of its government. The first should precede the second. The organization of the government of a municipal corporation presupposes necessarily the previous existence of the said corporation at the time its government is organized (Mejia vs Balolong, GR No L1925, 1948). Carreon vs Carreon (April 30, 1965) HELD: The existence of the City of Dagupan as a corporate body is incompatible with the continued existence of the municipal government of Dapitan. Sec 86 of RA 3811 creating the City of Dagupan out of the municipality of Dapitan can only mean that the municipal officials become city officials upon approval of the city charter. So when petitioners filed their certificates of candidacy for the positions in the city government, they filed their certificates to the same positions that they were holding, and could not be considered resigned therefrom under the old Election Code. The existence of a municipal corporation may be proved by its record of incorporation or charter. In the absence of a charter, its incorporation may be shown by parol evidence, at least to prove a de facto existence. Its existence may also be shown by reputation or by long use of its corporate powers, or by legislative grants necessarily implying a town corporation. CHAPTER 4: Alteration and dissolution of Municipal Corporations A municipal corporation cannot, without legal authorization, exercise its powers beyond its own corporate limits. It is therefore necessary that it must have its boundaries fixed, definite and certain, in order that they may be identified and that all may know the exact scope or section of territory and geographical division embraced in the corporate limits and over which the local corporation has jurisdiction. An incorporation is void where the boundaries of the municipal corporation are not described with certainty. With respect to disputes as to jurisdiction of municipal governments over areas or barangays, these shall be heard and decided by the Sangguniang Panlalawigan of the province where the municipalities concerned are situated. Where the areas or barangays in dispute are claimed by municipalities situated in different provinces, the Sangguniang Panlalawigan involved shall jointly hear and decide the dispute. In case no settlement is reached within 60 days from the date was referred to the Sangguniang Panlalawigan concerned, said dispute shall

7 D2010

Local Government
be elevated to the Regional Trial Court by the province which first took cognizance of the dispute. The case shall be decided by the said court within 1 year from the start of the proceedings and appeal may be taken from the decision within the time and manner prescribed by the Rules of Court (Sec 79, Local Government Code). Boundary disputes within barangays within the same city or municipality shall be heard and decided by the Sangguniang Panlungsod or Sangguniang Bayan concerned for the purpose of affording the parties an opportunity to reach an amicable settlement. In case no amicable settlement is reached within 60 days from the date the dispute was referred to the Sangguniang Panlungsod or Sangguniang Bayan, the case shall be heard and decided by the Regional Trial Court by the of the province or city where the barangays concerned are located within 1 year from the start of court proceedings. Appeal may be taken decision within the time and manner prescribed by the Rules of Court (Sec 80, Local Government Code). In the absence of any Constitutional prohibition, the Congress may alter or dissolve municipal corporations by: 1. fixing, altering, or changing the boundaries of municipal corporations for the purpose of enlarging or decreasing its territory; dividing a municipal corporation into 2 or more separate municipalities; merging or consolidating 2 or more separate municipalities into 1; annexing one municipality to another; repealing its charter. 4. On the title to the property of the annexed territory- The annexing territory shall acquire title to the property of the annexed territory at the time of the annexation unless the annexing statute provides otherwise. When the annexed territory forms part of a municipality from which it is taken, the legislature may provide for the payment of compensation for the indebtedness incurred on account of the property taken. Public buildings and improvements in the annexed territory are not required to be paid for by the annexing territory, as they have already been paid for by the annexed territory. However, if any indebtedness on these exists, the annexing territory must be required to share in the payment of said indebtedness. On the debts and obligations of the annexed territory- Those contracted prior to the annexation shall be assumed by the annexing territory in the absence of any provision to the contrary.

5.

Effects of division 1. On the legal existence of the original corporation- The corporate existence of the original municipality is extinguished. On the property, powers and rights of the original corporation- Unless the law provides otherwise, when a municipal corporation is divided into 2 or more municipalities, each municipality acquires title to all the property, powers, rights and obligations falling within its territorial limits.

2.

2. 3. 4. 5.

Effects of annexation or consolidation 1. On the legal existence of the territory annexedUnless otherwise provided, the annexation of one municipal corporation to another will dissolve the annexed territory. On the laws and ordinances of the annexed corporations- In the absence of a provision to the contrary, the annexed territory shall be become subject to the laws and ordinances by which the annexing corporation is governed. On the right of officers or employees of the annexed corporation to continue to hold their offices- Subject to what the legislature provides upon annexation, the officers and employees of the annexed territory shall terminate their official relation with their offices.

Limitations on the power to dissolve municipal corporations 1. by an express provision in the Constitution prohibiting the legislature to dissolve municipal corporations; those provisions in the Constitution affording protection to the rights, property and contracts of the inhabitants.

2.

2.

3.

Unless the Constitution has conferred such power, the courts have no power to dissolve municipal corporations. However, they can declare the act of the legislature creating a municipal corporation illegal. Municipal corporations cannot bring about their own dissolution by a mere surrender of their charter. A municipal corporation is not ipso facto dissolved or destroyed by non-user of its powers in whole or in part,

D2010 8

UP College of Law
or for its failure for a number of years to exercise the functions if a municipality. In such cases, the municipal corporation would be suspended but not civilly dead, since its dormant functions could be revived without action on the part of the sovereignty from which in theory of law, corporate life originally came. In the absence of a statute, a municipal corporation is not dissolved by the mere failure to elect or appoint its officers and agents to conduct its government. The officers do not constitute a corporation. Rather, the inhabitants of the designated locality are the corporators. Vilas vs City of Manila (42 Phil 953) HELD: The juristic identity of the corporation has in no wise been affected by the change of sovereignty, and in law, the present city is in every legal sense the successor of the old. As such, it is entitled to the property and property rights of the predecessor corporation and is subject to all of its liabilities. The mere change of the sovereign authority governing a country does not necessarily dissolve the municipal corporation organized under the former sovereign. Only such governmental functions as are incompatible with the present sovereignty may be considered suspended. Dissolution of a municipal corporation through the repeal of its charter or otherwise, without any provision for reincorporation, destroys and puts an end to the existence of the corporation. Its territory and inhabitants are then subject to such governmental functions as the State may impose subject to Constitutional limitations. CHAPTER FIVE: Legislative control and Executive supervision over Municipal Corporations Limitations on legislative control 1. Constitutional Limitations a. Those relating to the protection of private property b. Those preventing the impairment of contractual obligations c. Those prescribing a uniformity of law and prohibiting special or local legislation Limitations depending on the nature of the rights and powers exercised by the municipality a. a) In matters that relate to private powers or rights, the corporation is the agent of the inhabitants, and is assumes the character and relations of private individuals, and it is not subject to the absolute control of the legislature. In matters that relate to public or governmental powers or rights, the corporation is the agent of the State and is subject to its control. Legislative control over the municipal charter Unless prohibited by the Constitution, the charter, being a creation of the State may be modified, amended or repealed by the legislature when it is deemed necessary or advisable (US vs Joson, 26 Phil 1) Provisions of the charter relating to public or governmental matters do not constitute a contract within the impairment clause of the Constitution, so the legislature has absolute power to amend or alter them at will. Provisions of the charter granting rights, powers or privileges to the municipality for its own private interest may be considered as a contract, falling within the protection of the impairment clause. Legislative control over the form of government The legislature can change the form of government of a municipal corporation since it is a mere incident of their creation or existence, which the legislature has absolute and unlimited control over. The right of local self government is not inherent in municipalities and the legislature has the absolute power to deprive them of their rights to govern themselves in local affairs. In order to deprive the people of the right of local self government or any power vested by law in the municipality, it must be upon express terms of the legislature (City of Manila vs Manila Electric Railroad and Light Co, 26 Phil 89). Legislative control over municipal officers Subject to the Constitution, the legislature has absolute discretion in providing for the manner of selection and appointment of the officers to administer the public and governmental affairs of the municipal corporation (i.e. justices of the peace, policemen), but the municipality may not be deprived of the right to select the officers who will administer the private affairs of the municipality (i.e. caretakers of municipal parks or streets). The distinction lies in the nature of their functions. Legislative control over municipal property

2.

9 D2010

Local Government
Property which the municipality has acquired in its public or governmental capacity is under the complete control of the legislature. The State, at its pleasure, may modify or withdraw the power of the municipality to hold and manage its property. It may take such property without compensation, conditionally or unconditionally, despite protest from the citizens. It may require the municipality to spend its funds for the acquisition and maintenance of such property and provide for any regulation for the use and management thereof which it may deem to be for the public interest. Property which the municipality acquired in its private or proprietary capacity is held by it in the same character as a private individual. The legislature is subject to the constitutional restrictions when dealing with such property and cannot control it, except through a valid exercise of police power or by taking of private property upon payment of just compensation. Clinton vs Rapids, et al (24 Iowa 465) HELD: Public streets are not the private property of the city; it cannot alienate or use them for other than legitimate purposes. And over the use of such property acquired, dedicated under the statutes to the public use, the legislature, so far as regards the rights of public corporations, possesses an unlimited control and therefore can divert the use of the same for some other public purposes. In this case, the legislature can authorize the railroad company to lay its tracks on the streets of the city. Proprietors of Mt. Hope Cemetery vs City of Boston (153 Mass 509) HELD: The legislature has no power to require the city to transfer the cemetery to the corporation without compensation, because a cemetery is within the class of property which the city owns in its private or proprietary character. Ownership of such class of property is protected under the Constitution in that it cannot be taken without the payment of just compensation to the city. Province of Zamboanga del Norte vs City of Zamboanga (March 28, 1968) HELD: RA No 3039 which provides that all buildings, properties and assets belonging to the former Province of Zamboanga and located within the City of Zamboanaga are transferred to the city free of charge, is valid insofar as it affects lots used as the capitol site, school site and its grounds, hospital and leprosarium sites because they are truly private in nature. With respect to the 26 lots, the province of Zamboanga del Norte cannot be deprived of its share in their value inasmuch as said lots are patrimonial properties since they are not being utilized for distinctly governmental purposes. Legislative control over public utilities Unless the Constitution provides otherwise, municipal regulations over the operation of a water, light or gas company prevail against national or state regulations, because these involve the exercise of the proprietary or business functions of the municipality. However, the service and rates of the public utility may be subject to state regulation. Legislative control over municipal contracts Municipal contracts are regulated or controlled by the general legislation applicable to either all municipal corporations created thereafter or existing ones. The legislature may prescribe the methods of making municipal contracts or impose restrictions thereon by requiring bonds from the persons contracting with the municipality. It may regulate municipal employment, such as by determining the number of working hours of laborers. Execution of municipal contracts is subject to the requirements and restrictions in Secs 606-608 of the Revised Administrative Code. Legislative control over municipal funds and revenues The legislature has absolute control over the public revenues of a municipality such as those derived from taxation. It may require the expenditure of these funds for public purposes, but it has no control over public revenues over which the municipality or its creditors have already acquired a vested right. However, no such right arises until after an actual appropriation is made. The legislature may not control the private revenues of a municipal corporation because these posses the character of trust funds equitably belonging to the inhabitants. CHAPTER SIX: Executive and Ministerial supervision over Local Governments The President shall exercise general supervision over local governments to ensure that local affairs are administered according to the law. General supervision includes the power to order an investigation of the conduct of local government officials when necessary. This general supervision shall be exercised primarily

D2010 10

UP College of Law
through the Ministry of Local Government (now the DILG). There is a difference between control and supervision. Supervision means overseeing or the power or authority of an officer to see that his subordinate officers perform their duties, and to take such action as prescribed by law if they should fail to perform those duties. Control means the power of an officer to alter, modify, nullify or set aside what a subordinate had done in the performance of his duties and to substitute the judgment of the former for the latter (Mondano vs Silvosa, 51 OG 3427). This power of general supervision does not authorize the President to remove or suspend a local official without any ground or on grounds not authorized by law (Planas vs Gil, 67 Phil 62). Lacson vs Roque (49 OG 93) HELD: Suspension of the Mayor of the City of Manila on the ground of a pending charge against him for libel was illegal because it was not based on grounds provided for in law. To be a misconduct in office, the act must be committed in relation to the performance of his duties. Jover vs Borra (49 OG 2765) HELD: The act of the President when, for not valid reasons, he relieved the Mayor of Iloilo City of his office and designated an Acting Mayor in his stead, is illegal. Although the office of the mayor is appointive, Congress fixed the term of office of the Mayor to 6 years, and this implies that the said officer can continue in office until the expiration of his term unless removed for cause as provided by law. The Presidents action in this case amounted to an assertion of absolute control and not merely supervision. Mondano vs Silvosa (supra) HELD: Suspension of the Mayor of Mainit, Surigao by the Provincial Governor under an alleged authority of the President pending the hearing of charges of rape and concubinage is illegal because the charges preferred against him were not based on grounds provided for by law. The alleged act of the Mayor did not involve misconduct in office. Neither was he convicted yet of a crime involving moral turpitude. Local Government Unit is a political subdivision constituted by law, possessing substantial control over its own affairs. In a unitary system of government, it is an intrasovereign subdivision of one sovereign nation, and is not intended to be imperium in imperio (empire within an empire). The 1987 Philippine Constitution does not prescribe federalism. Autonomy does not contemplate the creation of mini-states. A Chartered City is a political body corporate, endowed with faculties of municipal corporations exercised through its city government in conformity with law and its proper corporate name; may sue and be sued, enter into contracts and be contracted with. No plebiscite is necessary when creating a national government agency such as the Metropolitan Manila Development Authority (MMDA). Plebiscite is only required for the creation of local government units. Dual nature/Two-Fold powers of Local Governments 1. Public, Governmental or Political springs from sovereignty; exercised in administering the powers of the State and promoting public welfare; include legislative, judicial, public and political aspects

Public character as regards the State: agent of the government 2. Corporate, Private or Proprietary arising from existence as legal persons, not as public agencies; exercised for the special benefit and advantage of the community; include ministerial, private and corporate aspects

Private character as regards the State: promote local necessities and convenience for its own community LGUs as agents and delegates of the National government Municipal Corporations (MC) derive their powers and rights from the legislature they can only exercise delegated legislative powers conferred by Congress as the national lawmaking body, therefore they cannot defy Congress will, nor modify or violate it. As agents, they are vested with the power of subordinate legislation, wherein the delegate cannot be superior to the principal or exercise higher powers.

Compendium on Local Autonomy and Local Government, Chapters 1-3 (Agra)


CHAPTER ONE: Nature of Local Governments

11 D2010

Local Government
MCs have no power to impose tax on natl govt instrumentalities, or otherwise retard, impede, burden or in any manner control the operations of constitutional law enacted by Congress to execute powers vested in the natl govt. Otherwise, mere creatures of the State can defeat national policies. Exercise of governmental functions As agencies of the State, MCs enjoy sovereign immunity from suit when engaged in governmental functions. However, they are subject to suit even in the performance of these functions if their charter so provides. Under the Real Property Tax Code, LGUs have no choice but to collect real property tax this means it is the national government expressing itself through the legislative branch that is levying the tax. The LGUs are merely constituted as agents to fix the rates. As agencies of the State for the promotion and maintenance of local self-government, MCs are endowed with police power in order to effectively accomplish and carry out the declared objects of their creation. The National Police Commission (NAPOLCOM) exercises administrative control and supervision over PNP officers and members while local chief executives exercise operational supervision and direction. Since the latter are only acting as representatives of NAPOLCOM, they are answerable to the former for their actions. However, unless the acts of the local executives are countermanded by NAPOLCOM, their acts are considered valid and binding. Although mandatory, deputization of local executives is not automatic and selfexecutory it requires a formal act of the NAPOLCOM. Delegated powers of local governments (not inherent) 1. Police Power inherent in the State, but not MCs; valid delegation by Natl Legislature necessary, since the latter is the repository of the inherent powers of the State Power to Tax must always yield to a legislative act; however, may be exercised pursuant to the direct authority conferred by the 1987 Philippine Constitution Power of Eminent Domain conferring statues cannot be broadened or constricted by implication Municipal authorities are in a better position to determine the evils sought to be prevented by the inclusion or incorporation of particular provisions in enacting a particular statute and therefore to pass the appropriate ordinance to attain the object of the law. Although the general law requires a majority vote in enacting ordinances, sanggunian may provide for higher requisite vote in amending specific ordinances. Corporate existence of LGUs Municipality [created under E.O. 265 (1949), later declared unconstitutional] which has performed governmental acts and been given State recognition and acknowledgment is a regular de jure municipality. [LGC of 1991 cured defect by stating that municipal districts organized pursuant to E.O.s and presidential issuances, which have their respective seats of office at the the of the Codes effectivity are to be considered regular de jure municipalities.] Attack on legal existence of a body politic: Quo Warranto must be timely filed; quo warranto suit against corporation for forfeiture of charter must be commenced within 5 years from time that act complained of was done or committed. Any other direct proceeding which must be brought in the name of the Republic. To change the name of a local government unit, a plebiscite is required to be conducted in the unit or units directly effected. Cannot be effected by mere resolution of sanggunian. Since the LGC is silent, sanggunian panlalawigan has no authority to change the name of its province authority lies with Congress. Territorial jurisdiction of political subdivisions Definition of territorial boundaries of LGU is a standard provision of the law creating them. Such provision, however, is not a grant of ownership of National Govt properties within the boundaries to the LGU. Jurisdiction refers to the sphere of political authority, not the area of ownership. An express grant is necessary to effect transfer of ownership. Council sessions must be conducted within territorial jurisdiction, except for instances such as lahar having wiped out the barangay.

2.

3.

As agents of the community

D2010 12

UP College of Law
Punong barangay cannot alter boundaries on his/her own. Power of LGUs to issue fishing privileges in the municipal waters was granted for revenue purposes. Commercial fishing operation within the 15-km. area of municipal waters is not prohibited provided: (a) activity is not undertaken in water areas within7 km. from shoreline; (b) allowed only in waters 7 or more fathoms deep. CHAPTER TWO: Local Autonomy and Centralization Local autonomy is not the same as decentralization. Local autonomy can only mean a measure of decentralization of functions of government. Decentralization devolution of national administration to the local levels in which local officials remain accountable to the central govt in the manner law may provide. LGU is autonomous in the sense that is is given more powers, authority, responsibilities and resource. Power which used to be highly centralized is thereby deconcentrated. Two levels of decentralization: 1. Decentralization of Administration/Administrative Autonomy central government delegates administrative powers to political subdivisions to achieve the ff. purposes: broaden local power base make units more responsive and accountable ensure full development of local governments as self-reliant communities break the monopoly of National Government over managing local affairs relieve National Government from the burden of managing local affairs chart its own destiny and shape its future with minimum intervention from central authorities; amounts to self-immolation. Autonomous government becomes accountable to constituency, not central authorities. President of the Republic Omission of phrase as may be provided for by law in relation to power of general supervision over LGUs in 1987 Constitution merely serves to underscore LGUs autonomy from Congress and to break the latters control over local government affairs. Basic law did not intend to deprive legislature of all authority over municipal corporations, particularly concerning discipline. Department Secretaries Secretary of Justice review and revoke tax ordinances with regard to constitutionality or legality; cannot substitute own judgment nor replace the same with another version nor declare it unjust, excessive, oppressive or confiscatory since they involve exercise of judgment or discretion as to wisdom and reasonableness; power is of supervision, not control Secretary of DILG administrative supervision over local govt units; need not confirm the decision of a local chief executive imposing penalty of removal against appointive officials Congress Retains control of the LGU, although in significantly reduced degree under present Constitution. Basic relp between national legislature and local governments has not been enfeebled by the new provisions in 1987 constitution strengthening the policy of local autonomy. Local autonomy not self-executing It is subject to the passage of a local government code, local tax law, income distribution legislation and national representation law. Interpretations in favor of decentralization Where a law is capable of two interpretations, one in favour of centralized power in Malacanang and the other beneficial to local autonomy, scales must be weighed in favour of the latter. (Should be resolved in favour of the greater economy of the local government.)

Role of the President: general supervision, only to ensure that local affairs are administered according to law. He/She has no control over their acts in the sense that he/she can substitute their judgments with his/her own. Administrative Autonomy primarily pertains to: power and responsibility to deliver basic services 2. Decentralization of Power/Political Autonomy involves abdication of political power in favour of LGUs; autonomous government is free to

13 D2010

Local Government
The power to review must be guided by a liberal construction in favour of the supervised/lower LGU and in consonance with the principles of local autonomy. The value of local governments as institutions of democracy is measured by the degree of autonomy that they enjoy. Fiscal autonomy Pursuant to fiscal autonomy granted to LGUs, and although the LGC is silent as to the placement of idle or excess funds, the sanggunian may authorize the investment, placement in time deposit or any judicious utilization of such funds as far as general funds are concerned. Incidents of decentralization Under the principle of local autonomy, even though the power to abolish an office is not expressly conferred on provincial governments, it is necessarily implied from the power to create offices. R.A. 5185, which gives mayors the power to appoint all officials entirely paid out of city funds, and B.P. 337, empowering local executives with the authority to appoint all officers and employees of the city were not meant to vest the city mayors per se with comprehensive powers rather, they underscore the transfer of the power of appointment over local officials and employees from the President to the local governments and to highlight local autonomy. LGUs enjoy full autonomy in the operation and management of economic enterprises, such as public markets. A sanggunian may provide for a different scheme of allocating market stalls. LGUs may not provide for allowances to judges assigned within a given locality since this would counter the spirit of R.A. 6758, which standardizes the salaries of government personnel. LGU cannot create its own government owned or controlled corporation to provide basic services, since only Congress may create such corporation by special law. Also, it is the function of the LGu to provide such services. Department of Agriculture may opt to devolve the function of regulating the operation of commercial fishing vessels within municipal waters, in consonance with the spirit of local autonomy, to LGUs. Centralization and national integration Office of the President

When Sangguniang Kabataan (SK) elections are not conducted in certain areas, the President may appoint SK officials in exercise of his/her residual powers and the power to appoint other officers of the government whose appointments are not otherwise provided. Ordinances enacted by local legislative bodies must not be repugnant to or in conflict with existing Administrative Orders or local budget circulars issued by the Dept. of Budget and Management (DBM) for govt employees and officials. Classification or reclassification of public forest land is a prerogative of the President upon recommendation of the pertinent department head. Additional compensation outside the purview of R.A. 6758 (ex. medical benefits) only allowable if approved by President via administrative order. Sangguniang panlalawigan cannot authorize grant of representation and transportation allowance to all division chiefs (divisions only components of departments/offices) of the provincial gov;t, since it violates Admin. Order 42 (clarifying role of DBM; only allows grant of benefit to department heads or asst. department heads). Commission on Elections (COMELEC)

No local elective may be suspended during election period except upon prior written approval of the COMELEC. Omnibus Election Code states that LGUs must share in expenses for the election of barangay officials, therefore COMELEC may direct compliance therewith. Commission on Audit (COA)

Approval by COA of disbursements of local funds relates to administrative aspect of the matter of the officials accountability, but does not foreclose Ombudsmans authority to investigate and determine WON there is a crime to be prosecuted for which he/she is accountable. Compliance with COA rules and regulations does not necessarily mean that misappropriation of public funds was not committed. COA has power to compromise or release any unsettled claim or liability whenever government interest requires.

D2010 14

UP College of Law
Disposal of unserviceable property through public auction does not need approval by COA, but those disposed of through negotiated sale does. In consonance with its constitutional mandate as guardian of public funds, disposal of real property owned by LGU shall be subject to approval of COA regardless of value or cost involved and mode of divestment, even if Committee on Awards determines floor price that is disadvantageous to the LGU. In exercise of its contract review functions, COA may direct the adoption of the BIR zonal valuation if it is a more reasonable selling price as compared to the highest available bid during the public auction. COA may employ the services of private appraisers to determine the value of real property sought to be sold if the amount involved is substantial and the transaction is the subject of investigation by the Office of the Ombudsman. Civil Service Commission (CSC) It is the governor that has authority to impose preventive suspension against a respondent municipal elective official. DILG has to authority to impose such suspension. DILG shall be the lead national government agency to oversee/administer national government assistance to local government units in the implementation of devolved infrastructure programs. [NOTE: a basic element that determines centralization is that the effects of the concerns of a particular department are not confined to a particular region rather, they can be felt on a nationwide scale] Department of Transportation and Communications (DOTC), Telecommunications Office (TELOF)

Cannot devolve its regional office functions to the LGUs, considering the interdependence of various telecom systems on a nationwide scale. Land Transportation Office (LTO) Civil Aeronautics Board (CAB) Department of Environment and National Resources (DENR)

Has the power to approve or disapprove appointments set before it by inquiring into the eligibility of the person, but does not have the power to make appointments itself or to direct the appointing authority to change employment status of an employee, otherwise this will amount to an arrogation of power belonging to appointing authority. May accredit an LGU, subject to certain requirements where appointments of local officials are no longer subject to CSC approval. Cannot nullify a board resolution calling for a reorganization in the local government offices CSC has jurisdiction only insofar as the implementation of the reorganization is concerned. Proper forum to assail resolution are the regular courts (RTCs). Department of Interior and Local Government (DILG)

Concerned with enforcement of forestry laws, rules and regulations, and the protection, development and management of forest lands. DENR retains power of control over activities concerning community-based forestry projects (notwithstanding fact of devolution), being the national agency authorized by the Administrative Code of 1987 to protect and preserve the environment, which is a constitutional mandate. Therefore, its power cannot be encroached upon by LGU. Department of Justice (DOJ) (role of Secretary of Justice discussed above) Department of Budget and Management (DBM) Department of Finance (DOF)

General supervision is exercised by the President through the Secretary of Local Government. Application for authority to travel abroad by all local officials and employees (other than governors, city mayors or highly urbanized and independent component cities) shall be approved by the Secretary. Failure to obtain authority ma constitute gross negligence and dereliction of duty.

Secretary of Finance is the proper appointing authority for treasurers, and the proper disciplining authority to issue preventive suspension. Local legislative councils are bereft of authority to initiate administrative action against the treasurer. Department of Public Works and Highways (DPWH)

15 D2010

Local Government
General supervision over the implementation of the National Building Code, including appellate jurisdiction over the decisions and order of the local building officials remains with the Secretary of Public Works and Highways. Local chief executive possesses the authority to appoint an engineer who shall also act as local building official. Department of Energy (DOE) Department of Agriculture (DA) Department of Agrarian Reform (DAR) Department of Trade and Industry (DTI) Laguna Lake Development Authority (LLDA) Philippine Amusement and Gaming Corporation (PAGCOR) Philippine Charity Sweepstakes Office (PCSO) Public Estates Authority (PEA) Cooperative Development Authority (CDA) Philippine Ports Authority (PPA) Metropolitan Waterworks and Sewerage System (MWSS) Construction Industry Arbitration Commission (CIAC) Autonomous Region of Muslim Mindanao Peculiar to the 1987 Constitution, and contemplates the grant of political autonomy (according to Cordillera Broad Coalition vs. COA). Registration of motor vehicles whose effects are not confined to a region should not be devolved to the ARMM and should remain with the LTO. CAB has no authority to devolve or share any of its general and specific functions with the ARMM in the regulation of all entities engaged in air transportation or air commerce. However, ARMM may assist CAB in the performance of its functions. Regional government of ARMM may not enact a Regional Public Works Act which will absolutely dispense with public bidding since this would be contrary to public policy in the prosecution of public undertakings. May not also enact a Regional Police Force Law. Employee of Sothern Philippines Development Authority may refuse to be transferred to the ARMM and may claim and be entitled to separation pay. Cannot be obligated to transfer because of constitutional prohibition against involuntary servitude. Permanent vacancies in the Sangguniang Bayan within a province situated in ARMM are filled by the provincial governor, not regional governor. Cordillera Autonomous Region Peculiar to the 1987 Constitution, and contemplates the grant of political autonomy. (according to Cordillera Broad Coalition vs. COA) CHAPTER THREE: Devolution Devolution a mandatory process premised on the constitutional mandate that all local government units possess and enjoy local autonomy E.O. 503 effectively extends the period of devolution from 6 months as provided for in the LGC to 1 year in order to accommodate the smooth transition and address difficulties in the devolution process. Powers devolved to LGUs Specific grant necessary the enumeration of powers and services under sec. 17 of the LGC is not exclusive

Local autonomy and the judicial system The exercise of an authority or a power by a local government unit may be judicially inquired into and corrected only if it is capricious, whimsical, unjust or unreasonable, there having been a denial of due process or a violation of any other applicable constitutional guarantee. Internal acts of the Sangguniang Pampook of Region XII, which enjoyed administrative autonomy are subject to the jurisdictionof the courts. Courts have the power to inquire into the legality of the exercise of the power of eminent domain and to determine whether there is genuine necessity therefor. Courts, not the CSC, are the proper forum to assail and seek nullification of a resolution for reorganization in the local government offices. CSC has jurisdiction only insofar as the implementation of the reorganization is concerned. Provinces, cities, municipalities and barangays These LGUs enjoy administrative autonomy. The 1987 constitution ensures the autonomy of local governments and political subdivisions, and limits the Presidents power to general supervision over local governments.

D2010 16

UP College of Law
Power to discipline locally assigned national employees cannot be subject to the jurisdiction of the municipal council (ex. fire personnel are under Bureau of Fire Protection of DILG, fire protection not being a devolved function) Devolution has a corresponding effect on local budgeting processes. Although regulation of cockfighting has been devolved to local governments, a municipal government cannot issue a permit to a promoter to hold cockfights in places other than a licensed cockpit, since this would violate the law. Ports which are vital to the national interest and security which are linked to each other are expressly excluded from the coverage of devolution. Implementation of locally-funded communal irrigation projects (CIP) shall be devolved to LGUs. The allocation for locally funded CIPs will no longer be released to the National Irrigation Administration (NIA) as a consequence of devolution. However, foreign-assisted CIPs fall outside the coverage of devolution. Licensing and regulation of activities undertaken by travel agencies, tour operators and professional congress organizers should be transferred to LGUs. There is partial devolution in terms of the functions of the Cooperative Development Authority (CDA). The functions devolved are the promotion, organization and development of cooperatives. CDA retains the powers relating to registration of cooperatives and the issuance of rules and regulations, policies and guidelines. General supervision over the implementation of the National Building Code, including appellate jurisdiction over the decisions and order of the local building officials remains with the Secretary of Public Works and Highways. With regard to the compensation and position classification system, the Joint Commission on Local Government Personnel Administration was abolished and transferred to the appropriate office in the CSC. Absorption of personnel Devolution does not only involve the delegation of the powers to regulate but also the transfer of the necessary assets and personnel. Objectives To ensure the administrative and technical capabilities of the LGUs to provide the devolved basic services and facilities at the local level To ensure that the delivery of basic services is not duly prejudiced or disrupted.

Incidents of absorption Re-appointment of an employee to the same position held prior to the reorganization constitutes a reconsideration of any previous objection to his/her earlier appointment. CSC has no power to disapprove appointments made to those already employed but subsequently devolved. Devolved personnel do not lose their security of tenure. Even without appointment, they shall continue to hold the position to which they were last appointed. Exceptions Not administratively viable there is a duplication of functions when the duties and responsibilities of national government agency personnel are similar to those of incumbent local government employees, and their offices perform the same functions. However, this does not apply to a national employee who is responsible for or attached to a devolved asset (ex. drivers of devolved agencies). The LGU and National Govt may invoke this provision if both agree as to the duplication, but LGU may still opt to absorb the employee. Involuntary servitude employee may refuse to be transferred and may claim separation pay. Appointment not renewed an appointment not renewed by the DBM prior to the completion of the 6month process cannot be re-appointed on the basis alone of the city government affected Devolved personnel are considered local government personnel and should be covered by the existing Position Classification and Pay Plan. There shall be no diminution in pay or benefits, therefore they may continue to enjoy higher rates than their counterparts. Gradual equalization of salaries of all local government personnel must then be achieved to eliminate (or at least minimize) the aforesaid disparity. To achieve this, LGUs lower than special cities and first class provinces may adopt the salary schedule for higher LGUs subject to the following conditions:

17 D2010

Local Government
LGU is financially capable Salary schedule shall be uniformly applied Schedule of the highly urbanized cities and first class provinces shall not be higher than that adopted by the national government In implementing a new and higher salary schedule, the salary grade allocation of positions and salary steps of personnel shall be retained Adoption shall be subject to budgetary and general limitations on Personal Service Expenditures (Secs. 324-325, LGC) In case of component cities and municipalities, schedule shall not be higher than that of the province or city where they belong Adoption of higher salary schedule shall not alter existing classification of LGU concerned Assets transferred to LGUs pursuant to devolution cannot be recovered since the transfer is mandatory. However, a provincial government may continue funding a general hospital until the same is officially transferred to the DBM. Since devolved hospitals have become components of LGUs like other devolved national government agencies, they shall be covered by COA Circular No. 92-382. Memoranda of Agreement to effect devolution To effect devolution, a MOA must be effected between a National Government Agency and local governments concerned. The local chief executive, in order to enter into an agreement, must have the authority of the local legislative council. The DILG cannot revoke such agreement, rule on the alleged diminution of powers, nor pass upon the merits of the contents and provisions of the agreement. Oversight Committee The OC is the body empowered to formulate and issue the appropriate rules and regulations necessary for the efficient and effective implementation of all provisions of the LGC. Opinions rendered by it shall prevail over those of the DOJ. The DOJ is bereft of authority to review or pass upon the rulings of the OC, unless the latter itself requests. The OC, however, cannot expand nor constrict the law. It must always remain congruent to it. A resolution adopted by the OC is devoid of legal force and effect unless approved by the President and subsequently incorporated or embodied in an executive or administrative order.

LGUs may grant allowances/additional compensation to national government employees assigned in their localities at rates authorized by law, rules and regulations, subject to the following conditions: Annual income or finances of LGU as certified by the local treasurer concerned will allow such grant without exceeding the general limitations for personal services Budgetary requirements (Sec. 324 of LGC and R.A. 6758) have been satisfied and provided fully in the budget as certified by the Budget Officer and COA representative LGU has fully implemented the devolution of personnel/functions in accordance with the LGC

The mandatory absorption does not preclude affected LGUs from creating equivalent positions nor conferring functions different from the position previously held by the devolved employeee provided there is no diminution of pay and benefits, reduction in rank and impairment of tenure. Assignment of devolved personnel is an administrative decision of the LGU. A local chief executive may appoint one person or a committee to investigate, hear, make findings and submit recommendations incidental to administrative complaints against erring appointive local officials, including devolved personnel. Transfer of assets

Alfiler
See attachments

Tapales
See attachments

D2010 18

UP College of Law

CREATION OF LOCAL GOVERNMENT UNITS


Tan v. Comelec
BP 885 (an Act creating a new province of Negros del Norte) was enacted. It provides for the new provinces component cities, boundaries, and the conduct of a plebiscite for its creation. A plebiscite was held, pursuant to the requirement of said BP (shall be conducted in the proposed new province which are the areas affected), but the plebiscite was confined only to the inhabitants of the territory of the new province, to the exclusion of the voters from the rest of the province. (Negros Occidental some cities and municipalities of Negross Occidental = Negros del Norte) WON the petition questioning the constitutionality and validity of the plebiscite renders the case moot and academic because the result was in favor of the creation of the new province and such creation was now fait accompli. HELD: No. It is the legality of the plebiscite itself which is challenged. WON the province complied with the plebiscite requirement. HELD: No. Sec 3, Art XI provides that No province, city, municipality or barrio may be created, divided, merged, abolished, or its boundary substantially altered, except in accordance with the criteria established in the local government code, and subject to the approval by a majority of the votes in a plebiscite in the unit or units affected. This means that the rest of Negros Occidental must be included in the conduct of plebiscite as the boundaries of the existing province of Negros Occidental would necessarily substantially altered by the division of its existing boundaries in order that there can be created the proposed new province. WON the new province complied with the requirements of land area. HELD: No. Sec 197 of the LGC provides that a province may be created if it has a territory of at least three thousand five hundred square kilometers, xxx. The territory need not be contiguous if it comprises two or more islands. The use of the word territory clearly, reflects that it has reference only to the mass of land area and excludes the waters over which the political unit exercises control. It can be safely concluded that the word territory in the first paragraph of Section 197 is meant to be synonymous with "land area" only.

Torralba v. Municipality of Sibagat


BP 56, enacted February 1980, created the Municipality of Sibagat, Province of Agusan del Sur. Petitioners assail its validity for being violative of Section 3, Article XI, 1973 Constitution: No province, city, municipality, or barrio may be created except in accordance with the criteria established in the Local Government Code; that the LGC must first be enacted to determine the criteria for the creation of any province, city, municipality, or barrio and since no LGC [came into being only on 10 February 1983] had yet been enacted as of the date BP 56 was passed, the latter could not have possibly complied with any criteria when the Municipality was created. HELD: Section 3, Article XI, 1973 Constitution does not prohibit the modification of territorial and political subdivisions before the enactment of the LGC. It does not say that the LGC is a condition sine qua non for the creation of a municipality. The constitutional provision simply means that once said Code is enacted, the creation, modification or dissolution of LGUs should conform with the criteria thus laid down. The power to create a municipal corporation is legislative in nature. In the absence of any constitutional limitation, a legislative body may create any corporation it deems essential for the more efficient administration of government. The creation of the new Municipality was a valid exercise of legislative power vested by the 1973 Constitution in the Interim Batasang Pambansa. Pursuant to BP 56, a valid plebiscite has been conducted among the unit/s affected. The officials of the new Municipality have taken their oaths and are performing their functions. A de jure entity has been created.

Bai Sema v. Comelec


WON RA 9054 delegating to the ARMM Regional Assembly the power to create provinces, cities, municipalities and barangays, is constitutional. WON a province (created by the ARMM Regional Assembly under MMA Act 201) is entitled to one representative in the House of Representatives without need of a national law creating a legislative district for such province

19 D2010

Local Government
HELD: No. For Congress to delegate validly the power to create a province or city, it must also validly delegate at the same time the power to create a legislative district. Congress cannot delegate to the ARMM Regional Assembly the power to create legislative districts for the House of Representatives. Congress can delegate to local legislative bodies the power to create local government units, subject to reasonable standards and provided no conflict arises with any provision of the Constitution. However, under the Local Government Code, "only x x x an Act of Congress" can create provinces, cities or municipalities. Under Section 19, Article VI of RA 9054, Congress delegated to the ARMM Regional Assembly the power to create provinces, cities, municipalities and barangays within the ARMM. Congress made the delegation under its plenary legislative powers because the power to create local government units is not one of the express legislative powers granted by the Constitution to regional legislative bodies. In the present case, the question arises whether the delegation to the ARMM Regional Assembly of the power to create provinces, cities, municipalities and barangays conflicts with any provision of the Constitution. There is no provision in the Constitution that conflicts with the delegation to regional legislative bodies of the power to create municipalities and barangays, provided Section 10, Article X of the Constitution is followed. However, the creation of provinces and cities is another matter. Section 5 (3), Article VI of the Constitution provides, "Each city with a population of at least two hundred fifty thousand, or each province, shall have at least one representative" in the House of Representatives. Similarly, Section 3 of the Ordinance appended to the Constitution provides, "Any province that may hereafter be created, or any city whose population may hereafter increase to more than two hundred fifty thousand shall be entitled in the immediately following election to at least one Member x x x." Clearly, a province cannot be created without a legislative district because it will violate Section 5 (3), Article VI of the Constitution as well as Section 3 of the Ordinance appended to the Constitution. For the same reason, a city with a population of 250,000 or more cannot also be created without a legislative district. Thus, the power to create a province, or a city with a population of 250,000 or more, requires also the power to create a legislative district. Even the creation of a city with a population of less than 250,000 involves the power to create a legislative district because once the city's population reaches 250,000, the city automatically becomes entitled to one representative under Section 5 (3), Article VI of the Constitution and Section 3 of the Ordinance appended to the Constitution. Thus, the power to create a province or city inherently involves the power to create a legislative district.

Province of North Cotabato v. GRP


On 8/5/08, the government and the MILF were scheduled to sign a Memorandum of Agreement on the Ancestral Domain Aspect of the GRP-MILF Tripoli Agreement of Peace of 2001 in Malaysia. The Solicitor General, who represents respondents, summarizes the MOA-AD by stating that the same contained, among others, the commitment of the parties to pursue peace negotiations, protect and respect human rights, negotiate with sincerity in the resolution and pacific settlement of the conflict, and refrain from the use of threat or force to attain undue advantage while the peace negotiations on the substantive agenda are on-going. WON the creation of the MOA-AD violates the Petitioners right to information on matters of public concern HELD: (a) The MOA-AD is a matter of public concern, involving as it does the sovereignty and territorial integrity of the State, which directly affects the lives of the people at large (b) The right to information contemplates inclusion of negotiations leading to the consummation of the transaction (c) The right includes (1) the right of the people to demand information and (2) the duty of the officialdom to give information even if nobody demands. (d) E.O No 3 contemplates not just the conduct of a plebiscite to effectuate continuing consultations, and such establishes petitioners right to be consulted on the peace agenda, corollary to the Constitutional right. (e) Presidential Adviser on the Peace Process (PAPP) Esperon committed grave abuse of discretion when he failed to carry out the pertinent consultation.

D2010 20

UP College of Law
At least three pertinent laws animate these constitutional imperatives and justify the exercise of the peoples right to be consulted on relevant matters relating to the peace agenda. 1. E.O. No. 3 itself is replete with mechanics for continuing consultations on both national and local levels and for a principal forum for consensus-building. In fact, it is the duty of the Presidential Adviser on the Peace Process to conduct regular dialogues to seek relevant information, comments, advice, and recommendations from peace partners and concerned sectors of society. The Local Government Code of 1991 requires all national offices to conduct consultations before any project or program critical to the environment and human ecology including those that may call for the eviction of a particular group of people residing in such locality, is implemented therein. The MOA-AD is one peculiar program that unequivocally and unilaterally vests ownership of a vast territory to the Bangsamoro people, which could pervasively and drastically result to the diaspora or displacement of a great number of inhabitants from their total environment. Republic Act No. 8371 or the Indigenous Peoples Rights Act of 1997 provides for clear-cut procedure for the recognition and delineation of ancestral domain, which entails, among other things, the observance of the free and prior informed consent of the Indigenous Cultural Communities/Indigenous Peoples. Notably, the statute does not grant the Executive Department or any government agency the power to delineate and recognize an ancestral domain claim by mere agreement or compromise. HELD: Yes. In Paredes v. Executive Secretary the constitutional provision on the need for a majority of the votes cast in the plebiscite in the unit or units affected would be satisfied even if "those voters who are not from the barangay to be separated were excluded in the plebiscite." It cannot be argued therefore that the plebiscite held in the areas affected to constitute Metropolitan Manila in the referendum on February 27, 1975 was not a sufficient compliance with the constitutional provision. With the voters in such four cities and thirteen municipalities, now composing Metropolitan Manila, having manifested their will, the constitutional provision relied upon by petitioners has been satisfied. It is to be noted likewise that at the time of such plebiscite in February, 1975, there was no Local Government Code. Article VIII, Section 2 of the Constitution expressly recognized the juridical entity known as Metropolitan Manila. Such express constitutional affirmation of its existence in the fundamental law calls for the dismissal of these petitions, there being no legal justification for the declaration of unconstitutionality of Presidential Decree No. 824. Nor was it the first time that there has been acknowledgment in law of the creation of Metropolitan Manila. Under PD 824: "The Commission, the General Manager and any official of the Commission shall be under the direct supervision and control of the President. Notwithstanding any provision in this Decree, the President shall have the power to revoke, amend or modify any ordinance, resolution or act of the Commission, the General and the Commissioners." It may give rise to doubts as to its validity insofar as it confers the power of control on the President. That control he certainly exercises under the present Constitution over the ministries. His power over local governments does not go that far. It extends no further than general supervision. These doubts, however, do not suffice to nullify such a provision. Succinctly put, that construction that would save is to be preferred as against one that will destroy. To show fidelity to this basic principle of construction is to lend substance to the equally basic doctrine that the constitution enters into and forms part of every statute. Accordingly, the presidential power of control over acts of the Metro Manila Commission is limited to those that may be considered national in character. Where, however, the acts of the Metro Manila Commission may be considered as properly appertaining to local government functions,

2.

3.

The invocation of the doctrine of executive privilege as a defense to the general right to information or the specific right to consultation is untenable. The various explicit legal provisions fly in the face of executive secrecy. In any event, respondents effectively waived such defense after it unconditionally disclosed the official copies of the final draft of the MOA-AD, for judicial compliance and public scrutiny.

Lopez v. Comelec
WON PD 824 (enacted November 1975) creating Metropolitan Manila is valid.

21 D2010

Local Government
the power of the President is confined to general supervision.

PRESUMPTION OF CONSTITUTIONALITY
Alvarez v. Guingona
RA 7720 converted the Municipality of Santiago, Isabela into an independent component city. Petitioners claim that the said municipality has not met the minimum average annual income required under the LGC. They assert that Internal Revenue Allotments are not actually income but merely transfers and/or budgetary aid from the national government and that they fluctuate, increase or decrease, depending on factors like population, land and equal sharing. Another contention of the petitioner is that RA 7330 originated not from Congress but from the Senate. Apparently, RA 7330 originated from HB 8817 which was filed on April 18, 1993. After the third reading, the bill was transmitted to the Senate on January 18, 1994. Meanwhile, a counterpart bill SB 1243 was filed on May 19, 1993. On February 23, 1994, HB 8817 was transmitted to the senate. The committee recommended that HB 8817 be approved without amendment, taking into consideration that the house bill was identical to the senate bill. HELD:RA 7720 is valid. Internal Revenue Allotments are items of income because they form part of the gross accretion of funds of the LGU. They are included in computing the average annual income required to become a city. The IRAs regularly and automatically accrues to the local treasury without need of any further action on the part of the LGU. Thus they constitute income which the local government can invariably rely upon as the source of much needed funds. The filing in the Senate of a substitute bill in anticipation of its receipt of the bill from the House, does not contravene the constitutional requirement that a bill of local application should originate in the House of Representatives, for as long as the Senate does not act thereupon until it receives the House bill. Nor does the Constitution prohibit the filing in the Senate of a substitute bill in anticipation of its receipt of the bill from the House, so long as action by the Senate as a body is withheld pending receipt of the House bill. Every law, including RA No. 7720,has in its favor the presumption of constitutionality It is a well-entrenched jurisprudential rule that on the side of every law lies the presumption of constitutionality. Consequently, for RA No. 7720 to be nullified, it must be shown that there is a clear and unequivocal breach of the Constitution, not merely a doubtful and equivocal one; in other words, the grounds for nullity must be clear and beyond reasonable doubt.

POWERS AND FUNCTIONS


A. Governmental powers and functions HELD: the municipality cannot be held liable for the torts committed by its regular employee, who was then engaged in the discharge of governmental functions. The test of liability of the municipality depends on whether or not the driver, acting in behalf of the municipality, is performing governmental or proprietary functions. The municipal corporations exist in a dual capacity, and their functions are twofold. In one they exercise the right springing from sovereignty, and while in the performance of the duties pertaining thereto, their acts are political and governmental. Their officers and agents in such capacity, though elected or appointed by them, are nevertheless public functionaries performing a public service, and as such they are officers, agents, and

Municipality of San Fernando v. Firme


A collision occurred involving (1) a passenger jeepney driven by Bernardo Balagot and owned by the Estate of Macario Nieveras, (2) a gravel and sand truck driven by Jose Manandeg and owned by Tanquilino Velasquez and (3) a dump truck of the petitioner Municipality of San Fernando and driven by Alfredo Bislig. Several passengers of the jeepney including Laureano Bania Sr. died as a result of the injuries they sustained and 4 others suffered physical injuries.

D2010 22

UP College of Law
servants of the state. In the other capacity the municipalities exercise a private, proprietary or corporate right, arising from their existence as legal persons and not as public agencies. Their officers and agents in the performance of such functions act in behalf of the municipalities in their corporate or individual capacity, and not for the state or sovereign power." It has already been remarked that municipal corporations are suable because their charters grant them the competence to sue and be sued. Nevertheless, they are generally not liable for torts committed by them in the discharge of governmental functions and can be held answerable only if it can be shown that they were acting in a proprietary capacity. The driver of the dump truck of the municipality insists that he was on his way to the Naguilian river to get a load of sand and gravel for the repair of San Fernando's municipal streets. In the absence of any evidence to the contrary, the regularity of the performance of official duty is presumed. Hence, the municipality cannot be held liable for the torts committed by its regular employee, who was then engaged in the discharge of governmental functions. Hence, the death of the passenger tragic and deplorable though it may be imposed on the municipality no duty to pay monetary compensation B. Proprietary powers and functions The North Cemetery is a patrimonial property of the City of Manila which was created by resolution of the Municipal Board. The City of Manila prescribes the procedure and guidelines for the disposition of burial lots and plots within the cemetery through Admin. Order No. 5, s.1975. With said acts of dominion, the cemetery is no doubt within the class of property which the City of Manila owns in its proprietary or private character. There is no dispute that the burial lot was leased in favor of the private respondents. Hence, obligation arising from contracting parties. Thus, a lease contract executed by the lessor and lessee remains as the law between them. Therefore a breach of contractual provision entitles the other party to damages even if no penalty of such breach is prescribed in the contract. Hence, the breach of a contractual obligation between the City of Manila and plaintiff, involving property which is patrimonial in character entitles the latter to damages.

City of Manila v. IAC


Vicencio Sto. Domingo, deceased husband of plaintiff Irene Domingo was buried in a lot of the North Cemetery which lot was leased by the City of Manila to Irene from 1971 to 2021. Irene paid the full rental thereof. In January of 1978, the cemetery authorities exhumed and removed the remains of Vivencio from the lot which were then placed in a bag and kept in the bodega of the cementery. When Irene went to the cemetery for All Souls Day, she was shocked to learn that the remains of her husband were not anymore in the lot as the same had been rented out to another lessee. HELD: The City of Manila is a political body corporate and as such endowed with the faculties of municipal corporations to be exercised by and through its city government in conformity with law, and its proper corporate name. It may sue and be sued and contract and be contracted with. Its powers are two fold in character, public, governmental or political on one hand, and corporate private and proprietary on the other hand.

23 D2010

Local Government

Part II Decentralization; Local Autonomy; Powers of Municipal Corporations


ARTICLES/REFERENCES
Public Corporations, Chapters IV VII (Martin)
Chapters IV VI, see above Sec. 11. Selection and Transfer of Local Government Site, Offices and Facilities The law or ordinance creating or merging local government units shall specify the seat of government from where governmental and corporate services shall be delivered. Factors relevant in selection of seat of government: geographical centrality, accessibility, availability of transportation and communication facilities, drainage and sanitation, development and economic progress, and other relevant considerations. Transfer of seat of government: When conditions and developments in the LGU concerned have significantly changed Public hearing and two-thirds vote of all the members of the Sanggunian No transfer shall be made outside the territorial boundaries of the LGU. The old site and improvements thereon may be disposed of by sale or lease or converted to such other use as the Sanggunian concerned may deem beneficial to the LGU concerned and its inhabitants.

CHAPTER SEVEN: Powers of municipal corprorations Sources of Powers of Municipal Corporations: 1. 2. 3. 4. Constitution Statutes of the State Charter In some states which adhere to it, the doctrine of the inherent right of self-government, with respect to certain municipal matters

Classification of municipal powers 1. Express Powers are those granted in express word by the special charter or the general law under which the corporation is organized Implied Powers are those powers which arise by natural implication from the grant of express powers or by necessary inference from the purposes or functions of the corporation Inherent Powers are those which are necessary and inseparable from every corporation, and which come into existence as a matter of course as soon as a municipality is created. They are the common-law powers of a corporation Legislative Power is the authority to make laws. This power is generally vested in the common council. Executive Power is the authority to enforce laws, or appoint the agents charged with the duty of such enforcement; generally vested in the mayor and the heads of the designated departments and other officers created by law.

2.

3.

4.

Transfer, relocation, and conversion to other uses of local government offices and facilities: Public hearings conducted for the purpose i. Concurrence of the majority of all the members of the Sanggunian

5.

Sec. 12. Government Centers. Government center - where offices, agencies, or branches of the National Government, LGUs, or

Chapter II Book I , LGC (Sec 6 24)


Sec 6 10, see above

D2010 24

UP College of Law
government-owned or controlled corporations may be located. LGUs, in designating such a center, shall take into account the existing facilities of national and local agencies and offices which may serve as the government center. The National Government, LGU, or government-owned or controlled corporation concerned shall bear the expenses for the construction of its buildings and facilities in the government center. Sec. 13. Naming of Local Government Units and Public Places, Streets and Structures. By the Sanggunian Of LGUs, public places, streets, and structures within their territorial jurisdiction (see below for detailed list) In consultation with PHC o In changes of name of public schools, upon recommendation of the local school board o In changes of names of publc hospitals, health centers, and other health facilities, upon recommendation of the local health board With notice to the Office of the President, the representative of the legislative district concerned, and the Bureau of Posts Ratified in a plebiscite conducted for the purpose in the political unit directly affected Public vocational or technical schools and other post-secondary and tertiary schools Provincial hospitals, health centers, and other health facilities Any other public place or building owned by the provincial government.

The Sanggunians of highly urbanized cities and independent component cities (i.e., component cities whose charters prohibit their voters from voting for provincial elective officials) may change the names of the following: City barangays, upon the recommendation of the sangguniang barangay concerned City roads, avenues, boulevards, thoroughfares, and bridges Public elementary, secondary and vocational or technical schools, community colleges and nonchartered colleges City hospitals, health centers and other health facilities Any other public place or building owned by the city government.

The Sanggunians of component cities and municipalities may change the names of the following: City and municipal barangays, upon recommendation of the sangguniang barangay concerned City, municipal and barangay roads, avenues, boulevards, thoroughfares, and bridges City and municipal public elementary, secondary and vocational or technical schools, postsecondary and other tertiary schools City and municipal hospitals, health centers and other health facilities Any other public place or building owned by the municipal government.

Limitations: Cannot be named after a living person Justifiable reason Not oftener than once in ten years The name of a LGU or a public place, street or structure with historical, cultural, or ethnic significance shall not be changed, unless by a unanimous vote of the Sanggunian concerned and in consultation with the PHC.

Sec. 14. Beginning of Corporate Existence. An LGUs corporate existence commences upon the election and qualification of its chief executive and a majority of the members of its Sanggunian, UNLESS some other time is fixed therefor by the law or ordinance creating it. Sec. 15. Political and Corporate Nature of Local Government Units. Every LGU is a body politic and corporate endowed with powers to be exercised by it in conformity with law.

The Sangguniang Panlalawigan may change the names of the following: Component cities and municipalities, upon the recommendation of the Sanggunian concerned (i.e., Sanggunian of the component city and municipality Provincial roads, avenues, boulevards, thoroughfares, and bridges

25 D2010

Local Government
As such, LGUs exercise powers as a political subdivision of the National Government as a corporate entity representing the inhabitants of its territory Health and social welfare services which include maintenance of barangay health center and day-care center Services and facilities related to general hygiene and sanitation, beautification, and solid waste collection Maintenance of katarungang pambarangay Maintenance of barangay roads and bridges and water supply systems Infrastructure facilities such as multi-purpose hall, multipurpose pavement, plaza, sports center, and other similar facilities Information and reading center Satellite or public market, where viable

Sec. 16. General Welfare. Every LGU shall exercise powers expressly granted necessarily implied therefrom necessary, appropriate, or incidental for its efficient and effective governance essential to the promotion of the general welfare.

For Municipalities Extension and on-site research services and facilities related to agriculture and fishery activities which include dispersal of livestock and poultry, fingerlings, and other seedling materials for aquaculture; palay, corn, and vegetable seed farms; medicinal plant gardens; fruit tree, coconut, and other kinds of seedling nurseries; demonstration farms; quality control of copra and improvement and development of local distribution channels, preferably through cooperatives inter-barangay irrigation systems; water and soil resources utilization and conservation projects; and enforcement of fishery laws in municipal waters including the conservation of mangroves Pursuant to national policies and subject to supervision, control and review of the DENR, implementation of community-based forestry projects which include integrated social forestry programs and similar projects; management and control of communal forest with an area not exceeding fifty (50) square kilometers; establishment of tree parks, greenbelts, and similar forest development projects Subject to the provisions of Title Five, Book I of the LGC, health services which include the implementation of programs and projects on primary health care, maternal and child care, and communicable and non-communicable disease control services; access to secondary and tertiary health services; purchase of medicines, medical supplies, and equipment needed to carry out the services herein enumerated Social welfare services which include programs and projects on child and youth welfare, family

LGUs shall ensure and support, among other things, the preservation and enrichment of culture, promote health and safety, enhance the right of the people to a balanced ecology, encourage and support the development of appropriate and self-reliant scientific and technological capabilities, improve public morals, enhance economic prosperity and social justice, promote full employment among their residents, maintain peace and order, and preserve the comfort and convenience of their inhabitants. Sec. 17. Basic Services and Facilities. LGUs shall endeavor to be self-reliant and continue exercising the powers and discharging the duties and functions currently vested upon them discharge the functions and responsibilities of national agencies and offices devolved to them pursuant to the LGC exercise such other powers and discharge such other functions and responsibilities as are necessary, appropriate, or incidental to efficient and effective provision of the basic services and facilities

Basic services and facilities (not an exclusive list) For Barangays Agricultural support services which include planting materials distribution system and operation of farm produce collection and buying stations

D2010 26

UP College of Law
and community welfare, women's welfare, welfare of the elderly and disabled persons; community-based rehabilitation programs for vagrants, beggars, street children, scavengers, juvenile delinquents, and victims of drug abuse; livelihood and other pro-poor projects; nutrition services; and family planning services Information services which include investments and job placement information systems, tax and marketing information systems, and maintenance of a public library Solid waste disposal system or environmental management system and services or facilities related to general hygiene and sanitation Municipal buildings, cultural centers, public parks including freedom parks, playgrounds, and other sports facilities and equipment, and other similar facilities Infrastructure facilities intended primarily to service the needs of the residents of the municipality and which are funded out of municipal funds including, but not limited to, municipal roads and bridges; school buildings and other facilities for public elementary and secondary schools; clinics, health centers and other health facilities necessary to carry out health services; communal irrigation, small water impounding projects and other similar projects; fish ports; artesian wells, spring development, rainwater collectors and water supply systems; seawalls, dikes, drainage and sewerage, and flood control; traffic signals and road signs and similar facilities Public markets, slaughterhouses and other municipal enterprises Public cemetery Tourism facilities and other tourist attractions, including the acquisition of equipment, regulation and supervision of business concessions, and security services for such facilities Sites for police and fire stations and substations and the municipal jail cooperatives, and other collective organizations, as well as the transfer of appropriate technology Industrial research and development services, as well as the transfer of appropriate technology Pursuant to national policies and subject to supervision, control and review of the DENR, enforcement of forestry laws limited to community-based forestry projects, pollution control law, small-scale mining law, and other laws on the protection of the environment; and mini-hydro electric projects for local purposes Subject to the provisions of Title Five, Book I of this Code, health services which include hospitals and other tertiary health services Social welfare services which include programs and projects on rebel returnees and evacuees; relief operations and, population development services Provincial buildings, provincial jails, freedom parks and other public assembly areas, and other similar facilities Infrastructure facilities intended to service the needs of the residents of the province and which are funded out of provincial funds including, but not limited to, provincial roads and bridges; inter-municipal waterworks, drainage and sewerage, flood control, and irrigation systems; reclamation projects; and similar facilities Programs and projects for low-cost housing and other mass dwellings, except those funded by the Social Security System (SSS), Government Service Insurance System (GSIS), and the Home Development Mutual Fund (HDMF): Provided, That national funds for these programs and projects shall be equitably allocated among the regions in proportion to the ratio of the homeless to the population Investment support services, including access to credit financing Upgrading and modernization of tax information and collection services through the use of computer hardware and software and other means Inter-municipal telecommunications services, subject to national policy guidelines Tourism development and promotion programs

For Provinces Agricultural extension and on-site research services and facilities which include the prevention and control of plant and animal pests and disease; dairy farms, livestock markets, animal breeding stations, and artificial insemination centers; and assistance in the organization of farmers' and fishermen's

For Cities All the services and facilities of the municipality and province

27 D2010

Local Government
Adequate communication and transportation facilities Support for education, police and fire services and facilities Public works and infrastructure projects and other facilities, programs and services funded by the National Government under the annual General Appropriations Act, other special laws, pertinent executive orders, and those wholly or partially funded from foreign sources, are not covered under this section, except in those cases where the local government unit concerned is duly designated as the implementing agency for such projects, facilities, programs, and services. of rank, salary or tenure Regional offices of national agencies or offices whose functions are devolved to LGUs shall be phased out within one year from the approval of the LGC. National agencies and offices may establish such field units as may be necessary for monitoring purposes and providing technical assistance to LGUs. The properties, equipment, and other assets of these regional offices shall be distributed to the LGUs in the region in accordance with the rules and regulations issued by the Oversight Committee.

The designs, plans, specifications, testing of materials, and the procurement of equipment and materials from both foreign and local sources necessary for the provision of the foregoing services and facilities shall be undertaken by the LGU concerned, based on national policies, standards and guidelines. Devolution Act by which the National Government confers power and authority upon the various local government units to perform specific functions and responsibilities National agencies or offices shall devolve to local government units the responsibility for the provision of basic services and facilities within six (6) months after the effectivity of the LGC. o Includes the transfer to LGUs of the records, equipment, and other assets and personnel of national agencies and offices corresponding to the devolved powers, functions, and responsibilities. o Personnel of said national agencies or offices shall be absorbed by the LGUs to which they belong or in whose areas they are assigned to the extent that it is administratively viable as determined by the Oversight Committee o The rights accorded to personnel pursuant to civil service law, rules and regulations shall not be impaired Regional directors who are career executive service officers and other officers of similar rank in the said regional offices who cannot be absorbed by the LGU shall be retained by the National Government, without any diminution

The National Government or the next higher level of local government unit may provide or augment the basic services and facilities assigned to a lower level of local government unit when: Services or facilities are not made available If made available, they are inadequate to meet the requirements of its inhabitants

Funding of basic services and facilities From the share of local government units in the proceeds of national taxes and other local revenues and funding support from the National Government, its instrumentalities and government-owned or controlled corporations which are tasked by law to establish and maintain such services or facilities To ensure the active participation of the private sector in local governance, LGUs may, by ordinance, sell, lease, encumber, or otherwise dispose of public economic enterprises owned by them in their proprietary capacity Costs may also be charged for the delivery of basic services or facilities enumerated in this Section. Any fund or resource available for the use of local government units shall be first allocated for the provision of basic services or facilities before applying the same for other purposes, unless otherwise provided in the LGC

Sec. 18. Power to Generate and Apply Resources. LGUs proprietary powers: to establish an organization that shall be responsible for the efficient and effective

D2010 28

UP College of Law
implementation of their development plans, program objectives and priorities to create their own sources of revenue and to levy taxes, fees, and charges which shall accrue exclusively for their use and disposition and which shall be retained by them to have a just share in national taxes which shall be automatically and directly released to them without need of any further action to have an equitable share in the proceeds from the utilization and development of the national wealth and resources within their respective territorial jurisdictions including sharing the same with the inhabitants by way of direct benefits to acquire, develop, lease, encumber, alienate, or otherwise dispose of real or personal property held by them in their proprietary capacity and to apply their resources and assets for productive, developmental, or welfare purposes, in the exercise or furtherance of their governmental or proprietary powers and functions and thereby ensure their development into self-reliant communities and active participants in the attainment of national goals. fair market value at the time of the taking of the property. Sec. 20. Reclassification of Lands. City or municipality Through an ordinance passed by the Sanggunian After public hearing for the purpose In the following cases: o when the land ceases to be economically feasible and sound for agricultural purposes as determined by DA o where the land shall have substantially greater economic value for residential, commercial, or industrial purposes, as determined by the Sanggunian concerned: Limited to the following percentages of the total agricultural land area at the time of the passage of the ordinance o Highly urbanized and independent component cities - 15% o Component cities and first to the third class municipalities - 10% o Fourth to sixth class municipalities - 5% BUT President upon recommendation of NEDA may authorize a city or municipality to reclassify lands in excess of the abovementioned limits when public interest so requires Agricultural lands distributed to agrarian reform beneficiaries under R.A. No. 6657 (CARL) shall not be affected. Conversion of such lands into other purposes shall be governed by Section 65 of CARL.

Sec. 19. Eminent Domain. LGUs exercise of power of eminent domain, requisites: Through the chief executive Pursuant to an ordinance For public use, or purpose, or welfare for the benefits of the poor and the landless Upon payment of just compensation Pursuant to the provisions of the Constitution and pertinent laws Valid and definite offer made to the owner that was not accepted

Sec. 65, CARL Conversion of Lands After the lapse of five (5) years from its award, when the land ceases to be economically feasible and sound for agricultural purposes, or the locality has become urbanized and the land will have a greater economic value for residential, commercial or industrial purposes, the DAR, upon application of the beneficiary or the landowner, with due notice to the affected parties, and subject to existing laws, may authorize the reclassification or conversion of the land and its disposition: Provided, That the beneficiary shall have fully paid his obligation. LGUs shall continue to prepare their respective comprehensive land use plans enacted through zoning

LGU may immediately take possession of the property upon Filing of the expropriation proceedings Deposit with the proper court of at least 15% of the FMV of the property based on the current tax declaration of the property to be expropriated

The amount to be paid for the expropriated property shall be determined by the proper court, based on the

29 D2010

Local Government
ordinances which shall be the primary and dominant bases for the future use of land resources. Requirements for food production, human settlements, and industrial expansion shall be taken into consideration in the preparation of such plans. When approval by a national agency (e.g., DA) is required for reclassification, such approval shall not be unreasonably withheld. Failure to act on a proper and complete application for reclassification within three (3) months from receipt of the same shall be deemed as approval thereof. Sec. 21. Closure and Opening of Roads. Requisites Pursuant to an ordinance With provisions for the maintenance of public safety For the establishment of shopping malls, Sunday, flea or night markets, or shopping areas where goods, merchandise, foodstuffs, commodities, or articles of commerce may be sold and dispensed to the general public Any local street, road, thoroughfare, or any other public place By a duly enacted ordinance

Sec. 22. Corporate Powers. LGUs corporate powers: To have continuous succession in its corporate name To sue and be sued To have and use a corporate seal To acquire and convey real or personal property To enter into contracts To exercise such other powers as are granted to corporations, subject to the limitations provided in this Code and other laws

In cases of permanent closure: Ordinance must be approved by at least twothirds of all the members of the Sanggunian An adequate substitute for the public facility that is subject to closure must be provided when necessary BUT no freedom park shall be closed permanently without provision for its transfer or relocation to a new site (adequate substitute always necessary) o A property permanently withdrawn from public use may be used or conveyed for any purpose for which other real property belonging to the LGU concerned may be lawfully used or conveyed

Corporate seals LGUs may continue using, modify, or change their existing corporate seals Newly established LGUs or those without corporate seals may create their own corporate seals which shall be registered with the DILG Any change of corporate seal shall also be registered with DILG.

Contracts Unless otherwise provided in the LGC, no contract may be entered into by the local chief executive in behalf of the LGU without prior authorization by the Sanggunian concerned. A legible copy of the contract shall be posted at a conspicuous place in the provincial capitol or the city, municipal or barangay hall.

Temporary closures During an actual emergency, or fiesta celebrations, public rallies, agricultural or industrial fairs, or an undertaking of public works and highways, telecommunications, and waterworks projects Any national or local road, alley, park, or square Duration shall be specified by the local chief executive concerned in a written order No national or local road, alley, park, or square shall be temporarily closed for athletic, cultural, or civic activities not officially sponsored, recognized, or approved by the LGU

Fiscal autonomy LGUs shall enjoy full autonomy in the exercise of their proprietary functions and in the management of their economic enterprises, subject to the limitations provided in the LGC and other applicable laws.

Sec. 23. Authority to Negotiate and Secure Grants.

D2010 30

UP College of Law
Local chief executives may negotiate and secure financial grants or donations in kind from local and foreign assistance agencies upon authority of the Sanggunian, in support of the basic services or facilities enumerated under Sec. 17 without necessity of securing clearance or approval therefor from any department, agency, or office of the National Government of from any higher LGU o BUT projects with national security implications shall be approved by the national agency concerned o When the national agency fails to act on the request for approval within thirty (30) days from receipt thereof, the same shall be deemed approved report of nature, amount, and terms to both Houses of Congress and the President within thirty (30) days upon signing of such grant agreement or deed of donation First: attached to the main trunk of municipal authority; relates to ordinances and regulations necessary to carry into effect and discharge the powers and duties conferred upon the municipal council by law. Second: much more independent of the specific functions of the council which are enumerated by law; authorizes ordinances that seem to be necessary and proper to provide for the health and safety, promote the prosperity, improve the morals, peace, good order, comfort and convenience of the municipality and the inhabitants, and for the protection of property. General Rule: Ordinances passed by virtue of the implied power of the general welfare clause must be: Reasonable Consonant with the general powers and purposes of the corporation Not inconsistent with the laws or policy of the State

Sec. 24. Liability for Damages. LGUs and their officials are not exempt from liability for death or injury to persons or damage to property.

The powers of the municipal corporations are to be construed strictissimi juris, and any doubt or ambiguity must be construed against the municipality. Exercise of police power may be judicially inquired into and corrected only if it is capricious, whimsical, unjust or unreasonable, there having been a denial or due process or a violation of any other applicable constitutional guarantee. Zoning A zoning ordinance or regulation is a valid exercise of police power and has the effect of nullifying or superseding contractual obligations. The rule of nonimpairment of contracts is not absolute it must be reconciled with the legitimate exercise of police power. Laws and reservation of essential attributes of sovereign power are read into contracts agreed upon by parties and they form part of, and are read into, every contract, unless clearly excluded in cases where exclusion is allowed. Police power cannot be surrendered or bargained away through the medium of a (lease) contract earlier executed. Police power may be activated anytime. Financial assistance LGU may use unappropriated available public funds for extending financial assistance to qualified (indigent) bereaved families. Public purpose is not unconstitutional

Compendium on Local Government, Chapters 5 6 (Agra)


CHAPTER FIVE: Police Power, Power of Eminent Domain, General Powers and Authority A. Police power, general welfare clause Police power is inherent in the State, but not in municipal corporations. There must be a valid delegation of such power by the National Legislature (which is the repository of inherent powers of the State) in order for the MC to exercise such power. MCs exercise such power under the general welfare clause. The power is broad and is said to be commensurate with but not exceeding the duty to provide for the real needs of the people in their health, safety, comfort and convenience, and consistently as may be with private rights. Police power is said to be the most essential, insistent, and illimitable of powers, and in a sense, the greatest and most powerful attribute of government. To secure the general welfare of the State and the fundamental aim of government, the rights of the individual may be subordinated. Two Branches of the General Welfare Clause

31 D2010

Local Government
merely because it incidentally benefits a limited number of persons. (However, may not be applied to heirs of deceased local government officials as financial assistance.) Improper exercise of police power The prohibition of establishment of legitimate enterprises (such as night clubs and cabarets) is not valid. Under B.P. 337, local governments are only empowered to regulate their operations. Confiscation of products A city mayor has no authority to cause the seizure/confiscation of meat products in contravention of a city ordinance, as it is a violation of due process requirements. B. Power of eminent domain ordinance invalid for being unnecessary considering there are other available lots. Role of national government agencies The approval of the national government is not required for local governments to exercise its power of eminent domain. Stages in actions for expropriation First: Determination of authority of plaintiff to exercise such power, and the propriety of its exercise in the context of the facts involved. Ends with an order (if not dismissal) of condemnation declaring the lawful right of plaintiff to take the property. Such order is a final order. Second: Determination by the court of the just compensation as of the date of the filing of the complaint, with the assistance of not more than three commissioners. Such order fixing the just compensation shall be final as well. Public purpose or use Under the new concept, public use means public advantage, convenience or benefit which tends to contribute to the general welfare and prosperity of the whole community. Right to take property Only after the deposit of just compensation. Complaint for eminent domain A complaint which failed to mention the existence of a valid and definite offer, and that such offer was not accepted but alleges that repeated negotiations were made but failed is sufficient to show cause of action. C. General powers and authority

As exercised by LGUs, it is only a delegated power. The statutes conferring such power cannot be broadened or constricted by implication. As a right, it is founded on genuine necessity, and the necessity must be of public character and for the public good. Therefore, LGUs may not capriciously choose what private property should be taken. Courts have the power to inquire into the legality of the exercise of the rights and to determine whether there is genuine necessity therefore. Requisites for the Valid Exercise of Eminent Domain: Ordinance must be passed authorizing the local chief executive to subject a certain property to expropriation Public use, purpose or welfare of poor and landless Payment of just compensation Valid and definite offer to pay property, which was not accepted

A municipal ordinance authorizing the mayor to file expropriation proceedings must be approved by the provincial board. Role of higher/supervising local government Sangguniang panlalawigan has the power to declare a municipal ordinance providing for the exercise of eminent domain invalid on the SOLE GROUND that it is beyond the power of the sangguniang bayan or the mayor to issue. Therefore, the SP cannot declare the

Restraint of Trade Sangguniang Panlalawigan cannot totally ban the buying and selling of all kinds of liquor since this is tantamount to restraint of trade. Granting it may be done, it must be expressly provided for by the law. However, in order to promote general welfare the State may interfere with personal liberty, property, business and occupations. Thus, a person may be subjected to certain kinds of restraints and burdens in order to secure the general welfare of the State. Compulsory Processes the contempt power of the national legislature is sui generis, as its exercise is a

D2010 32

UP College of Law
matter of self-preservation (it asserts its authority as one of the three independent and coordinate branches of the govt, independent of the judicial branch and punishes contempt) and local legislative bodies cannot correctly claim to possess it for the same reasons the national legislature does. As the contempt power and subpoena power partake of a judicial nature, they cannot be implied in the grant of legislative power. If there is no express statutory basis, it would run afoul of the doctrine of the doctrine of separation of powers. This must be considered an exception to Sec. 4 of B.P. 337, which provides for liberal rules of interpretation in favour of local autonomy. LGUs cannot proclaim religious or local holidays such power rests within the President Reclassificaton of Land the authority of the Sanggunian is limited to the reclassification of agricultural lands. The power of cities and municipalities to reclassify agricultural land into commercial, industrial and residential status is only for the purpose of assessment and real property taxation. Reclassification power lodged with the LGU; act of allocating lands to different activities or classes of land uses, evolved and enacted through local planning and zoning processes. DAR approval is not necessary. Land Conversion power lodged with DAR; actual change in land use and takes into account tenants and farmworkers, if any, and ascertainment of disturbance compensation. HOWEVER, such power to issue conversion clearance and/or approve/disapprove applications can only be exercised on or after June 15, 1988, the date of the effectivity of the Comprehensive Agrarian Reform Law (CARL). Role of DAR the power of DAR to approve or disapprove conversions is limited to the applications for reclassification submitted by land owners or tenant beneficiaries. DARs authority to convert agricultural land should be exercised in conjunction with the devolved powers of the LGUs to reclassify such land. However, once a landholding has been acquired at redistributed to qualified beneficiaries pursuant to CARL, it is excluded from the authority to LGUs to reclassify. Nothing in the LGC shall be construed to repeal, modify or amend the CARL. Prescribing penal provisions Sangguniang barangay cannot provide for the [enalty of imprisonment for violations of barangay ordinances An LGU must comply with the legal conditions imposed on a donation. LGC only provides for the imposition of a fine. Forfeiture of salary likewise cannot be validly prescribed. Sangguniang barangay cannot enact an ordinance identical to an ordinance of the city of municipality but with a lesser penalty, since the former is inconsistent with the latter. Contracting Loans LGU cannot contract external or foreign loans since LGC only provides for guarantee by the President of local or domestic loans. Conduct of Legislative Inquiry municipal mayor cannot require that all heads of departments and EEs obtain his clearance and permission before appearing before any governmental entity. Scholarship Grants scholarship fund may only be applied to schools within its jurisdiction. Legislative Voting Requirement local legislative council cannot provide for more than majority vote for the passage of appropriations ordinances, since LGC requires only simple majority. However, sanggunian may provide for different vote requirement for other certain ordinances. Change of Name of Government Center prior consultation with Philippines Historical Commission required Requiring Performance Reports may not be reqd by sangguniang panlalawigan because it may cause work disruption, and such function is essentially executive, not legislative. Regulation of Property public property is outside the commerce of man, therefore cannot be the subject of lease or contract, and constructions thereon can be summarily abated by the LGU. The power to regulate public property is with the LGU. Public Plaza Public Streets road lots in a private subdivision are private property, hence local governments must first acquire them by donation, purchase or expropriation if they wish to utilize them Land reclaimed by the PEA for and on behalf of the State is no longer part of the public domain and public use.

33 D2010

Local Government
Public Markets Buildings Disposal of Real Property is Proper When: Public bidding is conducted Price offered by vendee is higher than the appraised value Deed of sale is signed by the local chief executive, with the authority of the sanggunian Certification is issued that said lot is no longer needed for public use as duly verified by the auditor Logging Activities DENRs powers cannot be encroached upon by the LGU Quarrying Activities provincial governors authority to grant and issue quarry permits extends only to public lands Fishing, Fishery Privileges Maintaining Dumpsites must not endanger environment, health, safety and welfare of residents Littering fine may be validly imposed Power to Enter into Contracts Contracts entered into by local chief executives have the force of law between parties and should be complied with. A chief executive acting pursuant to a resolution already adopted by the council in signing the deed of sale to qualified buyers determined after public bidding was exercising a purely ministerial duty incidental to his functions. Council/Sanggunian authorization is a condition sine qua non for the validity of a contract entered into by a local chief executive. Authorization may take the form of a resolution. Traffic Regulation temporary street closures may be done through an ordinance Issuance of Permits the issuance, revocation or cancellation of permits is a discretionary act subject to strict implementation as to its scope Authority to issue business permits are subject to the regulatory powers of the city mayor Rejection of application a mayor may refuse the granting of a permit only if there are valid reasons embodied in an appropriate ordinance or national law; in the absence of such law or ordinance, the mayor may not validly refuse to grant the permit to a legitimate enterprise due to the principle of free enterprise and competition; also, issuance may not be withheld based on none-payment of taxes and imposts Cancellation of permit may not be revoked if operator was not informed of a specific violation of the LGC, as it will be violative of due process

Transfer of Property to Local Governments lots covered by a Certificate of Land Ownership Award (CLOA) issued pursuant to CARL can only be transferred or acquired through the DAR in order to be reallocated to another beneficiary. The term government does not contemplate its political subdivisions. Use of Land a municipality may change the use of a piece of land if there are no such express restrictions in the contract to sell/contract of sale; it may also enter into a joint venture agreement with a private entity embodied in a MOA signed by the local chief executive, a representative of the private entity and ratified by the sanggunian; HOWEVER, a chapel may not be constructed on land owned by the govt based on the separation of church and state As a LESSOR LGU may file an action for illegal detention and demand eviction for violation of lease contract and non-payment of rentals Abatement of Public Nuisance Local government officials cannot seek cover under the general welfare clause authorizing the abatement of nuisances without judicial proceedings. This tenet applies to a nuisance per se, which affects the immediate safety of persons and property, and may be summarily abated under the undefined law of necessity. If it be a nuisance per accidens, it may be proven in a hearing conducted for that purpose. It is not per se a nuisance warranting its abatement without judicial intervention. While the Sangguniang Bayan may provide for the abatement of a nuisance, it cannot declare a particular thing as a nuisance per se and order its condemnation. It can only be so adjudged by judicial determination. Land Use Planning

D2010 34

UP College of Law
Franchises power to issue National Franchises lies with the National Government, devolution is only with respect to regulatory powers within the jurisdictions concerned. However, the grant of franchises for the establishment, construction, operation and maintenance of public markets and bus/jeepney terminals are within the concern of the sanggunians. Although the municipality has the authority to grant franchises, the authority to collect franchise tax is under the power of the province and not the municipality. Coal Corporations power of regulation lies with the DOE Casinos, Gambling power of LGUs to suppress gambling refers only to illegal gambling Demolition Padlocking of Premises Local Infrastructure Projects authority of LGUs to undertake reclamation projects is limited to those funded out of local funds; projects funded by the National Govt are lodged with the PEA; regular courts are prohibited from issuing writs to stop any person, entity, government official or LGU from proceeding with or continuing the execution or implementation of amn infrastructure project approved by the President through the Executive Secretary (P.D. 3-A). Creation of LGUs the power to create political subdivisions is a function of the legislature (ex. conversion of municipal districts into regular municipalities) Internal Revenue Allotments (IRA) is included in computation of average annual income (part of general income of govt units) Population requirement of 5,000 for the creation of a barangay within a highly urbanized city is mandatory (but does not apply to those already existing) Registered voters of highly urbanized cities shall be prohibited from voting in elections at the provincial level, unless reclassification occurs after ratification of 1987 Constitution, but before effectivity of LGC of 1991. Plebiscite - to be conducted in the political units directly affected, (1) meaning residents of the political entity who would be economically dislocated by the separation have a right to vote, and (2) referring to the plurality of political units which would participate; whole unit must participate, not merely those that form part of the new unit Applies only to new LGUs created for the first time under the 1987 Constitution therefore no plebiscite is necessary in the case of a municipal corporation which has attained de facto status at the time the 1987 Constitution took effect. In case of a negative vote, sub-province shall continue to be part of the original province, to be represented by officials of the original province. There is no law authorizing the holding of special elections for the first set of barangay officials of newly-created barangays, therefore, such election cannot be conducted together with the SK elections.

Abolition of LGUs enactment of an ordinance by sangguniang panlalawigan or panlungsod concerned and the conduct of a plebiscite required Principle of non-user only applies to private corporation law; does not apply to municipal corporations Conversion of LGUs bills of local application must originate from the House of Representatives and initiate the legislative process which would culminate in the enactment of a Statute. A technical description similar to those in Torrens titles is not a condition sine qua non (it would defeat the purpose of the LGC). What is required is a reasonable ascertainment of the area. Reapportionment of Legislative Districts may be made by a special law, such as the enactment by special law of a charter of a new city. Congress may increase its own composition through legislative enactment. Settling Boundary Disputes Between barangays referred to sangguniang bayan or panlungsod concerned; Office of the President has no jurisdiction Between adjacent municipalities within a province elevated to sangguniang panlalawigan Between independent component city and a municipality, or highly urbanized cities and municipalities jointly referred to respective sanggunians

Barangay Clearances barangays are only authorized to issue clearances for business and impose reasonable

35 D2010

Local Government
fees, but are not allowed to issue business permits or licenses. Barangay clearance cannot be denied on grounds other than those specified in the appropriate ordinance. Warrants of Arrest mayors are no longer authorized to issue such warrants Election Activities COMELEC has exclusive jurisdiction over cases involving the enforcement of the Election Code; the transfer of officers and employees within the election period is prohibited, except for the purpose of coping with emergencies and efficiency in the government service Abolition of an Office express power to create local offices (absent any contrary provision), impliedly carries with it the power to abolish said office in GOOD FAITH. Relocation of Homeless responsibility of both LGU and National Housing Authority Inclusion in Special Economic Zones R.A. 7227 provides that the creation of the Subic Special Economic Zone is subject to the concurrence of concerned municipalities and cities by resolution. However, the Subic Authority shall prevail in conflicts concerning matters affecting the zone. Assistance to Sectors public purpose is not unconstitutional merely because it incidentally benefits a limited number of persons Creation of Fishery Resources Management Council by way of Ordinance Appointment to Local Offices punong barangays may appoint purok leaders, provided the appropriate ordinance has been enacted and the council thereafter approves the appointment. Projects, Countrywide Development Fund sanggunian is authorized to compel a congressman to seek its prior approval before the implementation of any projects, as the LGC provides that national projects must be approved by the sanggunian prior to their implementation. However, an appropriations ordinance is not required to facilitate the release of funds from the Countrywide Development Fund of Representatives of Congress. Legislation, however, must be enacted specifying the infrastructure and priority projects. Issuance of Bonds any security issued or guaranteed by the govt or any of its political subdivisions is exempt from registration; exempt securities Ultra Vires Acts of Private Organizations local chief executives may file a complaint against such organizations with the appropriate national government unit pursuant to a resolution adopted by the sanggunians Additional Insurance Benefits council by way of ordinance may not increase insurance benefits of municipalities officials and EEs since this would unreasonable add up to the tax burden of the inhabitants, resulting in the violation of the principle forbidding the appropriation of public funds for private purposes Imposition of Curfew may be exercised pursuant to police power, but not by the local chief executive alone. Sanggunian enact an ordinance determine the necessity, reasonableness, condition and procedures. Inter-Local Govt Cooperation local government may group themselves and procure equipment from domestic and foreign sources for purposes commonly beneficial to them, provided national policies, standards and guidelines are followed. Incorporation of Stock Corporations only natural persons can become incorporators of such corporations; prohibition extends to its local officials being agents of the province (principal). CHAPTER SIX: Inter-Local Government Relations Local Separation of Powers Doctrine of Separation of Powers and System of Checksand-Balance apply to local governments. It is the duty of public officers to enforce ordinances not otherwise repealed by the council nor annulled by the courts. Local chief executive (LCE) may validly enter into a contract only with council authorization. Prior authorization is not the same as pre-approval of contracts. Local councils do not possess the authority to pre-approve contracts after prior authorization has been given. Prior authorization also does not mean prior authorization for the payment of obligations. LCE cannot require that all requests for appropriations be endorsed by him before they can be enacted. Designation of members of the Personnel Selection Board as determined by the sanggunian by resolution must be approved by the LCE (merely ministerial).

D2010 36

UP College of Law
Mayor has no administrative supervision over sanggunian EEs; authority to approve applications for LOAs of sanggunian members and appointive EEs rests with the vice LCE. Signature of governor required in the resolution adopted by sangguniang panlalawigan approving or disapproving the ordinance or resolution enacted by sangguniang pankungsod/bayan. While authority to regulate traffic and use of streets rests with the sanggunian, the execution of an ordinance relating to it is the responsibility of the LCE. The authority of the LCE of the higher supervising unit to impose preventive suspension is purely ministerial, since the disciplinary authority over erring local legislative officials of the supervised unite is the sanggunian of the higher unit. Sanggunian is in the best position to determine rates of mayors permit fees to be levied are just, reasonable, and not confiscatory. Additional functions maybe given the vice mayor only as may be provided by law or ordinance, otherwise he may validly refuse. LCE is mandated by law to represent the LGU, no sanggunian authorization necessary. Power of appointment of the vice-mayor is limited to officials and EEs of the sanggunian, as well as EEs of the office of the vice mayor. Authority to discipline municipal sanggunian members and EEs lies with the vice LCE, since he had administrative supervision over EEs, being the one with the power to appoint the same. Barangay Audit Reports to be transmitted to the vice mayor, not the LCE, for information and appropriate action. Mayor is administrator when drawing checks in the settlement of obligations. Barangay kagawads perform tasks assigned pursuant to a valid resolution, and may be required to submit monthly accomplishment forms. Province and component city/municipality Declaration by the sangguniang panlalawigan that a particular city or municipal ordinance or resolution is invalid for being beyond the power conferred upon its respective sanggunians, is equivalent to a disapproval of the subject ordinance or resolution. When a province sells delinquent properties at a public auction, it was not only acting on its behalf but also on behalf of the municipalities concerned. Therefore when the province buys such lot (no other bidder), the municipalities may be considered co-owners thereof to the extent of their respective shares in the real property taxes and penalties thereon. The exercise of the power to tax by a (component) city also granted to a province deprives the province from imposing a similar tax thus exclusive power is granted to the city to collect and levy the subject tax, fees and charges. Real property assessments made by city or municipal assessors are not subject to approval by the provincial assessor the latter merely exercises technical supervision. A municipality is entitled to a share in the proceeds from the real property tax and lease rentals of subject property collected by the province. The power to levy tax on sand and gravel exclusively belongs to the province although a component city or municipality has a share in the proceeds if the said tax. If franchise tax is imposed by the province, a component city of municipality has no share. City/municipality and barangay The power to regulate facilities rests upon the unit which owns the same. In interpreting ambiguous provisions of the LGC, resolution is in favour of the lower LGU. Authority to enter into contracts involving barangay roads within a municipality rests with the barangay affected, not the municipality. The municipality only exercises regulatory powers over municipal roads. Barangay ordinances are subject to review by the city or municipal councils and not the other way around. A municipal mayor does not have authority to control the disbursement of barangay funds and internal revenue allotments, nor withhold the share of the barangay from the internal revenue allotment on the basis of an election protest. He/She also does not have

37 D2010

Local Government
the authority to control barangay projects. Such prerogatives belong to the punong barangay. A city/municipality possesses the POWER to SUPERVISE over component barangays. Such power, however, does not permit infringement upon the legislative powers of the lower LGU to the extent of dictating changes on the policies or decisions. The higher sanggunian must be guided by liberality of construction and fundamental principles of local autonomy. This power does not include the power to restrain, nor does it mean that the sangguniang bayan may invalidate any ordinance enacted by the sangguniang barangay. It may only point out the defect. Approval of vouchers is merely ministerial on the part of the mayor after the mayor after the treasurer (1) has certified the availability of funds and (2) an appropriate ordinance has been enacted and was subsequently approved by him/her. The mayor may not require a punong barangay to personally present the barangay payroll. Only a city and a municipality may issue business permits and licenses. Barangay has no such power. The new sharing scheme provided for by the LGC does not take place automatically. An ordinance must be enacted by the sangguniang bayan or panlungsod concerned in order to install it. Appointment of the barangay treasurer only needs the concurrence of the sangguniang bayan concerned. Confirmation from the sangguniang bayan is not required. Services of the municipal engineer may be secured by a barangay in the implementation of barangay infrastructure projects after prior representation has been made to the LCE concerned. Sangguniang bayan may suspend all barangay officials, which authorizes the mayor to appoint temporary replacements, provided such appointees possess all the necessary qualifications and none of the disqualifications provided by law. Where there is no law which authorizes the holding of special elections to fill-in the positions created by the incorporation of new barangays, the mayor may fill up the vacancies, there being permanent vacancies. If there are no permanent vacancies, neither the mayor nor the punong barangay have the authority to appoint officials. Past and present administrations The newly-elected LCE must abide by the contractual obligations made by the former administration since the party-in-interest is the LGU. However, an authority to negotiate loans granted by the local legislative council to the past LCE does not extend to the newly-elected official. The previous LCE, not the succeeding one, has the authority to observe and evaluate the performance of the employee concerned, where the act complained of was done during the previous administration.

LOCAL GOVERNMENT UNITS VIS A VIS NATIONAL GOVERNMENT


A. Power of general Supervision or modifies or sets aside a tax ordinance, he is not permitted to substitute his own judgment for the judgment of the local government that enacted the measure. The acts of Secretary Drilon in setting aside the Manila Revenue Code, was of mere supervision, not control: he did not replace the MRC with his own version. He did not pronounce the ordinance unwise or unreasonable as a basis for its annulment. He did not say that in his judgment it was a bad law.

Drilon v. Lim
Section 187 LGC authorizes the Secretary of Justice to review the constitutionality or legality of the tax ordinance and, if warranted, to revoke it on either or both of these grounds. Pursuant to this, Secretary of Justice Drilon declared Ordinance No. 7794 (Manila Revenue Code), null and void for non-compliance with the prescribed procedure in the enactment of tax ordinances and for containing certain provisions contrary to law and public policy. HELD: Section 187 of LGC is valid. Under this provision, when the Secretary of Justice alters

D2010 38

UP College of Law
All he did in reviewing the said measure was determine if the petitioners were performing their functions in accordance with law, that is, with the prescribed procedure for the enactment of tax ordinances and the grant of powers to the city government under the LGC. On the other hand, an officer in control lays down the rules in the doing of an act. If they are not followed, he may, in his discretion, order the act undone or re-done by his subordinate or he may even decide to do it himself. Supervision does not cover such authority. The supervisor or superintendent merely sees to it that the rules are followed, but he himself does not lay down such rules, nor does he have the discretion to modify or replace them. If the rules are not observed, he may order the work done or re-done but only to conform to the prescribed rules. He may not prescribe his own manner for the doing of the act. He has no judgment on this matter except to see to it that the rules are followed. All he is permitted to do is ascertain the constitutionality or legality of the tax measure, without the right to declare that, in his opinion, it is unjust, excessive, oppressive or confiscatory. He has no discretion on this matter. 6. must be general and consistent with public policy.

PD 1605 does not allow either the removal of license plates or the confiscation of driver's licenses for traffic violations committed in Metropolitan Manila. There is nothing in the following provisions of the decree authorizing the Metropolitan Manila Commission to impose such sanctions. In fact, the provisions prohibit the imposition of such sanctions in Metropolitan Manila. That the municipal enactment must not violate existing law explains itself. Local political subdivisions are able to legislate only by virtue of a valid delegation of legislative power from the national legislature (except only that the power to create their own sources of revenue and to levy taxes is conferred by the Constitution itself). They are mere agents vested with the power of subordinate legislation. As delegates of the Congress, the LGU cannot contravene but must obey at all times the will of their principal. The enactments in question, which are merely local in origin, cannot prevail against the PD 1605, which has the force and effect of a statute. The measures do not merely add to the requirement of PD 1605 but, worse, impose sanctions the decree does not allow and in fact, prohibits. In so doing, the ordinances disregard and violate and in effect partially repeal the law. Nowhere is the removal of license plates directly imposed by the decree or at least allowed by it to be imposed by the Commission.

Solicitor General v. Metropolitan Manila Authority


SC ruled that the confiscation of license plates by the MMC was not among the powers conferred upon it by its charter (PD 1605). It was also observed by the SC that confiscation of the drivers licenses was not directly prescribed or allowed by PD 1605. SC received several letters-complaint that peoples licenses were confiscated. MMA issued Ordinance No. 11 authorizing itself "to detach the license plate or tow and impound attended or unattended or abandoned motor vehicles illegally parked or obstructing the flow of traffic in Metro Manila." HELD: Ordinance null and void for being an invalid exercise of delegated legislative power. Test for a valid municipal ordinance: 1. 2. 3. 4. 5. must not contravene the Constitution or any statute; must not be unfair or oppressive; must not be partial or discriminatory; must not prohibit but may regulate trade; must not be unreasonable; and

Ganzon v. CA
The petitions of Mayor Ganzon originated from a series of administrative complaints, ten in number, filed against him by various city officials sometime in 1988, on various charges, among them, abuse of authority, oppression, grave misconduct, disgraceful and immoral conduct, intimidation, culpable violation of the Constitution, and arbitrary detention. He was placed in preventive suspension for 3 times by the respondent Secretary of Local Government on different occasions based on different administrative complaints filed against him. Mayor Ganzon assailed the power of the respondent to suspend him alleging that the 1987 Constitution no longer allows the President, as the 1935 and 1973 Constitutions did, to exercise the power of suspension and/or removal over local officials. HELD: The Sec of Local Government, as the alter ego of the President, has the power to suspend local officials.

39 D2010

Local Government
Autonomy does not contemplate making mini-states out of local government units, as in the federal governments of the USA. Autonomy, in the constitutional sense, is subject to the guiding star, though not control, of the legislature, albeit the legislative responsibility under the Constitution - and as the "supervision clause" itself suggest - is to wean local government units from over dependence on the central government. It is noteworthy that under the Charter, "local autonomy" is not instantly self-executing, but subject to, among other things, the passage of a local government code, a local tax law, income distribution legislation, and a national representation law, and measures designed to realize autonomy at the local level. It is also noteworthy that in spite of autonomy, the Constitution places the local government under the general supervision of the Executive. It is noteworthy finally, that the Charter allows Congress to include in the local government code provisions for removal of local officials, which suggest that Congress may exercise removal powers, and as the existing Local Government Code has done, delegate its exercise to the President. Autonomy, however, is not meant to end the relation of partnership and interdependence between the central administration and local government units, or otherwise, to usher in a regime of federalism. The Charter has not taken such a radical step. Local governments, under the Constitution, are subject to regulation, however limited, and for no other purpose than precisely, albeit paradoxically, to enhance self-government. However, the Court held that the successive suspensions were excessive and not proper. What bothers the Court, and what indeed looms very large, is the fact that since the Mayor is facing ten administrative charges, the Mayor is in fact facing the possibility of 600 days of suspension, in the event that all ten cases yield prima facie findings. The Court is not of course tolerating misfeasance in public office (assuming that Ganzon is guilty of misfeasance) but it is certainly another question to make him serve 600 days of suspension, which is effectively, to suspend him out of office. The plain truth is that this Court has been ill at ease with suspensions, for the above reasons, and so also, because it is out of the ordinary to have a vacancy in local government. The sole objective of a suspension, as we have held, is simply "to prevent the accused from hampering the normal cause of the investigation with his influence and authority over possible witnesses" or to keep him off "the records and other evidence." It is a means, and no more, to assist prosecutors in firming up a case, if any, against an erring local official. Under the Local Government Code, it can not exceed sixty days, which is to say that it need not be exactly sixty days long if a shorter period is otherwise sufficient, and which is also to say that it ought to be lifted if prosecutors have achieved their purpose in a shorter span.

Mactan Cebu International Airport Authority v. Marcos et al


The Officer of the Treasurer of Cebu City demanded payment for realty taxes on parcels of land belonging to petitioner MCIAA. Petitioner objected invoking its tax exemption. It also asserted that it is an instrumentality of the government performing governmental functions, citing section 133 of the LGC which puts limitations on the taxing powers of LGUs. The city refused insisting that petitioner is a GOCC performing proprietary functions whose tax exemption was withdrawn by Sections 193 and 234 of the LGC. HELD: There can be no question that under Section 14 RA 6958 the petitioner is exempt from the payment of realty taxes imposed by the National Government or any of its political subdivisions, agencies, and instrumentalities. Nevertheless, since taxation is the rule and exemption is the exception, the exemption may thus be withdrawn at the pleasure of the taxing authority. Tax exemptions or incentives granted to or presently enjoyed by natural or juridical persons, including government-owned and controlled corporations, Section 193 of the LGC prescribes the general rule, viz., they are withdrawn upon the effectivity of the LGC, except upon the effectivity of the LGC, except those granted to local water districts, cooperatives duly registered under R.A. No. 6938, non stock and non-profit hospitals and educational institutions, and unless otherwise provided in the LGC. The terms "Republic of the Philippines" and "National Government" are not interchangeable. The former is boarder and synonymous with "Government of the Republic of the Philippines" which the Administrative Code of the 1987 defines as the "corporate governmental entity though which the functions of the government are exercised through at the Philippines, including, saves as the contrary appears from the context, the various arms through which political authority is made effective in the Philippines, whether pertaining to the autonomous reason, the provincial, city, municipal or barangay subdivision or other forms of local government." These autonomous regions,

D2010 40

UP College of Law
provincial, city, municipal or barangay subdivisions" are the political subdivision. On the other hand, "National Government" refers "to the entire machinery of the central government, as distinguished from the different forms of local Governments." The National Government then is composed of the three great departments the executive, the legislative and the judicial. An "agency" of the Government refers to "any of the various units of the Government, including a department, bureau, office instrumentality, or government-owned or controlled corporation, or a local government or a distinct unit therein;" while an "instrumentality" refers to "any agency of the National Government, not integrated within the department framework, vested with special functions or jurisdiction by law, endowed with some if not all corporate powers, administering special funds, and enjoying operational autonomy; usually through a charter. This term includes regulatory agencies, chartered institutions and government-owned and controlled corporations". MCIAA is not an agency or instrumentality of the government but only a GOCC, thus, LGUs may tax them. B. Decentralization, local autonomy make them more effective partners in the pursuit of national development and social progress." At the same time, it relieves the central government of the burden of managing local affairs and enables it to concentrate on national concerns. The President exercises "general supervision" over them, but only to "ensure that local affairs are administered according to law." He has no control over their acts in the sense that he can substitute their judgments with his own. Decentralization of power, on the other hand, involves an abdication of political power in the favor of local governments units declared to be autonomous. In that case, the autonomous government is free to chart its own destiny and shape its future with minimum intervention from central authorities. According to a constitutional author, decentralization of power amounts to "self-immolation," since in that event, the autonomous government becomes accountable not to the central authorities but to its constituency. But the question of whether or not the grant of autonomy to Muslim Mindanao under the 1987 Constitution involves, truly, an effort to decentralize power rather than mere administration is a question foreign to this petition, since what is involved herein is a local government unit constituted prior to the ratification of the present Constitution. Hence, the Court will not resolve that controversy now, in this case, since no controversy in fact exists. We will resolve it at the proper time and in the proper case. Under the 1987 Constitution, local government units enjoy autonomy in these two senses An autonomous government that enjoys autonomy of the latter category is subject alone to the decree of the organic act creating it and accepted principles on the effects and limits of "autonomy." On the other hand, an autonomous government of the former class is, as we noted, under the supervision of the national government acting through the President (and the Department of Local Government). If the Sangguniang Pampook (of Region XII), then, is autonomous in the latter sense, its acts are, debatably, beyond the domain of this Court in perhaps the same way that the internal acts, say, of the Congress of the Philippines are beyond our jurisdiction. But if it is autonomous in the former category only, it comes unarguably under our jurisdiction. An examination of the very Presidential Decree creating the autonomous governments of Mindanao persuades us that they were never meant to exercise autonomy in the second sense, that is, in which the central government

Limbona v. Mangelin
Petitioner Speaker Alimbusat Limbona was the speaker of the regional legislative assembly of central Mindanao. He was invited to attend a conference and hence he advised acting secretary Alimbuyao to inform the assemblyman that there will be no session on such that he will be away. The Assembly held session in defiance of petitioner's advice. After declaring the presence of a quorum, all present voted that the seat of the speaker be declared vacant. The petitioner then went to court praying that judgment be rendered declaring the proceedings held by respondents during the session and his ouster as null and void. The respondents assails the jurisdiction of the Court to rule upon the issue. HELD: Autonomy is either decentralization of administration or decentralization of power. There is decentralization of administration when the central government delegates administrative powers to political subdivisions in order to broaden the base of government power and in the process to make local governments "more responsive and accountable," and "ensure their fullest development as self-reliant communities and

41 D2010

Local Government
commits an act of self-immolation. Presidential Decree No. 1618, in the first place, mandates that "[t]he President shall have the power of general supervision and control over Autonomous Regions." 33 the second place, the Sangguniang Pampook, their legislative arm, is made to discharge chiefly administrative services. Hence, we assume jurisdiction. And if we can make an inquiry in the validity of the expulsion in question, with more reason can we review the petitioner's removal as Speaker. The expulsion of the petitioner has no force and effect. In the first place, there is no showing that the Sanggunian had conducted an investigation, and whether or not the petitioner had been heard in his defense, assuming that there was an investigation, or otherwise given the opportunity to do so. In the second place, the resolution appears strongly to be a bare act of vendetta by the other Assemblymen against the petitioner arising from what the former perceive to be abduracy on the part of the latter. While it is within the discretion of the members of the Sanggunian to punish their erring colleagues, their acts are nonetheless subject to the moderating hand of this Court in the event that such discretion is exercised with grave abuse. those budgets. They are not formulated in the inner sanctums of an all-knowing DBM and unilaterally imposed on local governments whether or not they are relevant to local needs and resources. It is for this reason that there should be a genuine interplay, a balancing of viewpoints, and a harmonization of proposals from both the local and national officials. It is for this reason that the nomination and appointment process involves a sharing of power between the two levels of government.

Ganzon v. CA (supra)
See above

Cordillera Broad Coalition v. COA


EO 220, issued by the President in the exercise of her legislative powers under Art. XVIII, sec. 6 of the Constitution, created the Cordillera Administrative Region (CAR). Petitioners argue that the creation of CAR contravened the constitutional guarantee of local autonomy for provinces and cities composing it. HELD: It must be clarified that the constitutional guarantee of local autonomy in the Constitution refers to the administrative autonomy of local government units or, cast in more technical language, the decentralization of government authority. Local autonomy is not unique to the 1987 Constitution, it being guaranteed also under the 1973 Constitution. And while there was no express guarantee under the 1935 Constitution, the Congress enacted the Local Autonomy Act (R.A. No. 2264) and the Decentralization Act (R.A. No. 5185), which ushered the irreversible march towards further enlargement of local autonomy in the country. On the other hand, the creation of autonomous regions in Muslim Mindanao and the Cordilleras, which is peculiar to the 1987 Constitution, contemplates the grant of political autonomy and not just administrative autonomy to these regions. Thus, the provision in the Constitution for an autonomous regional government with a basic structure consisting of an executive department and a legislative assembly and special courts with personal, family and property law jurisdiction in each of the autonomous regions. CAR is a mere transitory coordinating agency that would prepare the stage for political autonomy for the Cordilleras. It fills in the resulting gap in the process of transforming a group of adjacent territorial and political subdivisions already enjoying local or administrative

San Juan v. Civil Service Commission


Petitioner governor San Juan recommended Santos to the position of Provincial Budget Officer for Rizal Province. However, in defiance to such recommendation, DBM Regional Officer Galvez appointed Almajosa instead. The governor protested. The DBM issued a memo ruling that petitioners protest is not meritorious as the DBM validly exercised its prerogative in filling-up the contested position since none of the petitioner's nominees met the prescribed requirements. HELD: When the Civil Service Commission interpreted the recommending power of the Provincial Governor as purely directory, it went against the letter and spirit of the constitutional provisions on local autonomy. If the DBM Secretary jealously hoards the entirety of budgetary powers and ignores the right of local governments to develop self-reliance and resoluteness in the handling of their own funds, the goal of meaningful local autonomy is frustrated and set back. Provincial and municipal budgets are prepared at the local level and after completion are forwarded to the national officials for review. They are prepared by the local officials who must work within the constraints of

D2010 42

UP College of Law
autonomy into an autonomous region vested with political autonomy. the power to destroy. The power to grant still includes the power to withhold or recall. True, there are certain notable innovations in the Constitution, like the direct conferment on the local government units of the power to tax, which cannot now be withdrawn by mere statute. By and large, however, the national legislature is still the principal of the local government units, which cannot defy its will or modify or violate it. Casino gambling is authorized by P.D. 1869. This decree has the status of a statute that cannot be amended or nullified by a mere ordinance. Hence, it was not competent for the Sangguniang Panlungsod of Cagayan de Oro City to enact Ordinance No. 3353 prohibiting the use of buildings for the operation of a casino and Ordinance No. 3375-93 prohibiting the operation of casinos. For all their praiseworthy motives, these ordinances are contrary to P.D. 1869 and the public policy announced therein and are therefore ultra vires and void.

Magtajas v. Pryce Properties Corporation


In 1992, representatives from PPC made representations with the Pagcor on the possibility of setting up a casino in Pryce Plaza Hotel in Cagayan de Oro City. On November 1992, the parties executed a contract of lease involving the ballroom of the hotel which would be converted into a casino. Sangguniang Panglungsod of CDO passed several ordinances prohibiting the establishment of gambling casinos. PPC and PAGCOR assail the authority to pass such ordinances. HELD: Ordinances should not contravene a statute. The rational for this requirement is obvious. Municipal governments are only agents of the national government. Local councils exercise only delegated legislative powers conferred on them by Congress as the national lawmaking body. The delegate cannot be superior to the principal or exercise powers higher than those of the latter. It is a heresy to suggest that the local government units can undo the acts of Congress, from which they have derived their power in the first place, and negate by mere ordinance the mandate of the statute. Municipal corporations owe their origin to, and derive their powers and rights wholly from the legislature. It breathes into them the breath of life, without which they cannot exist. As it creates, so it may destroy. As it may destroy, it may abridge and control. Unless there is some constitutional limitation on the right, the legislature might, by a single act, and if we can suppose it capable of so great a folly and so great a wrong, sweep from existence all of the municipal corporations in the State, and the corporation could not prevent it. We know of no limitation on the right so far as to the corporation themselves are concerned. They are, so to phrase it, the mere tenants at will of the legislature. This basic relationship between the national legislature and the local government units has not been enfeebled by the new provisions in the Constitution strengthening the policy of local autonomy. Without meaning to detract from that policy, we here confirm that Congress retains control of the local government units although in significantly reduced degree now than under our previous Constitutions. The power to create still includes

Taule v. Santos
On June 18,1989, the Federation of Associations of Barangay Councils (FABC) of Catanduanes, composed of eleven (11) members convened with six members in attendance for the purpose of holding the election of its officers. The election proceeded with petitioner Ruperto Taule declared as president. The governor protested to the Secretary of DILG the election of officers on the ground of irregularities. Taule assailed the power of the Secretary of DILG to decide FABC election contests. HELD: The Secretary of Local Government is not vested with jurisdiction to entertain any protest involving the election of officers of the FABC. There is no question that he is vested with the power to promulgate rules and regulations as set forth in Section 222 of the LGC and the Administrative Code. There is neither a statutory nor constitutional provision expressly or even by necessary implication conferring upon the Secretary of Local Government the power to assume jurisdiction over an election protect involving officers of the katipunan ng mga barangay. Presidential power over local governments is limited by the Constitution to the exercise of general supervision "to ensure that local affairs are administered according to law." The general supervision is exercised by the President through the Secretary of Local Government. In administrative law, supervision means overseeing or the power or authority of an officer to see that the subordinate officers perform their duties. If the latter

43 D2010

Local Government
fails or neglects to fulfill them the former may take such action or step as prescribed by law to make them perform their duties. Control, on the other hand, means the power of an officer to alter or modify or nullify or set aside what a subordinate officer had done in the performance of his duties and to substitute the judgment of the former for that of the latter. The fundamental law permits the Chief Executive to wield no more authority than that of checking whether said local government or the officers thereof perform their duties as provided by statutory enactments. Hence, the President cannot interfere with local governments so long as the same or its officers act within the scope of their authority. Supervisory power, when contrasted with control, is the power of mere oversight over an inferior body; it does not include any restraining authority over such body. Construing the constitutional limitation on the power of general supervision of the President over local governments, We hold that Secretary has no authority to pass upon the validity or regularity of the election of the officers of the katipunan. To allow the Secretary to do so will give him more power than the law or the Constitution grants. It will in effect give him control over local government officials for it will permit him to interfere in a purely democratic and non-partisan activity aimed at strengthening the barangay as the basic component of local governments so that the ultimate goal of fullest autonomy may be achieved. In fact, his order that the new elections to be conducted be presided by the Regional Director is a clear and direct interference by the Department with the political affairs of the barangays which is not permitted by the limitation of presidential power to general supervision over local governments. Indeed, it is the policy of the state to ensure the autonomy of local governments. To deny the Secretary of Local Government the power to review the regularity of the elections of officers of the katipunan would be to enhance the avowed state policy of promoting the autonomy of local governments. The RTCs have the exclusive original jurisdiction to hear the protest. P400,000 for the implementation of the program. However, Commission on Audit disapproved said resolution and disbursement of funds. The reasons it gave were: 1)the resolution has no connection to alleged public safety, general welfare, safety, etc. of the inhabitants of Makati; 2)it will only benefit a few individuals. Public funds should only be used for public purposes. The issue is WON Res. No. 60, reenacted as Res. No. 243, is a valid exercise of the police power under the general welfare clause. HELD: Yes. Police power is a governmental function, an inherent attribute of sovereignty inherent in the state but not in municipal corporations. Before a municipal corporation may exercise such power, there must be a valid delegation of such power by the legislature. Municipal corporations exercise police power under the general welfare clause. Under Sec. 7 of BP 337, every local government unit shall exercise the powers expressly granted, those necessarily implied therefrom, as well as necessary and proper for governance such as to promote health and safety, enhance prosperity, improve morals, and maintain peace and order in the LGU, and preserve the comfort and convenience of the inhabitants therein. Police power is the power to prescribe regulations to promote the health, morals, peace, education, good order or safety and general welfare of the people. It is the most essential, insistent, and illimitable of powers. The police power of a municipal corporation is broad, and has been said to be commensurate with, but not to exceed, the duty to provide for the real needs of the people in their health, safety, comfort, and convenience as consistently as may be with private rights. It extends to all the great public needs, and, in a broad sense includes all legislation and almost every function of the municipal government. Thus, it is inadvisable to frame any definition which shall absolutely indicate the limits of police power. Public purpose is not unconstitutional merely because it incidentally benefits a limited number of persons. The care for the poor is generally recognized as a public duty. The support for the poor has long been an accepted exercise of police power in the promotion of the common good. There is no violation of the equal protection clause in classifying paupers as subject of legislation because the classification is reasonable. Precious to the hearts of our legislators, down to our local councilors, is the welfare of the paupers. Thus, statutes have been passed giving rights and benefits to the disabled, emancipating the tenant-farmer from the bondage of the soil, housing the urban poor, etc.

Binay v. Domingo
Petitioner Municipality of Makati, through its Council, approved Resolution No. 60. This resolution aims to extend P500 burial assistance to poor, bereaved families, the funds to be taken out of the unappropriated available funds in the municipal treasury. The Metro Manila Commission approved Res. No. 60. Thereafter, the Municipal secretary certified a disbursement of

D2010 44

UP College of Law
Res. No. 60 of Makati is a paragon of the continuing program of our government towards social justice. compensation of a certain area from a private cemetery to benefit paupers who are charges of the municipal corporation. Instead of building a public cemetery for this purpose, the city passes the burden to private cemeteries. The said expropriation without compensation is not covered by the Q.C. Charter which empowers the city council to prohibit the burial of the dead within the center of population of the city and to provide for their burial in a proper place subject to the provisions of general law regulating burial grounds and cemeteries. When the Local Government Code, B.P. Blg. 337 states that a Sangguniang panlungsod may "provide for the burial of the dead in such place and in such manner as prescribed by law or ordinance" it simply authorizes the city to provide its own city owned land or to buy or expropriate private properties to construct public cemeteries. Moreover, the questioned ordinance was passed after Himlayang Pilipino, Inc. had incorporated, received necessary permits and commenced operating. The sequestration of 6% of the cemetery cannot be considered as having been impliedly acknowledged by the private respondent when it accepted the permits to commence operations.

City Government of Quezon City v. Ericta


Sec. 9 0f Ordinance No. 6118 requires that every memorial park cemetery shall set aside at least 6% of its total area for charity burial. For several years, this was not enforced. However, the Q.C. Council passed a resolution instructing the City Engineer to stop selling the memorial park lots owned by those who failed to donate the required 6% space for paupers burial. Respondent Himlayang Pilipino filed with the CFI a petition for declaratory relief, prohibition and mandamus with preliminary injunction seeking to annul Sec. 9 of the said ordinance, claiming that it is contrary to the Constitution, the Q.C. Charter, the Local Autonomy Act, and the Revised Admin. Code. The Court declared said section null and void. Petitioners argue that the taking of the respondent's property is a valid and reasonable exercise of police power and that the land is taken for a public use as it is intended for the burial ground of paupers. They further argue that the Quezon City Council is authorized under its charter, in the exercise of local police power. On the other hand, respondent Himlayang Pilipino, Inc. contends that the taking or confiscation of property is obvious because the questioned ordinance permanently restricts the use of the property such that it cannot be used for any reasonable purpose and deprives the owner of all beneficial use of his property. Is Section 9 of the ordinance in question a valid exercise of the police power? HELD: NO, it is not a mere police regulation but an outright confiscation. It deprives a person of his private property without due process of law, nay, even without compensation. An examination of the Q.C. Charter does not reveal any provision that would justify the ordinance in question except the provision granting police power to the City. The power to regulate neither includes the power to prohibit nor the power to confiscate. The ordinance not only confiscates but also prohibits the operation of a memorial park cemetery. Police power is usually exercised in the form of mere regulation or restriction in the use of liberty or property for the promotion of the general welfare. It does not involve the taking or confiscation of property with the exception of a few cases not obtaining here. There is no reasonable relation between the setting aside of at least 6% of the total area of an private cemeteries for charity burial grounds of deceased paupers and the promotion of health, morals, good order, safety, or the general welfare of the people. The ordinance is actually a taking without

Villanueva v. Castaneda
The Municipal Council of San Fernando adopted Res. No. 218 authorizing 24 members of the Fernandino United Merchants and Traders Association to construct permanent stalls and sell in the vicinity of the public market of San Fernando, Pampanga along Mercado Street. The action was protested and the CFI held that the land occupied by the petitioners/stallholders, being public in nature, was beyond the commerce of man and could not be the subject of private occupancy. The decision, however, was not enforced for the petitioners were not evicted from the place and were even assigned space allotments for which they paid daily fees to the municipal government. Thereafter, the Association of Concerned Citizens and Consumers of San Fernando filed a petition for the immediate implementation of Res. No. 29 to restore the property to its original and customary use as a public plaza. Respondent Macalino, as officerin-charge of the office of the mayor of San Fernando, issued a resolution directing the municipal treasurer and engineer to demolish the stalls. Petitioners, claiming that the area in question was leased to them by the municipal government, filed a case for prohibition with the CFI which was denied. Do the petitioners/stallholders have a right to the said land?

45 D2010

Local Government
HELD: NO, the place in question is a public plaza which is beyond the commerce of man and cannot be the subject of lease or any other contractual undertaking. In Muyot vs. de la Fuente, it was held that the City of Manila could not lease a portion of a public sidewalk, being likewise beyond the commerce of man. In Espiritu vs. Municipal Council of Pozorrubio, the Supreme Court held: The town plaza cannot be used for the construction of market stalls or residences, and such structures constitute a nuisance subject to abatement according to law. Town plazas are properties of public dominion, to be devoted to public use and to be made available to the public in general. They are outside the commerce of man and cannot be disposed of or even leased by the municipality to private parties. The occupation of stallholders (now almost 200) has caused health, safety and sanitation problems. It has deprived the stallholders in the public market of much business and has denied to the people the proper use of the public plaza. These problems are covered by police power as delegated to the municipality under the general welfare clause. This authorizes the municipal council to enact such ordinances, not repugnant to law, necessary to discharge the powers and duties conferred upon it by law and such as shall seem necessary and proper to provide for the health and safety, promote the prosperity, improve the morals, peace, good order, comfort, and convenience of the municipality and the inhabitants thereof, and for the protection of property therein. This authority was validly exercised through the adoption of a resolution by the municipal council of San Fernando. Even assuming a valid lease of the property in dispute, the resolution should have effectively terminated the agreement for it is settled that the police power cannot be surrendered or bargained away through the medium of a contract. In fact, every contract affecting the public interest suffers a congenital infirmity in that it contains an implied reservation of the police power as a postulate of the existing legal order. This power can be activated at any time to change the provisions of the contract, or even abrogate it entirely, for the promotion or protection of the general welfare. Such an act will not violate the impairment clause, which is subject to and limited by the paramount police power. occupants thereof, among them appellants Gonzales and Josue, started filling it. Each of the appellants constructed a mixed residential and commercial building on Lot 2. Thereafter, then President Magsaysay issued Proclamation No. 144, entitled "Reserving for Street Widening and Parking Space Purposes Certain Parcels of the Public Domain Situated in the Municipality of Malabon, Province of Rizal, Island of Luzon." Lots 1 and 2 were specifically withdrawn from sale or settlement and reserved for the purposes stated in the Proclamation. The Municipal Council of Malabon then passed Resolutions authorizing the filing of ejectment cases against appellants so that Proclamation No, 144 could be implemented. Separate complaints were also filed against appellants for the recovery of the portions of Lot 2 they were occupying. Appellants disputed the right of the Government to recover the lots for these reasons: 1) they already filed sales applications with the Bureau of Lands; 2) they had a municipal permit to construct a building as well as a business license duly issued by the Office of the Mayor of Malabon; and (3) the lot occupied by them was not needed by the Municipality of Malabon in the widening of F. Sevilla Boulevard and the setting aside of lots for parking does not redound to the public benefit. The Trial Court ordered the appellants to reconvey the lots to the government. Is Proc. No. 144 lawful and valid? HELD: Yes! Proc. No. 144 was issued by the President in response to several resolutions passed by the Municipal Council of Malabon, Rizal, to address the increasing vehicular traffic along F. Sevilla Blvd. The Municipal Council had proposed to widen the road and reserve an area for parking space to ease up traffic problems. The public has much to gain from the proposed road widening and from establishment of a municipal parking area. Traffic congestion constitutes a threat to the health, welfare, safety and convenience of the people and it can only be substantially relieved by widening streets and providing adequate parking areas. Under the Land Transportation and Traffic Code, parking in designated areas along public streets or highways is allowed which clearly indicates that provision for parking spaces serves a useful purpose. Appellants, however, allege that the resulting benefits, if any, will be confined to people who have cars, hence there would be lacking the essential feature of property reserved for public use or benefit. This conception is flawed since the number of users is not the yardstick in

Republic v. Gonzalez
The Republic of the Philippines is the owner of 2 parcels of land in Taong Malabon, Metro Manila (Lots 1 and 2). The said property was formerly a deep swamp until the

D2010 46

UP College of Law
determining whether property is properly reserved for public use or public benefit. To constitute public use, the public in general should have equal or common rights to use the land or facility involved on the same terms, however limited in number the people who can actually avail themselves of it at a given time. There is nothing in Proc. No. 144 which excludes non-car-owners from using a widened street or a parking area should they in fact happen to be driving cars; the opportunity to avail of the use thereof remains open to the public. Prior to the issuance of Proc. No. 144, appellants had applied for sales applications with the Bureau of Lands over the said lots. By doing so, they are deemed to have admitted ownership by the National Government since the said application can only be filed in respect of public land, not private land. These applications were either not yet approved or were already rejected by the Bureau of Lands at the time the proclamation was issued. Thus, no private rights had accrued and become vested in appellants. The lots remained public lands and were subject to the free disposition and control of the Government. While appellants had secured municipal permits for the construction of buildings on the lands in dispute, the Court held that the disposition and management of lands of the public domain were directly under the executive control of the Director of Lands, and not of local government officials. Thus, the Malabon Municipal Mayor exceeded his authority in allowing the use of lands of the public domain to appellants. residential building owned by Mr. Tepoot is adjacent to the funeral parlor, said residential building is being rented by a certain Mr. Asiaten who actually devotes it to his laundry business with machinery thereon. On appeal, the CA reversed the lower court by annulling the building permit issued to the petitioner. It disagreed with the lower court's determination that Tepoot's building was commercial and ruled that although it was used by Tepoot's lessee for a laundry business, it was a residential lot as reflected in the tax declaration, thus paving the way for the application of Ordinance No. 363. HELD: Petitioners operation of a funeral home constitutes permissible use within the district in Davao City. The testimony of City Councilor Vergara shows that Mr. Tepoot's building was used for a dual purpose both as a dwelling and as a place where a laundry business is conducted. But while its commercial aspect has been established by the presence of laundry paraphernalia, its use as a residence, other than being declared for taxation purposes as such, was not fully substantiated. The reversal by the CA of the TCs decision was based on Tepoot's building being declared for taxation purposes as residential. However, a tax declaration is not conclusive of the nature of the property for zoning purposes. A property may be declared by its owner as residential for real estate taxation purposes but it may well be within a commercial zone. A discrepancy may thus exist in the determination of the nature of property for real estate taxation purposes vis-a-vis the determination of a property for zoning purposes. A tax declaration only enables the assessor to identify the same for assessment levels. In fact, a tax declaration does not bind a provincial/city assessor, for under Sec. 22 of the Real Estate Tax Code, appraisal and assessment are based on the actual use irrespective of "any previous assessment or taxpayer's valuation thereon," which is based on a taxpayer's declaration. In fact, a piece of land declared by a taxpayer as residential may be assessed by the provincial or city assessor as commercial because its actual use is commercial. The finding that Mr. Tepoot's building is commercial is strengthened by the fact that the Sangguniang Panlungsod has declared the questioned area as commercial. Consequently, even if Tepoot's building was declared for taxation purposes as residential, once a local government has reclassified an area as commercial, that determination for zoning purposes must prevail. While the commercial character of the questioned vicinity has been declared thru the ordinance, private respondents

Patalinhug v. CA
The Sangguniang Panlungsod of Davao City enacted Ordinance No. 363 otherwise known as the "Expanded Zoning Ordinance of Davao City" which required that funeral parlors shall be built not less than 50 meters from any residential structures, churches, and other institutional buildings. A building permit was issued in favor of petitioner for the construction of a funeral parlor. Thereafter, petitioner commenced its construction. Acting on the complaint of several residents of Brgy. Agdao, Davao City that the construction of petitioner's funeral parlor violated Ordinance No. 363, since it was allegedly situated within a 50-meter radius from the Iglesia ni Kristo Chapel and several residences (the nearest residential structure, owned by Mr. Tepoot is only 8 inches to the south). Private respondents filed a case for the declaration of nullity of the building permit. The court dismissed the complaint finding that: 1) the residential building and Iglesia ni Kristo chapel are 63.25 meters and 55.95 m respectively from the funeral parlor; 2) Although the

47 D2010

Local Government
have failed to present convincing arguments to substantiate their claim that Cabaguio Avenue, where the funeral parlor was constructed, was still a residential zone. Unquestionably, the operation of a funeral parlor constitutes a "commercial purpose." The declaration of the said area as a commercial zone thru a municipal ordinance is an exercise of police power to promote the good order and general welfare of the people in the locality. Corollary thereto, the state, in order to promote the general welfare, may interfere with personal liberty, with property, and with business and occupations. Thus, persons may be subjected to certain kinds of restraints and burdens in order to secure the general welfare of the state and to this fundamental aim of government, the rights of the individual may be subordinated. The ordinance which regulates the location of funeral homes has been adopted as part of comprehensive zoning plans for the orderly development of the area covered thereunder. C. Intergovernmental relations forms of assistance to the local government unit. Such assistance shall be extended at no extra cost to the local government unit concerned. (d) National agencies and offices including governmentowned or controlled corporations with field units or branches in a province, city, or municipality shall furnish the local chief executive concerned, for his information and guidance, monthly reports including duly certified budgetary allocations and expenditures. Sec. 26. Duty of National Government Agencies in the Maintenance of Ecological Balance. It shall be the duty of every national agency or government-owned or controlled corporation authorizing or involved in the planning and implementation of any project or program that may cause pollution, climatic change, depletion of non-renewable resources, loss of cropland, rangeland, or forest cover, and extinction of animal or plant species, to consult with the local government units, nongovernmental organizations, and other sectors concerned and explain the goals and objectives of the project or program, its impact upon the people and the community in terms of environmental or ecological balance, and the measures that will be undertaken to prevent or minimize the adverse effects thereof. Sec. 27. Prior Consultations Required. No project or program shall be implemented by government authorities unless the consultations mentioned in Sections 2 (c) and 26 hereof are complied with, and prior approval of the sanggunian concerned is obtained: Provided, That occupants in areas where such projects are to be implemented shall not be evicted unless appropriate relocation sites have been provided, in accordance with the provisions of the Constitution. Article Two. Relations with the Philippine National Police Sec. 28. Powers of Local Chief Executives over the Units of the Philippine National Police. The extent of operational supervision and control of local chief executives over the police force, fire protection unit, and jail management personnel assigned in their respective jurisdictions shall be governed by the provisions of Republic Act Numbered Sixty-nine hundred seventy-five (R.A. No. 6975), otherwise known as "The Department of the Interior and Local Government Act of 1990", and the rules and regulations issued pursuant thereto. Article Three. Inter-Local Government Relations

Book I, Chapters 3 and 4, LGC (sec 25 36)


CHAPTER THREE: Intergovernmental Relations Article One. National Government and Local Government Units Sec. 25. National Supervision over Local Government Units. (a) Consistent with the basic policy on local autonomy, the President shall exercise general supervision over local government units to ensure that their acts are within the scope of their prescribed powers and functions. The President shall exercise supervisory authority directly over provinces, highly urbanized cities, and independent component cities through the province with respect to component cities and municipalities; and through the city and municipality with respect to barangays. (b) National agencies and offices with project implementation functions shall coordinate with one another and with the local government units concerned in the discharge of these functions. They shall ensure the participation of local government units both in the planning and implementation of said national projects. (c) The President may, upon request of the local government unit concerned, direct the appropriate national agency to provide financial, technical, or other

D2010 48

UP College of Law
Sec. 29. Provincial Relations with Component Cities and Municipalities. The province, through the governor, shall ensure that every component city and municipality within its territorial jurisdiction acts within the scope of its prescribed powers and functions. Highly urbanized cities and independent component cities shall be independent of the province. Sec. 30. Review of Executive Orders. (a) Except as otherwise provided under the Constitution and special statutes, the governor shall review all executive orders promulgated by the component city or municipal mayor within his jurisdiction. The city or municipal mayor shall review all executive orders promulgated by the punong barangay within his jurisdiction. Copies of such orders shall be forwarded to the governor or the city or municipal mayor, as the case may be, within three (3) days from their issuance. In all instances of review, the local chief executive concerned shall ensure that such executive orders are within the powers granted by law and in conformity with provincial, city, or municipal ordinances. (b) If the governor or the city or municipal mayor fails to act on said executive orders within thirty (30) days after their submission, the same shall be deemed consistent with law and therefore valid. Sec. 31. Submission of Municipal Questions to the Provincial Legal Officer or Prosecutor. In the absence of a municipal legal officer, the municipal government may secure the opinion of the provincial legal officer, and in the absence of the latter, that of the provincial prosecutor on any legal question affecting the municipality. Sec. 32. City and Municipal Supervision over Their Respective Barangays. The city or municipality, through the city or municipal mayor concerned, shall exercise general supervision over component barangays to ensure that said barangays act within the scope of their prescribed powers and functions. Sec. 33. Cooperative Undertakings Among Local Government Units. Local government units may, through appropriate ordinances, group themselves, consolidate, or coordinate their efforts, services, and resources for purposes commonly beneficial to them. In support of such undertakings, the local government units involved may, upon approval by the sanggunian concerned after a public hearing conducted for the purpose, contribute funds, real estate, equipment, and other kinds of property and appoint or assign personnel under such terms and conditions as may be agreed upon by the participating local units through Memoranda of Agreement. CHAPTER FOUR: Relations with People's and Nongovernmental Organizations Sec. 34. Role of People's and Non-governmental Organizations. Local government units shall promote the establishment and operation of people's and nongovernmental organizations to become active partners in the pursuit of local autonomy. Sec. 35. Linkages with People's and Non-governmental Organizations. Local government units may enter into joint ventures and such other cooperative arrangements with people's and non-governmental organizations to engage in the delivery of certain basic services, capability-building and livelihood projects, and to develop local enterprises designed to improve productivity and income, diversity agriculture, spur rural industrialization, promote ecological balance, and enhance the economic and social well-being of the people. Sec. 36. Assistance to People's and Non-governmental Organizations. A local government unit may through its local chief executive and with the concurrence of the sanggunian concerned, provide assistance, financial or otherwise, to such people's and non-governmental organizations for economic, socially-oriented, environmental, or cultural projects to be implemented within its territorial jurisdiction.

POWERS OF MUNICIPAL CORPORATIONS/LOCAL GOVERNMENTS


Municipal Corporations, Chapters VIII and IX (Martin)
CHAPTER EIGHT: Powers of municipal corporations I. Nature of the Power Is inherent in the State but not in municipal corporations. In order that a municipal corporation may exercise police Police Power

49 D2010

Local Government
power, there must be a legislative grant which necessarily also sets the limits for the exercise of the power. The grant of authority to the sangguniang of the particular local government unit is what is known as the General Welfare Clause In Barangays, the GWC is found in Section 91 (a) of the LGC In Municipalities, the GWC is found in Section 149 (a) of the LGC For Provinces, the GWC is found in Section 208 (a) of the LGC 2 Branches of the General Welfare Clause General Legislative Power the power to enact ordinances as may be necessary to carry into effect and discharge the responsibilities conferred upon it by law Power Power Proper the authority to enact such ordinances as shall be necessary and proper to promote health and safety, enhance the prosperity and general welfare, improve the morals and maintain peace and order in the particular local government unit and preserve the comfort and convenience of the inhabitants therein Requisites for the Exercise of Police Power First that the interest of the public generally as distinguished from those of a particular class require such interference, and Second that the means is reasonably necessary for the accomplishment of the purpose and not unduly oppressive upon individuals Liberal Interpretation of the General Welfare Clause A restrictive view of the general welfare clause is not favored. The view that the scope of the police power, and therefore of the general welfare clause, has been fixed by traditional delineations is not quite accurate. Police power has not received a full and complete definition; it is elastic and must be responsive to various social conditions; it is not confined within the narrow circumscription of precedents resting on past conditions; it must follow the legal progress of a democratic way of life Exercise of Powers Beyond Boundary Ordinarily, the powers of a municipal corporation can be exercised only within its own territorial limits which are contiguous. However, statutes sometimes authorize it to exercise police powers beyond its territorial boundaries, especially for the preservation of public health (i.e., City of Manila which is empowered by statute to extend its ordinances over the Bay of Manila, three miles beyond the city limits and over any vessel floating within that distance) Power of Municipal Corporations to Legislate on the Same Subject Covered by Law A municipal corporation, under the authority of its charter, may adopt ordinances upon subjects already covered by general law as long as the ordinance is not repugnant to nor in conflict with the latter. The reason for the above principle is that municipal authorities are in a better position to determine the evils sought to be prevented by the legislature in enacting a particular statute and, therefore, to pass the appropriate ordinance to attain the main object of the law Effect of conviction or acquittal under either Statute or Ordinance Punishing the Same Act The Constitution expressly provides, that if an act is punished by a law and an ordinance, conviction or acquittal under either shall constitute a bar to another prosecution for the same act. Hence, where it appears that a person has already been prosecuted and convicted or acquitted of an act made punishable by an ordinance he can invoke the defense of double jeopardy, should the government prosecute him further under the statute punishing the same act. II. Nature of the Power A municipal corporation can only exercise the right of eminent domain when the same has been conferred upon it by the legislature, expressly or by necessary implication, since the municipal corporation has no more right than any other corporation to condemn property. But although there is no inherent power in a municipal corporation to acquire property thru condemnation proceedings, in nearly all jurisdictions the legislature has conferred the power of eminent domain on cities, towns, and villages either by express provision or in a general statute or in the municipal charter. Power of Eminent Domain

D2010 50

UP College of Law
Who may exercise the right The right of eminent domain may be exercised either directly by the legislature or through the medium of corporate bodies, or of individual enterprises, by virtue of a delegation of the power. The legislature, unless limited by constitutional restriction, is entirely free to use its discretion in the selection of agents to exercise the power Purposes of Expropriation The power may only be exercised for public purposes (for examples see Section 2106[f] and Section 2245 of the Revised Administrative Code) Power of Eminent Domain as exercised by Municipal Corporation must be strictly construed Whether the power is directly exercised by the legislature or immediately thru municipal corporations or other public agencies, the purpose or use for which the property authorized to be appropriated must be specified. The power cannot be enlarged by doubtful construction. The power of eminent domain, being in derogation of private property rights, is justified only by a clear public necessity of an urgent public policy. The power must be strictly construed against the municipal corporation. The purpose and procedure prescribed for its exercise must be strictly followed in all substantial matters. This rule of strict construction is mandatory and should within reasonable limits, be inflexibly adhered to and applied. III. Power of Taxation form whatever upon goods and merchandise carried into the municipality, or out of the same, and any attempt of an unreasonable charge for wharfage, use of bridges or otherwise, shall be void (case of Laoag Producers vs. Mun. of Laoag, says this has already been impliedly repealed by the Local Autonomy Act) In no case shall collection of municipal taxes be left to any person Except as allowed by law, municipal funds, shall be devoted exclusively to local public purpose

4. 5.

Power to License Where only the power to license is given to municipal corporations, the presumption is that it is regulatory, rather than for raising of revenue. The terms in which a municipality is empowered to grant license are expected to indicate with sufficient precision whether the power could be exercised for revenue-raising purposes or merely for regulation Distinction between the Power to License and the Power to Tax Power to License is a police measure; the power to Tax is a revenue measure Distinction between a license tax and a general tax A license tax is not a tax upon property, but it is a burden imposed for the right to exercise a franchise or a privilege which could be withheld or forbidden altogether. The sum charged is merely used as the mode of computing the amount to be paid for the exercise of the privilege. Kinds of Municipal License 1. 2. 3. Licenses for the regulation of useful occupations or enterprises Licenses for the regulation or restriction of nonuseful occupations or enterprises Licenses for revenue only

Nature of the Power as exercised by Municipal Corporations The power to tax, although inherent in sovereignty, is not possessed by municipal corporations by the mere fact that they are creations of the State. The Power must be expressly conferred upon them, either in their charters or in a general law. Requisites of Municipal Taxation 1. Municipal revenue obtainable by taxation shall be derived from such sources only as are expressly authorized by law Taxation shall be just and in each municipality uniform It shall not be within the power of the municipal council to impose a tax in any

Effectivity of Tax Ordinance A tax ordinance shall go into effect on the 15th day after its passage, unless the ordinance shall provide otherwise: Provided, however, that the Secretary of Finance shall have the authority to suspend the effectivity of any ordinance within one hundred and twenty days after its passage, if, in his opinion, the tax or fee therein levied or

2. 3.

51 D2010

Local Government
imposed is unjust, excessive, oppressive, or confiscatory, and when the said secretary exercises this authority the effectivity of such ordinance shall be suspended. Condonation of Taxes Municipal council has no right to condone taxes already accrued. This power has been withheld from municipal councils to prevent abuse Under the law, it is the provincial treasurer who can condone municipal taxes (Reyes vs. Cornista, 49 OG 931) Special Assessment; Its distinguishing Features Like general taxes, special assessments are enforced in proportionate contributions, but instead of being imposed at regularly recurring periods to provide continuous revenue, special assessments are levied only occasionally as required. They are imposed not upon the general body of citizens, but upon a limited class of persons who are interested in local government, and also are specially benefited in a local improvement to the extend of the assessment. They are imposed and collected as an equivalent, actual or presumed, of the benefits and are required to pay for the cost of the improvement. Distinction between a Tax and Special Assessment 1. 2. 3. 4. SA can be levied only on land SA cannot be made a personal liability of the person assessed SA is based wholly on benefits SA is exceptional both as to time and locality The legislature can neither compel nor authorize a municipal corporation to spend any of its funds for a private purpose, and consequently, since practically every undertaking of a municipality does or may require the expenditures of money, a municipal corporation cannot, even with the express legislative sanction, embark in any private enterprise or assume any functions which are not in a legal sense public. II. Power to Borrow Money

Nature of the Power The power to borrow does not belong to a municipal corporation as an incident of its creation. It exists only when conferred by express constitutional or legislative grant or by force of legislative vestment of power, coupled with the imposition of duties which are incapable of exercise and performance without the borrowing of money Limitation on the Power to Borrow A limit to municipal indebtedness may be fixed either by statute or Constitution beyond which no obligation could be incurred by the municipality The purpose of limiting municipal indebtedness is to protect persons residing in municipalities from the abuse of their creditors and the consequent operation of burdensome if not ruinous taxes III. Power to Issue Municipal Bonds

CHAPTER NINE: Powers of municipal corporations I. Power to Appropriate and Spend Money

Nature of the Power Unless expressly authorized by its charter or statute, a municipal corporation cannot appropriate or give away the public money of the municipality as pure donations to any person, corporation, or private institution, under the control of the city and having no connection with it In the Philippines, the power of municipal corporations to appropriate and spend money is among the many powers expressly authorized in their charters Limitation on the Power to Appropriate and Spend

Subject to Judicial Examination. There are cases holding that the power to issue bonds is inherent in the municipal corporation. Upon examination, however, most of the cases will be found as sustaining the implied rather than the inherent power of a municipality to issue bonds The power to issue negotiable paper will be implied from the express power to borrow money IV. Power to Contract

Nature and Extent of Power The rules that municipal corporations may exercise those powers which are expressly granted, or those essential to the carrying out of declared objects and the purposes of the corporation, is applicable with respect to the power of the municipality to bind itself by contract

D2010 52

UP College of Law
This power, however, is not to be construed as authorizing the making of contracts of all description, but only such as are necessary and usually fit and proper, to enable the corporation to secure or carry into effect the purposes for which it was created Nature of the Duty of Municipal Authorities in the Execution of Contracts The duty of the officers of the municipal corporation in the letting of municipal contracts is not merely ministerial but of a judicial and discretionary nature. In the absence of fraud or palpable abuse of discretion, courts have no power to control their action Requisites Necessary for the Validity of Municipal Contracts a. First, that the Corporation has the express, implied, or inherent power to enter into the particular contract Second, that the contract is entered into by the proper department, board, committee, officer or agent Power to Acquire, Hold, and Dispose of Property exercise a like power over their properties of a public nature in violation of the trust in which such properties are held. Power to Mortgage Where property not charged with a trust or public use is held by the corporation without legislative restriction as to its sale, it may mortgage it to secure any debt or obligation that it has the power to create or enter into. The power to mortgage, if not expressly given or denies, can be considered an incident of the power to hold and dispose of property, and to make contracts VI. Power to Sue and Be Sued

Nature of the Power The power to sue and be sued is an essential attribute of Municipal Corporation. A municipality, like any other complete corporation or person, may appeal to the court for the vindication of its right; and for any wrong done by it, it may be sued by the injured party. The power to sue and be sued by the injured party includes the right to settle or compromise claims of the municipality. Municipality may file suit in behalf of the inhabitants In many instances, a municipal corporation may bring a representative action in behalf of its inhabitants to protect, preserve, or establish a common right. Venue of Action An action against a municipal corporation is inherently local. In the absence of any statutory provision to the contrary, it must be brought in the place or province in which the municipality is situated. The reasons for this are: That the municipal corporation remain statutory in some place, and therefore, must be sued where they are found; that the welfare of the municipal bodies and of their citizens require that their officers should be permitted to remain at home and discharge their public duties instead of being called elsewhere to attend litigation. The venue of action in the place of the municipality applies whether the cause of action arises from the proprietary or governmental conduct of the municipality.

b.

V.

Nature of Power While the power to acquire property for public purposes is deemed inherent in municipal corporations or one necessarily implied from other express powers of municipal corporations, in the Philippines, this power is expressly authorized in the corporate charters Municipal Property, Classified In the Philippines, properties of provinces, cities, and municipalities are divided into property for public use and patrimonial property Grants of Land in favor of municipal corporations by the State; Requisites Two requisites are necessary in order to presume a grant of land to municipal corporations, namely, that the land is one which the municipality itself can exclusively own; and that the land is used to meet public necessity Municipality may alienate its property Municipal Corporations possess the incidental or implied power to alienate or dispose of their real or personal properties of a private nature. Conversely, they cannot

Compendium on Local Autonomy and Local Government, Chapter 5 (Agra)


See above

53 D2010

Local Government Book I, Chapter 2, LGC (Sec 6 24)


See above A. Police Power dangerous buildings. Respondent City Engineer and Building Official can, therefore, validly issue the questioned condemnation and demolition orders. This is also true with the Mayor who can approve or deny the condemnation orders as provided in Section 276 of the Compilation of Ordinances of the City of Manila.There was no grave abuse of discretion on the part of the respondent City Engineer because the orders were made only after thorough ocular inspections were conducted by the City's Building Inspectors. The respondent Mayor's act of approving the condemnation orders was likewise done in accordance with law. Also, the protest made by petitioners was submitted 3 months after the notices of condemnation were issued, and clearly beyond the seven days prescribed for doing so.

Binay v. Domingo (supra)


See above

Chua Huat v. CA
Manuel Uy and Sons, Inc. requested del Rosario, the City Engineer and Building Official of Manila, to condemn the dilapidated structures located in Paco, Manila, all occupied by petitioners. The said official issued notices of condemnation to petitioners. The condemnation orders stated that the subject buildings were found to be in dangerous condition and therefore condemned, subject to the confirmation of the Mayor as required by Section 276 of the Compilation of Ordinances of the City of Manila. It was stated that the notice was not an order to demolish as the findings of the City Engineer are still subject to the approval of the Mayor. The orders were based on inspection reports made by the Office of the City Engineer which showed that the buildings suffered from structural deterioration of as much as 80%. The Mayor confirmed the condemnation orders. Petitioners protested against the notices of condemnation. Later, the City Engineer issued a demolition order to the petitioners. The petitioners filed a Petition for Prohibition, with PI or TRO against the City Mayor, City Engineer, Building Officer and Manuel Uy and Sons Inc. The Court issued the TRO and required respondents to comment. Respondents prayed that the petition be dismissed claiming that: (1) the power to condemn buildings and structures in the City of Manila falls within the exclusive domain of the City Engineer pursuant to Sections 275 and 276 of its Compilation of Ordinances (also Revised Ordinances 1600); and (2) the power to condemn and remove buildings and structures is an exercise of the police power granted the City of Manila to promote public safety. HELD: The power to condemn buildings and structures in the City of Manila falls within the exclusive jurisdiction of the City Engineer, who is at the same time the Building Officials. The Compilation of Ordinances of the City of Manila and the National Building Code, also provide the authority of the Building Officials, with respect to

Tatel v. Municipality of Virac


Due to the complaints received from the residents of barrio Sta. Elena against the disturbance caused by the operation of the abaca bailing machine inside petitioners warehouse which emitted obnoxious odor and dust, a committee was appointed by the municipal council of Virac to investigate the matter. The committee noted that the warehouse was near residential houses and that the inflammable materials inside created danger to the lives and properties of the people. Resolution 29 was passed by the Municipal Council declaring the warehouse as a public nuisance within the purview of Article 694 of the CC. The petitioners MR was denied. Petitioner filed a petition for prohibition with preliminary injunction with the CFI enjoining them from enforcing Resolution 29 of the Council. The municipal officials contend that the warehouse was constructed in violation of Ordinance No. 13, prohibiting the construction of warehouses near a block of houses either in the poblacion or barrios without maintaining the necessary distance of 200 meters from said block of houses to avoid loss of lives and properties by accidental fire. Petitioner contends that said ordinance is unconstitutional, contrary to the due process and equal protection clause of the Constitution and null and void for not having been passed in accordance with law. The court ruled in favor of the municipal council and held that the ordinance was a legitimate and valid exercise of police power by the municipal council. HELD: Ordinance No. 13 was passed by the Municipal Council of Virac in the exercise of its police power. Municipal corporations are agencies of the State for the promotion and maintenance of local self-government

D2010 54

UP College of Law
and as such are endowed with the police powers in order to effectively accomplish and carry out the declared objects of their creation. Its authority emanates from the general welfare clause under the Administrative Code. For an ordinance to be valid, it must not only be within the corporate powers of the municipality to enact but must also be passed according to the procedure prescribed by law, and must be in consonance with certain well established and basic principles of a substantive nature. These principles require that a municipal ordinance (1) must not contravene the Constitution or any statute (2) must not be unfair or oppressive (3) must not be partial or discriminatory (4) must not prohibit but may regulate trade (5) must be general and consistent with public policy, and (6) must not be unreasonable. Ordinance No. 13, meets these criteria. The purpose of the ordinance is to avoid loss of life and property in case of accidental fire which is one of the primordial and basic obligation of any government. Its purpose is well within the objectives of sound government. No undue restraint is placed upon the petitioner or for anybody to engage in trade but merely a prohibition from storing inflammable products in the warehouse because of the danger of fire to the lives and properties of the people residing in the vicinity. As to the contention, that warehouses similarly situated as that of the petitioner were not prosecuted, the mere fact that the municipal authorities have not proceeded against other warehouses in the municipality allegedly violating Ordinance 13 is no reason to claim that the ordinance is discriminatory. A distinction must be made between the law itself and the manner in which said law is implemented by the agencies in charge with its administration/enforcement. There is no valid reason for the petitioner to complain, in the absence of proof that the other bodegas mentioned by him are operating in violation of the ordinance and that the complaints have been lodged against the bodegas concerned without the municipal authorities doing anything about it. merely a time, place and manner (TPM) regulation that was content-neutral. Since the content of the speech was not relevant, it held that it does not impose any prior restraint. The overbreadth argument of petitioners was also dispensed with by the Court by saying that BP 880 only regulates the exercise of the right to peaceful assembly and petition only to the extent needed to avoid a clear and present danger of the substantive evils Congress has the right to preventAs to the delegation of powers to the mayor, the law provides a precise and sufficient standard the clear and present danger test stated in Sec. 6(a) [of BP 880]. The reference to imminent and grave danger of a substantive evil in Sec. 6(c) substantially means the same thing and is not an inconsistent standard. However, the CPR serves no valid purpose if it means the same thing as maximum tolerance and is illegal if it means something else For this reason, the so-called calibrated preemptive response policy has no place in our legal firmament and must be struck down as a darkness that shrouds freedom. It merely confuses our people and is used by some police agents to justify abuses. B. Power of Taxation: Local Taxes and Real Property Tax

Sec 5 -7, Art X, 1987 Constitution


Section 5. Each local government unit shall have the power to create its own sources of revenues and to levy taxes, fees, and charges subject to such guidelines and limitations as the Congress may provide, consistent with the basic policy of local autonomy. Such taxes, fees, and charges shall accrue exclusively to the local governments. Section 6. Local government units shall have a just share, as determined by law, in the national taxes which shall be automatically released to them. Section 7. Local governments shall be entitled to an equitable share in the proceeds of the utilization and development of the national wealth within their respective areas, in the manner provided by law, including sharing the same with the inhabitants by way of direct benefits.

Bayan v. Ermita
Petitioners attacked the constitutionality of the Public Assembly Act (BP 880) and the Calibrated Preemptive Response (CPR) policy by the Arroyo administration which enforced a no permit, no rally policy and authorized the preemptive dispersal of rallies deemed unlawful by the executive branch. Petitioners based their arguments primarily on the ground of freedom of expression. The Court upheld the constitutionality of BP 880. It found out that the procedure prescribed in the statute does not impose an absolute ban on rallies, but

Sec 128 283, LGC


See attachments i. Local Taxation

Basco v. PAGCOR 55 D2010

Local Government
PAGCOR, a government owned and controlled corporation, is exempted by its charter from payment of all kinds of taxes except for 5% franchise tax. Petitioners assail the grant of such exemption on the ground that it infringes on the right of the City of Manila to impose local fees and taxes. The issue is whether or not PAGCOR which is exempted by its charter from payment of all kinds of taxes except for the 5% franchise tax is subject to the local fees and taxes imposed by the City of Manila. Ruling: yes. The taxing power of local government units must always yield to an act of Congress. Local government has no inherent power of tax but merely derives such power from Congress. In fact, local government units have no power to tax instrumentalities of the National Government. Under its charter, PAGCOR is empowered to operate and regulate gambling casinos. With its regulatory power, it becomes an instrumentality of the National Government and hence, entitled to exemption from local taxes. 53 of the same decree. The manufacture and the export of cement do not fall under the said provision for it is not a mineral product. Exemptions are construed strictly against the taxpayer.

Tuzon and Mapagu v. CA


The issue in this case is the liability of petitioners (Mayor Tuzon and Mapagu) for damages for refusing to issue to petitioner Jurado a mayor's permit and license to operate his palay-threshing business for failure to comply with the 1% donation. The Sangguniang Bayan of Camalaniugan, Cagayan adopted a resolution soliciting 1% donation from the thresher operatorsto help finance the continuation of the construction of the Sports and Nutrition Center Building. Although the validity of the resolution was not in issue, the SC observed that: it seems to make the donation obligatory and a condition precedent to the issuance of the mayor's permit. This goes against the nature of a donation, which is an act of liberality and is never obligatoryIf, on the other hand, it is to be considered a tax ordinance, then it must be shown in view of the challenge raised by the private respondents to have been enacted in accordance with the requirements of the Local Tax Code. Petitioners acted within the scope of their authority and in consonance with their honest interpretation of the resolution in question. It was not for them to rule on its validity. In the absence of a judicial decision declaring it invalid, its legality would have to be presumed. As executive officials of the municipality, they had the duty to enforce it as long as it had not been repealed by the Sangguniang Bayan or annulled by the courts. ii. Franchises

Philippine Petroleum Corp v. Municipality of Pililia Rizal


Philippine Petroleum Corporation is a business enterprise engaged in the manufacture of lubricated oil basestock which is a petroleum product, with its refinery plant situated at Malaya, Pililla, Rizal. The municipality of Pililia enacted a municipal tax ordinance which covers the business of petitioner. Petitioner questions the imposition in view of several provincial circulars directing the municipality officers to refrain from collecting the said taxes. The validity of the ordinance is undisputed for it is an exercise of the constitutional power of LGUs to levy taxes. To allow the continuous effectivity of the prohibition set forth in the circulars would be tantamount to restricting their power to tax by mere administrative issuances. Administrative regulations must be in harmony with the provisions of the law. It is an ancient rule that exemptions from taxation are construed in strictissimi juris against the taxpayer and liberally in favor of the taxing authority.

PLDT v. City of Davao


PLDT assails the franchise tax being imposed by the City of Davao. Its argument is based on its franchise in relation to RA 7925 particularly the provision on the equality of treatment in the telecommunications industry. It argues that because Smart and Globe are exempt from the franchise tax, it follows that it must likewise be exempt from the tax being collected by the City of Davao because the grant of tax exemption to Smart and Globe ipso facto extended the same exemption to it. In approving R.A. No. 7925, Congress did not intend it to operate as a blanket tax exemption to all telecommunications entities. Applying the rule of strict construction of laws granting tax exemptions and the rule that doubts should be resolved in favor of municipal corporations in interpreting statutory provisions on

Floro Cement Corp. v. Gorospe


The municipality of Lugait, province of Misamis Oriental, seeks to collect from Floro Cement manufacturers and exporter's taxes. Floro Cement says that it is exempted by virtue of PD 463 which prevents LGUs from levying taxes on mineral products. Ruling: the Court has consistently held that it is not a mineral product but rather a manufactured product. The exemption mentioned in P.D. No. 463 refers only to machineries, equipment, tools for production, etc., as provided in Sec.

D2010 56

UP College of Law
municipal taxing powers, R.A. 7925 cannot be considered as having amended petitioner's franchise so as to entitle it to exemption from the imposition of local franchise taxes. considered real property for taxation purposes because they are installed at a specific location with a character of permanency. The LBAA also pointed out that the owner of the bargesFELS, a private corporationis the one being taxed, not NPC. A mere agreement making NPC responsible for the payment of all real estate taxes and assessments will not justify the exemption of FELS; such a privilege can only be granted to NPC and cannot be extended to FELS. Fels appealed to the CBAA. The CBAA reversed and ruled that the power barges belong to NPC; since they are actually, directly and exclusively used by it, the power barges are covered by the exemptions under Section 234(c) of R.A. No. 7160. As to the other jurisdictional issue, the CBAA ruled that prescription did not preclude the NPC from pursuing its claim for tax exemption in accordance with Section 206 of R.A. No. 7160. Upon MR, the CBAA reversed itself. WON the petitioner may be assessed real property taxes Held: Yes The CBAA and LBAA maintain that power barges are real property and are thus subject to real property tax. Tax assessments by tax examiners are presumed correct and made in good faith, with the taxpayer having the burden of proving otherwise. Factual findings of administrative bodies, which have acquired expertise in their field, are generally binding and conclusive upon the Court. Power barges are categorized as immovable property by destination (Article 415(9) NCC), being in the nature of machinery and other implements intended by the owner for an industry or work which may be carried on in a building or on a piece of land and which tend directly to meet the needs of said industry or work. Petitioners maintain that the power barges are exempt from real estate tax under Section 234 (c) of R.A. No. 7160 because they are actually, directly and exclusively used by petitioner NPC, a government- owned and controlled corporation engaged in the supply, generation, and transmission of electric power. SC affirms the findings of the LBAA and CBAA that the owner of the taxable properties is petitioner FELS, which is the entity being taxed by the local government. As stipulated under Section 2.11, Article 2 of the Agreement: Ownership of power barges. POLAR shall own the Power Barges and all the fixtures, fittings, machinery and

City Government of QC v. Bayantel


Bayantel claims exemption from real property taxes being imposed by Quezon City. It anchors its claim on its legislative franchise. According to the court, the franchise distinguishes between two (2) sets of properties, be they real or personal, owned by the franchisee, namely, (a) those actually, directly and exclusively used in its radio or telecommunications business, and (b) those properties which are not so used. The power to impose realty taxes by the LGUs only covers the second category. Bayantels franchise being national in character, the "exemption" thus granted in its charter applies to all its real or personal properties found anywhere within the Philippine archipelago. Although the enactment of the LGC withdrew Bayantels exemption, the amendatory law of Bayantels franchise reenacted the exemption.

FELS Energy v. Province of Batangas


NPC entered into a lease contract with Polar Energy, Inc. over 3x30 MW diesel engine power barges moored at Balayan Bay in Calaca, Batangas. The contract (Energy Conversion Agreement) was for a period of 5 years. Article 10 states that NPC shall be responsible for the payment of taxes; other than (i) taxes imposed or calculated on the basis of the net income of POLAR and Personal Income Taxes of its employees and (ii) construction permit fees, environmental permit fees and other similar fees and charges. Polar Energy then assigned its rights under the Agreement to Fels despite NPCs initial opposition. FELS received an assessment of real property taxes on the power barges from the Provincial Assessor of Batangas City. FELS referred the matter to NPC, reminding it of its obligation under the Agreement to pay all real estate taxes. FELS gave NPC the full power and authority to represent it in any conference regarding the real property assessment of the Provincial Assessor. NPC filed a petition with the LBAA. The LBAA ordered Fels to pay the real estate taxes. The LBAA ruled that the power plant facilities, while they may be classified as movable or personal property, are nevertheless

57 D2010

Local Government
equipment on the Site used in connection with the Power Barges which have been supplied by it at its own cost. POLAR shall operate, manage and maintain the Power Barges for the purpose of converting Fuel of NAPOCOR into electricity. FELS cannot escape liability from the payment of realty taxes by invoking its exemption in Section 234 (c) of R.A. No. 7160. Indeed, the law states that the machinery must be actually, directly and exclusively used by the government owned or controlled corporation; nevertheless, petitioner FELS still cannot find solace in this provision because Section 5.5, Article 5 of the Agreement provides: Operation. POLAR undertakes that until the end of the Lease Period, subject to the supply of the necessary Fuel pursuant to Article 6 and to the other provisions hereof, it will operate the Power Barges to convert such Fuel into electricity in accordance with Part A of Article 7. It is a basic rule that obligations arising from a contract have the force of law between the parties. Not being contrary to law, morals, good customs, public order or public policy, the parties to the contract are bound by its terms and conditions. Applying the rule of strict construction of laws granting tax exemptions, and the rule that doubts should be resolved in favor of provincial corporations, we hold that FELS is considered a taxable entity. The mere undertaking of petitioner NPC under Section 10.1 of the Agreement, that it shall be responsible for the payment of all real estate taxes and assessments, does not justify the exemption. The privilege granted to petitioner NPC cannot be extended to FELS. The covenant is between FELS and NPC and does not bind a third person not privy thereto, in this case, the Province of Batangas. It must be pointed out that the protracted and circuitous litigation has seriously resulted in the local governments deprivation of revenues. The power to tax is an incident of sovereignty and is unlimited in its magnitude, acknowledging in its very nature no perimeter so that security against its abuse is to be found only in the responsibility of the legislature which imposes the tax on the constituency who are to pay for it. The right of local government units to collect taxes due must always be upheld to avoid severe tax erosion. This consideration is consistent with the State policy to guarantee the autonomy of local governments and the objective of the Local Government Code that they enjoy genuine and meaningful local autonomy to empower them to achieve their fullest development as self-reliant communities and make them effective partners in the attainment of national goals. The power to tax is the most potent instrument to raise the needed revenues to finance and support myriad activities of the local government units for the delivery of basic services essential to the promotion of the general welfare and the enhancement of peace, progress, and prosperity of the people.

Digitel v. Province of Pangasinan


Section 137 LGC withdrew any exemption from the payment of franchise tax by authorizing the LGUs to impose a franchise tax on businesses at a rate not exceeding 50% of 1% of the gross annual receipts of the business. Section 232 also authorizes the imposition of an ad valorem tax on real property by the LGUs within the Metropolitan Manila Area wherein the land, building, machinery and other improvement not thereinafter specifically exempted. Digitel was granted, under Provincial Ordinance No. 1892, a provincial franchise to install, maintain and operate a telecommunications system within Pangasinan. Under the Sec 6 of the provincial franchise, the grantee is required to pay franchise and real property taxes. The Sangguniang Panlalawigan also enacted Provincial Tax Ordinance 1 (Real Property Tax Ordinance of 1992). Section 4 expanded the application of Sec. 6 of the provincial franchise of Digitel to include machineries and other improvements, not thereinafter exempted,. Provincial Tax Ordinance No 4 was then enacted. Sections 4, 5 and 6 positively imposed a franchise tax on businesses enjoying a franchise within the province of Pangasinan. Digitel was granted by RA 7678 a legislative franchise. Under Section 5 of its legislative franchise, DIGITEL became liable for the payment of a franchise tax as may be prescribed by law of all gross receipts of the telephone or other telecommunications businesses transacted under it by the grantee, as well as real property tax on its real estate, and buildings exclusive of this franchise.

D2010 58

UP College of Law
Province of Pangasinan found that Digitel had a franchise tax deficiency for the years of 1992, 1993 and 1994. In the interregnum, on 16 March 1995, Congress passed RA 7925 (The Public Telecommunications Policy Act of the Philippines). Section 23 (Equality of Treatment in the Telecommunications Industry), provided for the ipso facto application to any previously granted telecommunications franchises of any advantage, favor, privilege, exemption or immunity granted under existing franchises, or those still to be granted, to be accorded immediately and unconditionally to earlier grantees. Digitel opposed Pangasinans claim on the ground that: prior to the approval of its legislative franchise, its operation of a telecommunications system was done under a Facilities Management Agreement it had previously executed with the DOTC. Thus, all revenues generated from the operation of the facilities inured to the DOTC and all the fees received by petitioner DIGITEL were purely for services rendered. under its legislative franchise, the payment of a franchise tax to the BIR would be in lieu of all taxes on said franchise or the earnings therefrom. The Province of Pangasinan filed a Complaint for Mandamus, Collection of Sum of Money and Damages before Branch 68 of the RTC of Lingayen, Pangasinan. The trial court decided the Province. Digitel maintains that its legislative franchise being an earlier enactment, by virtue of Section 23 of Republic Act No. 7925, the ipso facto, immediate and unconditional application to it of the tax exemption found in the franchises of Globe, Smart and Bell. WON Digitel is exempt from the payment of provincial franchise tax in view of Section 23 of RA 7925 in relation to the exemptions enjoyed by other telephone companies? Held: No Prior to the enactment of its legislative franchise, Digitel did not enjoy and exemption from the payment of franchise and real property taxes. In fact the provincial franchise made Digitel liable for the payment of such taxes. In the case of PLDT v. City of Davao, SC already clarified the confusion brought about by the effect of Section 23 of Republic Act No. 7925 that the word exemption as used in the statute pertains merely to an exemption from regulatory or reporting requirements of the DOTC or the NTC and not to the grantees tax liability. Congress did not intend Section 23 to operate as a blanket tax exemption to all telcos. PLDTs theory will leave the Government with the burden of having to keep track of all granted telecommunications franchises, lest some companies be treated unequally. It is different if Congress enacts a law specifically granting uniform advantages, favor, privilege, exemption, or immunity to all telecommunications entities. R.A. No. 7925 is a legislative enactment designed to set the national policy on telecommunications and provide the structures to implement it to keep up with the technological advances in the industry and the needs of the public. The thrust of the law is to promote gradually the deregulation of the entry, pricing, and operations of all public telecommunications entities and thus promote a level playing field in the telecommunications industry. There is nothing in the language of 23 nor in the proceedings of both the House of Representatives and the Senate in enacting R.A. No. 7925 which shows that it contemplates the grant of tax exemptions to all telecommunications entities, including those whose exemptions had been withdrawn by the LGC. In view of the passage of RA 7716 abolishing the franchise tax imposed on telecommunications companies effective 1 January 1996 and in its place is imposed a 10% VAT, the in-lieu-of-all-taxes clause/provision in the legislative franchises of Globe, Smart and Bell, among others, has now become functus officio, made inoperative for lack of a franchise tax. Therefore, from 1 January 1996, Digitel ceased to be liable for national franchise tax and in its stead is imposed a 10% VAT in accordance with Section 108 of the Tax Code. WON Digitel is exempt from payment of real estate tax under its legislative franchise. Held: Yes SECTION 5. Tax Provisions. The grantee shall be liable to pay the same taxes on its real estate, buildings, and personal property exclusive of this franchise as other persons or corporations are now or hereafter may be required by law to pay x x x. SC qualifies that such exemption solely applies to those real properties actually, directly and exclusively used by the grantee in its franchise. The present issue actually boils down to a dispute between the inherent taxing power of Congress and the

59 D2010

Local Government
delegated authority to tax of the local government borne by the 1987 Constitution. In the PLDT v. City of Davao, SC sustained the power of Congress to grant exemptions over and above the power of the local governments delegated taxing authority notwithstanding the source of such power. Had Congress intended to tax each and every real property of Digitel, regardless of whether or not it is used in the business or operation of its franchise, it would not have incorporated a qualifying phrase, which such manifestation admittedly is. The fact that Republic Act No. 7678 was a later piece of legislation can be taken to mean that Congress, knowing fully well that the Local Government Code had already withdrawn exemptions from real property taxes, chose to restore such immunity even to a limited degree. Said exemption, however, merely applies from the time of the effectivity of petitioner DIGITELs legislative franchise and not a moment sooner. iii. Real Property Taxation and Special Education Fund Tax WON the Ministry of Finance could legally promulgate Regulations prescribing a rate of penalty on delinquent taxes other than that provided for under PD 464, also known as the Real Property Tax Code. Held: No The subject Regulations must be struck down for being repugnant to Section 66 of P.D. No. 464 or the Real Property Tax Code, which is the law prevailing at the time material to this case. Under Section 66 of P.D. No. 464, the maximum penalty for delinquency in the payment of real property tax shall in no case exceed 24% of the delinquent tax Section 4(c) of the challenged Joint Assessment Regulations No. 1-85 and Local Treasury Regulations No. 2-85 issued by respondent Secretary of Finance provides that the penalty of two percent (2%) per month of delinquency or twenty-four percent (24%) per annum as the case may be, shall continue to be imposed on the unpaid tax from the time the delinquency was incurred up to the time that the delinquency is paid for in full. The penalty imposed under the assailed Regulations has no limit inasmuch as the 24% penalty per annum shall be continuously imposed on the unpaid tax until it is paid for in full unlike that imposed under Section 66 of the Real Property Tax Code where the total penalty is limited only to twenty-four percent of the delinquent tax. The Court harbors doubts on the veracity of petitioners contention that the Regulations at issue are sanctioned by E.O. No. 73. The underlying principle behind E.O. No. 73, is to advance the date of effectivity of the application of the Real Property Tax Values of 1984 from 01 January 1988, the original date it was intended by E.O. No. 1019 to take effect for purposes stated therein, to 01 January 1987. E.O. No. 73 did not, in any way, alter the structure of the real property tax assessments as provided for in P.D. No. 464 or the Real Property Tax Code. E.O. No. 73 did not touch at all on the topic of amendment of rates of delinquent taxes or the amendment of rates of penalty on delinquent taxes. E.O. No. 73, particularly in Section 2 thereof, has merely designated the Minister of Finance to promulgate the rules and regulations towards the implementation of E.O. No. 73, particularly on the application of the Real

Sec of Finance v. Ilarde


Cipriano P. Cabaluna, Jr., was the Regional Director of Regional Office No. VI of the DOF. He co-owns with his wife certain properties in Jaro, Iloilo City. Private respondent failed to pay the land taxes for the years 1986 to 1992. A breakdown of the computation of the delinquent taxes showed that more than 24% of the delinquent taxes were charged and collected from private respondent by way of penalties. Cabaluna paid his land taxes and the receipts were issued to him by the City Treasurers Office with the notation paid under protest. After Cabalunas retirement, he filed a formal protest with the City Treasurer of Iloilo wherein he contends that the computation was erroneous since the rate of penalty exceed 24% in contravention of Section 66 of P.D. No. 464 (Real Property Tax Code). Section 66 of P.D. No. 464 fixed the maximum penalty for delinquency in the payment of real estate taxes at 24% of the delinquent tax. The Assistant City Treasurer Rizalina Tulio turned down the protest, citing Sec. 4(c) of Joint Assessment Regulations No. 1-85 and Local Treasury Regulations No. 2-85 of the DOF.

D2010 60

UP College of Law
Property Values as of 31 December 1984, which is the general purpose for enacting said executive order. PD 464 in general and Section 66 in particular, remained to be good law. To accept petitioners premise would be tantamount to saying that EO 73 has repealed or amended PD 464. Repeal of laws should be made clear and expressed. Repeals by implication are not favored for a law cannot be deemed repealed unless it is clearly manifest that the legislature so intended it. The failure to add a specific repealing clause indicates that the intent was not to repeal any existing law, unless an irreconcilable inconsistency and repugnancy exist in the terms of the new and old laws. SC found no such inconsistency or repugnancy between EO 73 and Section 66 of PD 464. A regulation which is in itself invalid for being contrary to law cannot be validated by any act of endorsement of any official, much less, by a subordinate of the official who issued such regulation. Estoppel, certainly, cannot make an invalid regulation valid. The penalties imposed by respondents City Treasurer and Assistant City Treasurer of Iloilo City on the property of private respondent are valid only up to 24% of the delinquent taxes. The excess penalties paid should, in view of that, be refunded by the latter. However, from 01 January 1992 onwards, the proper basis for the computation of the real property tax payable, including penalties or interests, if applicable, must be Rep. Act No. 7160 inasmuch as Section 534 had expressly repealed P.D. No. 464 or the Real Property Tax Code. Section 5(d) of Rep. Act No. 7160 provides that rights and obligations existing on the date of effectivity of the new Code and arising out of contracts or any source of prestation involving a local government unit shall be governed by the original terms and conditions of the said contracts or the law in force at the time such contracts were vested. Section 39. Rates Of LevyThe provincial, city or municipal board or council shall fix a uniform rate of real property tax applicable to their respective localities. Benguet Corp contends that LGUs are without any authority to levy realty taxes on mines per PD 463 and the Local Tax Code. WON it is the LGUs and not the National Government which levy realty taxes Held: No, it is the national government While the local government units are changed with fixing the rate of real property taxes, it does not necessarily follow from that authority the determination of the imposition of the tax. LGUs have no alternative but to collect taxes as mandated by Section 38 ( Incidence of Real Property Tax) of the Real Property Tax Code. It is clear that is the national government that levies real property tax. When LGUs are required to fix the rates, they are merely constituted as agents of the national government in the enforcement of the Real Property Tax Code. There is even no delegation of power to speak of because the national government has already imposed realty tax in Section 38, leaving only the enforcement to be done by the local governments. Realty tax has always been imposed by the law-making body. It is enforced throughout the Philippines and not merely in a particular municipality or city but the proceeds of the tax accrue to the province, city, municipality and barrio where the municipality is situated. (Section 68 of PD 464) IN contrast, a local tax is imposed by the municipality or city council pursuant to the Local Tax Code. The provisions of the Local Tax Code and PD 463 alleged by the corporation regarding the absence of power to levy realty taxes by LGUs on mines are mere limitations on the taxing power of the LGUs and are not pertinent to the issue in the case at bar. It cannot affect the imposition of real property tax by the national government.

Benguet Corp v. CBAA National Development Co. v. Cebu City


Benguet Corporation has bunkhouses used by its rankand-file employees for residential purposes. The Provincial Assessor of Benguet assessed real property tax on these bunkhouses. According to him, the tax exemption of bunkhouses under PD 745 was already withdrawn by PD 1955. The main contention if Benguet Corp is that the realty taxes are local taxes because these taxes are levied by LGUs per Section 39 of PD 464. National Development Company (NDC) is a GOCC authorized to engage in commercial, industrial, mining, agricultural and other enterprises necessary or contributory to economic development or important to public interest. It also operates subsidiary corporations one of which is National Warehousing Corporation (NWC).

61 D2010

Local Government
In 1939, the President issued Proclamation No. 430 reserving Block no. 4, Reclamation Area No. 4, of Cebu City for warehousing purposes under the administration of NWC. In 1940, a warehouse with a floor area of 1,940 square meters more or less, was constructed thereon. In 1947, EO 93 dissolved NWC with NDC taking over its assets and functions. In 1948, Cebu City assessed and collected from NDC real estate taxes on the land and the warehouse thereon. By the first quarter of 1970, a substantial amount of the taxes were paid under protest. NDC asked for a full refund contending that the land and the warehouse belonged to the Republic and therefore exempt from taxation. WON the NDC is exempt from real estate taxes Held: Yes To come within the ambit of the exemption provided in Art. 3, par. (a), of the Assessment Law, it is important to establish that the property is owned by the government or its unincorporated agency, and once government ownership is determined, the nature of the use of the property, whether for proprietary or sovereign purposes, becomes immaterial. What appears to have been ceded to NWC (later transferred to NDC), in the case before Us, is merely the administration of the property while the government retains ownership of what has been declared reserved for warehousing purposes under Proclamation No. 430. A reserved land is defined as a public land has been withheld or kept back from sale or disposition. The government does not part with its title by reserving them, but gives notice to the world that it desires them for a certain purpose.Absolute disposition of land is not implied from reservation; it merely means a withdrawal of a specified portion of the pubic domain from disposal under the land laws and the appropriation for the time being to some particular use or purpose of the general government.As its title remains with the Republic, the reserved land is clearly covered by the tax exemption provision. However, as regards the warehouse constructed on a public reservation, a different rule should apply because "[t]he exemption of public property from taxation does not extend to improvements on the public lands made by pre-emptioners, homesteaders and other claimants, or occupants, at their own expense, and these are taxable by the state . . ." Consequently, the warehouse constructed on the reserved land by NWC (now under administration by NDC), indeed, should properly be assessed real estate tax as such improvement does not appear to belong to the Republic.

Province of Tarlac v. Judge Alcantara


Tarlac Enterprises Inc .is the owner of a parcel of land in Mabini, Tarlac, an ice drop factory in said land, machinery shed and other machinery. These properties were declared for purposes of taxation in the Provincial Assessors Office. The Provincial Treasurer found that real estate taxes for the years 1974 until 1992 including penalties were not yet paid. Therefore, the Provincial Treasurer filed a complaint praying that the company pay the said sum as well as damages. The court rendered the decision dismissing the complaint. It ruled that P.D. No. 551 expressly exempts private respondent from paying the real property taxes demanded, it being a grantee of a franchise to generate, distribute and sell electric current for light. The court held that in lieu of said taxes, private respondent had been required to pay 2% franchise tax in line with the intent of the law to give assistance to operators such as the private respondent to enable the consumers to enjoy cheaper rates. WON Tarlac Enterprises, Inc. is exempt from the payment of real property tax under Sec. 40 (g) of P.D. No. 464 in relation to P.D. No. 551, as amended. Held: No The phrase "in lieu of all taxes and assessments of whatever nature" in the second paragraph of Sec. 1 of P.D. No. 551 does not expressly exempts private respondent from paying real property taxes. Said proviso is modified and delimited by the phrase "on earnings, receipts. income and privilege of generation, distribution and sale" which specifies the kinds of taxes and assessments which shall not be collected in view of the imposition of the franchise tax. Said enumerated items upon which taxes shall not be imposed, have no relation at all to, and are entirely different from real properties subject to tax. If the intention of the law is to exempt electric franchise grantees from paying real property tax and to make the 2% franchise tax the only imposable tax, then said enumerated items would not have been added when PD 852 was enacted to amend P.D. No. 551. The legislative authority would have simply stopped after the phrase "national or local authority" by putting therein a period. On the contrary, it went on to enumerate what should not be subject to tax thereby delimiting the extent of the exemption.

D2010 62

UP College of Law
It bears emphasis that P.D. No. 551 as amended by P.D. No. 852 deals with franchise tax and tariff on fuel oils and the "earnings, receipts, income and privilege of generation, distribution and sale of electric current" are the items exempted from taxation by the imposition of said tax or tariff duty. On the other hand, the collection complaint filed by petitioner specified only taxes due on real properties. While P.D. No. 551 was intended to give "assistance to the franchise holders by reducing some of their tax and tariff obligations," to construe said decree as having granted such franchise holders exemption from payment of real property tax would unduly extend the ambit of exemptions beyond the purview of the law. P.D. No. 551 is not as all-encompassing as said provision of the Local Autonomy Act for it enumerates the items which are not taxable by virtue of the payment of franchise tax. It has always been the rule that "exemptions from taxation are construed in strictissimi juris against the taxpayer and liberally in favor of the taxing authority" primarily because "taxes are the lifeblood of government and their prompt and certain availability is an imperious need." Thus, to be exempted from payment of taxes, it is the taxpayer's duty to justify the exemption "by words too plain to be mistaken and too categorical to be misinterpreted. Private respondent has utterly failed to discharge this duty. iv. Shares of LGUs in National Taxes departments and agencies, including state universities and colleges, GOCCs and LGUs will identify and implement measures in FY 1998 that will replace total expenditures by at least 25% of authorized regular appropriations for non-personal services items. Section 4 also provided that pending assessment by the Development Budget Coordinating Committee of the emerging fiscal situation, the amount equivalent to 10% of the IRA to LGUs shall be withheld. President Estrada issued AO 43, amending Section 4 by reducing to 5% the IRA to be withheld. WON Section 1 of AO 372, insofar as it "directs" LGUs to reduce their expenditures by 25% is valid Held: Yes Section 1 of AO 372, insofar as it directs LGUs to reduce expenditures by at least 25% is a valid exercise of the Presidents power of general supervision over LGUs as it is advisory only. Supervisory power, when contrasted with control, is the power of mere oversight over an inferior body; it does not include any restraining authority over such body. (Taule vs. Santos) Under existing law, LGU, in addition to having administrative autonomy, enjoy fiscal autonomy as well. Fiscal autonomy means that local governments have the power to create their own sources of revenue in addition to their equitable share in the national taxes released by the national government, as well as the power to allocate their resources in accordance with their own priorities. It extends to the preparation of their budgets, and local officials in turn have to work within the constraints thereof. Local fiscal autonomy does not however rule out any manner of national government intervention by way of supervision, in order to ensure that local programs, fiscal and otherwise, are consistent with national goals. However, under the Constitution, the formulation and the implementation of such policies and programs are subject to "consultations with the appropriate public agencies, various private sectors, and local government units." The President cannot do so unilaterally. WON withholding a part of LGUs IRA is valid Held: No Section 4 is invalid because it interferes with local autonomy, particularly local fiscal autonomy. Local autonomy signified a more responsive and accountable local government structure instituted through a system of decentralization. A basic feature of local fiscal autonomy is the automatic release of the share of LGUs in the national internal revenue. This is mandated by the

1987 Constitution
Section 6. Local government units shall have a just share, as determined by law, in the national taxes which shall be automatically released to them. Section 7. Local governments shall be entitled to an equitable share in the proceeds of the utilization and development of the national wealth within their respective areas, in the manner provided by law, including sharing the same with the inhabitants by way of direct benefits.

Sec 284-294, LGC


See attachments

Pimentel v. Aguirre
President Ramos issued Administrative Order 372 (Adoption of Economic Measures in Government for Fiscal Year 1998). Section 1 provided that all government

63 D2010

Local Government
Constitution. Section 4 orders the withholding of 10% of IRA pending he assessment and evaluation by the Development Budget Coordinating Committee of the emerging fiscal situation. Such withholding contravenes the Constitution and the law. Although temporary, it Is equivalent to a holdback, which means something held back or withheld, often temporarily. Hence, the temporary nature of the retention does not matter. Any retention is prohibited. C. Abatement of Nuisance Held: No (Tan Gin entitled to just compensation but not to damages) It is not disputed that the quonset building, which is being used for the storage of copra, is located outside the zone for warehouses. It is referred to in Ordinance as a non-conforming structure, which should be relocated. In the event that an immediate relocation of the building can not be accomplished, Sec 16 of the Ordinance provides: A certificate of non-conformance for all nonconforming uses shall be applied for by the owner or agent of the property involved within 12mo from the approval of this Ordinance, otherwise the nonconforming use may be condemned or removed at the owner's expense. Even granting that petitioner failed to apply for a Certificate of Non-conformance, the provision should not be interpreted as authorizing the summary removal of a non-conforming building by the municipal government. For if it does, it must be struck down for being in contravention of the requirements of due process, as originally held by the CA. The enforcement and administration of the provisions of the Ordinance resides with the Zoning Administrator , who may call upon the City Fiscal to institute the necessary legal proceedings to enforce the provisions of the Ordinance. Violation of a municipal ordinance does not empower municipal mayor to avail of extra-judicial remedies. . On the contrary, the Local Government Code imposes upon him the duty "to cause to be instituted judicial proceedings in connection with the violation of ordinances". Respondents can not seek cover under the general welfare clause authorizing the abatement of nuisances without judicial proceedings. That tenet applies to a nuisance per se or one which affects the immediate safety of persons and property and may be summarily abated under the undefined law of necessity. The storage of copra in the quonset building is a legitimate business. By its nature, it can not be said to be injurious to rights of property, of health or of comfort of the community. If it be a nuisance per accidens it may be so proven in a hearing conducted for that purpose. It is not per se a nuisance warranting its summary abatement without judicial intervention. While the Sangguniang Bayan may provide for the abatement of a nuisance (Local Government Code, Sec. 149 [ee]), it can not declare a particular thing as a nuisance per se and order its condemnation. The nuisance can only be so adjudged by judicial determination.

Estate of Francisco v. CA
A quonset was constructed by the American Liberation Forces in 1944. It was purchased in 1946 by Gregoria Francisco. It stands on a lot owned by the PPA and faces the municipal wharf. By virtue of Proclamation No. 83 issued by President Elpidio Quirino, said land was declared for the exclusive use of port facilities. The PPA issued to Tan Gin San, spouse of Gregoria Francisco, a permit to occupy the lot where the building stands for a period of one (1) year, to expire on 31 December 1989. Tan Gin used the quonset for the storage of copra. On May 1989, the Mayor notified Tan Gin San to remove or relocate its Quonset building citing Ordinance No. 147, noting its antiquated and dilapidated structure; and stressing the "clean-up campaign on illegal squatters and unsanitary surroundings along Strong Boulevard." Likewise, according to the Mayor, the building was outside the warehouse zone; hence it was a nonconfirming structure. The notifications remained unheeded; thus, the Mayor ordered the demolition on 24 May 1989. Petitioner sought a Writ of Prohibition with Injunction and Damages before the RTC of Basilan, which denied the petition and upheld the power of the Mayor to order the demolition without judicial authority pursuant to Ordinance 147. On 6 September 1989, petitioner's quonset building was completely demolished. The CA reversed the RTC and ruled that the mayor was not vested with power to order summarily without any judicial proceeding to demolish the Quonset building which was not a nuisance per se. Upon reconsideration, however, the CA reversed itself and ruled that the deficiency was remedied when petitioner filed a petition for prohibition and injunction and was heard on oral argument. WON it was proper for the Mayor to summarily, without judicial process, order the demolition of the nonconforming structure.

D2010 64

UP College of Law
Petitioner was in lawful possession of the lot and quonset building by virtue of a permit from the PPA when demolition was effected. It was not squatting on public land. It was entitled to an impartial hearing before a tribunal authorized to decide whether the quonset building did constitute a nuisance in law. There was no compelling necessity for precipitate action. The public officials of Isabela, Basilan, transcended their authority in abating summarily petitioner's quonset building. They had deprived petitioner of its property without due process of law. set aside the order which granted a writ of preliminary mandatory injunction and dissolved the writ issued. WON the Mayor validly issued the closure order? Held: YES The following circumstances militate against the maintenance of the writ of preliminary injunction sought by petitioner: 1. No mayor's permit had been secured. The mayor of a town has as much responsibility to protect its inhabitants from pollution, and by virtue of his police power, he may deny the application for a permit to operate a business or otherwise close the same unless appropriate measures are taken to control and/or avoid injury to the health of the residents of the community from the emissions in the operation of the business. 2. The Acting Mayor called the attention of petitioner to the pollution emitted by the fumes of its plant whose offensive odor "not only pollute the air in the locality but also affect the health of the residents in the area," so that petitioner was ordered to stop its operation until further orders. 3. This action of the Acting Mayor was in response to the complaint of the residents of Barangay Guyong, Sta. Maria, Bulacan, directed to the Provincial Governor through channels. 4. The closure order of the Acting Mayor was issued only after an investigation was made by Marivic Guina who in her report observed that the fumes emitted by the plant goes directly to the surrounding houses and that no proper air pollution device has been installed. 5. Petitioner failed to produce a building permit from the municipality of Sta. Maria, but instead presented a building permit issued by an official of Makati on March 6, 1987. 6. While petitioner was able to present a temporary permit to operate by the then National Pollution Control Commission on December 15, 1987, the permit was good only up to May 25, 1988. Petitioner had not exerted any effort to extend or validate its permit much less to install any device to control the pollution and prevent any hazard to the health of the residents of the community. Court takes note of the plea of petitioner focusing on its huge investment in this dollar-earning industry. It must be stressed however, that concomitant with the need to promote investment and contribute to the growth of the

Technology Developers Inc. v. CA


Petitioner received a letter from acting mayor Pablo N. Cruz, ordering the full cessation of the operation of the petitioner's plant located at Guyong, Sta. Maria, Bulacan. The letter requested Plant Manager Armando Manese to bring with him to the office of the mayor the following: a) Building permit; b) Mayor's permit; c) Region IIIPollution of Environment and Natural Resources AntiPollution Permit; and of other document. Petitioner undertook to comply with respondent's request for the production of the required documents. Petitioner commenced to secure "Region III-DENR AntiPollution Permit," although among the permits previously secured prior to the operation of petitioner's plant was a "Temporary Permit to Operate Air Pollution Installation" issued by the then National Pollution Control Commission. Petitioner's attention having been called to its lack of mayor's permit, it sent its representatives to the office of the mayor to secure the same but were not entertained. Without previous and reasonable notice upon petitioner, respondent ordered the Municipality's station commander to padlock the premises of petitioner's plant, thus effectively causing the stoppage of its operation. Petitioner instituted an action for certiorari, prohibition, mandamus with preliminary injunction against private respondent. The judge found that petitioner is entitled to the issuance of a writ of preliminary injunction upon posting of a bond worth P50,000. During the MR, the Provincial Prosecutor presented his evidence prepared by Marivic Guina, Due to the manufacturing process and nature of raw materials used, the fumes coming from the factory may contain particulate matters which are hazardous to the health of the people. As such, the company should cease operating until such a time that the proper air pollution device is installed and operational." The lower court then

65 D2010

Local Government
economy is the equally essential imperative of protecting the health, nay the very lives of the people, from the deleterious effect of the pollution of the environment. There should be no quarrel over permit fees for fishpens, fishcages and other aqua-culture structures in the Laguna de Bay area. Section 3 of Executive Order No. 927 provides for the proper sharing of fees collected. In view of the foregoing, this Court holds that Section 149 of RA 7160, otherwise known as the Local Government Code of 1991, has not repealed the provisions of the charter of the LLDA, Republic Act No. 4850, as amended. Thus, the Authority has the exclusive jurisdiction to issue permits for the enjoyment of fishery privileges in Laguna de Bay to the exclusion of municipalities situated therein and the authority to exercise such powers as are by its charter vested on it. D. Power of Eminent Domain

Laguna Lake Development Authority v. CA


Issue: Which agency of the Government - the LLDA or the towns and municipalities comprising the region should exercise jurisdiction over the Laguna Lake and its environs insofar as the issuance of permits for fishery privileges is concerned? Held: LLDA Section 4 (k) of RA 4850, the provisions of PD 813, and Section 2 of EO 927, specifically provide that the LLDA shall have exclusive jurisdiction to issue permits for the use or all surface water for any projects or activities in or affecting the said region, including navigation, construction, and operation of fishpens, fish enclosures, fish corrals and the like. On the other hand, RA 7160 has granted to the municipalities the exclusive authority to grant fishery privileges in municipal waters. The Sangguniang Bayan may grant fishery privileges to erect fish corrals, oyster, mussels or other aquatic beds or bangus fry area within a definite zone of the municipal waters. The provisions of RA7160 do not necessarily repeal the laws creating the LLDA and granting the latter water rights authority over Laguna de Bay and the lake region. The Local Government Code of 1991 does not contain any express provision which categorically expressly repeal the charter of the Authority. It has to be conceded that there was no intent on the part of the legislature to repeal Republic Act No. 4850 and its amendments. The repeal of laws should be made clear and expressed. Where there is a conflict between a general law and a special statute, the special statute should prevail since it evinces the legislative intent more clearly that the general statute. The special law is to be taken as an exception to the general law in the absence of special circumstances forcing a contrary conclusion. Considering the reasons behind the establishment of the Authority, which are enviromental protection, navigational safety, and sustainable development, there is every indication that the legislative intent is for the Authority to proceed with its mission.

Rule 67 Expropriation, Rules of Court


Section 1. The complaint. The right of eminent domain shall be exercised by the filing of a verified complaint which shall state with certainty the right and purpose of expropriation, describe the real or personal property sought to be expropriated, and join as defendants all persons owning or claiming to own, or occupying, any part thereof or interest therein, showing, so far as practicable, the separate interest of each defendant. If the title to any property sought to be expropriated appears to be in the Republic of the Philippines, although occupied by private individuals, or if the title is otherwise obscure or doubtful so that the plaintiff cannot with accuracy or certainty specify who are the real owners, averment to that effect shall be made in the complaint. (1a) Sec. 2. Entry of plaintiff upon depositing value with authorized government depositary. Upon the filing of the complaint or at any time thereafter and after due notice to the defendant, the plaintiff shall have the right to take or enter upon the possession of the real property involved if he deposits with the authorized government depositary an amount equivalent to the assessed value of the property for purposes of taxation to be held by such bank subject to the orders of the court. Such deposit shall be in money, unless in lieu thereof the court authorizes the deposit of a certificate of deposit of a government bank of the Republic of the Philippines payable on demand to the authorized government depositary. If personal property is involved, its value shall be provisionally ascertained and the amount to be deposited shall be promptly fixed by the court.

D2010 66

UP College of Law
After such deposit is made the court shall order the sheriff or other proper officer to forthwith place the plaintiff in possession of the property involved and promptly submit a report thereof to the court with service of copies to the parties. (2a) Sec. 3. Defenses and objections. If a defendant has no objection or defense to the action or the taking of his property, he may file and serve a notice of appearance and a manifestation to that effect, specifically designating or identifying the property in which he claims to be interested, within the time stated in the summons. Thereafter, he shall be entitled to notice of all proceedings affecting the same. If a defendant has any objection to the filing of or the allegations in the complaint, or any objection or defense to the taking of his property, he shall serve his answer within the time stated in the summons. The answer shall specifically designate or identify the property in which he claims to have an interest, state the nature and extent of the interest claimed, and adduce all his objections and defenses to the taking of his property. No counterclaim, cross-claim or third-party complaint shall be alleged or allowed in the answer or any subsequent pleading. A defendant waives all defenses and objections not so alleged but the court, in the interest of justice, may permit amendments to the answer to be made not later than ten (10) days from the filing thereof. However, at the trial of the issue of just compensation, whether or not a defendant has previously appeared or answered, he may present evidence as to the amount of the compensation to be paid for his property, and he may share in the distribution of the award. (n) Sec. 4. Order of expropriation. If the objections to and the defenses against the right of the plaintiff to expropriate the property are overruled, or when no party appears to defend as required by this Rule, the court may issue an order of expropriation declaring that the plaintiff has a lawful right to take the property sought to be expropriated, for the public use or purpose described in the complaint, upon the payment of just compensation to be determined as of the date of the taking of the property or the filing of the complaint, whichever came first. A final order sustaining the right to expropriate the property may be appealed by any party aggrieved thereby. Such appeal, however, shall not prevent the court from determining the just compensation to be paid. After the rendition of such an order, the plaintiff shall not be permitted to dismiss or discontinue the proceeding except on such terms as the court deems just and equitable. (4a) Sec. 5. Ascertainment of compensation. Upon the rendition of the order of expropriation, the court shall appoint not more than three (3) competent and disinterested persons as commissioners to ascertain and report to the court the just compensation for the property sought to be taken. The order of appointment shall designate the time and place of the first session of the hearing to be held by the commissioners and specify the time within which their report shall be submitted to the court. Copies of the order shall be served on the parties. Objections to the appointment of any of the commissioners shall be filed with the court within ten (10) days from service, and shall be resolved within thirty (30) days after all the commissioners shall have received copies of the objections. (5a) Sec. 6. Proceedings by commissioners. Before entering upon the performance of their duties, the commissioners shall take and subscribe an oath that they will faithfully perform their duties as commissioners, which oath shall be filed in court with the other proceedings in the case. Evidence may be introduced by either party before the commissioners who are authorized to administer oaths on hearings before them, and the commissioners shall, unless the parties consent to the contrary, after due notice to the parties to attend, view and examine the property sought to be expropriated and its surroundings, and may measure the same, after which either party may, by himself or counsel, argue the case. The commissioners shall assess the consequential damages to the property not taken and deduct from such consequential damages the consequential benefits to be derived by the owner from the public use or purpose of the property taken, the operation of its franchise by the corporation or the carrying on of the business of the corporation or person taking the property. But in no case shall the consequential benefits assessed exceed the consequential damages assessed, or the owner be deprived of the actual value of his property so taken. (6a) Sec. 7. Report by commissioners and judgment thereupon. The court may order the commissioners to report when any particular portion of the real estate shall have been passed upon by them, and may render judgment upon such partial report, and direct the commissioners to proceed with their work as to

67 D2010

Local Government
subsequent portions of the property sought to be expropriated, and may from time to time so deal with such property. The commissioners shall make a full and accurate report to the court of all their proceedings, and such proceedings shall not be effectual until the court shall have accepted their report and rendered judgment in accordance with their recommendations. Except as otherwise expressly ordered by the court, such report shall be filed within sixty (60) days from the date the commissioners were notified of their appointment, which time may be extended in the discretion of the court. Upon the filing of such report, the clerk of the court shall serve copies thereof on all interested parties, with notice that they are allowed ten (10) days within which to file objections to the findings of the report, if they so desire. (7a) Sec. 8. Action upon commissioners report. Upon the expiration of the period of ten (10) days referred to in the preceding section, or even before the expiration of such period but after all the interested parties have filed their objections to the report or their statement of agreement therewith, the court may, after hearing, accept the report and render judgment in accordance therewith; or, for cause shown, it may recommit the same to the commissioners for further report of facts; or it may set aside the report and appoint new commissioners; or it may accept the report in part and reject it in part; and it may make such order or render such judgment as shall secure to the plaintiff the property essential to the exercise of his right of expropriation, and to the defendant just compensation for the property so taken. (8a) Sec. 9. Uncertain ownership; conflicting claims. If the ownership of the property taken is uncertain, or there are conflicting claims to any part thereof, the court may order any sum or sums awarded as compensation for the property to be paid to the court for the benefit of the person adjudged in the same proceeding to be entitled thereto. But the judgment shall require the payment of the sum or sums awarded to either the defendant or the court before the plaintiff can enter upon the property, or retain it for the public use or purpose if entry has already been made. (9a) Sec. 10. Rights of plaintiff after judgment and payment. Upon payment by the plaintiff to the defendant of the compensation fixed by the judgment, with legal interest thereon from the taking of the possession of the property, or after tender to him of the amount so fixed and payment of the costs, the plaintiff shall have the right to enter upon the property expropriated and to appropriate it for the public use or purpose defined in the judgment, or to retain it should he have taken immediate possession thereof under the provisions of section 2 hereof. If the defendant and his counsel absent themselves from the court, or decline to receive the amount tendered, the same shall be ordered to be deposited in court and such deposit shall have the same effect as actual payment thereof to the defendant or the person ultimately adjudged entitled thereto. (10a) Sec. 11. Entry not delayed by appeal; effect of reversal. The right of the plaintiff to enter upon the property of the defendant and appropriate the same for public use or purpose shall not be delayed by an appeal from the judgment. But if the appellate court determines that plaintiff has no right of expropriation, judgment shall be rendered ordering the Regional Trial Court to forthwith enforce the restoration to the defendant of the possession of the property, and to determine the damages which the defendant sustained and may recover by reason of the possession taken by the plaintiff. (11a) Sec. 12. Costs, by whom paid. The fees of the commissioners shall be taxed as a part of the costs of the proceedings. All costs, except those of rival claimants litigating their claims, shall be paid by the plaintiff, unless an appeal is taken by the owner of the property and the judgment is affirmed, in which event the costs of the appeal shall be paid by the owner. (12a) Sec. 13. Recording judgment, and its effect. The judgment entered in expropriation proceedings shall state definitely, by an adequate description, the particular property or interest therein expropriated, and the nature of the public use or purpose for which it is expropriated. When real estate is expropriated, a certified copy of such judgment shall be recorded in the registry of deeds of the place in which the property is situated, and its effect shall be to vest in the plaintiff the title to the real estate so described for such public use or purpose. (13a) Sec. 14. Power of guardian in such proceedings. The guardian or guardian ad litem of a minor or of a person judicially declared to be incompetent may, with the approval of the court first had, do and perform on behalf of his ward any act, matter, or thing respecting the expropriation for public use or purpose of property belonging to such minor or person judicially declared to be incompetent, which such minor or person judicially declared to be incompetent could do in such proceedings if he were of age or competent. (14a)

D2010 68

UP College of Law Sec 19, LGC


Sec. 19. Eminent Domain. A local government unit may, through its chief executive and acting pursuant to an ordinance, exercise the power of eminent domain for public use, or purpose, or welfare for the benefits of the poor and the landless, upon payment of just compensation, pursuant to the provisions of the Constitution and pertinent laws: Provided, however, That the power of eminent domain may not be exercised unless a valid and definite offer has been previously made to the owner, and such offer was not accepted: Provided, further, That the local government unit may immediately take possession of the property upon the filing of the expropriation proceedings and upon making a deposit with the proper court of at least fifteen percent (15%) of the fair market value of the property based on the current tax declaration of the property to be expropriated: Provided, finally, That, the amount to be paid for the expropriated property shall be determined by the proper court, based on the fair market value at the time of the taking of the property. Thus, the Sangguniang Panlalawigan was without the authority to disapprove Municipal Resolution for the Municipality of Bunawan clearly has the power to exercise the right of eminent domain and its Sangguniang Bayan the capacity to promulgate said resolution. The resolution is, therefore, valid and binding and could be used as lawful authority to petition for the condemnation of petitioners' property.

Province of Camarines Sur v. CA


The Province of Camarines Sur authorized its governor by resolution to expropriate a piece of land contiguous to the provincial capitol site in order to establish a pilot farm for non-food and non-traditional agricultural crops and a housing project for provincial government employees. The governor filed expropriation cases in the RTC and then filed motions for the issuance of writs of possessions. The owners, San Joaquins, moved to dismiss the complaints for inadequacy of the price offered for the property. The RTC granted the writs of possession. The San Joaquins appealed to the CA which set aside the order allowing CamSur to take possession. It ordered the RTC to suspend further proceedings until after CamSur shall have obtained the requisite approval of the Dept. of Agrarian Reform to convert the classification of the land from agricultural to non-agricultural. WON a local government unit needs the approval of the DAR to reclassify land before it can expropriate it Held: No. A reading of previous jurisprudence shows that the power of eminent domain is superior to the power to distribute lands under the land reform program. The local government units power of expropriation is a delegated power. In delegating the power, the legislature may retain certain control or impose certain restraints on the exercise thereof by the local governments. While such delegated power may be a limited authority, it is complete within its limits. Moreover, the limitations on the exercise of the delegated power must be clearly expressed, either in the law conferring the power or in other legislations. The Local Government Code does not intimate in the least that local government units must first secure the

Moday v. CA
The Sangguniang Bayan of the Municipality of Bunawan passed a resolution authorizing its municpal mayor to initiate expropriation proceedings against he property of Percival Moday for a farmers center and government sports facilities. The resolution was approved by the mayor and transmitted to the Sangguniang Panlalawigan for approval. The latter disapproved it and returned it with comment to the Municipality stating that expropriation was unnecessary because there are still available lots in Bunawan for the purpose. The municipality filed a motion to take possession of the land and this was granted by the RTC. The petitioner attacks the validity of the resolution because it was not approved by the province. WON a municipalitys resolution authorizing its mayor to expropriate was rendered invalid by its disapproval by the province Held: No. The law, as expressed in Section 153 of B.P. Blg. 337, grants the Sangguniang Panlalawigan the power to declare a municipal resolution invalid on the sole ground that it is beyond the power of the Sangguniang Bayan or the Mayor to issue.

69 D2010

Local Government
approval of the Department of Land Reform for the conversion of lands from agricultural to non-agricultural use, before they can institute the necessary expropriation proceedings. Likewise, there is no provision in the Comprehensive Agrarian Reform Law which expressly subjects the expropriation of agricultural lands by local government units to the control of the Department of Agrarian Reform. Statutes conferring the power of eminent domain to political subdivisions cannot be broadened or constricted by implication. To sustain the Court of Appeals would mean that the local government units can no longer expropriate agricultural lands needed for the construction of roads, bridges, schools, hospitals, etc, without first applying for conversion of the use of the lands with the Department of Agrarian Reform, because all of these projects would naturally involve a change in the land use. In effect, it would then be the Department of Agrarian Reform to scrutinize whether the expropriation is for a public purpose or public use. Ordinarily, it is the legislative branch of the local government unit that shall determine whether the use of the property sought to be expropriated shall be public, the same being an expression of legislative policy. There is also an ancient rule that restrictive statutes, no matter how broad their terms are, do not embrace the sovereign unless the sovereign is specially mentioned as subject thereto. The orders of the CA nullifying the trial court's order allowing the Province of Camarines Sur to take possession of private respondents' property and requiring the Province of Camarines Sur to obtain the approval of the Department of Agrarian Reform to convert or reclassify private respondents' property from agricultural to non-agricultural use are set aside. real property and the value of said property is determinative of jurisdiction. And the property involved in this case is P1,740 or less than P20,000. Therefore, it ruled, jurisdiction is with the MTC. Which court has jurisdiction over actions for eminent domain? Held: RTC. Actions for eminent domain are incapable of pecuniary estimation and thus jurisdiction properly lies with the RTC. In determining whether an action is one the subject matter of which is not capable of pecuniary estimation, the criterion is of first ascertaining the nature of the principal action or remedy sought. If it is primarily for the recovery of a sum of money, the claim is considered capable of pecuniary estimation. However, where the basic issue is something other than the right to recover a sum of money, or where the money claim is purely incidental to, or a consequence of, the principal relief sought, such actions are cases where the subject of the litigation may not be estimated in terms of money, and are cognizable exclusively by RTC. The rationale of the rule is plainly that the second class of cases, besides the determination of damages, demands an inquiry into other factors which the law has deemed to be more within the competence of RTC. Actions for eminent domain involve two phases. The first is concerned with the determination of the authority of the plaintiff to exercise the power of eminent domain and the propriety of its exercise in the context of the facts involved in the suit.

Barangay San Roque v. Heirs of Pastor


Barangay San Roque filed an expropriation suit with the Municipal Trial Court against the properties of the respondents. It was dismissed by the MTC for lack of jurisdiction. The lower court reasoned that an action for eminent domain is incapable of pecuniary estimation and jurisdiction should, therefore, lie with the RTC. The case was then filed with the RTC but it was again dismissed for lack of jurisdiction. The RTC was of the view that an action for eminent domain involved title to

It ends with an order, if not of dismissal of the action, of condemnation declaring that the plaintiff has a lawful right to take the property sought to be condemned, for the public use or purpose described in the complaint, upon the payment of just compensation to be determined as of the date of the filing of the complaint. The second phase of the eminent domain action is concerned with the determination by the court of the just compensation for the property sought to be taken. This is done by the Court with the assistance of not more than three (3) commissioners. The order fixing the just compensation on the basis of the evidence before, and

D2010 70

UP College of Law
findings of, the commissioners would be final, too. It would finally dispose of the second stage of the suit, and leave nothing more to be done by the Court regarding the issue. It should be stressed that the primary consideration in an expropriation suit is whether the government or any of its instrumentalities has complied with the requisites for the taking of private property. Hence, the courts determine: 2. the authority of the government entity, the necessity of the expropriation, and the observance of due process. 3. Thus, the following essential requisites must concur before an LGU can exercise the power of eminent domain: 1. An ordinance is enacted by the local legislative council authorizing the local chief executive, in behalf of the LGU, to exercise the power of eminent domain or pursue expropriation proceedings over a particular private property. The power of eminent domain is exercised for public use, purpose or welfare, or for the benefit of the poor and the landless. There is payment of just compensation, as required under Section 9, Article III of the Constitution, and other pertinent laws. A valid and definite offer has been previously made to the owner of the property sought to be expropriated, but said offer was not accepted.

In the main, the subject of an expropriation suit is the governments exercise of eminent domain, a matter that is incapable of pecuniary estimation. True, the value of the property to be expropriated is estimated in monetary terms, for the court is duty-bound to determine the just compensation for it. This, however, is merely incidental to the expropriation suit. Indeed, that amount is determined only after the court is satisfied with the propriety of the expropriation.

4.

Municipality of Paranaque v. VM Realty Corp


The Municipality of Paranaque filed a complaint for expropriation against V.M. Realty Corporation over two parcels of land for the purpose of providing socialized housing. It authorized its mayor to file the complaint by a resolution. Later, the Municipality passed an Ordinance for the same purpose. It is claimed that by res judicata, the municipality is barred from filing another expropriation proceeding. WON authorization by a resolution is sufficient for compliance with the requisites under the LGC Held: No. The power of eminent domain is lodged in the legislative branch of government, which may delegate the exercise thereof to LGUs, other public entities and public utilities. An LGU may therefore exercise the power to expropriate private property only when authorized by Congress and subject to the latter's control and restraints, imposed "through the law conferring the power or in other legislations." And Section 19 of the LGC requires authorization by ordinance.

The terms "resolution" and "ordinance" are not synonymous. A municipal ordinance is different from a resolution. An ordinance is a law, but a resolution is merely a declaration of the sentiment or opinion of a lawmaking body on a specific matter. An ordinance possesses a general and permanent character, but a resolution is temporary in nature. Additionally, the two are enacted differently. A third reading is necessary for an ordinance, but not for a resolution, unless decided otherwise by a majority of all the Sanggunian members. WON res judicata may apply HELD: All the requisites for the application of res judicata are present in this case. Be that as it may, the principle of res judicata, which finds application in generally all cases and proceedings, cannot bar the right of the State or its agent to expropriate private property. The very nature of eminent domain, as an inherent power of the State, dictates that the right to exercise the power be absolute and unfettered even by a prior judgment or res judicata. The scope of eminent domain is plenary and, like police power, can "reach every form of property which the State might need for public use." "All separate interests of individuals in property are held of the government under this tacit agreement or implied reservation. Notwithstanding the grant to individuals, the eminent domain, the highest and most exact idea of property, remains in the government, or in the aggregate body of the people in their sovereign capacity; and they have the

71 D2010

Local Government
right to resume the possession of the property whenever the public interest requires it." 47 Thus, the State or its authorized agent cannot be forever barred from exercising said right by reason alone of previous noncompliance with any legal requirement. Our ruling that petitioner cannot exercise its delegated power of eminent domain through a mere resolution will not bar it from reinstituting similar proceedings, once the said legal requirement and, for that matter, all others are properly complied with. However, while the principle of res judicata does not denigrate the right of the State to exercise eminent domain, it does apply to specific issues decided in a previous case. 3. to this rule. This case should similarly be treated with liberality. Here an examination of the Citys Comment and Opposition to the Motion to Dismiss leave no room for doubt that petitioner had indeed made a valid and definite offer. Also, attached to the complaint and made an integral part of it is the Ordinance authorizing the Mayor to expropriate the property. The rule is that a motion to dismiss hypothetically admits the truth of the facts alleged in the complaint. And the Ordinance staes that the city government has made valid and definite offer.

City of Cebu v. CA
The City of Cebu authorized its mayor by both resolution and city ordinance to expropriate the property of Merlita Cardeno. Cardeno filed a motion to dismiss asseverating that the allegations in the Citys complaint do no show compliance with the condition precedent of a valid and definite offer. The allegation states, that repeated negotiations had been made with the defendant to have the aforementioned property purchased by the plaintiff through negotiated sale without resorting to expropriation, but said negotiations failed. She argues that, by definition, negotiations run the whole range of acts preparatory to concluding an agreement, from the preliminary correspondence; the fixing of the terms of the agreement; the price; the mode of payment; obligations of the parties may conceive as necessary to their agreement." Thus, "negotiations" by itself may pertain to any of the foregoing and does not automatically mean the making of "a valid and definite offer." WON the City has complied with the condition precedent of a valid and definite offer Held: Yes. 1. The ambiguity in the complaint is not a ground for a motion to dismiss but rather a bill of particulars. The RTC and CA should not have strictly applied the rule that in a motion to dismiss, no evidence may be allowed and the issue should only be determined in the light of allegations of the complaint. Jurisprudence has established exceptions

(The court did not decide whether or not negotiations is compliance with the requirement.)

Francia v. Municipality of Meycauyan


The Municipality of Meycauayan filed a complaint for expropriation against the petitioners to obtain property located at a highway junction to be used as a public terminal. The Municipality deposited the fifteen percent of the fair market value of the property based on the current tax declaration and the RTC granted it the writ of possession. The petitioner claims that the RTC committed grave abuse of discretion in issuing the writ without first conducting a hearing to determine the existence of a public purpose. WON determination of public purpose is a prerequisite to the issuance of a writ of possession Held: No. The Local Government Code states that local government unit may immediately take possession of the property upon the filing of the expropriation proceedings and upon making a deposit with the proper court of at least fifteen percent (15%) of the fair market value of the property based on the current tax declaration of the property to be expropriated. The requisites before a local government unit may enter into the possession of the property are only that (1) a complaint for expropriation must have been filed sufficient in form and substance in the proper court and (2) a deposit of 15% of the propertys fair market value based on its current tax declaration must have been made.

2.

D2010 72

UP College of Law
The law does not make the determination of a public purpose a condition precedent to the issuance of a writ of possession. Filstream is the property owner of parcels of lands located in Antonio Rivera St., Tondo II Manila. It filed an ejectment suit against the occupants (private respondents) on the grounds of termination of lease and non-payment of rentals. The ejectment suit became final and executory after the CA affirmed the decision of the RTC. During the pendency of the ejectment proceedings private respondents filed a complaint for Annulment of Deed of Exchange against Filstream. The City of Manila came into the picture when it approved Ordinance No. 7813 authorizing Mayor Alfredo Lim to initiate acquisition through legal means of certain parcels of land. Subsequently, the City of Manila approved Ordinance No. 7855 declaring the expropriation of certain parcels of land situated along Antonio River which formed part of the properties of Filstream. The said properties were sold and distributed to qualified tenants pursuant to the Land Use Development Program of the City of Manila. The City of Manila then filed a complaint for eminent domain seeking to expropriate lands in Antonio Rivera St. the Court issued a Writ of Possession. Filstream is claiming that Ordinance No. 7813 is a void enactment for it was approved without a public hearing and violative of the constitutional guarantee against impairment of obligations and contracts, the price was too low. HELD: Expropriation is not valid. The City of Manila did not comply with RA 7279, Secs. 9 and 10. The City of Manila has the power to expropriate private property in the pursuit of its urban land reform and housing program which must be done pursuant to the provisions of the Constitution and pertinent laws. There are limitations with respect to the order of priority in acquiring private lands and in resorting to expropriation proceedings as a means to acquire the same. Private lands rank last in the order of priority for purposes of socialized housing. In the same vein, expropriation proceedings are to be resorted to only when the other modes of acquisition have been exhausted. Also, Petitioner takes exception to the resolutions of respondent CA which ordered the dismissal of its Petition for Certiorari for non-compliance with Sec 2(a) of Rule 6 of the Revised Internal Rules of the CA by failing to attach to its petition other pertinent documents and papers and for attaching copies of pleadings which are blurred and unreadable. Petitioner argues that the respondent court erred in giving more premium to form rather than substance. A strict adherence to the

Heirs of Ardona v. Reyes


The Philippine Tourism Authority filed 4 complaints with the CFI Cebu City for the expropriation of some 282 hectares of rolling land situated in barangays Malubog and Babag, Cebu City for the development into integrated sport complexes of selected and well-defined geographic areas with potential tourism value. The defendants (now petitioners) have the common allegation that the taking is not impressed with public use under the Constitution, that there is no specific provision authorizing the taking for tourism purposes. Also, they are claiming that the lands are under the land reform program thus it is the Court of Agrarian Relations and not the CFI that has jurisdiction. The Court issued writs of possession authorizing the PTA to take immediate possession upon deposit with the Philippine National Bank an amount equivalent to 10% of the value of the properties. HELD: Expropriation is valid. The States power of eminent domain extends to the expropriation of land for tourism purposes although this specific objective is not expressed in the Constitution. The policy objectives of the framers can be expressed only in general terms. The programs to achieve these objectives vary from time to time and according to place. The concept of public use is not limited to traditional purposes; it does not strictly mean use by the public. Expropriation of several barangays for provocation of tourism and construction of sports and hotel complex constitutes expropriation for public use. Also, the government is authorized to take immediate possession, control and disposition of the property notwithstanding the issues pending in court upon deposit with the PNB of an amount equivalent to 10% of the value of the property. Defendants (now petitioners) failed to show that area being expropriated is land reform area. In fact, only 8,970 square meters (less than 1hec.-just 2 property owners) of 283 hectares is part of the Operation Land Transfer. This 8,970 sq. m. is not even within the sports complex proper but forms part of the 32 hectares resettlement area where the petitioners would be provided with proper housing, schools, etc.

Filstream International Inc. v. CA

73 D2010

Local Government
technical and procedural rules in this case would defeat rather than meet the ends of justice as it would result in the violation of the substantial rights of the petitioner. At stake in the appeal is the exercise of their properties rights over the disputed premises which have been expropriated and have in fact been ordered condemned in favor of the City of Manila. In effect, the dismissal of their appeal in the expropriation proceedings based on those grounds is tantamount to a deprivation of property without due process of law as it would automatically validate the expropriation proceedings appealed Resolution certified by the Dept. of Justice but failed to do so on account of typhoon Loleng. Nonetheless, the Court held that the petition should be dismissed as the appeal of the petitioner with the Sec. of Justice was already time-barred. Sec. 187 of the 1991 LGC states that an appeal of a tax ordinance or revenue measure should be made to the Sec. of Justice within 30 days from the effectivity of the assailed ordinance shall not be suspended. In the case at bar, Mun. Ord, No. 28 took effect in Oct. 1996. Petitioner filed its appeal only in Dec. 1997. The periods stated in Sec. 187 LGC are mandatory. Being a revenue measure, the collection of which is of paramount importance thus it is essential that the validity of revenue measures is not left uncertain for a considerable length of time. Petitioners cannot gripe that there was practically no public hearing conducted as its objections to the proposed measure were not considered by the Sanguniang Bayan. Public hearings are conducted by legislative bodies to allow interested parties to ventilate their views on a proposed law or ordinance. However, the views are not binding on the legislative body and it is not compelled by law to adopt the same. Also, even on the substantive points raised, the petition must fail. Sec. 6c.04 of the 1993 Mun. Rev. Code & Sec 191 of the LGC limiting the percentage of increase that can be imposed apply to tax rates, not rentals. Neither can it be said that the rates were not uniformly imposed. The ordinance covered 3 public markets. However, it excluded Bagong Munisipyo from the increase since it is only a makeshift, dilapidated place intended for transient peddlers.

Hagonoy Market Vendors Association v. Mun of Hagonoy Bulacan


On Oct. 1, 1996, the Sanguniang Bayan of Hagonoy, Bulacan enacted an ordinance, Kautusan Blg. 28 which increased the stall rentals of market vendors in Hagonoy. Art.3 provided that it shall take effect upon approval. The subject ordinance was posted from Nov. 4-25, 1996. In the last week of Nov. 1997, the petitioners members were personally given copies of the approved Ordinance and were informed that it shall be enforced in Jan. 1998. On Dec. 8, 1997, the petitioners President filed an appeal with the Secretary of Justice assailing the constitutionality of the tax ordinance. Petitioner claimed it was unaware of the posting of the ordinance. Respondent opposed the appeal. It contended that the ordinance took effect on October 6, 1996 and that the ordinance, as approved, was posted as required by law. Hence, it was pointed out that petitioners appeal, made over a year later, was already time-barred. The Secretary of Justice dismissed the appeal on the ground that it was filed out of time beyond the 30 days from the effectivity of the Ordinance on Oct. 1, 1996 as prescribed under Sec.187 of the 1991 LGC. After its motion for reconsideration was denied, petitioner appealed to the CA, claiming the Sec. erred and should have overlooked the technicality and ruled on its petition on the merits. CA dismissed its petition for being formally deficient as it was not accompanied by certified true copies of the assailed Resolutions of the Sec. of Justice. HELD: Ordinance is valid & petition should be dismissed for being time-barred. The petitioner insists that it had good reasons for its failure to comply with the rule and the CA erred in refusing to accept its explanation. This Court agrees with the petitioner. It is clear from the records that the petitioner exerted due diligence to get the copies of its

RP v. CA
Notes: Lot to be expropriated for the broadcast operation and use of VP. Deposited amount provisionally fixed. 9 years after Court ordered condemnation and payment of just compensation. NG failed to pay. Santos (1) pay current zonal value or (2) return expropriated property HELD: no right to recover On Sept. 19, 1969, petitioner instituted expropriation proceedings before RTC Bulacan covering a total of 554,980 sq.m. of contiguous land situated along

D2010 74

UP College of Law
MacArthur Highway to be utilized for the continued broadcast operation and use of radio trasnsmitter facilities for the Voice of the Philippines project. Petitioner made a deposit of P517,558.80, the sum provisionally fixed as being the reasonable value of the property. On Feb. 26, 1979 or nine years after the institution of the expropriation proceedings, the trial courts issued an order condemning the properties of the defendants and ordering the plaintiff to pay the defendants just compensation. It would appear that the National Government failed to pay the respondents the just compensation pursuant to the foregoing decision. The respondents then filed a manifestation with a motion seeking payment for the expropriated property. In response, the court issued a writ of execution for the implementation thereof. Meanwhile, Pres. Estrada issued Proc. No. 22 transferring 20 hec. of the expropriated land to the Bulacan State University. Despite the courts order, the Santos heirs remained unpaid and no action was on their case until petitioner filed its manifestation and motion to permit the deposit in court of the amount P4,664,000 by way of just compensation. The Santos heirs submitted a countermotion to adjust the compensation from P6/sq.m. as previously fixed to its current zonal value of P5,000/sq.m. or to cause the return of the expropriated property. The RTC Bulacan ruled in favor of the Santos heirs declaring its previous decision to be unenforceable on the ground of prescription in accordance with Sec. 6, Rule 39 of the 1964/1997 ROC which states that a final and executory judgment or order may be executed on motion within 5 years from the date of its entry. HELD: Expropriation is valid; private respondents have no right to recover the property. The right of eminent domain is the ultimate right of the sovereign power to appropriate any property within its territorial sovereignty for a public purpose. Expropriation proceedings are not adversarial for the condemning authority is not required to assert any conflicting interest in the property. Thus, by filing the action, the condemnor in effect merely serves notice that it is taking title and possession of the property and the defendant asserts title/ interest in the property not to prove a right of possession but to prove a right to compensation for the taking. Petitioner has occupied and has exercised dominion over the property pursuant to the judgment. The exercise of such rights vested to it as the condemnee has amounted to atleast a partial compliance of the 1979 decision thereby preempting any claim of bar by prescription on grounds of non-execution. In determining the public use, 2 approaches are used actual use by the public and public benefit/ advantage. Respondents question the public nature of the utilization since its present use differs from the original use contemplated in the 1979 decision. This is of no moment, the property has assumed a public character upon its expropriation. Petitioner is well within its rights to alter and decide the use of that property. The constitutional limitation of the just compensation is considered to be the sum equivalent to the market value of the property fixed at the time of the actual taking by the government. However, between the actual taking of the property and the actual payment, legal interests may accrue in order to place the owner in a position as good as but not better than the position he was in before the taking occurred. E. Reclassification of Lands

CARP Act
See attachments

Sec 9 and 11, RA 8435


SECTION 9. Delineation of Strategic Agriculture and Fisheries Development Zones. - The Department, in consultation with the Department of Agrarian Reform, the Department of Trade and Industry, the Department of Environment and Natural Resources, Department of Science and Technology, the concerned LGUs, the organized farmers and fisherfolk groups, the private sector and communities shall, without prejudice to the development of identified economic zones and free ports, establish and delineate, based on sound resource accounting, the SAFDZ within one (1) year from the effectivity of this Act. All irrigated lands, irrigable lands already covered by irrigation projects with firm funding commitments, and lands with existing or having the potential for growing high-value crops so delineated and included within the SAFDZ shall not be converted for a period of five (5) years from the effectivity of this Act: Provided, however, That not more than five percent (5%) of the said lands located within the SAFDZ may be converted upon compliance with existing laws, rules, regulations, executive orders and issuances, and administrative orders relating to land use conversion: Provided, further, That thereafter: 1) a review of the SAFDZ, specifically on the productivity of the areas, improvement of the quality of life of farmers and fisherfolk, and efficiency and

75 D2010

Local Government
effectiveness of the support services shall be conducted by the Department and the Department of Agrarian Reform, in coordination with the Congressional Oversight Committee on Agricultural and Fisheries Modernization; 2) conversion may be allowed; if at all, on a case-to-case basis subject to existing laws, rules, regulations, executive orders and issuances, and administrative orders governing land use conversion; and, 3) in case of conversion, the land owner will pay the Department the amount equivalent to the government's investment cost including inflation. SECTION 10. Preparation of Land Use and Zoning Ordinance. - Within one (1) year from the finalization of the SAFDZ, in every city and municipality, all cities and municipalities shall have prepared their respective land use and zoning ordinance incorporating the SAFDZ, where applicable. Thereafter, all land use plans and zoning ordinances shall be updated every four (4) years or as often as may be deemed necessary upon the recommendation of the Housing and Land Use Regulatory Board and must be completed within the first year of the term of the mayor. If the cities/municipalities fail to comply with the preparation of zoning and land use plans, the DILG shall impose the penalty as provided for under Republic Act No. 7160. SECTION 11. Penalty for Agricultural Inactivity and Premature Conversion. - Any person or juridical entity who knowingly or deliberately causes any irrigated agricultural lands seven (7) hectares or larger, whether contiguous or not, within the protected areas for agricultural development, as specified under Section 6 in relation to Section 9 of this Act, to lie idle and unproductive for a period exceeding one (1) year, unless due to force majeure, shall be subject to an idle land tax of Three Thousand Pesos (P3,000.00) per hectare per year. In addition, the violator shall be required to put back such lands to productive agricultural use. Should the continued agricultural inactivity, unless due to force majeure, exceed a period of two (2) years, the land shall be subject to escheat proceedings. Any person found guilty of premature or illegal conversion shall be penalized with imprisonment of two (2) to six (6) years, or a fine equivalent to one hundred percent (100%) of the government's investment cost, or both, at the discretion of the court, and an accessory penalty of forfeiture of the land and any improvement thereon. In addition, the DAR may impose the following penalties, after determining, in an administrative proceedings, that violation of this law has been committed: a) Cancellation or withdrawal of the authorization for land use conversion; and b) Blacklisting, or automatic disapproval of pending and subsequent conversion applications that they may file with the DAR.

Fortich v. Corono
This concerns the motion for reconsideration of the courts resolution dated Nov. 17, 1998 and motion to refer the case to the Court en banc. In the previous case, the Court voted 2-2 on the separate MRs, as a result of which the decision was affirmed. The Court noted in a resolution dated Jan. 27, 1999 that the movants have no legal personality to seek redress before the Court as their motion to intervene was already denied and that the motion to refer the case to the Court en banc is akin to a second MR which is prohibited. In this motion, both respondents and intervenors prayed the case be referred to the court en banc inasmuch as their earlier MR was resolved by a vote of 2-2, the required number to carry a decision under the Constitution, 3votes, was not met. HELD: The issues presented before the Court by the movants are matters of no extraordinary import to merit the attention of the Court en banc. Specifically the issue of whether or not the power of the local government units to reclassify lands is subject to the approval of DAR is no longer novel, this having been decided in Province of Camarines Sur v. CA wherein the Court held that local government units need not obtain approval of the DAR to convert lands from agricultural to non-agricultural use. Intervenors insist that they are the real parties in interest inasmuch as they have already been issued certificates of land ownership award and that while they are seasonal farmers, they have been identified by DAR as qualified beneficiaries of property. The issuance of CLOA to them does not grant them the requisite standing in view of the nullity of the Win-Win Resolution. No legal rights can emanate from a resolution that is null and void. The same is void and has no legal effect considering that the March 29, 1996 decision of the Office of the President had already become final and executory even prior to the filing of the MR which became the basis of the said WinWin Resolution.

Roxas and Co v. IAC D2010 76

UP College of Law
Pres. Cory promulgated Proc. No. 131 instituting CARP and EO No. 229 providing for mechanisms necessary to implement CARP. Later when Congress formally convened, it passed RA 6657 or the Comprehensive Agrarian Reform Law. Petitioner Roxas is the property owner of Haciendas Palico, Banilad and Caylaway. Before CARL took effect, petitioner voluntarily offered to sell Hacienda Caylaway pursuant to EO 229. The other two were placed under compulsory acquisition by respondent DAR in accordance with CARL. Petitioner instituted a case with DAR Adjudication Bd. To cancel the CLOAs issued alleging that the place where the haciendas are located was declared a tourist zone and that the land is not suitable for agricultural production. DARAB held that the case involved the prejudicial question of whether the property was subject to agrarian reform and should be submitted to the Office of the Sec. of Agrarian Reform. Petitioner then filed a case with the CA questioning the expropriation of the properties under CARL and the denial of due process in the acquisition of the land. CA dismissed the petition on the ground of failure to exhaust administrative remedies. HELD: This Court can take cognizance of petitioners petition despite his failure to exhaust administrative remedies since his action falls under the exception to the doctrine of exhaustion of remedies since there is no other plain, speedy and adequate remedy. The CLOAs to the farmers were issued without just compensation to the petitioner. The transfer of possession/ ownership to the government is conditioned upon the receipt of the landowner of the corresponding payment/ deposit by the DAR. Until then, the title should remain with the landowner. The law provides that the deposit must be made only in cash or LBP bonds. Resp. DARs opening of a trust account deposit in petitioners name with the Land Bank of the Phil. does not constitute payment under the law. The acquisition proceedings over the 3 haciendas are invalid. (1) There was improper service of Notice of Acquisition Sec. 16 of CARL requires that the said notice be sent to the landowner by personal delivery or registered mail. The Revised Rules of Proc. of DARAB states that notices and pleadings are served on private corporations through its president, manager, secretary, cashier, agent or any of its directors or partners. Jaime Pimentel to whom the Notice was served was neither of those. (2) Parcels of land were not properly identified before they were taken by DAR under the law, a landowner may retain not more than 5 hec. the right to choose the retained land pertains to the landowner. Upon the receipt of Notice of Acquisition, petitioner corporation had no idea which portions of its estate were subject to compulsory acquisition. Court has no jurisdiction to rule on the reclassification of land from agricultural to non-agricultural. DARs failure to observe due process does not ipso facto give the Court the power to adjudicate over petitioners application for conversion of its haciendas from agricultural to non. The agency charged with the mandate of approving applications for conversion is DAR. F. Closure and Opening of Roads

Sec 21, LGC


Sec. 21. Closure and Opening of Roads. (a) A local government unit may, pursuant to an ordinance, permanently or temporarily close or open any local road, alley, park, or square falling within its jurisdiction: Provided, however, That in case of permanent closure, such ordinance must be approved by at least two-thirds (2/3) of all the members of the sanggunian, and when necessary, an adequate substitute for the public facility that is subject to closure is provided. (b) No such way or place or any part thereof shall be permanently closed without making provisions for the maintenance of public safety therein. A property thus permanently withdrawn from public use may be used or conveyed for any purpose for which other real property belonging to the local government unit concerned may be lawfully used or conveyed: Provided, however, That no freedom park shall be closed permanently without provision for its transfer or relocation to a new site. (c) Any national or local road, alley, park, or square may be temporarily closed during an actual emergency, or fiesta celebrations, public rallies, agricultural or industrial fairs, or an undertaking of public works and highways, telecommunications, and waterworks projects, the duration of which shall be specified by the local chief executive concerned in a written order: Provided, however, That no national or local road, alley, park, or square shall be temporarily closed for athletic, cultural, or civic activities not officially sponsored, recognized, or approved by the local government unit concerned. (d) Any city, municipality, or barangay may, by a duly enacted ordinance, temporarily close and regulate the use of any local street, road, thoroughfare, or any other public place where shopping malls, Sunday, flea or night markets, or shopping areas may be established and where goods, merchandise, foodstuffs, commodities, or

77 D2010

Local Government
articles of commerce may be sold and dispensed to the general public. the greater convenience the new road has been giving the public.

Cabrera v. CA
The Provincial Board of Catanduanes adopted Resolution No. 158 which provided for the closure of an old road leading to the new Capitol Bldng. and giving the owners of the properties traversed by the new road equal area from the old road adjacent to the respective remaining portion of their property. Pursuant thereto, Deeds of Exchange were executed under which the Province of Catanduanes conveyed to the private respondents portions of the closed road. In 1978, part of the northern end of the old road fronting the petitioners house was planted with vegetables by one of the private respondents. Another private respondent converted a part of the old road into a piggery farm. Learning about Res. 158, petitioner filed a complaint for Restoration of Public Road and/or Abatement of Nuisance, Annulment of Resolutions and Documents for Damages. Petitioner alleges that the land fronting his house was a public road owned by the Province of Catanduanes in its governmental function and therefore beyond the commerce of man. He contends that Res. No. 158 and the deeds of exchange were invalid as so too was the closure of the northern portion of the said road. HELD: Closure is valid. The closure of a provincial road is within the powers of the Provincial Bd. It is the authority competent to determine whether or not a certain property is still necessary to public use. Such power to vacate a street is discretionary and such will not ordinarily be controlled or interfered with by the courts absent a plain case of abuse or fraud. The Board has the duty of maintaining such roads for the comfort and convenience of the inhabitants of the province. This authority is inferable from the grant by the national legislature of the funds to the Province for the construction of roads. Petitioner is not entitled to damages. The general rule is that one whose property does not abut on the closed section of a street has no right of compensation for the closing or vacation of the street. To warrant recovery, the property owner must show that the situation is such that he has sustained special damages differing in kind and not merely in degree from those sustained by the public generally. The construction of the new road was undertaken under the general welfare clause. Whatever inconvenience petitioner suffered pales in significance to

MMDA v. Bel Air Village Association Inc.


MMDA is a government agency tasked with the delivery of basic services in Metro Manila, while BAVA is a nonstock non-profit corporation whose members are home owners in Bel-Air Village, a private subdivision in Makati City. BAVA is the registered owner of Neptune Street, a road inside Bel-Air Village. On 26 Dec 1995 BAVA received from MMDA Chairman Prospero Oreta a notice requesting it to open Neptune Street to public vehicular traffic starting 2 Jan 1996. BAVA was also informed that the perimeter wall separating the subdivision from adjacent Kalayaan Avenue would be demolished. On 2 Jan 1996, BAVA instituted a petition before the RTC Makati City Br 136 for the issuance of a TRO and preliminary injunction enjoining the opening of said street and the demolition of the wall. The TRO was issued the following day. But on 23 Jan 1996, after due hearing, it denied the preliminary injunction. BAVA appealed. CA conducted an ocular inspection of Neptune Street and on 13 Feb 1996 it issued a writ of preliminary injunction. It later on granted the petition (making the writ permanent), reasoning that MMDA had no authority to order the opening of a private subdivision road nor to demolish the subdivisions perimeter wall. MR was denied. WON MMDA has the authority to open Neptune Street to public traffic without an ordinance enacted by Makati City HELD: No MMDA is not a local government which is a political subdivision of a nation or state which is constituted by law and has substantial control of local affairs. MMDAs charter RA 7924 limited MMDAs function to the delivery of seven basic services, one of which is transport and traffic management. It is limited to the following acts: formulation, coordination, regulation, implementation, preparation, management, monitoring, setting of policies, installation of a system and administration. There is no syllable in RA 7924 that grants the MMDA police power, let alone legislative power. All its functions are administrative in nature. MMDA is not the same entity as its predecessor, the Metro Manila Commission because the charter of MMC (PD 824) shows that MMC possessed greater powers which were not bestowed on MMDA. Unlike MMC, MMDA has no power to enact ordinances for the welfare of the community. It is the LGUs, acting through their respective legislative councils,

D2010 78

UP College of Law
that possess legislative and police power. In this case, the Sangguniang Panlungsod of Makati City did not pass any ordinance or resolution ordering the opening of Neptune Street; hence, its proposed opening by MMDA is illegal. confirmed by the fact that both ordinances as well as the BAVA Articles of Incorporation treat Jupiter Street as the boundary line making it clear that Jupiter Street was intended for use by both commercial and residential blocks from the very beginning. WON the MMC Ordinance, raised by Ayala Corporation as an affirmative defense, was a legitimate exercise of police power HELD: Yes The National Government itself, through the MMC, had reclassified Jupiter Street into a high density commercial zone pursuant to its Ordinance 81-01. Hence, the petitioners have no cause of action on the strength alone of the deed restrictions. This is not to say that restrictive easements are invalid or ineffective. As far as Bel-Air Village is concerned, they are valid and enforceable. But they are, like all contracts, subject to the overriding demands, needs, and interests of the greater number as the State may determine in the legitimate exercise of police power. The sanctity of contract cannot be raised as a deterrent to police power, which is designed precisely to promote health, safety, peace, and enhance the common good, at the expense of contractual rights, whenever necessary. Absent any showing of capriciousness or arbitrariness on the part of MMC, the ordinance reclassifying Jupiter Street should be upheld.

Sangalang v. IAC
Buyers of lots in Bel-Air Village, owned and operated by Makati Development Corporation (later on merged with Ayala Corporation) were required to comply with certain deed restrictions, one of which is that the lots shall be used only for residential purposes. On 4 Apr 1975 the municipal council of Makati enacted Ordinance 81 (zonification of Makati), wherein Bel-Air Village was classified as a Class A Residential Zone, with its boundary extending to the center line of Jupiter Street. A subsequent Ordinance 81-01 of the Metro Manila Commission however showed that Bel-Air Village was simply bound by Jupiter Street, and the area on the other side was classified as a High Intensity Commercial Zone. Due to the reclassification, commercial establishments appeared along Jupiter Street, in violation of the deed restrictions. The Office of the Mayor of Makati wrote the Bel-Air Village Association, Inc. (BAVA), directing that, in the interest of public welfare, certain streets should be opened to ease traffic congestion. The Makati municipal officials allegedly opened, destroyed and removed the gates forcibly, and opened the entire length of Jupiter Street to public traffic. Ayala Corporation donated the entire Jupiter Street to BAVA, but even before 1978 the Makati Police and the security force of BAVA were already the ones regulating the traffic along Jupiter Street after the gates were opened in 1977. Residents of Jupiter Street (the Sangalang spouses, Gaston spouses, and Briones spouses) and the Bel-Air Village Association, Inc. (BAVA) filed separate actions against Ayala Corporation and the lot owners who allowed their lots to be used for commercial purposes. Three of the five trial courts in the five petitions ruled in favor of the petitioners, while two were dismissed on procedural and substantive grounds. On appeal, the Court of Appeals dismissed all five petitions, on the ground that the ordinances, both being valid as a legitimate exercise of police power, allowed the use of Jupiter Street for commercial purposes. WON there was a violation of the deed restrictions HELD: No There was no violation of the deed restrictions as Jupiter Street, which was the common boundary for the commercial and residential zones, was never exclusively for residential or commercial purposes only. This is

Macasiano v. Diokno
On 13 Jun 1990, the Municipality of Paraaque passed Ordinance 86 (Series of 1990) which authorized the closure of J. Gabriel, G.G. Cruz, Bayanihan, Lt Garcia Extension and Opena Streets in Baclaran and the establishment of a flea market thereon. This ordinance was approved by the municipal council pursuant to the Metro Manila Commission Ordinance 2 (Series of 1979) authorizing and regulating the use of certain city and/or municipal streets, roads and open spaces within Metro Manila as sites for flea market and/or vending areas under certain terms and conditions. The municipal council on 20 Jun 1990 issued a resolution authorizing Paraaque Mayor Walfrido Ferrer to enter into contract with any service cooperative for the establishment, operation, maintenance and management of flea markets and/or vending areas. On 8 Aug 1990, Palanyag Kilusang Bayan for Service (Palanyag), a service cooperative was made a flea market operator subject to municipality dues for such operation. On 13 Sep 1990, Brig Gen Levy Macasiano (PNP Superintendent of the Metropolitan Traffic Command) ordered the destruction and confiscation of stalls along G.G. Cruz and J Gabriel

79 D2010

Local Government
Streets. B/Gen Macasiano on 16 Oct 1990 gave notice to Palanyag that it had 10 days to discontinue the flea market; otherwise the market stalls would be dismantled. The Municipality and Palanyag filed with RTC Makati Br 62 (Judge Roberto Diokno presiding) a petition for prohibition and mandamus with damages and a prayer for preliminary injunction. RTC issued a TRO against Macasiano pending the hearing, and later on upheld the validity of the ordinance. WON an ordinance or resolution issued by the municipal council of Paraaque authorizing the lease and use of public streets or thoroughfares as sites for flea markets is valid HELD: No Said streets which were closed off are local roads used for public service and are therefore considered public properties of the municipality. Properties of the local government which are devoted to public service are deemed public and under the absolute control of Congress. Local governments have no authority to control or regulate the use of public properties unless specific authority is vested upon them by Congress. Sec 10, Loc Gov Code (BP 337, which was the applicable law at the time)empowers the local governments to close roads, but this provision should be read and interpreted in accordance with basic principles already established by law: 1. Art 424, CC: Properties of public dominion devoted to public use and made available to the public in general are outside the commerce of man and cannot be disposed of or leased by the LGU to private persons. 2. To withdraw a property from public use, the requirement of due process should be complied with, and circumstances should show that the property is no longer intended or necessary for public use or public service. (When the property is withdrawn from public use, it becomes patrimonial property of the LGU.) The Municipality has not shown that it has complied with the conditions precedent set by the Metropolitan Manila Authority for the latter to approve of such ordinance. (Conditions are that the streets are not used for vehicular traffic and that majority of the residents are not opposed to the establishment of a flea market/vending area, and that there is a time schedule in during which the flea market would operate.) Furthermore, the powers of a LGU should be subservient to paramount considerations of health and well-being of the members of the community. It is of public notice that the streets along Baclaran are congested with people, houses and traffic brought about by the proliferation of vendors occupying the streets. Thus, allowing the flea market would be contrary to the local governments sworn obligation to enact measures that will enhance the public health and safety of the municipalitys inhabitants. G. Corporate Powers Sec. 22. Corporate Powers. (a) Every local government unit, as a corporation, shall have the following powers: (1) To have continuous succession in its corporate name; (2) To sue and be sued; (3) To have and use a corporate seal; (4) To acquire and convey real or personal property; (5) To enter into contracts; and (6) To exercise such other powers as are granted to corporations, subject to the limitations provided in this Code and other laws. (b) Local government units may continue using, modify, or change their existing corporate seals: Provided, That newly established local government units or those without corporate seals may create their own corporate seals which shall be registered with the Department of the Interior and Local Government: Provided, further, That any change of corporate seal shall also be registered as provided herein. (c) Unless otherwise provided in this Code, no contract may be entered into by the local chief executive in behalf of the local government unit without prior authorization by the sanggunian concerned. A legible copy of such contract shall be posted at a conspicuous place in the provincial capitol or the city, municipal or barangay hall. (d) Local government units shall enjoy full autonomy in the exercise of their proprietary functions and in the management of their economic enterprises, subject to the limitations provided in this Code and other applicable laws. H. Liability for damages Sec. 24. Liability for Damages. Local government units and their officials are not exempt from liability for death or injury to persons or damage to property.

D2010 80

UP College of Law
I. Authority to negotiate and secure grants; receive donations; float bonds; build-operatetransfer

Sec. 23. Authority to Negotiate and Secure Grants. Local chief executives may, upon authority of the sanggunian, negotiate and secure financial grants or donations in kind, in support of the basic services or facilities enumerated under Section 17 hereof, from local and foreign assistance agencies without necessity of securing clearance or approval therefor from any department, agency, or office of the National Government of from any higher local government unit: Provided, That projects financed by such grants or assistance with national security implications shall be approved by the national agency concerned: Provided, further, That when such national agency fails to act on the request for approval within thirty (30) days from receipt thereof, the same shall be deemed approved. The local chief executive shall, within thirty (30) days upon signing of such grant agreement or deed of donation, report the nature, amount, and terms of such assistance to both Houses of Congress and the President. J. Mayors power over the police: Operational control; suspension

RA 8551
See attachments

Sec 28, LGC


Sec. 28. Powers of Local Chief Executives over the Units of the Philippine National Police. The extent of operational supervision and control of local chief executives over the police force, fire protection unit, and jail management personnel assigned in their respective jurisdictions shall be governed by the provisions of Republic Act Numbered Sixty-nine hundred seventy-five (R.A. No. 6975), otherwise known as "The Department of the Interior and Local Government Act of 1990", and the rules and regulations issued pursuant thereto.

81 D2010

Local Government

Part III Elective Officials, Vacancies, Succession, Disciplinary Actions


ELECTIVE OFFICIALS
Sec 39 75, LGC
Qualifications of elective officials (Sec. 39) 1. 2. citizen of the Philippines registered voter in the barangay, municipality, city, or province or district (only in the case of a member of the sangguniang panlalawigan, sangguniang panlungsod, or sangguniang bayan) where he intends to be elected a resident therein for at least one (1) year immediately preceding the day of the election able to read and write Filipino or any other local language or dialect. Age requirements for candidates, as of election day: a. governor, vice-governor, or member of the sangguniang panlalawigan, or mayor, vice-mayor or member of the sangguniang panlungsod of highly urbanized cities - at least 23 years old b. mayor or vice-mayor of independent component cities, component cities, or municipalities - at least 21 years old c. punong barangay , member of the sangguniang panlungsod, sangguniang bayan, or sangguniang barangay - at least 18 years old d. sangguniang kabataan - at least 15 years old but not more than 21 years old The residence requirement is rooted in the desire that officials of districts or localities be acquainted with the needs, difficulties, and other matters vital to the common welfare of the constituents. The actual, physical and personal presence is substantial enough to show his intention to fulfill the duties of mayor and for the voters to evaluate his qualifications for the mayorship. A very legalistic, academic and technical approach to the residence requirement does not satisfy the rationale for the said requirement (Torayno v. COMELEC [2000]). Disqualifications of elective officials (Sec. 40) 1. Those sentenced by final judgment for an offense involving moral turpitude or for an offense punishable by one (1) year or more of imprisonment, within two (2) years after serving sentence Those removed from office as a result of an administrative case Those convicted by final judgment for violating the oath of allegiance to the Republic Those with dual citizenship Fugitives from justice in criminal or non-political cases here or abroad Permanent residents in a foreign country or those who have acquired the right to reside abroad and continue to avail of the same right after the effectivity of this Code; and The insane or feeble-minded. Other grounds for disqualification: a. Vote-buying, upon determination in a summary administrative proceeding. (Nolasco v. COMELEC [1997]). b. Previously-elected official. Any elective official who has resigned from his office, which he previously occupied but has caused to become vacant due to his resignation (by accepting an appointive office or for whatever reason), is disqualified from running in a special election (from RA 8295, An act providing for the proclamation of a

3. 4. 5.

2. 3. 4. 5. 6.

7. 8.

Certificates of candidacy to be taken at face value COMELEC may not deny due course or cancel a certificate without proper proceedings. To receive and acknowledge receipt of the certificates of candidacy is a ministerial duty of COMELEC. The COMELEC does not have discretion to give or not to give due course to the certificate. It may not look into matters not appearing on their face (Cipriano v. COMELEC [2004]). Rationale of residency requirement

D2010 82

UP College of Law
lone candidate for any elective office in a special election, and for other purposes). Politically-motivated acts. Any person who, directly or indirectly, coerces, bribes, threatens, harasses, intimidates or actually causes, inflicts or produces any violence, injury, punishment, torture, damage, loss or disadvantage to any person or persons aspiring to become a candidate or that of the immediate member of his family, his honor or property that is meant to eliminate all other potential candidate, where evidence of guilt is strong (also from RA 8295). Official Governor, vice-governor, mayor, vice-mayor, punong barangay Sangguniang kabataan chairman Members of the sangguniang panlalawigan, panlungsod, bayan Members of the sangguniang barangay How elected At large, by all the qualified voters in their respective units By the registered voters of the katipunan ng 1 kabataan By the qualified voters in their district At large

c.

Other members of the various sanggunian 1. The presidents of the leagues of sanggunian members of component cities and municipalities shall serve as ex officio members of the sangguniang panlalawigan concerned. The presidents of the "liga ng mga barangay and the pederasyon ng mga sangguniang kabataan" elected by their respective chapters shall serve as ex officio members of the sangguniang panlalawigan, sangguniang panlungsod, and sangguniang bayan There shall be one (1) sectoral representative from the women, one (1) from the workers, and one (1) from any of the following sectors: the urban poor, indigenous cultural communities, disabled persons, or any other sector as may be determined by the sanggunian concerned within ninety (90) days prior to the holding of the next local elections as may be provided for by law.

Term of Office (Sec. 43) 1. For all elective officials except barangay officials: 3 years, starting from noon of June 30, 1992 or such date as may be provided for by law. a. all local officials first elected during the local elections immediately following the ratification of the 1987 Constitution shall serve until noon of June 30, 1992 For barangay officials and members of the sangguniang kabataan: 3 years, which shall begin after the regular election of barangay officials on the second Monday of May 1994. No local elective official shall serve for more than 3 consecutive terms in the same position. Voluntary renunciation of the office for any length of time shall not be considered as an interruption in the continuity of service

2.

3.

2.

3.

4.

Vacancies and Succession of Elective OFficials Permanent Vacancies (Sec. 44) Arises when an elective local official: 1. 2. 3. 4. 5. 6.
1

See also Sec. 8, Art. X, 1987 Constitution: The term of office of elective local officials, except barangay officials, which shall be determined by law, shall be three years and no such official shall serve for more than three consecutive terms. Voluntary renunciation of the office for any length of time shall not be considered as an interruption in the continuity of his service for the full term for which he was elected. Elections (Sec. 41, 42) Unless otherwise provided by law, the elections for local officials shall be held every 3 years on the second Monday of May.

fills a higher vacant office refuses to assume office fails to qualify dies is removed from office voluntarily resigns

Sec. 424, LGC: Katipunan ng Kabataan. The katipunan ng kabataan shall be composed of all citizens of the Philippines actually residing in the barangay for at least 6 months, who are 15 but not more than 21 years of age, and who are duly registered in the list of the sangguniang kabataan or in the official barangay list in the custody of the barangay secretary.

83 D2010

Local Government
7. is otherwise permanently incapacitated to discharge the functions of his office concerned

Filling of vacancy by automatic succession (Sec. 44) Permanent Vacancy 1. Office of the governor, mayor 2. Offices of governor & vice-governor, or mayor & vice-mayor Successor Vice-governor, vice-mayor - Highest-ranking sanggunian member, or in case of his permanent inability, - The second highestranking sanggunian member Highest-ranking sanggunian member, or in case of his permanent inability, - The second highestranking sanggunian member General Rule: the appointee shall come from the same political party as that of the sanggunian member who caused the vacancy and shall serve the unexpired term of the vacant office. To ensure this, a nomination and a certificate of membership of the appointee from the highest official of the political party concerned are conditions sine qua non. Effects if nomination and certificate are absent: a. b. any appointment without such shall be null and void ab initio ground for administrative action against the official responsible therefor

3. Office of the punong barangay

Exceptions: In case the permanent vacancy is caused by a sanggunian member who does not belong to any political party, the local chief executive shall, upon recommendation of the sanggunian concerned, appoint any qualified person to fill the vacancy In case of vacancy in the representation of the youth and the barangay in the sanggunian, said vacancy shall be filled automatically by the official next in rank of the organization concerned. Temporary Vacancy in the Office of the Local Chief Executive (Sec. 46) Arises when the local chief executive (governor, mayor, or punong barangay): 1. 2. 3. 4. is on leave of absence is on travel abroad is suspended from office is otherwise temporarily incapacitated to perform his duties for physical or legal reasons

For purposes of succession, ranking in the sanggunian shall be determined on the basis of the proportion of votes obtained by each winning candidate to the total number of registered voters in each district in the immediately preceding local election. A tie between or among the highest ranking sanggunian members shall be resolved by the drawing of lots The successors as defined herein shall serve only the unexpired terms of their predecessors. Filling of vacancy by appointment (Sec. 45) Resorted to when automatic succession provided in Sec. 44 does not apply Permanent Vacancy 1. Sangguniang panlalawigan; sangguniang panlungsod of highly urbanized cities and independent component cities 2. Sangguniang panlungsod of component cities; sangguniang bayan 3. Sangguniang barangay Appointing Power President, through the Executive Secretary

General rule: the vice-governor, city/municipal vicemayor, or the highest ranking sangguniang barangay member shall automatically exercise the powers, and perform the duties and functions, of the local chief executive. Exception: The power to appoint/suspend/dismiss employees, which can be exercised only if the period of temporary incapacity exceeds 30 working days. General rule: The local chief executive cannot authorize any local official to assume the powers/duties/functions

Governor

City or municipal mayor, upon recommendation of the sangguniang barangay

D2010 84

UP College of Law
of his office, other than the vice-governor, city/municipal vicemayor, or highest ranking sangguniang barangay member. Exception: If the local chief executive is traveling within the country but outside his territorial jurisdiction for a period not exceeding 3 consecutive days, he may designate in writing the officer-in-charge. The authorization shall specify the powers and functions that the officer-in-charge shall exercise, which does not include the power to appoint/suspend/dismiss EEs. Exception to the exception: If the local chief executive fails/refuses to issue the authorization, the vicegovernor, city/municipal vice-mayor, or highest ranking sangguniang barangay member has right to assume the powers/duties/functions of the office on the 4th day of absence. The temporary incapacity shall terminate upon submission to the appropriate sanggunian of a written declaration by the local chief executive concerned that he has reported back to office. In cases where the temporary incapacity is due to legal causes, the local chief executive concerned shall also submit necessary documents showing that said legal causes no longer exist. Acting governor cannot simultaneously act as vicegovernor. A vice-governor who is concurrently an acting governor is actually a quasi-governor. Being the acting governor, the vice cannot continue to simultaneously exercise the duties of the latter office, since the nature of the duties of the governor for a full-time occupant to discharge them. Hence, there is an inability on the part of the regular presiding officer (vice-governor) to preside during the sanggunian sessions, which calls for the election of a temporary presiding officer (Gamboa v. Aguirre [1999]). The governor has the power to fill vacancy in the Sangguniang Bayan caused by a member not belonging to any political party. It is the same manner as where the member belonged to a political party. Where there is no political party to make the nomination, the Sanggunian, where the vacancy occurs, must be considered authority for making the recommendation. The appointing authority is limited to the appointment of those recommended to the governor. The recommendation is a condition sine qua non for the validity of the appointment (Farias v. Barba [1996]). Leaves of Absence (Sec. 47) LOA of 1. Governor; mayor of a highly urbanized city or an independent component city 2. Vice-governor; city / municipal vice-mayor 3. Members of the sanggunian and its employees 4. Mayor or component city / municipality 5. Punong barangay 6. Sangguniang barangay members To be approved by The President or his duly authorized representative

Local chief executive concerned Vice-governor or vicemayor concerned Governor City / municipal mayor Punong barangay

Whenever the application for leave of absence hereinabove specified is not acted upon within five (5) working days after receipt thereof, the application for leave of absence shall be deemed approved. Local Legislation Where local legislative power vests (Sec. 48) 1. 2. 3. 4. sangguniang panlalawigan for the province sangguniang panlungsod for the city sangguniang bayan for the municipality sangguniang barangay for the barangay

Presided over by the vice-governor, vice-mayor, or punong barangay. And because the presider is not a member of the Sanggunian, he can vote only in case of a tie (Perez v. Dela Cruz [1969]). Should the vice-governor, vice-mayor or punong barangay be unable to preside, the members present and constituting a quorum shall elect from among themselves a temporary presiding officer. The temporary presiding officer shall certify, within 10 days from the passage of ordinances enacted and resolutions adopted, such ordinances and resolutions in the session over which he temporarily See also Sec. 20, Art. X, 1987 Constitution. Within its territorial jurisdiction and subject to the provisions of this Constitution and national laws, the organic act of autonomous regions shall provide for legislative powers over: 1. 2. Administrative organization; Creation of sources of revenues;

85 D2010

Local Government
3. 4. 5. 6. 7. 8. 9. Ancestral domain and natural resources; Personal, family, and property relations; Regional urban and rural planning development; Economic, social, and tourism development; Educational policies; Preservation and development of the cultural heritage; and Such other matters as may be authorized by law for the promotion of the general welfare of the people of the region. the roll of the members and thereafter announce the results Where there is no quorum, the presiding officer may declare a recess until such time as a quorum is constituted, or a majority of the members present may adjourn from day to day Majority of the members present may compel the immediate attendance of any member absent without justifiable cause by designating a member of the sanggunian to be assisted by the police force assigned in the territorial jurisdiction of the LGU concerned, to arrest the absent member and present him at the session If there is still no quorum despite this, no business shall be transacted. The presiding officer, upon proper motion duly approved by the members present, shall then declare the session adjourned for lack of quorum Sessions (Sec. 52) Regular Sessions. The sanggunian shall, by resolution, fix the day, time, and place of its regular sessions on the first session day immediately after the election of its members. The minimum numbers of regular sessions shall be: once a week for the sangguniang panlalawigan, sangguniang panlungsod, and sangguniang bayan twice a month for the sangguniang barangay

Internal Rules of Procedure (Sec. 50) Every sanggunian must adopt or update its existing rules of procedure, on the first regular session following the election of its members and within 90 days thereafter The rules of procedure shall provide for the following: 1. 2. The organization of the sanggunian and the election of its officers the creation of standing committees (eg., committees on appropriations, women and family, human rights, youth and sports development, environmental protection, cooperatives, etc.), their general jurisdiction, their chairpersons, and their members The order and calendar of business for each session The legislative process The parliamentary procedures which include the conduct of members during sessions The discipline of members for disorderly behavior and absences without justifiable cause for four (4) consecutive sessions, a. the penalty of suspension or expulsion shall require the concurrence of at least two-thirds (2/3) vote of all the sanggunian members b. a member convicted by final judgment to imprisonment of at least one (1) year for any crime involving moral turpitude shall be automatically expelled from the sanggunian; and Such other rules as the sanggunian may adopt

3. 4. 5. 6.

Special Sessions. Special sessions may be called by the local chief executive or by a majority of the members of the sanggunian whenever public interest so demands. In the case of special sessions, a written notice to the members shall be served personally at the member's usual place of residence at least 24 hours before the special session is held. And unless otherwise concurred in by two-thirds (2/3) vote of the sanggunian members present, there being a quorum, no other matters may be considered at a special session except those stated in the notice. General rule: all sessions shall be open to the public Exception: unless a closed-door session is ordered by an affirmative vote of majority of the members present, there being a quorum Reasons for closed-door session: public interest, reasons of security, decency, or morality.

7.

Quorum (Sec. 53) A majority of all the members of the sanggunian who have been elected and qualified shall constitute a quorum to transact official business Should a question of quorum be raised during a session, the presiding officer shall immediately proceed to call

D2010 86

UP College of Law
No two (2) sessions, regular or special, may be held in a single day. Each sanggunian shall keep a journal and record of its proceedings, which may be published upon resolution of the sanggunian concerned. Full disclosure of conflict of interest of members (Sec. 51) Conflict of interest refers in general to one where it may be reasonably deduced that a member of a sanggunian may not act in the public interest due to some private, pecuniary, or other personal considerations that may tend to affect his judgment to the prejudice of the service or the public. Every sanggunian member shall, upon assumption to office, make a full disclosure of: his business and financial interests professional relationship, or any relation by affinity or consanguinity within the fourth civil degree business interest, financial connection, or professional relationship described herein Approval and Vetoing of Ordinances (Sec. 54, 55) 1. Ordinances enacted by the sangguniang panlalawigan, panlungsod, or bayan: a. Every ordinance enacted by the sangguniang panlalawigan, panlungsod, or bayan shall be presented to the local chief executive (i.e., the governor or mayor). b. If the local chief executive approves the same, he shall affix his signature on each and every page thereof. c. Otherwise, he shall veto it within 15 days (in case of a province) or 10 days (in case of a city or municipality) and return the same with his written objections to the sanggunian, which may proceed to reconsider the same. i. Grounds for vetoing: the ordinance is ultra vires, or prejudicial to the public welfare ii. Item-vetoing: the local chief executive shall have the power to veto any particular item or items of 1. an appropriations ordinance 2. an ordinance or resolution adopting a local development plan and public investment program, or 3. an ordinance directing the payment of money or creating liability

with any person, firm, or entity affected by any ordinance or resolution under consideration by the sanggunian of which he is a member, which relationship may result in conflict of interest. Such relationship shall include: Ownership of stock or capital, or investment, in the entity or firm to which the ordinance or resolution may apply Contracts or agreements with any person or entity which the ordinance or resolution under consideration may affect

The disclosure required under this Act shall be made in writing and submitted to the secretary of the sanggunian or the secretary of the committee of which he is a member. The disclosure shall form part of the record of the proceedings and shall be made: before the member participates in the deliberations on the ordinance or resolution under consideration before voting on the ordinance or resolution on second and third readings, if the member did not participate during the deliberations when a member takes a position or makes a privilege speech on a matter that may affect the

In such a case, the veto shall not affect the item or items which are not objected to. The vetoed item or items shall not take effect unless the sanggunian overrides the veto; otherwise, any item/s in the appropriations ordinance of the previous year corresponding to those vetoed shall be deemed reenacted. iii. The local chief executive may veto an ordinance or resolution only once.

87 D2010

Local Government
d. The sanggunian concerned may override the veto of the local chief executive by two-thirds (2/3) vote of all its members, thereby making the ordinance or resolution effective for all legal intents and purposes, even without the approval of the local chief executive concerned. If the ordinance is not returned by the local chief executive within the time prescribed, the ordinance shall be deemed approved as if he had signed it. Ordinances enacted by the sangguniang barangay a. Must be signed by the punong barangay upon approval by the majority of all the sanggunian members. b. The punong barangay has no veto power.

2.

e.

Review of Ordinances and Resolutions (Sec. 56-58) Ordinances and resolutions by the sangguniang panlungsod / sangguniang bayan Sangguniang panlalawigan 1. Approved ordinances 2. Resolutions approving a. local development plans b. public investment programs formulated by local development councils Within 3 days after approval Ordinances by the sangguniang barangay Sangguniang panlungsod / bayan All barangay ordinances

Reviewed by

What may be reviewed

Period for submitting copies by the secretary of the sanggunian concerned Period for review

Within 10 days after enactment

Manner of review

Ground/s for invalidating, and action to be taken

Within 30 days after receipt of copies of ordinances and resolutions 1. By itself, or 2. With the help of the provincial attorney / provincial prosecutor. The latter must submit his written comments or recommendations to the sangguniang panlalawigan within 10 days from receipt of the documents to be examined 1. If the sangguniang panlalawigan finds that such an ordinance or resolution is ultra vires, it shall declare such ordinance or resolution invalid in whole or in part. 2. The sangguniang panlalawigan shall enter its action in the minutes and shall advise the corresponding city or municipal authorities of the action it has taken. 3. Any attempt to enforce any ordinance or any resolution approving the local development plan and public investment program, after the disapproval thereof, shall be sufficient ground for the suspension or dismissal of the official or employee concerned.

Within 30 days from receipt of copies of the ordinances and resolutions By itself

1.

2.

3.

If the sangguniang panlungsod / bayan finds that such ordinance or resolution is inconsistent with law and city/municipal ordinances, it shall return the same with its comments and recommendations to the sangguniang barangay for adjustment, amendment, or modification In this case, the effectivity of the barangay ordinance is suspended until such time as the revision called for is effected. Any attempt to enforce any ordinance after the disapproval thereof, shall be sufficient ground for the suspension or dismissal of

D2010 88

UP College of Law
the official or employee concerned. If no action is taken within thirty (30) days from receipt of the ordinances, the same shall be deemed approved

Effect of inaction

If no action has been taken within 30 days after submission of such an ordinance or resolution, the same shall be presumed consistent with law and therefore valid

Effectivity of Ordinances or Resolutions (Sec. 59) 1. Ordinance or resolution approving the local development plan and public investment program

published once in a local newspaper of general circulation within the city In the absence of a local newspaper, the ordinance or resolution shall be published in any newspaper of general circulation Disciplinary Actions Grounds (Sec. 60) 1. 2. 3. Disloyalty to the Republic of the Philippines; Culpable violation of the Constitution; Dishonesty, oppression, misconduct in office, gross negligence, or dereliction of duty; Commission of any offense involving moral turpitude or an offense punishable by at least prision mayor; Abuse of authority; Unauthorized absence for fifteen (15) consecutive working days, except in the case of members of the Sangguniang Panlalawigan, Sangguniang Panlungsod, Sangguniang bayan, and Sangguniang Barangay; Application for, or acquisition of, foreign citizenship or residence or the status of an immigrant of another country; and Such other grounds as may be provided in this Code and other laws.

Unless otherwise stated in the ordinance or resolution approving the local development plan and public investment program, the same shall take effect after ten (10) days from Posting, by the secretary, of a copy thereof in a bulletin board at the entrance of the provincial capitol / city, municipal, or barangay hall, which must be done not later than 5 days after its approval; and in at least 2 other conspicuous places in the LGU concerned

4.

The text of the ordinance or resolution shall be disseminated and posted in Filipino or English and in the language understood by the majority of the people in the LGU concerned The secretary to the sanggunian shall record such fact in a book kept for the purpose, stating the dates of approval and posting 2. Ordinances with penal sanctions

5. 6.

7.

The gist of all ordinances with penal sanctions shall be published in a newspaper of general circulation within the province where the local legislative body concerned belongs. In the absence of any newspaper of general circulation within the province, posting of such ordinances shall be made in all municipalities and cities of the province where the sanggunian of origin is situated. 3. Ordinances enacted by the local legislative bodies of highly urbanized and independent component cities

8.

Removal of elective officials based on these grounds may only be done by ORDER of proper court. Form and Filing of administrative complaint (sec. 61) Form of Complaint: It must be VERIFIED Where to File: (1) Against an elective official of a province, highlyurbanized city, independent component city or component city Office of the President (OP); (2) Against an elective municipal official Sangguniang Panlalawigan, appealable to the Office of the President (OP);

The main features of the ordinance or resolution duly enacted or adopted shall, in addition to being posted, be

89 D2010

Local Government
(3) Against an elective barangay official Sangguniang Panlungsod or Bayan as the case may be, non appealable. Notice of hearing (Sec. 62) The notice of hearing must be sent to the elective official complained of within seven (7) days from filing of verified complaint which indicates that: An administrative complaint is lodged against him; Requiring him to submit his VERIFIED ANSWER within fifteen (15) days from receipt of notice Duration of Preventive Suspension: Any single preventive suspension of local elective official SHALL NOT extend beyond sixty (60) days. In the event that several administrative cases are filed against an elective official, he cannot be preventively suspended for more than ninety (90) days within a single year on the same ground or grounds existing and known at the time of the first suspension.

What happens after preventive suspension: The suspended elective official shall be deemed REINSTATED IN OFFICE without prejudice to the continuation of the proceedings against him

The disciplining authority shall commence the investigation of the case within ten (10) days after receipt of the respondents answer. NOTE: (1) No investigation shall be held within ninety (90) days immediately prior to any LOCAL ELECTION, and no preventive suspension shall be imposed within the same period. (2) If a preventive suspension has been imposed prior to 90-day period immediately preceding local election, it shall be deemed AUTOMATICALLY LIFTED upon start of aforesaid period. Preventive suspension of local elective officials (Sec. 63) Who may impose: Against an elective official of a PROVINCE, HIGHLY-URBANIZED CITY, INDEPENDENT COMPONENT CITY The President thru the DILG Secretary; Against an elective municipal official or an elective official of a component city The Provincial Governor; Against an elective barangay official The Mayor

Duration of Administrative Proceedings (Rendition of Judgment): One hundred twenty (120) days from the time the respondent was formally notified of the case against him.

NOTE: If the delay in the proceedings of the case of the case is due to his FAULT, NEGLECT, or REQUEST, other than the appeal duly filed, the duration of such delay shall not be counted in computing the time of termination of the case. Any abuse of the exercise of the power of preventive suspension shall be penalized as ABUSE OF AUTHORITY.

Salary of respondent pending suspension (Sec 64) The suspended official shall not receive any salary or compensation during the pendency of the suspension If found NOT GUILTY after the proceeding The suspended official shall be paid FULL salary or compensation including other emoluments accruing during the suspension

If found GUILTY after the proceeding

When to impose: At any time after the issues have been joined, when the evidence of guilt is strong, and given the gravity of offense, then is great possibility that the continuance in office of the respondent could influence the witnesses or pose a threat to the safety and integrity of the records and other evidence. No compensation will be given

Rights of respondent official (Sec 65)

D2010 90

UP College of Law
1. 2. 3. To appear and defend himself in person or by counsel; To confront and cross-examine the witnesses against him; To require the attendance of witnesses and the production of documentary evidence in his favor through the compulsory process of sub poena or sub poena duces tecum Period to appeal: Within thirty (30) days from receipt of the decision.

Where to appeal: For the decisions of Sangguniang Panlungsod of Component Cities and 2 Sangguniang Bayan

Form and Notice of decision (Sec 66) The investigation of the case SHALL be terminated within 90 days from the start thereof. The disciplining authority shall render its decision within 30 days after the end of the investigation. Form of decision: It shall be in writing; It shall state clearly and distinctly the facts and the reasons for the decision.

Sangguniang Panlalawigan

Office of the President

For the decisions of Sangguniang Panlalawigan and the Sangguniang Panlungsod of Highly Urbanized Cities (HUC) and Independent Component Cities

Notice of decision: The notice together with the copies of the decision shall IMMEDIATELY be furnished the respondent and all interested parties.

Decisions of the Office of the President shall be final and executory. (see Calingin v. CA, July 12, 2004) Execution pending appeal (Sec 68) An appeal shall not prevent a decision from becoming final and executory.
3

Imposable penalty: It shall not exceed: (1) the unexpired term of the respondent; or (2) a period of six (6) months for every administrative offense It shall NOT be a bar to the candidacy of the respondent so suspended as long as he meets the qualifications required for the office Amounts to a bar to the candidacy of the respondent for ANY elective position

SUSPENSION

The provision is not explicit whether the decision of Sangguniang Bayan and Sangguniang Panlungsod of Component Cities after appeal to Sangguniang Panlalawigan can be further appealed to the Office of the President. But the author of this reviewer humbly believes that pursuant to the doctrine of exhaustion of administrative remedies in Administrative Law, it can be further appealed to the Office of the President. This view is also bolstered by the last sentence of the same provision which states that Decisions of the Office of the President shall be final and executory. The said statement begs the question: If the decision of the Office of the President is final and executory, how about that of the Sangguniang Panlalawigan?

REMOVAL

Administrative appeals (Sec 67)

This provision is poorly worded. The drafter might be drunk when he/she wrote this provision. The first and second sentences, as written, do not make any sense. One might ask regarding the first sentence, Why appeal if the decision will become final and executory notwithstanding the filing of an appeal? The author humbly submits that the second and third sentences of this provision should have been written or was intended to be written this way: The respondent shall be considered as having been placed under preventive suspension during the pendency of an appeal[.] in the event he wins such appeal. In the event the appeal results in an exoneration, he shall be paid his salary and such emoluments during the pendency of the appeal

91 D2010

Local Government
The respondent official shall be considered as having been placed under preventive suspension during the pendency of an appeal in the event he wins such appeal. In the event the appeal results in EXONERATION, he shall be paid his salary and such other emoluments during the pendency of the appeal. this point and shall be ruled with finality within 15 days from filing of such protest or challenge. COMELEC then verifies and authenticates the signatures gathered. Representatives of the petitioners as well as the official sought to be recalled shall be duly notified and shall have the right to participate therein as mere observers. COMELEC shall announce the acceptance of candidates to the position and thereafter prepare the list of candidates which shall include the name of the official sought to be recalled.

4.

5.

Recall By whom exercised (Sec 69) It shall be exercised by the registered voters of a local government unit to which the local elective official subject to such recall belongs. Ground: Loss of Confidence Initiation of Recall Process (Sec 70) Contents of the Petition: 1. 2. The names and addresses of the petitioner written in legible form and their signatures The barangay, city or municipality, local legislative district and the province to which the petitioners belong The name of the official sought to be recalled A brief narration of the reasons and justifications therefore.

Election on recall (Sec 71) Prescribed date of Recall Election: Shall not be later than thirty (30) days upon the completion of the procedure for initiation of recall process Shall not be later than forty-five (45) days upon the completion of the procedure for initiation of recall process

For recall of barangay, city, or municipal officials

3. 4. Process: 1.

For recall of provincial officials

2.

3.

Petition of a registered voter in the LGU concerned, supported by percentage of registered voters during the election in which the local official sought to be recalled is elected filed with the COMELEC thru its office in the LGU concerned. (Percentage decreases as population of people in area increases. Also, the supporting voters MUST sign the petition) Within 15 days after filing, COMELEC must certify the sufficiency of the required number of signatures. Failure to obtain required number automatically nullifies the petition. Within 3 days of certification of sufficiency form and substance of the petition, the COMELEC shall provide the official with a copy of the petition and causes its publication for 3 weeks (once a week) in a national newspaper and a local newspaper of general circulation. Petition must also be posted for 10 to 20 days at conspicuous places. Protest should be filed at

The official sought to be recalled shall automatically be considered as duly registered candidate or candidates to the pertinent positions and, like other candidates, shall be entitled to be voted upon. Effectivity of recall (Sec 72) The recall of an elective local official shall be effective only upon the election and proclamation of the candidate receiving the highest number of votes cast during the recall election. Should the official sought to be recalled receive the highest number of votes, confidence in him is thereby affirmed and he shall continue in office. Prohibition from resignation (Sec 73) The elective local official sought to be recalled shall not be allowed to resign while the recall process is in progress. Limitation on recall (sec 74)

D2010 92

UP College of Law
Any elective local official may be the subject of a recall election only once during his term of office No recall shall take place within one (1) year from the date of the officials assumption to office or one (1) year immediately preceding a regular local election. Expenses incident to recall election (Sec 75) All expenses incident to recall elections shall be borne by the COMELEC. For this purpose, there shall be included in the annual General Appropriations Act a contingency fund for the conduct of recall elections. A. Qualifications to prove that the petitioner temporarily left her residence to pursue any calling, profession or business to satisfy the principle of animus revertendi. The fact that she occasionally visits Kananga through the years does not signify an intention to continue her residence therein. Having thus been established as an Ormoc City resident, she is disqualified from running as Leyte governor since Ormoc Citys charter prohibits its voters from voting for provincial elective officials, since it is treated like a highly urbanized city which is outside the supervisory power of the province to which it is geographically attached. The provision in the charter connotes two prohibitions: one from running and the other from voting for any provincial elective official. (Since there was a vacancy in the Office of the Governor, the vice-governor assumed office.)

Sec 39, LGC


See above

Frivaldo v. Comelec (1989)


Juan Frivaldo assumed office as Sorsogon governor in 1988. On 27 Oct 1988 the League of Municipalities Sorsogon Chapter, represented by its president Salvador Estuye (also suing in his personal capacity) filed with the COMELEC a petition for the annulment of Frivaldos election on the ground that he was not a FIipino citizen. Frivaldo admitted he was naturalized in the US, but explained that such was only to protect himself against President Marcos. He said he could not have repatriated himself since the Special Committee on Naturalization (created by LOI 270) had not yet been organized. He argued that his oath in his certificate of candidacy that he was a natural-born citizen was a sufficient act of repatriation, and that his active participation in the 1987 elections had divested him of American citizenship under US laws, thus restoring his Filipino citizenship. WON Frivaldo was a Filipino citizen at the time of his election as Sorsogon governor HELD: No The alleged forfeiture of his American citizenship due to his active participation in Philippine politics does not automatically restore his Filipino citizenship. If the Committee tasked to handle the repatriation had not yet been constituted, it only meant that Frivaldo should have waited until this was done, or else he should have sought reacquisition of his Filipino citizenship through other means (i.e., direct act of Congress or applied for naturalization). Frivaldo was then not a Filipino citizen, and thus disqualified to vote and run for office since Filipino citizenship is one of the qualifications for voting and for being a candidate for local elective office. (There

Abella v. Comelec
Benjamin Abella was a candidate for Leyte governor for the 1988 elections. Emeterio Larrazabal was also a candidate for Leyte governor, but was disqualified for lack of residence. On the day before the elections, his wife Adelina Larrazabal then filed her candidacy as his substitute. The following day Silvestre de la Cruz, a registered voter of Tacloban City, filed a petition to disqualify her, on the ground that she misrepresented her residence as Kananga, Leyte when in fact she was a resident of Ormoc City like her husband. She was however proclaimed as the winning candidate, and thus assumed office while the hearings on her disqualification case continued. She was later on declared by the COMELEC as disqualified from running as governor. The incumbent vice-governor Leopoldo Petilla then took his oath as Leyte Governor and assumed office, which COMELEC resolved by ordering Petilla to maintain status quo ante and refrain from assuming the office of governor. WON Adelina Larrazabal was qualified to run as governor of Leyte HELD: No Adelina Larrazabal lacked the required residence because she has established her residence at Ormoc City since 1975 and not at Kananga. Her attempt to purportedly change her residence one year before the election by registering at Kananga clearly shows that she considers herself as an Ormoc City resident. There is no evidence

93 D2010

Local Government
was then a vacancy in the Office of the Governor, thus the elected vice-governor assumed office.) failed to Answer and present evidence; COMELEC subsequently cancelled Labos certificate of candidacy. Labo then moved to stay the implementation of the COMELEC decision, which was granted with COMELEC issuing an Order that Labo may still continue to be voted upon as mayor in the 1992 elections, but it resolved to suspend Labos proclamation in the event he wins the elections. WON Labo is a Filipino citizen and hence qualified to be Mayor HELD: No Labo failed to submit proof that he has reacquired his Philippine citizenship. He claims that he has reacquired his Filipino citizenship by citing his application for reacquisition of Philippine citizenship filed before the OSG, but despite favorable recommendation by the Solicitor General, the Special Committee on Naturalization had yet to act upon his application for repatriation. In the absence of any official action or approval by the proper authorities, a mere application for repatriation does not, and cannot, amount to an automatic reacquisition of the applicants Philippine citizenship. (Note: Second-placer Ortega is not entitled to be proclaimed as Mayor because he was not the choice of the sovereign will. The ineligibility of a candidate receiving majority votes does not entitle the eligible candidate receiving the next highest number of votes to be declared elected.)

Labo v. Comelec (1989)


Ramon Labo Jr married an Australian citizen in the Philippines. As the spouse of an Australian citizen, he was not required to meet normal requirements for the grant of citizenship, and was granted Australian citizenship on 28 Jul 1976. His marriage was later declared void for being bigamous. He returned to the Philippines using an Australian passport and obtained an Alien Certificate of Registration (ACR). He applied for a change in status from immigrant to returning Filipino citizen, but was denied by the Commission on Immigration and Deportation since he has not applied for reacquisition of his Filipino citizenship. He later on ran for and won as Mayor of Baguio City. The second-placer Luis Lardizabal filed a petition for quo warranto, alleging that Labo is disqualified from holding public office on the ground that Labo was an alien, and asking that Labos proclamation as Mayor be annulled. WON Labo is a Filipino citizen and hence qualified to be Mayor HELD: No Labo is not a Filipino citizen because he has lost his Filipino citizenship by all three modes specified in the Constitution: (1) naturalization in a foreign country, (2) express renunciation of citizenship, and (3) subscribing to an oath of allegiance to support the Constitution or laws of a foreign country. Even if it be assumed that his Australian citizenship was annulled because his marriage was void for being bigamous, that circumstance alone does not automatically restore his Philippine citizenship. It does not appear that Labo sought to reacquire his Filipino citizenship through any of the three modes provided by our laws: direct act of Congress, naturalization or repatriation. (Also, the earlier contrary COMELEC decision can also be reversed since res judicata does not apply to questions of citizenship.) Not being a Filipino citizen, Labo is disqualified to run for mayor. (There was then a vacancy in the Office of Mayor, thus the Vice Mayor assumed office.)

Frivaldo v. Comelec (1996)


On 20 Mar 1995, Juan Frivaldo filed his certificate of candidacy for Sorsogon governor in the 1995 elections. Three days later, another Sorsogon governor candidate, Raul Lee filed for the cancellation of his Frivaldos certificate of candidacy on the ground that Frivaldo was not a Filipino citizen. COMELEC granted the petition. Frivaldo filed a MR, which was not acted upon until after the elections. His candidacy then continued and was thus voted upon (he received the highest number of votes). It was only after the elections that the COMELEC en banc affirmed the resolution of its Second Division. By order of COMELEC, Lee was proclaimed as governor at 830 PM on 30 Jun 1995. On 6 Jul 1995, Frivaldo filed a petition with the COMLEC praying for the annulment of Lees proclamation, claiming that since he took his oath of allegiance to the Philippines at 2PM on 30 Jun 1995, there was no more impediment to his being proclaimed as governor. In the alternative, he averred that it was the Vice-Governor-elect that should assume the office and not Lee.

Labo v. Comelec (1992)


Ramon Labo Jr filed his certificate of candidacy for mayor of Baguio City in the 1992 elections. Roberto Ortega also filed his certificate of candidacy for the same office, and also subsequently filed for Labos disqualification before the COMELEC on the ground that Labo was an alien. Labo

D2010 94

UP College of Law
WON Frivaldo was qualified to be elected as Sorsogon governor HELD: Yes The law does not specify any particular date or time when the candidate must possess citizenship. Sine Frivaldo reassumed his citizenship on 30 June 1995 the very day the term of office of governor began he was therefore already qualified to be proclaimed, to hold such office and to discharge the functions and responsibilities as of said date. Furthermore, Frivaldos repatriation retroacted to the date of the filing of his application on 17 Aug 1994, since laws which create new rights are given retroactive effect. And even if Frivaldo was previously declared as not a Filipino citizen by the Supreme Court, such decisions declaring the acquisition or denial of citizenship cannot govern a persons future status with finality, because a person may subsequently reacquire, or for that matter lose, his citizenship under any of the modes recognized by law for the purpose. SC adds, And let it not be overlooked that Frivaldos demonstrated tenacity and sheer determination to reassume his nationality of birth despite several setbacks speak more loudly, in spirit, in fact and in truth than any legal technicality, of his consuming intention and burning desire to re-embrace his native Philippines even now at the ripe old age of 81 years. Justice Davide DISSENTED, saying that Section 39 of the Local Government Code of 1991 refers to no other than the qualifications of candidates for elective local offices and their election. Hence, in no way may the section be construed to mean that possession of qualifications should be reckoned from the commencement of the term of office of the elected candidate. Also, it is only upon taking the oath of allegiance that the applicant is deemed ipso jure to have reacquired Philippine citizenship. If the decree had intended the oath taking to retroact to the date of the filing of the application, then it should not have explicitly provided otherwise. under the principle of jus soli. He was also a natural-born Filipino citizen by virtue of the 1935 Philippine Constitution, as his father and mother were Filipinos at the time of his birth. At the age of 6, his parents brought him to the Philippines using an American passport. His parents registered his as an alien with the Philippine Bureau of Immigration and was thus issued an Alien Certificate of Registration (ACR). When he attained the age of majority, he registered himself as a voter, and voted in the elections of 1992, 1995 and 1998. On 7 May 1998, COMELEC granted Mamarils petition and cancelled Manzanos certificate of candidacy on the ground that he was a dual citizen and thus disqualified from running for any elective position. On 8 May 1998, Manzano moved for reconsideration of the decision, and the motion remained pending even until after the elections. On 31 Aug 1998 the COMELEC en banc declared that Manzano was qualified to run for vice mayor. (1) WON Manzano possesses dual citizenship, and (2) WON he is disqualified from running for and being elected as vice mayor HELD: (1) Yes and (2) No Dual citizenship is different from dual allegiance. The former arises when, as a result of the concurrent application of the different laws of two or more states, a person is simultaneously considered a national by the said states. Such a person, ipso facto and without any voluntary act on his part, is concurrently considered a citizen of both states. Dual allegiance, on the other hand, refers to the situation in which a person simultaneously owes, by some positive act, loyalty to two or more states. While dual citizenship is involuntary, dual allegiance is the result of an individuals volition. In including Section 5 in Article IV of the Constitution (Dual allegiance of citizens is inimical to the national interest and shall be dealt with by law.), the concern of the Constitutional Commission was not with dual citizens per se but with naturalized citizens who maintain their allegiance to their countries of origin even after their naturalization. Hence, the phrase dual citizenship in RA 7160 (Local Government Code of 1991) Sec 40(d) and RA 7854 (Makati City Charter) Sec 20 must be understood as referring to dual allegiance. Thus, persons with mere dual citizenship do not fall under this disqualification. The oath of allegiance contained in Manzanos certificate of candidacy is sufficient to constitute renunciation of his American citizenship, effectively removing any disqualification he might have as a dual citizen. There is no merit to the contention that to be effective, such

Mercado v. Manzano
In the 1998 elections, 3 candidates for vice-mayor competed for the post: Eduardo Manzano, Ernesto Mercado and Gabriel Daza III. Manzano won the elections but his proclamation was suspended due to a pending petition for disqualification filed by Ernesto Mamaril who alleged that Manzano was not a Filipino citizen but was instead a US citizen. Manzano was born in San Francisco, California, USA and thus acquired US citizenship by operation of the US Constitution and laws

95 D2010

Local Government
renunciation should have been made upon Manzanos reaching the age of majority, since no law requires the election of Philippine citizenship to be made upon attainment of the age of majority. Manzanos oath of allegiance to the Philippines, when considered with the fact that he has spent his youth and adulthood, received his education, practiced his profession as an artist, and taken part in past elections in this country, leaves no doubt of his election of Philippine citizenship. (1) year immediately preceding the day of the election; and able to read and write Filipino or any other local language or dialect. (Sec. 39(a), LGC) The term residence is to be understood not in its common acceptation as referring to dwelling or habitation, but rather to domicile or legal residence, that is, the place where a party actually or constructively has his permanent home, where he, no matter where he may be found at any given time, eventually intends to return and remain (animus manendi). A domicile of origin is acquired by every person at birth. It is usually the place where the childs parents reside and continues until the same is abandoned by acquisition of new domicile (domicile of choice). Petitioner lost his domicile of origin in Oras by becoming a U.S. citizen after enlisting in the U.S. Navy in 1965. From then on and until November 10, 2000, when he reacquired Philippine citizenship, petitioner was an alien without any right to reside in the Philippines save as our immigration laws may have allowed him to stay as a visitor or as a resident alien. It is not true, as petitioner contends, that he reestablished residence in this country in 1998 when he came back to prepare for the mayoralty elections of Oras by securing a Community Tax Certificate in that year and by constantly declaring to his townmates of his intention to seek repatriation and run for mayor in the May 14, 2001 elections. Evidence shows that when petitioner entered the country in 1998, he did so as a visa-free balikbayan visitor whose stay as such was valid for one year only. Hence, petitioner can be held to have waived his status as an alien and as a non-resident only on November 10, 2000 upon taking his oath as a citizen of the Philippines under R.A. No. 8171. He lacked the requisite residency to qualify him for the mayorship of Oras, Eastern, Samar.

Coquilla v. Comelec
Coquilla was born of Filipino parents in Oras, Eastern Samar. He resided there until 1965, when he joined the United States Navy. He was subsequently naturalized as a U.S. citizen. On October 15, 1998, petitioner came to the Philippines and took out a residence certificate but still made several trips to the US. On November 10, 2000, he was repatriated and he took his oath as a citizen of the Philippines. On February 27, 2001, after his application for registration as a voter of Butnga, Oras, Eastern Samar was approved, he filed his certificate of candidacy stating therein that he had been a resident of Oras, Eastern Samar for two (2) years. Respondent Alvarez (incumbent mayor of Oras and who was running for reelection) sought the cancellation of petitioners certificate of candidacy on the ground that the Coquilla had made a material misrepresentation in his certificate of candidacy by stating that he had been a resident of Oras for two years when in truth he had resided therein for only about six months since November 10, 2000, when he took his oath as a citizen of the Philippines. The COMELEC was unable to render judgment on the case before the elections on May 14, 2001. Meanwhile, petitioner was elected by a margin of 379 votes, was proclaimed mayor, and subsequently took his oath of office. HELD: Petitioner had NOT been a resident of Oras, Eastern Samar at least one (1) year before the elections held on May 14, 2001 as he represented in his certificate of candidacy. An elective local official must be a citizen of the Philippines; a registered voter in the barangay, municipality, city, or province or, in the case of a member of the sangguniang panlalawigan, sangguniang panlungsod, or sangguniang bayan, the district where he intends to be elected; a resident therein for at least one

Case of Mayor Jesse Robredo


CA case B. Disqualifications

Sec 40, LGC


See above

Caasi v. CA
Merito Miguel was elected as the municipal mayor of Bolinao, Pangasinan in the local elections of January 18,

D2010 96

UP College of Law
1988. Caasi filed a disqualification case against Miguel on the ground that he is a green card holder - hence, a permanent resident of the United States of America, not of Bolinao in violation of Sec. 68 of the Omnibus Election Code. In his "Application for Immigrant Visa and Alien Registration" in 1984, Miguel's answer to Question No. 21 therein regarding his "Length of intended stay (if permanently, so state)," Miguel's answer was, "Permanently." The green card that was subsequently issued identified him in clear bold letters as a RESIDENT ALIEN. On the back of the card, the upper portion, the following information is printed: Person identified by this card is entitled to reside permanently and work in the United States. HELD: Miguel is disqualified under Sec. 68 of the Omnibus Election Code on the ground that he is a green card holder. An immigrant is a person who moves into a country for the purpose of permanent residence. Miguel's immigration to the United States in 1984 constituted an abandonment of his domicile and residence in the Philippines. He entered the United States with the intention to live there permanently as evidenced by his application for an immigrant's (not a visitor's or tourist's) visa. As a resident alien in the U.S., Miguel owes temporary and local allegiance to the U.S., the country in which he resides. Section 68 of the Omnibus Election Code of the Philippines provides that any person who is a permanent resident of or an immigrant to a foreign country shall not be qualified to run for any elective office unless said person has waived his status as permanent resident or immigrant of a foreign country in accordance with the residence requirement provided for in the election laws. To be "qualified to run for elective office" in the Philippines, the law requires that the candidate who is a green card holder must have "waived his status as a permanent resident or immigrant of a foreign country." Miguels act of filing a certificate of candidacy for elective office in the Philippines did not of itself constitute a waiver of his status as a permanent resident or immigrant of the United States. The waiver of his green card should be manifested by some act or acts independent of and done prior to filing his candidacy for elective office in this country. Without such prior waiver, he was disqualified to run for any elective office. The records of this case are starkly bare of proof that he had waived his status as such before he ran for election as municipal mayor of Bolinao on January 18, 1988. In addition, residence in the municipality where he intends to run for elective office for at least one (1) year at the time of filing his certificate of candidacy, is one of the qualifications that a candidate for elective public office must possess. Miguel did not possess that qualification because he was a permanent resident of the United States and he resided in Bolinao for a period of only three (3) months (not one year) after his return to the Philippines in November 1987 and before he ran for mayor of that municipality on January 18, 1988. The Court therefore holds that he was disqualified to become a candidate for that office.

Marquez v. Comelec
At the time private respondent Eduardo Rodriguez filed his certificate of candidacy for Governor of Quezon, a criminal charge against him for ten (10) counts of insurance fraud or grand theft of personal property was still pending before the Municipal Court of Los Angeles Judicial District, County of Los Angeles, State of California, U.S.A. A warrant issued by said court for his arrest, it is claimed, has yet to be served on private respondent on account of his alleged "flight" from that country. Private respondent was proclaimed Governorelect of Quezon on 29 May 1992. Petitioner Bienvenido Marquez, the defeated candidate for the elective position of Governor in the Province of Quezon in the 11th May 1992 elections, argues that Rodriguez is disqualified from being a candidate by virtue Section 40(e) of the Local Government Code which states that a fugitive from justice in criminal or non-political cases here or abroad is disqualified from running for any elective local position. Rodriguez argues that that term fugitive from justice is limited to one who has been convicted by final judgment. His basis is Article 73(b) of the Rules and Regulations Implementing the Local Government Code of 1991. It states that fugitive from justice refers to a person who has been convicted by final judgment." The issue in this case is the meaning of fugitive from justice. Note: the issue of WON Rodriguez is disqualified for being a fugitive from justice was decided by the Court in the succeeding case of Rodriguez v. COMELEC. HELD: The law needs no further interpretation and construction. Section 40(e) of Republic Act No. 7160 is clear and it disqualifies "fugitives from justice in criminal or non-political cases here or abroad" from seeking any

97 D2010

Local Government
elective local office. The phrase "fugitive from justice" includes not only those who flee after conviction to avoid punishment but likewise those who, after being charged, flee to avoid prosecution. This definition truly finds support from jurisprudence and it may be so conceded as expressing the general and ordinary connotation of the term. Article 73 of the Rules and Regulations Implementing the Local Government Code of 1991, to the extent that it confines the term "fugitive from justice" to refer only to a person (the fugitive) "who has been convicted by final judgment," is an inordinate and undue circumscription of the law. that animates one's flight from a particular jurisdiction. And obviously, there can only be an intent to evade prosecution or punishment when there is knowledge by the fleeing subject of an already instituted indictment, or of a promulgated judgment of conviction. Rodriguez' case just cannot fit in this concept. He arrived in the Philippines from the US on June 25, 1985 while the felony complaint in the Los Angeles Court was filed and the warrant of arrest was issued only on November 12, 1985, almost five (5) months after. It was clearly impossible for Rodriguez to have known about such felony complaint and arrest warrant at the time he left the US, as there was in fact no complaint and arrest warrant much less conviction to speak of yet at such time. What prosecution or punishment then was Rodriguez deliberately running away from with his departure from the US? The very essence of being a "fugitive from justice" under the MARQUEZ Decision definition, is just nowhere to be found in the circumstances of Rodriguez. The evidence of petitioner Rodriguez sufficiently proves that his compulsion to return to the Philippines was due to his desire to join and participate vigorously in the political campaigns against former President Ferdinand E. Marcos and not to evade any charge against him in the US. When, in good faith, a person leaves the territory of a state not his own, homeward bound, and learns subsequently of charges filed against him while in the relative peace and service of his own country, the fact that he does not subject himself to the jurisdiction of the former state does not qualify him outright as a fugitive from justice.

Rodriguez v. Comelec
(See Marquez v. COMELEC for background. The Marquez decision was promulgated for the 1992 elections. This case involves the 1995 elections)

In the May 8, 1995 election, Rodriguez and Marquez renewed their rivalry for the same position of governor. This time, Marquez challenged Rodriguez' candidacy via petition for disqualification before the COMELEC, based principally on the same allegation that Rodriguez is a "fugitive from justice." The COMELEC, allegedly having kept in mind the Marquez Decision definition of "fugitive from justice", found Rodriguez to be one. Such finding was essentially based on Marquez' documentary evidence consisting of an authenticated copy of the November 12, 1995 warrant of arrest issued by the Los Angeles Municipal Court against Rodriguez, and an authenticated copy of the felony complaint. Rodriguezs defense is that long before the felony complaint was allegedly filed, respondent was already in the Philippines and he did not know of the filing of the same nor was he aware that he was being proceeded against criminally. In a sense, thru this defense, respondent implicitly contends that he cannot be deemed a fugitive from justice, because to be so, one must be aware of the filing of the criminal complaint, and his disappearance in the place where the long arm of the law, thru the warrant of arrest, may reach him is predicated on a clear desire to avoid and evade the warrant. HELD: Rodriguez is NOT a fugitive from justice. To reiterate, a "fugitive from justice" includes not only those who flee after conviction to avoid punishment but likewise who, after being charged, flee to avoid prosecution (from Marquez Decision). The definition thus indicates that the intent to evade is the compelling factor

Dela Torre v. Comelec


De La Torre was disqualified by COMELEC from running for the position of Mayor of Cavinti, Laguna in the May 8, 1995 elections citing as the ground therefor, Section 40(a) of the Local Government Code which provides that those sentenced by final judgment for an offense involving moral turpitude or for an offense punishable by one (1) year or more of imprisonment within two (2) years after serving sentence are disqualified from running for any elective local position. De La Torre was found guilty (his conviction became final on January 18, 1991) for violation of P.D. 1612, otherwise known as the Anti-fencing Law. De La Torre, however, argues that the crime of fencing does not involve moral turpitude and that Section 40 (a) of the Local Government Code does not apply to his case inasmuch as the probation granted him by the MTC on December 21, 1994 which suspended

D2010 98

UP College of Law
the execution of the judgment of conviction and all other legal consequences flowing therefrom, rendered inapplicable Section 40 (a) as well. HELD: The crime of fencing involves moral turpitude. Blacks Law Dictionary of moral turpitude as an act of baseness, vileness, or depravity in the private duties which a man owes his fellowmen, or to society in general, contrary to the accepted and customary rule of right and duty between man and woman or conduct contrary to justice, honesty, modesty, or good morals. The elements of the crime of fencing, as defined under P.D. 1612, are: 5. 6. A crime of robbery or theft has been committed; The accused who is not a principal or accomplice in the crime of robbery or theft, buys, receives, possesses, keeps, acquires, conceals, sells or disposes, or buys and sells, or in any manner deals in any article, item, object or anything of value, which have been derived from the proceeds of the said crime; The accused knows or should have known that the said article, item, object or anything of value has been derived from the proceeds of the crime of robbery or theft; and There is, on the part of the accused, intent to gain for himself or for another. the fact in the performance of his duty to another or would govern his conduct upon assumption that such fact exists. As to WON a grant of probation affects Sec. 40(a)s applicability, the legal effect of probation is only to suspend the execution of the sentence. Petitioners conviction of fencing subsists and remains totally unaffected notwithstanding the grant of probation. In fact, a judgment of conviction in a criminal case ipso facto attains finality when the accused applies for probation, although it is not executory pending resolution of the application for probation.

Magno v. Comelec
A petition was filed by Montes for the disqualification of petitioner Nestor Magno as mayoralty candidate (in the 2001 election) on the ground that petitioner was previously convicted of direct bribery of which he was discharged after application for parole on March 5, 1998. COMELEC granted the petition of MONTES and declared MAGNO disqualified citing Section 12 of the Omnibus Election Code. WON the crime of direct bribery did not involve moral turpitude and WON that Local Government Code, (citing Section 40 ) and not the Omnibus Election Code that should apply in this situation. HELD: Direct bribery is a crime involving moral turpitude. The fact that the offender agrees to accept a promise or gift and deliberately commits an unjust act or refrains from performing an official duty in exchange for some favors, denotes a malicious intent on the part of the offender to renege on the duties which he owes his fellowmen and society in general. Also, the fact that the offender takes advantage of his office and position is a betrayal of the trust reposed on him by the public. It is a conduct clearly contrary to the accepted rules of right and duty, justice, honesty and good morals. The Local Government Code and not the Omnibus Election Code that applies in this situation. Article 12 of the Omnibus Election Code must yield to Article 40 of the Local Government Code. The Omnibus Election Code was approved on December 3, 1985 while the Local Government Code took effect on January 1, 1992. In case of irreconcilable conflict between two laws, the later enactment must prevail, being the more recent expression of legislative will. Legis posteriores priores contrarias abrogant. In accordance

7.

8.

Moral turpitude is deducible from the third element. Actual knowledge by the fence of the fact that property received is stolen displays the same degree of malicious deprivation of ones rightful property as that which animated the robbery or theft which, by their very nature, are crimes of moral turpitude. And although the participation of each felon in the unlawful taking differs in point in time and in degree, both the fence and the actual perpetrator/s of the robbery or theft invaded ones peaceful dominion for gain - thus deliberately reneging in the process private duties they owe their fellowmen or society in a manner contrary to accepted and customary rule of right and duty, justice, honesty, or good morals The same underlying reason holds even if the fence did not have actual knowledge, but merely should have known the origin of the property received. The words should know denote the fact that a person of reasonable prudence and intelligence would ascertain

99 D2010

Local Government
therewith, Section 40 of RA 7160 is deemed to have repealed Section 12 of BP 881. Section 40 of RA 7160, insofar as it governs the disqualifications of candidates for local positions, assumes the nature of a special law which ought to prevail. The court Citied David vs. COMELEC: RA 7160 is a codified set of laws that specifically applies to local government units. Section 40 thereof specially and definitively provides for disqualifications of candidates for elective local positions. It is applicable to them only. On the other hand, Section 12 of BP 881 speaks of disqualifications of candidates for any public office. The resolution of the COMELEC declaring Magno disqualified from the 2001 mayoral elections is hereby reversed and set aside. expiration of the term for which he was elected, is disqualified from being a candidate for a local elective position under Sec. 40(b) of the LGC. HELD: Sulong is not disqualified from holding the position of mayor of Lapuyan. Reyes vs. Commission on Elections cannot be applied to this case because the 1992 decision of the Sangguniang Panlalawigan has not until now become final. The filing of his motion for reconsideration prevented the decision of Sangguniang Panlalawigan from becoming final. While R.A. No. 7160 on disciplinary actions is silent on the filing of a motion for reconsideration, the same cannot be interpreted as a prohibition against the filing of a motion for reconsideration. Indeed, considering the failure of the Sangguniang Panlalawigan to resolve respondents motion, it is unfair to the electorate to be told after they have voted for respondent Sulong that after all he is disqualified. There being no prohibition against a motion for reconsideration there can thus also be no decision finding respondent guilty to speak of. Petition for certiorari is DISMISSED; Resolution of the COMELEC en banc is AFFIRMED

Lingating v. Comelec
Lingating filed a petition for disqualification against SULONG (May 2001 mayoral candidate) alleging that Sulong was administratively charged, found guilty (by a Sangguniang Panlalawigan February 4, 1992 decision) and ordered removed from office. Sulong denied that the decision had become final and executory since he filed a motion for reconsideration and/or notice of appeal thereof on February 18, 1992. He also denied having been removed from office. Because COMELEC was unable to render judgment before the May 2001 elections, Sulong was voted for and proclaimed mayor. COMELEC (First Division) then declared Sulong disqualified citing Section 40(b) of the LGC THEN the COMELEC en banc reversed the resolution of its First Division, dismissing the petition for lack of merit. The COMELEC en banc found that after having been found guilty by the Sangguniang Panlalawigan, Sulong was reelected mayor of Lapuyan Zamboanga del Sur in May 1992 & 1995 elections. Applying Aguinaldo vs. Santos ruling that re-election renders an administrative case moot and academic the COMELEC en banc held that the re-election of Sulong in the 1992 and 1995 elections would be tantamount to a condonation of the Sangguniang Panlalawigan decision finding him guilty of dishonesty, malversation of public funds etc. Lingating, in this Petition for Certiorari, contends that the COMELEC en banc erred in applying the ruling in Aguinaldo v. Commission on Elections. He cites Reyes v. Commission on Elections which held that an elective local executive officer, who is removed before the

Flores v. Drilon
The constitutionality of Sec. 13, par. (d) , of R.A. 7227, otherwise known as the "Bases Conversion and Development Act of 1992," under which respondent Mayor Richard J. Gordon of Olongapo City was appointed Chairman and Chief Executive Officer of the Subic Bay Metropolitan Authority (SBMA), is challenged in this petition. Petitioners, maintain that the proviso in par. (d) of Sec. 13 infringes on Sec. 7, first par., Art. IX-B, of the Constitution, which states that "no elective official shall be eligible for appointment or designation in any capacity to any public officer or position during his tenure, because the City Mayor of Olongapo City is an elective official and the subject posts are public offices; HELD: Gordon is ineligible for appointment as SBMA Chairman in light of the fact that he is the elective mayor of Olongapo. The proviso in Sec. 13 (d) of R.A. 7227 violates the constitutional proscription against appointment or designation of elective officials to other government posts.

D2010 100

UP College of Law
Sec. 7 of Art. IX-B of the Constitution expresses the policy against the concentration of several public positions in one person, so that a public officer or employee may serve full-time with dedication and thus be efficient in the delivery of public services. It is an affirmation that a public office is a full-time job. Since this is precisely what the constitutional proscription seeks to prevent, it needs no stretching of the imagination to conclude that the proviso contravenes Sec. 7, first par., Art. IX-B of the Constitution. The fact that the expertise of an elective official may be most beneficial to the higher interest of the body politic is of no moment. It is argued that Sec. 94 of the Local Government Code (LGC) permits the appointment of a local elective official to another post if so allowed by law or by the primary functions of his office. But, the contention is fallacious. Section 94 of the LGC is not determinative of the constitutionality of Sec. 13, par. (d), of R.A. 7227, for no legislative act can prevail over the fundamental law of the land. In any case, the view that an elective official may be appointed to another post if allowed by law or by the primary functions of his office ignores the clear-cut difference in the wording of the two (2) paragraphs of Sec. 7 Art. IX-B of the Constitution. While the second paragraph authorizes holding of multiple offices by an appointive official when allowed by law or by the primary functions of his position, the first paragraph appears to be more stringent by not providing any exception to the rule against appointment or designation of an elective official to the government post, except as are particularly recognized in the Constitution itself, e.g., the President as head of the economic and planning agency; the VicePresident, who may be appointed Member of the Cabinet; and, a member of Congress who may be designated ex officio member of the Judicial and Bar Council. The distinction being clear, the exemption allowed to appointive officials in the second paragraph cannot be extended to elective officials who are governed by the first paragraph. The phrase "shall be appointed" in the proviso unquestionably shows the intent to make the SBMA posts appointive and not merely adjunct to the post of Mayor of Olongapo City. Had it been the legislative intent to make the subject positions ex officio, Congress would have, at least, avoided the word "appointed" and, instead, "ex officio" would have been used. Therefore, as an incumbent elective official, respondent Gordon is ineligible for appointment to the position of Chairman of the Board and Chief Executive of SBMA. C. Election cases involving Local Elective Officials

Galido v. Comelec
Petitioner Galido and private respondent Galeon were candidates during the January 1988 local elections for mayor of Garcia-Hernandez, Bohol. Petitioner was proclaimed duly-elected Mayor. Private respondent filed an election protest before the RTC which upheld the proclamation of petitioner. Private respondent appealed RTC decision to the COMELEC. Its First Division reversed the RTC decision and declared private respondent the duly-elected mayor. After the COMELEC en banc denied the petitioners motion for reconsideration and affirmed the decision of its First Division. The COMELEC held that the 15 ballots in the same precinct containing the initial C after the name Galido were marked ballots and, therefore, invalid. Undaunted, petitioner filed the present petition for certiorari and injunction before SC and succeeded in getting a temporary restraining order. In his comment to the petition, private respondent moved for dismissal, citing Article IX (C), Section 2(2), paragraph 2 of the 1987 Constitution, that Final decisions, orders or rulings of the COMELEC in election contests involving elective municipal offices are final and executory, and not appealable. HELD: That decisions, final orders or rulings of the COMELEC in contests involving elective municipal and barangay offices are final, executory and not appealable, does not preclude a recourse to this Court by way of a special civil action of certiorari under Rule 65. Article IX (A), Section 7, 1987 Constitution: Unless otherwise provided by this Constitution or by law, any decision, order, or ruling of each (Constitutional) Commission may be brought to the Supreme Court on certiorari by the aggrieved party within thirty days from receipt thereof. The function of a writ of certiorari is to keep an inferior court or tribunal (such as the COMELEC in the exercise of its quasi-judicial powers) within the bounds of its jurisdiction or to prevent it from committing a grave abuse of discretion amounting to lack or excess of jurisdiction. Nevertheless, SC found no GAD in COMELECs part.

Rivera v. Comelec 101 D2010

Local Government
Petitioner Juan Garcia Rivera and private respondent Juan Mitre Garcia II were candidates for the position of Mayor of Guinobatan, Albay, during the local elections in January 1988. The Municipal Board of Canvassers proclaimed Rivera as Mayor by a majority of ten votes. Garcia filed an election protest with the RTC, Legazpi City which found Garcia to have obtained 6,376 votes as against Rivera's 6,222. Rivera appealed to the COMELEC which affirmed the RTC decision. Garcia commenced to discharge the duties and functions of Mayor of Guinobatan on 10 October 1990, by virtue of a writ of execution implementing the COMELEC decision of 6 September 1990. Rivera filed this petition for certiorari. HELD: The main thrust of the present petition for certiorari is that the respondent COMELEC en banc committed grave abuse of discretion when it affirmed the decision of its First Division, promulgated on 2 May 1990, annulling the proclamation of the petitioner as the duly elected Mayor of Guinobatan, Albay and when it did not exclude from the total votes of Garcia at least ten (10) votes which were allegedly misappreciated in Garcia's favor. We have closely scrutinized the challenged COMELEC decision and find that the said decision was not arrived at capriciously or whimsically by respondent COMELEC. A painstaking re-evaluation of the questioned 67 ballots was made by the COMELEC en banc. Moreover, the appreciation and re-evaluation of ballots are factual determinations. It is settled that in a petition for certiorari, findings of fact of administrative bodies are final unless grave abuse of discretion has marred such factual determinations. D. Term of Office: 3-Term Limit Rule starting from noon of June 30, 1992 or such date as may be provided for by law, except that of elective barangay officials: Provided, That all local officials first elected during the local elections immediately following the ratification of the 1987 Constitution shall serve until noon of June 30, 1992. (b) No local elective official shall serve for more than three (3) consecutive terms in the same position. Voluntary renunciation of the office for any length of time shall not be considered as an interruption in the continuity of service for the full term for which the elective official concerned was elected. (c) The term of office of barangay officials and members of the sangguniang kabataan shall be for three (3) years, which shall begin after the regular election of barangay officials on the second Monday of May 1994.

Rivera v. Comelec
In the May 2004 Synchronized National and Local Elections, respondent Marino "Boking" Morales ran as candidate for mayor of Mabalacat, Pampanga for the term commencing July 1, 2004 to June 30, 2007. Petitioners,filed with the COMELEC a petition to cancel respondent Morales Certificate of Candidacy on the ground that he was elected and had served three previous consecutive terms as mayor of Mabalacat. They alleged that his candidacy violated Section 8, Article X of the Constitution and Section 43 (b) of RA 7160. respondent Morales admitted that he was elected mayor of Mabalacat for the term commencing July 1, 1995 to June 30, 1998 (first term) and July 1, 2001 to June 30, 2004 (third term), but he served the second term from July 1, 1998 to June 30, 2001 only as a "caretaker of the office" or as a "de facto officer" since his proclamation as mayor was declared void by the Regional Trial Court (RTC) HELD: For the three-term limit for elective local government officials to apply, two conditions or requisites must concur, to wit: (1) that the official concerned has been elected for three (3) consecutive terms in the same local government post, and (2) that he has fully served three (3) consecutive terms. Respondent Morales was elected for the term July 1, 1998 to June 30, 2001. He assumed the position. He served as mayor until June 30, 2001. He was mayor for the entire period notwithstanding the Decision of the RTC in the electoral protest case filed by petitioner Dee ousting him (respondent) as mayor. Respondent Morales is now serving his fourth term. He has been mayor of

Sec 8, Art X, Constitution


Section 8, Art. X, Constitution: The term of office of elective local officials, except barangay officials, which shall be determined by law, shall be three years and no such official shall serve for more than three consecutive terms. Voluntary renunciation of the office for any length of time shall not be considered as an interruption in the continuity of his service for the full term for which he was elected.

Sec 43, LGC


Section 43. Term of Office. (a) The term of office of all local elective officials elected after the effectivity of this Code shall be three (3) years,

D2010 102

UP College of Law
Mabalacat continuously without any break since July 1, 1995. In just over a month, by June 30, 2007, he will have been mayor of Mabalacat for twelve (12) continuous years. The framers of the Constitution, by including this exception, wanted to establish some safeguards against the excessive accumulation of power as a result of consecutive terms. Therefore, having found respondent Morales ineligible, his Certificate of Candidacy dated December 30, 2003 should be cancelled. Not being a candidate, the votes cast for him SHOULD NOT BE COUNTED and must be considered stray votes.

Borja v. Comelec
Private respondent Jose T. Capco, Jr. was elected vicemayor of Pateros on January 18, 1988 for a term ending June 30, 1992. On September 2, 1989, he became mayor, by operation of law, upon the death of the incumbent, Cesar Borja. On May 11, 1992, he ran and was elected mayor for a term of three years which ended on June 30, 1995. On May 8, 1995, he was reelected mayor for another term of three years ending June 30, 1998. Capco filed a certificate of candidacy for mayor of Pateros relative to the May 11, 1998 elections. Petitioner Borja, Jr., also a candidate for mayor, sought Capcos disqualification on the theory that the latter would have already served as mayor for three consecutive terms by June 30, 1998 and would therefore be ineligible to serve for another term after that. The COMELEC en banc declared Capco eligible to run as he was not elected to the position of Mayor in the 1988 election, but succeeded by operation of law. Thus, it is not counted as 1 term for the computation of the 3 term limitation. HELD: A textual analysis supports the ruling of the COMELEC that Art. X, 8 contemplates service by local officials for three consecutive terms as a result of election. The first sentence speaks of the term of office of elective local officials and bars such official*s+ from serving for more than three consecutive terms. The second sentence, in explaining when an elective local official may be deemed to have served his full term of office, states that voluntary renunciation of the office for any length of time shall not be considered as an interruption in the continuity of his service for the full term for which he was elected. The term served must therefore be one for which *the official concerned+ was elected. The purpose of this provision is to prevent a circumvention of the limitation on the number of terms an elective official may serve. Conversely, if he is not serving a term for which he was elected because he is simply continuing the service of the official he succeeds, such official cannot be considered to have fully served the term now withstanding his voluntary renunciation of office prior to its expiration. To recapitulate, the term limit for elective local officials must be taken to refer to the right to be elected as well as the right to serve in the same elective position. Consequently, it is not enough that an individual has served three consecutive terms in an elective local office, he must also have been elected to the same position for

Montebon v. Comelec
Petitioners Montebon and Ondy and respondent Potencioso, Jr. were candidates for municipal councilor of the Municipality of Tuburan, Cebu for the May 14, 2007 Synchronized National and Local Elections. On April 30, 2007, petitioners and other candidates for municipal councilor filed a petition for disqualification against respondent with the COMELEC alleging that respondent had been elected and served three consecutive terms as municipal councilor in 1998-2001, 2001-2004, and 20042007. Thus, he is proscribed from running for the same position in the 2007 elections as it would be his fourth consecutive term. Respondent claimed that the service of his second term in 2001-2004 was interrupted on January 12, 2004 when he succeeded as vice mayor of Tuburan due to the retirement of Vice Mayor Petronilo L. Mendoza. Consequently, he is not disqualified from vying for the position of municipal councilor in the 2007 elections HELD: While it is undisputed that respondent was elected municipal councilor for three consecutive terms, the issue lies on whether he is deemed to have fully served his second term in view of his assumption of office as vice-mayor of Tuburan on January 12, 2004. Succession in local government offices is by operation of law. Section 44 of Republic Act No. 7160, provides that if a permanent vacancy occurs in the office of the vice mayor, the highest ranking sanggunian member shall become vice mayor. In this case, a permanent vacancy occurred in the office of the vice mayor due to the retirement of Vice Mayor Mendoza. Respondent, being the highest ranking municipal councilor, succeeded him in accordance with law. It is clear therefore that his assumption of office as vice-mayor can in no way be considered a voluntary renunciation of his office as municipal councilor. Thus, it is an involuntary severance from his office as municipal councilor, resulting in an interruption in the service of his 2001-2004 term.

103 D2010

Local Government
the same number of times before the disqualification can apply. interruption from being joined together to form a continuous service or consecutive terms After three consecutive terms, an elective official cannot immediate re-election for a fourth term, The prohibited election refers to the next regular election for a fourth term. The prohibited election refers to the next regular election for the same office following the same office following the third consecutive term. Any subsequent election, like a recall election, is no longer covered by the prohibition for two reasons: 1. A subsequent election like a recall election, is no longer an immediate reelection after the three consecutive terms. The intervening period constitutes an involuntary interruption in the continuity of service.

Adormeo v. Comelec
Mayor Adormeo was elected in two consecutive elections and served out the full terms. The third time he ran, he lost. However during the term of the winning candidate, there was a recall election, which Adormeo won. He served the unexpired term. Adormeo wants to run again in the next immediately succeeding elections. HELD: The term limit for elective local officials must be taken to refer to the right to be elected as well as the right to serve in the same elective position. Consequently, it is not enough that an individual has served three consecutive terms in an elective local office, he must also have been elected to the same position for the same number of times before the disqualification can apply. COMELECs ruling that private respondent was not elected for three (3) consecutive terms should be upheld. For nearly two years he was a private citizen. The continuity of his mayorship was disrupted by his defeat in the 1998 elections. The time between his second term and the recall election is sufficient interruption. Thus, there was no three consecutive terms as contemplated in the disqualifications in the LGC.

2.

Mendoza v. Comelec
Petitioners Melanio L. Mendoza and Mario E. Ibarra seek to declare respondent Leonardo B. Romans election as governor of Bataan on May 14, 2001 as null and void for allegedly being contrary to Art. X, 8 of the Constitution. HELD: The Constitution does not prohibit elective local officials from serving for more than three consecutive terms because, in fact, it excludes from the three-term limit interruptions in the continuity of service, so long as such interruptions are not due to the voluntary renunciation of the office by an incumbent. Hence, the period from June 28, 1994 to June 30, 1995, during which respondent Leonardo B. Roman served as governor of Bataan by virtue of a recall election held in 1993, should not be counted. Since on May 14, 2001 respondent had previously served as governor of Bataan for only two consecutive terms (1995-1998 and 19982001), his election on that day was actually only his third term for the same position. A recall term should not be considered as one full term, because a contrary interpretation would in effect cut short the elected officials service to less than nine years and shortchange his constituents. The desire to prevent monopoly of political power should be balanced against the need to uphold the voters obvious preference who, in the present case, is Roman who received 97 percent of the votes cast. E. Tenure of Office

Socrates v. Comelec
A recall was initiated for the position of Victorino Dennis M. Socrates who assumed office as Puerto Princesas mayor on June 30, 2001. Edward M. Hagedorn (Hagedorn for brevity) filed his certificate of candidacy for mayor in the recall election. A petition was filed to disqualify Hagedorn from running in the recall election and to cancel his certificate of candidacy on the ground that he is disqualified from running for a fourth consecutive term, having been elected and having served as mayor of the city for three (3) consecutive full terms immediately prior to the instant recall election for the same post. HELD: These constitutional and statutory provisions have two parts. The first part provides that an elective local official cannot serve for more than three consecutive terms. The clear intent is that only consecutive terms count in determining the three-term limit rule. The second part states that voluntary renunciation of office for any length of time does not interrupt the continuity of service. The clear intent is that involuntary severance from office for any length of time interrupts continuity of service and prevents the service before and after the

Osmena v. Comelec

D2010 104

UP College of Law
Petition assailing the constitutionality of R.A. 7056, which desynchronized the national and local elections. This, in effect, shortened the terms of local officials to be elected on November 1992. WON R.A. 7056 is unconstitutional. Held: Yes. R.A. 7056 contravenes Article XVIII, Sections 2 and 5 of the 1987 Constitution which provides for the synchronization of national and local elections. The said law, on the other hand, provides for the desynchronization of election by mandating that there be two separate elections in 1992. The term synchronization in the mentioned constitutional provision was used synonymously as the phrase holding simultaneously since this is the precise intent in terminating their Office Tenure on the same day or occasion. This common termination date will synchronize future elections to once every three years. R.A. 7056 also violated Sec. 2, Art. XVIII of the 1987 Constitution which provides that the local official first elected under the Constitution shall serve until noon of June 30, 1992. But under Sec. 3 of RA 7056, these incumbent local officials shall hold over beyond June 30, 1992 and shall serve until their successors shall have been duly elected and qualified. The Supreme Court, quoting Corpus Juris Secundum, states that it is not competent for the legislature to extend the term of officers by providing that they shall hold over until their successors are elected and qualified where the constitution has in effect or by clear implication prescribed the term and when the Constitution fixes the day on which the official term shall begin, there is no legislative authority to continue the office beyond that period, even though the successors fail to qualify within the time. R.A. 7056 also violated the clear mandate of Sec. 8, Art. X of 1987 Constitution which fixed the term of office of all elective local officials, except barangay officials, to three (3) years. If the local election will be held on the second Monday of November 1992 under RA 7056, those to be elected will be serving for only two years and seven months, that is, from November 30, 1992 to June 30, 1995, not three years. F. Vacancies and Succession

Jainal v. Comelec
Mayor Salip Aloy Jainal and Julhatab J. Talib were candidates for Mayor of Indanan, Sulu Jainal was proclaimed the winning candidate. Talib filed a preproclamation case with the COMELEC, praying for the annulment of certain election returns. Talib claimed that his official watchers were asked to leave the precincts before the counting and the preparation of the election returns. Furthermore, the election returns for these precincts did not bear the signatures of the members of the Board of Election, and the number of votes cast exceeded the number of voters. The COMELEC then issued a Resolution annulling the proclamation and ordering a recount. The COMELEC issued an Order directing the Vice-Mayor or any councilor to cease and desist from assuming the position of Acting Mayor. Said Order was issued during the pendency of the present petition and after Ahajan had already taken his oath and assumed office as Acting Mayor pursuant to resolutions issued by the COMELEC. WON the COMELEC Order is valid. Held: No. With the nullification of Jainals proclamation, the position of Municipal Mayor of Indanan, Sulu became vacant. Sec. 44 of R.A. No. 7160 and Art. 83, Rule XIV of the Implementing Rules of the LGC then apply the highest ranking sanggunian member shall become mayor, a tie between or among the highest ranking sanggunian members shall be resolved by the drawing of lots, and the successor shall serve only the unexpired terms of their predecessors. A permanent vacancy arises when an elective local official fills a higher vacant office, refuses to assume office, fails to qualify, dies, is removed from office, voluntarily resigns, or is otherwise permanently incapacitated to discharge the functions of his office. Ranking in the sanggunian shall be determined on the basis of the proportion of votes obtained by each winning candidate to the total number of registered voters in each district in the immediately preceding local election. The vacancy created by the nullification of Jainals proclamation is in the nature of a permanent vacancy and may be qualified as a "permanent incapacity to discharge the functions of his office. However, Ahajans assumption of the office of Mayor should be understood as subject to the result of the recount to be conducted in accordance with the issuances of the COMELEC.

Sec 44 47, LGC


See above

105 D2010

Local Government Labo v. Comelec (supra)


Ramon Labo, Jr. was disqualified as Mayor of Baguio City for non-compliance with the citizenship requirement of the Constitution. WON the person with the second highest number of votes cast may assume the position of Mayor. Held: No. The candidate who obtained the second highest number of votes cannot occupy the office that was vacant as a result of the disqualification of the candidate who obtained the highest number of votes. The simple reason is that, obtaining the second highest number of votes, he was obviously not the choice of the people. It would be extremely repugnant to the basic concept of the constitutionally guaranteed right to suffrage if a candidate who has not acquired the majority or plurality of votes is proclaimed a winner and imposed as a representative of the constituency, the majority of which have positively declared through their ballots that they do not choose him. position of Governor, the Court rules that, in order to obviate the dilemma resulting from an interregnum, the President, acting through her alter ego, the Secretary of Local Government, may remedy the situation. The Secretary of Government therefore had the authority to designate Menzon as Acting Vice-Governor. (3) The mode of succession provided for in permanent vacancies may likewise be observed in case of a temporary vacancy. Menzons appointment is in full accord with the intent of the LGC the contingency of having vacancies in office cannot be set aside. In a republican form of government, the majority rules through a chosen few, and if one of them is incapacitated or absent, the management of governmental affairs will be hampered. The right to be paid the salary attached to the Office of the Vice-Governor is indubitable Menzon is a de facto officer entitled to compensation.

Docena v. Sangguniang Panlalawigan of Eastern Samar


Luis Capito, elected as a member of the SPES, died in office. Agustin Docena was appointed by the Secretary of Local Government to replace him. Docena took his oath of office before Speaker Ramon Mitra of the House of Representatives and thereafter assumed office. Socrates Alar was subsequently appointed by the Secretary of Local Government to the position already occupied by Docena. In a First Indorsement signed by the Department of Local Government, Alars appointment was recalled on the basis of the earlier appointment of Docena. The SPES then passed Resolution No. 1, reiterating the recognition of Alar. WON Docenas appointment was permanent. Held: Yes. The appointment, having been issued by the Secretary of Local Government and accepted by Docena, had already become complete and enforceable for all legal intents and purposes at the time it was supposed to have been superseded by the appointment in favor of Alar. Docenas appointment was intended to be permanent and as such was valid for the unexpired portion of the term of the deceased SPES member. Docena had already acquired security of tenure in the position and could only be removed therefrom for the causes and conformably to the procedure prescribed by the LGC.

Menzon v. Petilla
As no Governor had been proclaimed in the province of Leyte, the Secretary of Local Government designated Vice-Governor Leopoldo Petilla as Acting Governor and Aurelio Menzon, a senior member of the Sangguniang Panlalawigan as Vice-Governor. Menzon took his oath of office before Sen. Alberto Romulo. However, in Resolution No. 505 of the Sangguniang Panlalawigan, Menzons appointment was held to be invalid. In the meantime, Adelina Larrazabal was proclaimed Governor. WON (1) there is a vacancy, (2) the Secretary of Local Government had the authority to designate, and (3) the designation of Menzon is valid. Held: Yes. (1) There is a vacancy when there is no person lawfully authorized to assume and exercise at present the duties of the office. The office of Vice-Governor was left vacant when Petilla was appointed as Governor. (2) Under CA No. 558 and the Revised Administrative Code of 1987, the President is empowered to make temporary appointments in certain public offices in case of any vacancies that may occur. Considering the silence of the LGC as regards the filling up of vacancies in the

D2010 106

UP College of Law

APPOINTIVE LOCAL OFFICIALS COMMON TO ALL MUNICIPALITIES, CITIES AND PROVINCES


Sec 469 490, LGC
Position 1. Secretary to the Sanggunian Necessity mandatory position Qualifications Qualifications a. Citizen of the Philippines b. Resident of the LGU concerned c. Of good moral character d. A holder of a college degree preferably in law, commerce or public administration from a recognized college or university, and e. A first grade civil service eligible or its equivalent. (Sec 469[b]) Duties a. Attend meetings of the sanggunian and keep a journal of its proceedings; b. Keep the seal of the LGU and affix the same with his signature to all ordinances, resolutions, and other official acts of the sanggunian and present the same to the presiding officer for his signature; c. Forward to the governor or mayor for approval, copies of ordinances enacted by the sanggunian and duly certified by the presiding officer; d. Forward to the sanggunian panlungsod or bayan or the sangguniang panlungsod of component cities or sangguniang bayan, copies of duly approved ordinances; e. Furnish certified copies of records of public character in his custody; f. Record in a book kept for the purpose, all ordinances and resolutions enacted or adopted by the sanggunian, with the dates of passage and publication thereof; g. Keep his office and all non-confidential records therein open to the public during the usual business hours; h. Translate into the dialect used by the majority of the inhabitants all ordinances and resolutions immediately after their approval, and cause the publication of the same; i. Take custody of the local archives and, where applicable, the local library and annually account for the same; and j. Exercise such other powers and perform such other duties and functions as may be prescribed by law or ordinance relative to his position. (Sec. 469[c]) a. Perform the duties provided for under Book II of the Code b. Advise the governor or mayor, sanggunian, and other local government and national officials regarding disposition of local government funds, and other matters relative to public finance; c. Take custody of and exercise proper management of the funds of LGU; d. Take charge of the disbursement of all local government funds and such other funds the custody of which may be entrusted to him; e. Inspect private commercial and industrial establishments in relation to the implementation of tax ordinances; f. Maintain and update the tax information system of the LGU; g. In the case of the provincial treasurer, exercise technical supervision over all treasury offices of component cities and municipalities; and h. Exercise such other powers and perform such other duties and functions as may be prescribed by law or ordinance. (Sec 470[d]) a. Assist the treasurer and perform such duties as the latter may assign to him. b. Administer oaths concerning notices and notifications to those delinquent in the payment of the real property tax and concerning official matters relating to the accounts of the treasurer or arising in the offices of the treasurer and the assessor. (Sec 471[c])

2. Treasurer

mandatory position Appointed by the Secretary of Finance from a list of at least 3 ranking, eligible recommendees of the governor or mayor, as the case may be (Sec 470 [a])

a. Citizen of the Philippines b. A resident of the LGU concerned c. Of good moral character d. A holder of a college degree preferably in commerce, public administration or law from a recognized college or university, and e. A first grade civil service eligible or its equivalent. f. Acquired experience in treasury or accounting service for at least 5 years in the case of the city or provincial treasurer, and 3 years in the case of the municipal treasurer. (Sec 470[c])

3. Treasurer

Assistant

optional position Appointed by the Secretary of Finance from a list of at least 3 ranking, eligible recommendees of the governor or mayor (Sec 471[a])

a. A citizen of the Philippines, b. Resident of the LGU concerned, c. Of good moral character, d. A holder of a college degree preferably in commerce, public administration, or law from a recognized college or university, e. A first grade civil service eligible or its equivalent, f. Acquired at least 5 years experience in the treasury or accounting service in the case of the city or provincial

107 D2010

Local Government
assistant treasurer, and 3 years in the case of the municipal assistant treasurer. (Sec 471[b]) a. Citizen of the Philippines, b. A resident of the LGU concerned, c. Of good moral character, d. A holder of a college degree preferably in civil or mechanical engineering, commerce, or any other related course from a recognized college or university, e. A first grade civil service eligible or its equivalent. f. Acquired experience in real property assessment work or in any related field for at least 5years in the case of the city or provincial assessor, and 3 years in the case of the municipal assessor. (Sec 472[a])

4. Assessor

mandatory position

5. Assistant Assessor

optional position

6. Accountant

mandatory position

a. Citizen of the Philippines, b. Resident of the LGU concerned, c. Of good moral character, d. A holder of a college degree preferably in civil or mechanical engineering, commerce, or any related course from a recognized college or university e. A first grade civil service eligible or its equivalent. f. Acquired experience in assessment or in any related field for at least 3years in the case of the city or provincial assistant assessor, and 1 year in the case of the city or provincial assistant assessor. (Sec 473[a]) a. Citizen of the Philippines, b. A resident of the LGU concerned, c. Of good moral character, d. A certified public accountant. e. Acquired experience in the treasury or accounting service for at least 5 years in the case of the provincial or city accountant, and 3 years in the case of the municipal accountant. (Sec

a. Take charge of the assessor's office, b. Perform the duties provided for under Book II of the Code, c. Ensure that all laws and policies governing the appraisal and assessment of real properties for taxation purposes are properly executed; d. Initiate, review, and recommend changes in policies and objectives, plans and programs, techniques, procedures and practices in the valuation and assessment of real properties for taxation purposes; e. Establish a systematic method of real property assessment; f. Install and maintain a real property identification and accounting system, g. Prepare, install and maintain a system of tax mapping; h. Conduct frequent physical surveys to verify and determine whether all real properties within the province are properly listed in the assessment rolls; i. Exercise the functions of appraisal and assessment primarily for taxation purposes of all real properties in the LGU; j. Prepare a schedule of the fair market value for the different classes of real properties; k. Issue certified copies of assessment records of real property and all other records relative to its assessment; l. Submit every semester a report of all assessments, cancellations and modifications of assessments to the local chief executive and the sanggunian; m. In the case of the assessor of a component city or municipality attend sessions of the local board of assessment appeals whenever his assessment is the subject of the appeal; n. In the case of the provincial assessor, exercise technical supervision and visitorial functions over all component city and municipal assessors, coordinate with component city or municipal assessors in the conduct of tax mapping operations and all other assessment activities, and provide all forms of assistance; (Sec 472[b]) o. Exercise such other powers and perform such other duties and functions as may be prescribed by law or ordinance. (Sec 472[c]) a. Assist the assessor and perform such other duties as the latter may assign to him b. Administer oaths on all declarations of real property for purposes of assessment. (Sec 473[b])

a. Take charge of both the accounting and internal audit services of the LGU; b. Install and maintain an internal audit system in the local LGU; c. Prepare and submit financial statements to the governor or mayor and to the sanggunian; d. Apprise the sanggunian and other local government officials on the financial condition and operations of the LGU; e. Certify to the availability of budgetary allotment to which

D2010 108

UP College of Law
474[a]) f. Incumbent chief accountant in the office of the treasurer shall be given preference in the appointment to the position of accountant.(Sec 474[c]) expenditures and obligations may be properly charged; f. Review supporting documents before preparation of vouchers to determine completeness of requirements; g. Prepare statements of cash advances, liquidation, salaries, allowances, reimbursements and remittances; h. Prepare statements of journal vouchers and liquidation of the same and other adjustments; i. Post individual disbursements to the subsidiary ledger and index cards; Maintain individual ledgers for officials and employees pertaining to payrolls and deductions; j. Record and post in index cards details of purchased furniture, fixtures, and equipment, including disposal thereof, if any; k. Account for all issued requests for obligations and maintain and keep all records and reports related thereto; l. Prepare journals and the analysis of obligations and maintain and keep all records and reports related thereto; and m. Exercise such other powers and perform such other duties and functions as may be provided by law or ordinance. (Sec 474[b]) a. Take charge of the budget office; b. Prepare forms, orders, and circulars embodying instructions on budgetary and appropriation matters for the signature of the governor or mayor, c. Review and consolidate the budget proposals of different departments and offices; d. Assist the governor or mayor in the preparation of the budget and during budget hearings; e. Study and evaluate budgetary implications of proposed legislation and submit comments and recommendations thereon; f. Submit periodic budgetary reports to the DBM; g. Coordinate with the treasurer, accountant, and the planning and development coordinator for the purpose of budgeting; h. Assist the sanggunian concerned in reviewing the approved budgets; i. Coordinate with the planning and development coordinator in the formulation of the local government unit development plan; (Sec 475 [b]) j. Exercise such other powers and perform such other duties and functions as may be prescribed by law or ordinance. (Sec 475[c]) a. Take charge of the planning and development office b. Formulate integrated economic, social, physical, and other development plans and policies for consideration of the local government development council; c. Conduct continuing studies, researches, and training programs necessary to evolve plans and programs for implementation; d. Integrate and coordinate all sectoral plans and studies undertaken by the different functional groups or agencies; e. Monitor and evaluate the implementation of the different development programs, projects, and activities in the local government unit concerned in accordance with the approved development plan; f. Prepare comprehensive plans and other development planning documents for the consideration of the local development council; g. Analyze the income and expenditure patterns, and formulate and recommend fiscal plans and policies for consideration of the finance committee of LGU; h. Promote people participation in development planning within the LGU; i. Exercise supervision and control over the secretariat of the local development council; and (Sec 476[b]) a. Act as the local building official (Sec 477[a])

7. Budget Officer

mandatory position

a. Citizen of the Philippines, b. Resident of the LGU concerned, c. Of good moral character, d. A holder of a college degree preferably in accounting, economics, public administration or any related course from a recognized college or university, e. A first grade civil service eligible or its equivalent. f. Acquired experience in government budgeting or in any related field for at least 5 years in the case of the provincial or city budget officer, and at least 3 years in the case of the municipal budget officer. (Sec 475[a])

8. Planning Development Coordinator

and

mandatory position

a. Citizen of the Philippines b. A resident of the LGU concerned c. Of good moral character d. A holder of a college degree preferably in urban planning, development studies, economics, public administration, or any related course from a recognized college or university e. A first grade civil service eligible or its equivalent f. Acquired experience in development planning or in any related field for at least 5 years in the case of the provincial or city planning and development coordinator, and 3 years in the case of the municipal planning and development coordinator. (Sec 476[a])

9. Engineer

mandatory position

a. Citizen of the Philippines,

109 D2010

Local Government
b. A resident of the LGU concerned, c. Of good moral character, d. A licensed civil engineer e. Acquired experience in the practice of his profession for at least 5 years in the case of the provincial or city engineer, and 3 years in the case of the municipal engineer. (Sec 477[a]) b. Initiate, review and recommend changes in policies and objectives, plans and programs, techniques, procedures and practices in infrastructure development and public works in general; c. Advise the governor or mayor on infrastructure, public works, and other engineering matters; d. Administer, coordinate, supervise, and control the construction, maintenance, improvement, and repair of roads, bridges, and other engineering and public works projects; e. Provide engineering services to the LGU, including investigation and survey, engineering designs, feasibility studies, and project management; f. In the case of the provincial engineer, exercise technical supervision over all engineering offices of component cities and municipalities; and (Sec 477[b]) g. Exercise such other powers and perform such other duties and functions as may be prescribed by law or ordinance. (Sec477[c]) a. Take charge of the office on health services, supervise the personnel and staff of said office, formulate program implementation guidelines and rules and regulations for the operation of the said b. Formulate measures for the consideration of the sanggunian and provide technical assistance and support to the governor or mayor in carrying out activities to ensure the delivery of basic services and provision of adequate health facilities; c. Develop plans and strategies, implement the same, particularly those which have to do with health programs and projects which the governor or mayor, is empowered to implement and which the sanggunian is empowered to provide for; d. Formulate and implement policies, plans, programs and projects to promote the health of the people; e. Advise the governor or mayor and the sanggunian on matters pertaining to health; f. Execute and enforce all laws, ordinances and regulations relating to public health; g. Recommend to the sanggunian, through the local health board, the passage of such ordinances as he may deem necessary for the preservation of public health; h. Recommend the prosecution of any violation of sanitary laws, ordinances or regulations; i. Direct the sanitary inspection of all business establishments selling food items or providing accommodations such as hotels, motels, lodging houses, pension houses, and the like, in accordance with the Sanitation Code; j. Conduct health information campaigns and render health intelligence services; k. Coordinate with other government agencies and nongovernmental organizations involved in the promotion and delivery of health services; l. In the case of the provincial health officer, exercise general supervision over health officers of component cities and municipalities; and m. Be in the frontline of health services delivery, particularly during and in the aftermath of man-made and natural disasters and calamities; and (Sec478[b]) n. Exercise such other powers and perform such other duties and functions as may be prescribed by law or ordinance. (Sec478[c]) a. Responsible for the civil registration program pursuant to the Civil Registry Law, the Civil Code, and other pertinent laws, rules and regulations (Sec479[b]) b. Take charge of the office of the civil registry c. Develop plans and strategies and implement the same,

10. Health Officer

mandatory position

a. Citizen of the Philippines, b. A resident of the LGU concerned, c. Of good moral character, and d. A licensed medical practitioner. e. Acquired experience in the practice of his profession for at least 5 years in the case of the provincial or city health officer, and 3 years in the case of the municipal health officer. (Sec478[a])

11. Civil Registrar

mandatory: city and municipal

a. Citizen of the Philippines, b. A resident of the LGU concerned, c. Of good moral character, d. A holder of a college degree from a recognized college or university,

D2010 110

UP College of Law
e. A first grade civil service eligible or its equivalent. f. Acquired experience in civil registry work for at least 5 years in the case of the city civil registrar and 3 years in the case of the municipal civil registrar. (Sec479[a]) particularly those which have to do with civil registry programs and projects which the mayor is empowered to implement and which the sanggunian is empowered to provide for d. Accept all registrable documents and judicial decrees affecting the civil status of persons; e. File, keep and preserve in a secure place the books required by law; f. Transcribe and enter immediately upon receipt all registrable documents and judicial decrees affecting the civil status of persons in the civil registry books; g. Transmit to the Office of the Civil Registrar- General duplicate copies of registered documents required by law; h. Issue certified transcripts or copies of any certificate or registered documents upon payment of the prescribed fees to the treasurer; i. Receive applications for the issuance of a marriage license and issue the license upon payment of the authorized fee to the treasurer; g. Coordinate with the NSO in conducting educational campaigns for vital registration and assist in the preparation of demographic and other statistics (Sec479[c]) h. Exercise such other powers and perform such other duties and functions as may be prescribed by law or ordinance. (Sec479[d]) a. Take charge of the office of the administrator b. Develop plans and strategies and implement the same particularly those which have to do with the management and administration-related programs and projects which the governor or mayor is empowered to implement and which the sanggunian is empowered to provide for; c. Assist in the coordination of the work of all the officials of the LGU, under the supervision, direction, and control of the governor or mayor, and convene the chiefs of offices and other officials of the local government unit; d. Establish and maintain a sound personnel program for the LGU designed to promote career development and uphold the merit principle in the local government service; e. Conduct a continuing organizational development of the LGU with the end in view of instituting effective administrative reforms; f. Be in the frontline of the delivery of administrative support services, particularly those related to the situations during and in the aftermath of man-made and natural disasters and calamities; g. Recommend to the sanggunian and advise the governor and mayor on all other matters relative to the management and administration of the LGU (Sec480[b]) h. Exercise such other powers and perform such other duties and functions as may be prescribed by law or ordinance. (Sec480[c]) a. Take charge of the office of legal services b. Formulate measures for the consideration of the sanggunian and provide legal assistance and support to the governor or mayor, in carrying out the delivery of basic services and provisions of adequate facilities c. Develop plans and strategies and implement the same, particularly those which have to do with programs and projects related to legal services which the governor or mayor is empowered to implement and which the sanggunian is empowered to provide d. Represent the local government unit in all civil actions and special proceedings wherein the LGU or any official thereof, in his official capacity, is a party; e. Draft ordinances, contracts, bonds, leases and other instruments, involving any interest of the LGU; and provide comments and recommendations on any instruments already

12. Administrator

mandatory: provincial and city, optional: municipal

a. Citizen of the Philippines, b. A resident of the LGU concerned, c. Of good moral character, d. A holder of a college degree preferably in public administration, law, or any other related course from a recognized college or university, e. A first grade civil service eligible or its equivalent f. Acquired experience in management and administration work for at least 5 years in the case of the provincial or city administrator, and 3 years in the case of the municipal administrator. g. Term is coterminous with that of his appointing authority (Sec480[a])

13. Legal Officers

mandatory: provincial and city, optional: municipal

a. Citizen of the Philippines, b. A resident of the LGU, c. Of good moral character, d. A member of the Philippine Bar e. Practiced his profession for at least 5 years in the case of the provincial and city legal officer, and 3years in the case of the municipal legal officer f. Term of the legal officer shall be coterminous with that of his appointing authority.(Sec481[a])

111 D2010

Local Government
drawn; f. Render his opinion in writing on any question of law when requested to do so by the governor, mayor, or sanggunian; g. Investigate or cause to be investigated any local official or employee for administrative neglect or misconduct in office, and recommend appropriate action; h. Investigate or cause to be investigated any person, firm or corporation holding any franchise or exercising any public privilege for failure to comply with any term or condition in the grant of such franchise or privilege, and recommending appropriate action to the governor, mayor or sanggunian, as the case may be; i. Initiate and prosecute in the interest of the LGU any civil action on any bond, lease or other contract upon any breach or violation thereof; j. Review and submit recommendations on ordinances approved and executive orders issued by component units; k. Recommend measures to the sanggunian and advise the governor or mayor as the case may be on all other matters related to upholding the rule of law ; l. Be in the frontline of protecting human rights and prosecuting any violations thereof, particularly those which occur during and in the aftermath of man-made or natural disasters or calamities; and (Sec481[b]) m. Exercise such other powers and perform such other duties and functions as may be prescribed by law or ordinance. (Sec481[c]) a. Take charge of the office for agricultural service b. Formulate measures and provide technical assistance and support in carrying out said measures to ensure the delivery of basic services and provision of adequate facilities relative to agricultural services as provided for under Section 17 c. Develop plans and strategies and implement the same, particularly those which have to do with agricultural programs and projects which the governor or mayor is empowered to implement and which the sanggunian us empowered to provide for d. Ensure that maximum assistance and access to resources in the production, processing and marketing of agricultural and aqua-cultural and marine products are extended to farmers, fishermen and local entrepreneurs; e. Conduct or cause to be conducted location-specific agricultural researches and assist in making available the appropriate technology arising out of and disseminating information on basic research on crops, preventive and control of plant diseases and pests, and other agricultural matters f. Assist in the establishment and extension services of demonstration farms or aqua-culture and marine products; g. Enforce rules and regulations relating to agriculture and aquaculture; h. Coordinate with government agencies and NGOs which promote agricultural productivity through appropriate technology compatible with environmental integrity; i. Be in the frontline of delivery of basic agricultural services, particularly those needed for the survival of the inhabitants during and in the aftermath of man-made and natural disasters; j. Recommend and advise on all matters related to agriculture and aqua-culture which will improve the livelihood and living conditions of the inhabitants; (Sec482[b]) k. Exercise such other powers and perform such other duties and functions as may be prescribed by law or ordinance. (Sec482[c]) a. Take charge of the office on social welfare and development services b. Formulate measures and provide technical assistance and

14. Agriculturist

mandatory: provincial; optional: city and municipal

a. Citizen of the Philippines, b. A resident of the LGU concerned, c. Of good moral character, d. A holder of a college degree in agriculture or any related course from a recognized college or university, e. A first grade civil service eligible or its equivalent. f. Practiced his profession in agriculture or acquired experience in a related field for at least 5 years in the case of the provincial and city agriculturist, and 3years in the case of the municipal agriculturist. (Sec482[a])

15. Social Welfare and Development Officer

mandatory: provincial and city; optional: municipal

a. Citizen of the Philippines, b. A resident of the LGU concerned, c. Of good moral character,

D2010 112

UP College of Law
d. A duly licensed social worker or a holder of a college degree preferably in sociology or any other related course from a recognized college or university, e. A first grade civil service eligible or its equivalent f. Acquired experience in the practice of social work for at least 5 years in the case of the provincial or city social welfare and development officer, and 3 years in the case of the municipal social welfare and development officer. (Sec483[a]) support in carrying out measures to ensure the delivery of basic services and provision of adequate facilities relative to social welfare and development services as provided for under Section 17 c. Develop plans and strategies and mplement the same particularly those which have to do with social welfare programs and projects which the governor or mayor is empowered to implement and which the sanggunian is empowered to provide for d. Identify the basic needs of the needy, the disadvantaged and the impoverished and develop and implement appropriate measures to alleviate their problems and improve their living conditions; e. Provide relief and appropriate crisis intervention for victims of abuse and exploitation and recommend appropriate measures to deter further abuse and exploitation; f. Assist the governor or mayor in implementing the barangay level program for the total development and protection of children up to six (6) years of age; g. Facilitate the implementation of welfare programs for the disabled, elderly, and victims of drug addiction, the rehabilitation of prisoners and parolees, the prevention of juvenile delinquency and such other activities which would eliminate or minimize the ill-effects of poverty; h. Initiate and support youth welfare programs that will enhance the role of the youth in nation-building; i. Coordinate with government agencies and NGOs which have for their purpose the promotion and the protection of all needy, disadvantaged, underprivileged or impoverished groups or individuals, particularly those identified to be vulnerable and high-risk to exploitation, abuse and neglect; j. Be in the frontline of service delivery, particularly those which have to do with immediate relief during and assistance in the aftermath of man-made and natural disaster and natural calamities; k. Recommend to the sanggunian and advise the governor or mayor on all other matters related to social welfare and development services which will improve the livelihood and living conditions of the inhabitants; (Sec483[b]) l. Exercise such other powers and perform such other duties and functions as may be prescribed by law or ordinance. (Sec483[c]) a. Take charge of the office on environment and natural resources b. Formulate measures and provide technical assistance and support in carrying out measures to ensure the delivery of basic services and provision of adequate facilities relative to environment and natural resources services as provided for under Sec 17; c. Develop plans and strategies and implement the same, particularly those which have to do with environment and natural resources programs and projects which the governor or mayor is empowered to implement and which the sanggunian is empowered to provide for; d. Establish, maintain, protect and preserve communal forests, watersheds, tree parks, mangroves, greenbelts and similar forest projects and commercial forest; e. Provide extension services to beneficiaries of forest development projects and technical, financial and infrastructure assistance; f. Manage and maintain seed banks and produce seedlings for forests and tree parks; g. Provide extension services to beneficiaries of forest development projects and render assistance for natural resources-related conservation and utilization activities consistent with ecological balance;

16. Environment and Natural Resources Officer

optional position

a. A citizen of the Philippines, b. Resident of the local government unit concerned, c. Of good moral character, d. Holder of a college degree preferably in environment, forestry, agriculture or any related course from a recognized college or university, e. A first grade civil service eligible or its equivalent f. Acquired experience in environmental and natural resources management, conservation, and utilization, of at least 5 years in the case of the provincial or city environment and natural resources officer, and 3 years in the case of the municipal environment and natural resources officer. (Sec484[a])

113 D2010

Local Government
h. Promote the small-scale mining and utilization of mineral resources, particularly mining of gold; i. Coordinate with government agencies and NGOs in the implementation of measures to prevent and control land, air and water pollution with the assistance of the DENR; j. Be in the frontline of the delivery of services concerning the environment and natural resources, particularly in the renewal and rehabilitation of the environment during and in the aftermath of man-made and natural calamities and disasters; k. Recommend to the sanggunian and advise the governor or mayor on all matters relative to the protection, conservation, maximum utilization, application of appropriate technology and other matters related to the environment and natural resources; and (Sec484[b]) l. Exercise such other powers and perform such other duties and functions as may be prescribed by law or ordinance. (Sec484[c]) a. Take charge of the office on architectural planning and design b. Formulate measures for the consideration of the sanggunian and provide technical assistance and support to the governor or mayor in carrying out measures to ensure the delivery of basic services and provision of adequate facilities relative to architectural planning and design as provided for under Section 17; c. Develop plans and strategies and implement the same, particularly those which have to do with architectural planning and design programs and projects which the governor or mayor is empowered to implement and which the sanggunian is empowered to provide for under this Code; d. Prepare and recommend for consideration of the sanggunian the architectural plan and design for the local government unit or a part thereof, including the renewal of slums and blighted areas, land reclamation activities, the greening of land, and appropriate planning of marine and foreshore areas; e. Review and recommend for appropriate action of the sanggunian, governor or mayor the architectural plans and design submitted by governmental and non-governmental entities or individuals, particularly those for undeveloped, underdeveloped, and poorly-designed areas; f. Coordinate with government and NGOs and individuals involved in the aesthetics and the maximum utilization of the land and water within the jurisdiction of the LGU, compatible with environmental integrity and ecological balance. g. Be in the frontline of the delivery of services involving architectural planning and design, particularly those related to the redesigning of spatial distribution of basic facilities and physical structures during and in the aftermath of man-made and natural calamities and disasters; h. Recommend to the sanggunian and advise the governor or mayor on all other matters relative to the architectural planning and design as it relates to the total socioeconomic development of the local government unit; and (Sec 485[b]) i. Exercise such other powers and perform such other duties and functions as may be prescribed by law or ordinance. (Sec485[c]) a. Take charge of the office on public information b. Formulate measures and provide technical assistance and support in providing the information and research data required for the delivery of basic services and provision of adequate facilities so that the public becomes aware of said services and may fully avail of the same; c. Develop plans and strategies and implement the same, particularly those which have to do with public information and research data to support programs and projects which the

17. Architect

optional position

a. Citizen of the Philippines, b. A resident of the LGU concerned, c. Of good moral character, d. A duly licensed architect. e. Practiced his profession for at least 5 years in the case of the provincial or city architect, and 3 years in the case of the municipal architect. (Sec485[a])

18. Officer

Information

optional position

a. Citizen of the Philippines, b. A resident of the LGU concerned, c. Of good moral character, d. A holder of a college degree preferably in journalism, mass communication or any related course from a recognized college or university, e. A first grade civil service eligible or

D2010 114

UP College of Law
its equivalent. f. Have experience in writing articles and research papers, or in writing for print, television or broadcast media of at least 3 years in the case of the provincial or city information officer, and at least 1 year in the case of municipal information officer. g. Term of the information officer is co-terminous with his appointing authority. (Sec486[a]) governor or mayor is empowered to implement and which the sanggunian is empowered to provide for; d. Provide relevant, adequate, and timely information to the LGU and its residents; e. Furnish information and data on LGUs to government agencies or offices as may be required by law or ordinance; and NGOs to be furnished to said agencies and organizations; f. Maintain effective liaison with the various sectors of the community on matters and issues that affect the livelihood and the quality of life of the inhabitants and encourage support for programs of the local and national government; g. Be in the frontline in providing information during and in the aftermath of manmade and natural calamities and disasters, with special attention to the victims thereof, to help minimize injuries and casualties during and after the emergency, and to accelerate relief and rehabilitation; h. Recommend and advise on all other matters relative to public information and research data as it relates to the total socioeconomic development of the LGU; (Sec486[b]) i. Exercise such other powers and perform such other duties and functions as may be prescribed by law or ordinance. (Sec486[c]) a. Take charge of the office for the development of cooperatives b. Formulate measures and provide technical assistance and support in carrying out measures to ensure the delivery of basic services and provision of facilities through the development of cooperatives, and in providing access to such services and facilities; c. Develop plans and strategies and implement the same, particularly those which have to do with the integration of cooperatives principles and methods in programs and projects which the governor or mayor is empowered to implement and which the sanggunian is empowered to provide for; d. Assist in the organization of cooperatives; e. Provide technical and other forms of assistance to existing cooperatives to enhance their viability as an economic enterprise and social organization; f. Assist cooperatives in establishing linkages with government agencies and NGOs involved in the promotion and integration of the concept of cooperatives in the livelihood of the people and other community activities; g. Be in the frontline of cooperatives organization, rehabilitation or viability-enhancement, particularly during and in the aftermath of man-made and natural calamities and disasters, to aid in their survival and, if necessary subsequent rehabilitation; h. Recommend and advise on all other matters relative to cooperatives development and viability- enhancement which will improve the livelihood and quality of life of the inhabitants; (Sec487[b]) i. Exercise such other powers and perform such other duties and functions as may be prescribed by law or ordinance. (Sec487[c]) a. Take charge of the office on population development b. Formulate measures and provide technical assistance and support in carrying out measures to ensure the delivery of basic services and provision of adequate facilities relative to the integration of the population development principles and in providing access to said services and facilities; c. Develop plans and strategies and implement the same, particularly those which have to do with the integration of population development principles and methods in programs and projects which the governor or mayor is empowered to implement and which the sanggunian is empowered to provide for;

19. Officer

Cooperative

optional: provincial and city

a. Citizen of the Philippines, b. A resident of the LGU concerned, c. Of good moral character, d. Holder of a college degree preferably in business administration with special training in cooperatives or any related course from a recognized college or university, e. First grade civil service eligible or its equivalent f. Have experience in cooperatives organization and management of at least 5 years in the case of the provincial or city cooperatives officer, and 3 years in the case of municipal cooperatives officer. (Sec487[a])

20. Officer

Population

optional position

a. Citizen of the Philippines, b. A resident of the LGU concerned, c. Of good moral character, d. A holder of a college degree with specialized training in population development from a recognized college or university, e. A first grade civil service eligible or its equivalent. f. Have experience in the implementation of programs on population development or

115 D2010

Local Government
responsible parenthood for at least 5 years in the case of the provincial or city population officer and 3 years in the case of the municipal population officer. (Sec488[a]) d. Assist the governor or mayor in the implementation of the Constitutional provisions relative to population development and the promotion of responsible parenthood; e. Establish and maintain an updated data bank for program operations, development planning and an educational program to ensure the people's participation in and understanding of population development; f. Implement appropriate training programs responsive to the cultural heritage of the inhabitants; (Sec488[b]) g. Exercise such other powers and perform such other duties and functions as may be prescribed by law or ordinance. (Sec488[c]) a. Take charge of the office for veterinary services; b. Formulate measures and provide technical assistance and support in carrying out measures to ensure the delivery of basic services and provision of adequate facilities pursuant to Section 17; c. Develop plans and strategies and implement the same particularly those which have to do with the veterinary-related activities which the governor or mayor is empowered to implement and which the sanggunian is empowered to provide for d. Advise the governor or the mayor on all matters pertaining to the slaughter of animals for human consumption and the regulation of slaughterhouses; e. Regulate the keeping of domestic animals; f. Regulate and inspect poultry, milk and dairy products for public consumption; g. Enforce all laws and regulations for the prevention of cruelty to animals; h. Take the necessary measures to eradicate, prevent or cure all forms of animal diseases; i. Be in the frontline of veterinary related activities, such as in the outbreak of highly-contagious and deadly diseases, and in situations resulting in the depletion of animals for work and human consumption, particularly those arising from and in the aftermath of man-made and natural calamities and disasters; j. Recommend and advise on all other matters relative to veterinary services which will increase the number and improve the quality of livestock, poultry and other domestic animals used for work or human consumption; (Sec489[b]) k. Exercise such other powers and perform such other duties and functions as may be prescribed by law or ordinance. (Sec489[c]) a. Take charge of the office on general services; b. Formulate measures and provide technical assistance and support in carrying out measures to ensure the delivery of basic services and provision of adequate facilities pursuant to Section 17 and which require general services expertise and technical support services; c. Develop plans and strategies and implement the same, particularly those which have to do with the general services supportive of the welfare of the inhabitants which the governor or mayor is empowered to implement and which the sanggunian is empowered to provide for; d. Take custody of and be accountable for all properties, real or personal, owned by the LGU and those granted to it in the form of donation, reparation, assistance and counterpart of joint projects; e. Assign building or land space to local officials or other public officials, who by law, are entitled to such space; f. Recommend the reasonable rental rates for local government properties, whether real or personal, which will be leased to public or private entities g. Recommend reasonable rental rates of private properties which may be leased for the official use of the local

21. Veterinarian

mandatory: provincial and city

a. Citizen of the Philippines, b. A resident of the local government concerned, c. Of good moral character, d. A licensed doctor of veterinary medicine, e. Have practiced his profession for at least 3 years in the case of provincial or city veterinarian and at least 1 year in the case of the municipal veterinarian. (Sec489[a])

22. General Services Officer

mandatory: provincial and city

a. Citizen of the Philippines, b. A resident of the local government unit concerned, c. Of good moral character, d. A holder of a college degree on public administration, business administration and management from a recognized college or university, e. A first grade civil service eligible or its equivalent. f. Have acquired experience in general services, including management of supply, property, solid waste disposal, and general sanitation, of at least 5 years in the case of the provincial or city general services officer, and at least 3 years in the case of the municipal general services officer. (Sec490[a])

D2010 116

UP College of Law
government unit; h. Maintain and supervise janitorial, security, landscaping and other related services in all local government public buildings and other real property i. Collate and disseminate information regarding prices, shipping and other costs of supplies and other items commonly used by the LGU; j. Perform archival and record management with respect to records of offices and departments of the LGU k. Perform all other functions pertaining to supply and property management performed by the local government treasurer; and enforce policies on records creation, maintenance, and disposal; l. Be in the frontline of general services related activities, such as the possible or imminent destruction or damage to records, supplies, properties, and structures and the orderly and sanitary clearing up of waste materials or debris, particularly during and in the aftermath of man-made and natural calamities and disasters; m. Recommend and advise on all other matters relative to general services; (Sec490[b]) n. Exercise such other powers and perform such other duties and functions as may be prescribed by law or ordinance. (Sec490[c])

De Rama v. CA
Mayor Conrado de Rama sought for the recall of 14 municipal employees on the ground that they were midnight appointees of the former mayor, in violation of Art. VII, Sec. 15 of the Constitution. The Civil Service Commission declared the appointments in accordance with law and valid (approved by the head of the CSC Field Office of Lucena City). ISSUE: Whether or not the appointments were unconstitutional. Held: No. RULING: Rule VI, Sec. 20 of the Omnibus Implementing Rules of the Revised Administrative Code provides that appointments may be recalled on the following grounds: (1) non-compliance with procedure, (2) failure to pass through Selection Board, (3) violation of agreement relative to promotion, (4 violation of other existing civil service laws. No law prohibits local elective officials from making appointments during the last days of his or her tenure. The midnight appointments prohibited by Art. VII, Sec. 15 of the Constitution applies only to appointments made by the President. There were no allegations of fraud on the part of the outgoing mayor or that the appointments were tainted by irregularities only belatedly, as a supplemental pleading on appeal, was it alleged that CSC procedures were not followed (rules on screening, posting of notices, merit and fitness). Failure

to raise these grounds in the original pleading constitutes a waiver. Upon the issuance of an appointment and the appointees assumption of position, he acquires a legal and not only equitable right to the position, which cannot be taken away either by revocation or by removal without cause and previous notice and hearing. There is no showing that any of the appointees were not qualified. They assumed their appointive positions this cannot be unilaterally revoked. Rule V, Sec. 9 of the Omnibus Implementing Rules of the Revised Administrative Code provides that an appointment accepted cannot be revoked by the appointing authority and shall remain in force and effect until disapproved by the CSC. a. Leagues of Local Barangay Units and Elective Officials

Sec 491 510, LGC


Article One. Liga ng Mga Barangay Sec. 491. Purpose of Organization. There shall be an organization of all barangays, to be known as the Liga ng mga Barangay, for the primary purpose of determining the representation of the liga in the sanggunians and for ventilating, articulating and crystallizing issues affecting barangay government administration and securing, through proper and legal means, solutions thereto.

117 D2010

Local Government
Sec. 492. Representation, Chapters, National Liga. Every barangay shall be represented in said liga by the punong barangay, or in his absence or incapacity, by a sanggunian member duly elected for the purpose among its members, who shall attend all meetings or deliberations called by the different chapters of the liga. The liga shall have chapters at the municipal, city, provincial and metropolitan political subdivision levels. The municipal and city chapters of the liga shall be composed of the barangay representatives of municipal and city barangays, respectively. The duly elected presidents of component municipal and city chapters shall constitute the provincial chapter or the metropolitan political subdivision chapter. The duly elected presidents of highly-urbanized cities, provincial chapters, the Metropolitan Manila chapter and metropolitan political subdivision chapters shall constitute the National Liga ng mga Barangay. Sec. 493. Organization. The liga at the municipal, city, provincial, metropolitan political subdivision, and national levels directly elect a president, a vice-president, and five (5) members of the board of directors. The board shall appoint its secretary and treasurer and create such other positions as it may deem necessary for the management of the chapter. A secretary-general shall be elected from among the members of the national liga and shall be charged with the overall operation of the liga on the national level. The board shall coordinate the activities of the chapters of the liga. Sec. 494. Ex Officio Membership in Sanggunians. The duly elected presidents of the liga at the municipal, city and provincial levels, including the component cities and municipalities of Metropolitan Manila, shall serve as exofficio members of the sangguniang bayan, sangguniang panlungsod, and sangguniang panlalawigan, respectively. They shall serve as such only during their term of office as presidents of the liga chapters, which in no case shall be beyond the term of office of the sanggunian concerned. Sec. 495. Powers, Functions and Duties of the Liga ng mga Barangay. The Liga ng mga Barangay shall: (a) Give priority to programs designed for the total development of the barangays and in consonance with the policies, programs and projects of the National Government; (b) Assist in the education of barangay residents for people's participation in local government administration in order to promote united and concerted action to achieve country wide development goals; (c) Supplement the efforts of government in creating gainful employment within the barangay; (d) Adopt measures to promote the welfare of barangay officials; (e) Serve as a forum of the barangays in order to forge linkages with government and non-governmental organizations and thereby promote the social, economic and political well-being of the barangays; and (f) Exercise such other powers and perform such other duties and functions which will bring about stronger ties between barangays and promote the welfare of the barangay inhabitants. Article Two. League of Municipalities Sec. 496. Purpose of Organization. There shall be an organization of all municipalities, to be known as the League of Municipalities, for the primary purpose of ventilating, articulating and crystallizing issues affecting municipal government administration and securing, through proper and legal means, solutions thereto. The league shall form provincial chapters composed of the league presidents for all component municipalities of the province. Sec. 497. Representation. Every municipality shall be represented in the league by the municipal mayor or, in his absence, by the vice mayor or a sanggunian member duly elected for the purpose by the members, who shall attend all meetings and participate in the deliberations of the league. Sec. 498. Powers, Functions and Duties of the League of Municipalities. The League of Municipalities shall: (a) Assist the National Government in the formulation and implementation of policies, programs and projects affecting municipalities as a whole; (b) Promote local autonomy at the municipal level; (c) Adopt measures for the promotion of the welfare of all municipalities and its officials and employees; (d) Encourage people's participation in local government administration in order to promote united and concerted action for the attainment of country wide development goals;

D2010 118

UP College of Law
(e) Supplement the efforts of the National Government in creating opportunities for gainful employment within the municipalities; (f) Give priority to programs designed for the total development of the municipalities in consonance with the policies, programs and projects of the National Government; (g) Serve as a forum for crystallizing and expressing ideas, seeking the necessary assistance of the National Government, and providing the private sector avenues for cooperation in the promotion of the welfare of the municipalities; and (h) Exercise such other powers and perform such other duties and functions as the league may prescribe for the welfare of the municipalities. Article Three. League of Cities Sec. 499. Purpose of Organization. There shall be an organization of all cities, to be known as the League of Cities, for the primary purpose of ventilating, articulating and crystallizing issues affecting city government administration and securing, through proper and legal means, solutions thereto. The league may form chapters at the provincial level for the component cities of a province. Highly-urbanized cities may also form a chapter of the league. The National League shall be composed of the presidents of the league of highly-urbanized cities and the presidents of the provincial chapters of the league of component cities. Sec. 500. Representation. Every city shall be represented in the league by the city mayor or, in his absence, by the city vice mayor or a sanggunian member duly elected for the purpose by the members, who shall attend all meetings and participate in the deliberations of the league. Sec. 501. Powers, Functions and Duties of the League of Cities. The League of Cities shall: (a) Assist the National Government in the formulation and implementation of the policies, programs and projects affecting cities as a whole; (b) Promote local autonomy at the city level; (c) Adopt measures for the promotion of the welfare of all cities and its officials and employees; (d) Encourage people's participation in local government administration in order to promote united and concerted action for the attainment of country wide development goals; (e) Supplement the efforts of the National Government in creating opportunities for gainful employment within the cities; (f) Give priority to programs designed for the total development of cities in consonance with the policies, programs and projects of the National Government; (g) Serve as a forum for crystallizing and expressing ideas, seeking the necessary assistance of the National Government and providing the private sector avenues for cooperation in the promotion of the welfare of the cities; and (h) Exercise such other powers and perform such other duties and functions as the league may prescribe for the welfare of the cities. Article Four. League of Provinces Sec. 502. Purpose of Organization. There shall be an organization of all provinces, to be known as the League of Provinces, for the primary purpose of ventilating, articulating and crystallizing issues affecting provincial and metropolitan political subdivision government administration and securing, through proper and legal means, solutions thereto. For this purpose, the Metropolitan Manila Area and any metropolitan political subdivision shall be considered as separate provincial units of the league. Sec. 503. Representation. Every province shall be represented in the league by the provincial governor, or in his absence, by the provincial vice mayor or a sanggunian member duly elected for the purpose by the members, who shall attend all meetings and participate in the deliberations of the league. Sec. 504. Powers, Functions and Duties of the League of Provinces. The league of Provinces shall: (a) Assist the National Government in the formulation and implementation of the policies, programs and projects affecting provinces as a whole; (b) Promote local autonomy at the provincial level; (c) Adopt measures for the promotion of the welfare of all provinces and its officials and employees;

119 D2010

Local Government
(d) Encourage people's participation in local government administration in order to promote united and concerted action for the attainment of countrywide development goals; (e) Supplement the efforts of the National Government in creating opportunities for gainful employment within the province; (f) Give priority to programs designed for the total development of the provinces in consonance with the policies, programs and projects of the National Government; (g) Serve as a forum for crystallizing and expressing ideas, seeking the necessary assistance of the national government and providing the private sector avenues for cooperation in the promotion of the welfare of the provinces; and (h) Exercise such other powers and perform such other duties and functions as the league may prescribe for the welfare of the provinces and metropolitan political subdivisions. Article Five. Provisions Common to all Leagues activities of the national league. The board of directors on the chapter or national level may create such other positions as may be deemed necessary for the management of the chapters and of the national league. The national board directors of the leagues for municipalities, cities or provinces shall coordinate programs, projects and activities of the chapter and the national-level league. Sec. 507. Constitution and By-laws of the Liga and the Leagues. All other matters not herein otherwise provided for affecting the internal organization of the leagues of local government units shall be governed by their respective constitution and by-laws which are hereby made suppletory to the provision of this Chapter: Provided, That said constitution and by-laws shall always conform to the provisions of the Constitution and existing laws. CHAPTER 2. Leagues and Federations of Local Elective Officials Sec. 508. Organization. (a) Vice governors, vice mayors, sanggunian members of barangays, municipalities, component cities, highly-urbanized cities and provinces, and other elective local officials of local government units, including those of the Metropolitan Manila area and any metropolitan political subdivisions, may form their respective leagues or federations, subject to applicable provisions of this Title and pertinent provisions of this Code; (b) Sanggunian members of component cities and municipalities shall form a provincial federation and elect a board of directors and a set of officers headed by the president. The duly elected president of the provincial federation of sanggunian members of component cities and municipalities shall be an ex officio member of the sangguniang panlalawigan concerned and shall serve as such only during his term of office as president of the provincial federation of sanggunian members of component cities and municipalities, which in no case shall be beyond the term of office of the sanggunian panlalawigan concerned. Sec. 509. Constitution and By-laws. The leagues or federations shall adopt a constitution and by-laws which shall govern their internal organization and operation: Provided, That said constitution and by-laws shall always conform to the provision of the Constitution and existing laws. Sec. 510. Funding. The leagues and federations may derive funds from contributions of individual league or

Sec. 505. Funding. (a) All leagues shall derive its funds from contributions of member local government units and from fund-raising projects and activities without the necessity of securing permits therefor: Provided, That the proceeds from said fund-raising projects and activities shall be used primarily to fund the projects for which the said proceeds have been raised, subject to the pertinent provisions of this Code and the Omnibus Election Code. (b) All funds of leagues shall be deposited as trust funds with its treasurer and shall be disbursed in accordance with the board of director's resolutions, subject to pertinent accounting and auditing rules and regulations: Provided, That the treasurer shall be bonded in an amount to be determined by the board of directors. The funds of a chapter shall be deposited as chapter funds and funds of the national league shall be deposited as national funds. Sec. 506. Organizational Structure. To ensure the effective and efficient administration, the leagues for municipalities, cities and provinces shall elect chapterlevel and national-level boards of directors and a set of officers headed by the president. A secretary-general shall be chosen from among the national league members to manage the day to day operation and

D2010 120

UP College of Law
federation members or from fund-raising projects or activities. The local government unit concerned may appropriate funds to support the leagues or federation organized pursuant to this Section, subject to the availability of funds. thereby leaving to the lawmakers full discretion to fix such term in accordance with the exigencies of public service. b. Private counsel/lawyers for elective local officials

David v. Comelec
Petitioners seek to declare as unconstitutional Sec. 43(c) of R.A. 7160, which limited the term of office of Barangay officials to three years. Petitioners contend that under Sec. 2 of RA 6653 the term of office of barangay officials shall be for five years. This is reiterated in R.A. 6679. Petitioners further aver that although Sec. 43 of RA 7160 reduced the term of office of all local elective officials to three years, such reduction does not apply to barangay officials because (1) RA 6679 is a special law applicable only to barangays while RA 7160 is a general law which applies to all other local government units; (2) RA 7160 does not expressly or impliedly repeal RA 6679 insofar as the term of barangay officials is concerned; (3) while Sec. 8 of Article X of the 1987 constitution fixes the term of elective local officials at three years, the same provision states that the term of barangay officials "shall be determined by law"; and (4) thus, it follows that the constitutional intention is to grant barangay officials any term, except three years. The COMELEC maintains that RA 7160 repealed all other special laws relied upon by the Petitioner. WON the term of the barangay officials should be limited only to three years. Held: Yes. R.A. 7160 was enacted later than RA 6679. In case of an irreconciliable conflict between two laws of different vintages, the later enactment prevails. Also, R.A. 7160 is a codified set of laws that specifically applies to local government units. It specifically provides that the term of office of barangay officials shall be for three years. With such particularity, the provision cannot be deemed a general law. Petitioners posit that by excepting barangay officials whose "term shall be determined by law" from the general provision fixing the term of "elective local officials" at three years, the Constitution thereby impliedly prohibits Congress from legislating a three year term for such officers. This is legally flawed. The Constitution did not expressly prohibit Congress from fixing any term of office for barangay officials. It merely left the determination of such term to the lawmaking body, without any specific limitation or prohibition,

Alinsug v. RTC-Negros Occidental


Zonsayda Alinsug was a regular employee of the Office of the Mayor of Escalante Negros Occidental. Mayor Ponsica issued Office Order No. 31, suspending Zonsayda for one month and one day for simple misconduct categorized as an act of insubordination. Zonsayda filed with the RTC a petition for injunction with damages. Mayor Ponsica, through private practitioner Samuel SM Lezama, claimed that Zonsayda had not yet exhausted administrative remedies and that her suspension was in accordance with law. Alinsug moved that the respondents be all declared in default on the ground that, since the respondents were sued in their official capacities, they should have been represented by either the municipal legal officer or the provincial legal officer or prosecutor as provided for by Sec. 481 (b) [i] and [3] of the Local Government Code. The respondents opposed the motion manifesting that the municipality of Escalante has no legal officer. WON private counsel may represent municipal officials sued in their official capacities. Held: Yes. It appears that the law allows a private counsel to be hired by a municipality only when the municipality is an adverse party in a case involving the provincial government or another municipality or city within the province. This has its apparent origin in De Guia v. The Auditor General where the Court held that the municipality's authority to employ a private attorney is expressly limited only to situations where the provincial fiscal would be disqualified to serve and represent it. But would these proscriptions include public officials? Not necessarily. It can happen that a government official, ostensibly acting in his official capacity and sued in that capacity, is later held to have exceeded his authority. On the one hand, his defense would have then been underwritten by the people's money which ordinarily should have been his personal expense. On the other hand, personal liability can attach to him without, however, his having had the benefit of assistance of a counsel of his own choice.

121 D2010

Local Government
In the discharge of governmental functions, municipal corporations are responsible for the acts of its officers, except if and when, the only to the extent that, they have acted by authority of the law, and in conformity with the requirements thereof. Also, a government official sued in his official capacity may engage the services of private counsel when the complaint contains other allegations and a prayer for moral damages, which, if due from the defendants, must be satisfied by them in their private capacity. The key then to resolving the issue of whether a local government official may secure the services of private counsel, in an action filed against him in his official capacity, lies on the nature of the action and the relief that is sought. While the petition below was filed against respondents as public officials, its allegations were also aimed at questioning certain acts that can well bring the case beyond the mere confines of official functions; thus The petition then went on to claim moral and exemplary damages, as well as litigation expenses, as shown by its prayer. Moral damages cannot generally be awarded unless they are the proximate result of a wrongful act or omission. Exemplary damages, on the other hand, are not awarded if the defendant had not acted in a wanton, oppressive or malevolent manner nor in the absence of gross or reckless negligence. A public official, who in the performance of his duty acts in such fashion, does so in excess of authority, and his actions would be ultra vires that can thereby result in an incurrence of personal liability. WON Atty. Mendiola has the authority to file the petition in behalf of the municipality. Held: No. Private attorneys cannot represent a province or municipality in lawsuits; only the provincial fiscal and the municipal attorney can represent a province or municipality in their lawsuits. The provision is mandatory. The municipality's authority to employ a private lawyer is expressly limited only to situations where the provincial fiscal is disqualified to represent it. The fact that the provincial fiscal was disqualified to handle the case must appear on record. In the instant case, there is nothing in the record to show that the provincial fiscal is disqualified; the appearance of herein private counsel is without authority of law. Also, the fiscal's refusal to represent the municipality is not a legal justification for employing the services of private counsel. A fiscal cannot refuse to perform his functions on grounds not provided for by law without violating his oath of office. Instead of engaging the services of a special attorney, the municipal council should request the Secretary of Justice to appoint an acting provincial fiscal in place of the provincial fiscal who has declined, (Sec. 1679, Administrative Code). Furthermore, even assuming that the representation of the municipality by Atty. Mendiola was duly authorized, said authority is deemed to have been revoked by the municipality when the latter, through the municipal mayor and without said counsel's participation, entered into a compromise agreement with the respondent. A client, by appearing personally and presenting a motion by himself, is considered to have impliedly dismissed his lawyer.

Municipality of Pililia v. CA
RTC rendered judgment in favor of the Municipality of Pililla, Rizal, against the Philippine Petroleum Corporation, ordering the latter defendant to pay the amount of P5,301,385.00 representing the tax on business due from the defendant under Section 9(A) of Municipal Tax Ordinance No. 1 of said municipality. When Atty. Mendiola ffiled a petition for certiorari with the SC, the PPC filed a motion questioning his authority to represent petitioner municipality. The CA dismissed the petition for having been filed by a private counsel in violation of law and jurisprudence but without prejudice to the filing of a similar petition by the Municipality of Pililla through the proper provincial or municipal legal officer.

Ramos v. CA
A petition was filed for the Declaration of Nullity of Municipal Ordinances and the contract of lease over a commercial arcade to be constructed in the municipality of Baliuag, Bulacan. Atty. Romanillos manifested that he was counsel for the municipality filing a motion to dismiss and an amended answer. The provincial attorney, Atty. Regalado, who first filed the answer, appeared as collaborating counsel. However, the Provincial Fiscal (Regalado) did not appear. It was Atty. Romanillos who worked the case for the municipality. The petitioners questioned the personality of Atty. Romanillos to appear as counsel of the municipality. In a joint statement, Atty. Romanillos withdrew as counsel for the municipality and Atty. Regalado, as collaborating

D2010 122

UP College of Law
counsel adopted the entire proceedings participated in/undertaken by Atty. Romanillos. The judge denied the petitioners motion to disqualify. WON private lawyer is authorized to represent the Municipality in its Lawsuits? Held: NO General Rule: Private counsel may not represent municipality or province in lawsuits. Section 1683 of the Revised Administrative Code provides that it is the duty of fiscal to represent provinces and provincial subdivisions in litigation EXCEPT in cases where: 1. 2. Original jurisdiction is vested in the SC The municipality or municipal district in question is a party adverse to the provincial government or to some other municipality or municipal district in the same province. He, or his wife, or child, is pecuniarily involved, as heir legatee, creditor or otherwise. EXCEPT that in interest of substantial justice, the municipality may adopt the work already performed in good faith by such private lawyer, which work is beneficial to it, provided: 1. 2. No injustice it thereby heaped on the adverse party; No compensation in any guise is paid therefor by said municipality to the private lawyer.

Unless so expressly adopted, the private lawyers work cannot bind the municipality. The proceedings already done are declared null and void for being participated in by unauthorized counsel. The Court believes that conferring legitimacy to the appearance of Atty. Romanillos would not cause substantial prejudice on petitioners. Requiring a new trial on the mere legal technicality that the municipality was not represented by a legally authorized counsel would not serve the interest of justice.

3.

Salalima v. Guingona
An administrative complaint was filed against Albay Governor Salalima, Vice-Governor Azafla, and several members of the Albay Sangguniang Panlalawigan because of the retainer contract for legal services entered into between the Province of Albay and Atty. Cornago and the Cortes & Reyna Law Firm (private lawyers), and the disbursement of public funds in payment thereof. WON respondents have incurred administrative liability in entering into the retainer agreement and making payments pursuant thereto. (The retainer was for a case filed by NPC against the province). Held: Yes Sec. 481 of the Local Government Code which is based on Section 1681 of the Revised Administrative Code requires the appointment of a legal officer to represent the local government unit in all civil actions and special proceedings wherein the local government unit or any official thereof, in his official capacity is a party; EXCEPT that in actions or proceeding where a component city or municipality is a party adverse to the provincial government or to another component city or municipality, a special legal officer may be employed to represent the adverse party. Local government units cannot be represented by private lawyers and it is solely the Provincial Fiscal who can rightfully represent them. Moreover, the entire

The above provision is complemented by Section 3, RA 2264, the Local Autonomy Law. The provision is mandatory. The municipality's authority to employ a private lawyer is expressly limited only to situations where the provincial fiscal is disqualified to represent it. This strict coherence to the letter of the law appears to have been dictated by the fact that the municipality should not be burdened with expenses of hiring a private lawyer and that the interests of the municipality would be best protected if a government lawyer handles its litigations. None of the exemptions are present in this case. In addition, for the exceptions to apply, the fact that the provincial fiscal was disqualified to handle the municipality's case must appear on record. There is nothing in the records to show that the provincial fiscal is disqualified to act as counsel for the Municipality. There is also no estoppel on the part of the plaintiffs because the legality of the representation of an unauthorized counsel may be raised at any stage of the proceedings. WON Collaboration with private counsel is allowed? Held: NO General rule: Private counsel may not represent the municipality/province even if only in collaboration with authorized government lawyers.

123 D2010

Local Government
transaction was attended by irregularities (i.e. No prior written approval of Solicitor General and COA before the disbursements were made, the resolution passed only authorized the governor to sign a retainer contract with the Cortes & Reyna Loaw Firm yet he also signed with Atty. Cornago which is a different entity, the province disbursed money to the Cortes & Reyna Law Firm although the latter did not appear as counsel for the Province in the SC case, the attorneys fees were unreasonable = P38.5 Million).

DISCIPLINARY ACTIONS
A. Sec 60 68, LGC Suspension is temporary. It may be imposed for no more than sixty days, a longer suspension is unjust and unreasonable, and nothing less than tyranny. Since the Mayor is facing ten administrative charges, the Mayor is in fact facing the possibility of 600 days of suspension, in the event that all ten cases yield prima facie findings. The Court is not of course tolerating misfeasance in public office but it is certainly another question to make him serve 600 days of suspension, which is effectively, to suspend him out of office. The Court is aware that only the third suspension is under questions, and that any talk of future suspensions is in fact premature. The fact remains, however, that Mayor Ganzon has been made to serve a total of 120 days of suspension and the possibility of sixty days more is arguably around the corner which amounts to a violation of the Local Government Code which brings to light a pattern of suspensions intended to suspend the Mayor the rest of his natural tenure. The Court is simply foreclosing what appears to us as a concerted effort of the State to perpetuate an arbitrary act.

See above

Ganzon v. CA and Artieda v. Santos (joint case)


A series of administrative complaints (10) were filed against Mayor Ganzon (Mayor of Iloilo City) by city officials on various charges: abuse of authority, oppression, grave misconduct and others. Secretary of Local Government, Hon. Santos, issued a preventive suspension order for 60 days. A second 60 day suspension was ordered but Mayor Ganzon was able to obtain a restraining order and a writ of preliminary injunction in the RTC. The second preventive suspension was not enforced. Amidst the two successive suspensions, Mayor Ganzon instituted an action for prohibition against the respondent in the RTC. Meanwhile, the respondent issued a third order for another 60 day preventive suspension (3rd time in 20 months), designating ViceMayor Malabor as acting mayor. Undaunted, Mayor Ganzon commenced before the CA, a petition for prohibition. The CA rendered judgment dismissing the cases. WON the several suspensions imposed upon Mayon Ganzon are proper. Held: No The plain truth is that this Court has been uncomfortable with suspensions because it is out of the ordinary to have a vacancy in local government. The sole objective of a suspension is simply "to prevent the accused from hampering the normal cause of the investigation with his influence and authority over possible witnesses" or to keep him off "the records and other evidence." It is a means, and no more, to assist prosecutors in firming up a case, if any, against an erring local official. Under the LGC (section 63), a suspension cannot exceed sixty days. It need not be exactly sixty days long if a shorter period is sufficient and it ought to be lifted if prosecutors have achieved their purpose in a shorter span.

Espiritu v. Melgar
Garing filed a sworn letter-complaint to the Secretary Santos of DILG, the Provincial Governor of Oriental Mindoro Espiritu and to the Presidential Action Center, charging Mayor Melgar of Naujan. Oriental Mindoro, with grave misconduct, oppression, abuse of authority, culpable violation of the Constitution and conduct prejudicial to the best interest of the public service. Melgar allegedly assaulted Garing and ordered his arrest and detention without filing any charges until his release the following day. Mayor Melgar submitted his answer wherein he said that while he was delivering a speech during a graduation ceremony, Garing suddenly clapped causing disturbance on the part of the audience. When the Mayor ended his speech, he instructed a policeman to investigate Garing. It appeared that Garing was drunk. The mayor informed Garing to go home but he refused to go and only did so the following morning.

D2010 124

UP College of Law
The Sangguniang Panlalawigan of Oriental Mindoro passed Resolution No 55, recommending to the Provincial Governor that the Mayor be preventively suspended for 45 days pending the investigation of the administrative complaint. When the mayor received the order of suspension, he filed a Petition for Certiorari with Preliminary Injunction with prayer for Restraining Order in the RTC. The RTC judge issued a writ of preliminary injunction enjoining Governor Espiritu from implementing the Order of suspension against Mayor Melgar. On appeal, Governor Espiritu contends that the trial judge erred in granting the preliminary injunction since the Governor is empowered to place an elective municipal official under preventive suspension pending decision of an administrative case against the elective municipal official. WON the governor has the power to suspend the mayor Held: Yes Under Section 63 LGC, the provincial governor is authorized by law to preventively suspend the municipal mayor anytime after the issues had been joined and any of the following grounds were shown to exist: 1. When there is reasonable ground to believe that the respondent has committed the act or acts complained of. When the evidence of culpability is strong. When the gravity of the offense so warrants. When the continuance in office of the respondent could influence the witnesses or pose a threat to the safety and integrity of the records and other evidence. suspension of Mayor Melgar was maintained by the TRO and therefore has already been served, he is deemed reinstated in office without prejudice to the continuation of the administrative investigation of the charges against him.

Aguinaldo v. Santos
Aguinaldo was the duly elected Governor of the province of Cagayan. After the December 1989 coup d'etat was crushed, the DILG Secretary Santos sent a telegram & letter to Governor Aguinaldo requiring him to show cause why he should not be suspended or removed from office for disloyalty to the Republic. A sworn complaint was also filed by Mayors of several municipalities in Cagayan against Aguinaldo for acts committed during the coup. Aguinaldo denied being privy to the planning of the coup or actively participating in its execution, though he admitted that he was sympathetic to the cause of the rebel soldiers. The Secretary suspended petitioner from office for 60 days from notice, pending the outcome of the formal investigation. Later, the Secretary rendered a decision finding petition guilty as charged and ordering his removal from office. The Vice Governor Vargas was installed as Governor. Aguinaldo appealed. While the case was pending before the SC, Aguinaldo filed his certificate of candidacy for the position of Governor of Cagayan. Three petitions for disqualification were filed against him on the ground that he had been removed from office. The Comelec granted the petition. Later, this was reversed on the ground that the decision of the Secretary has not yet attained finality and is still pending review with the Court. As Aguinaldo won by a landslide margin in the elections, the resolution paved the way for his eventual proclamation as Governor of Cagayan. WON the Secretary has the power to suspend or remove local government officials as alter ego of the President Held: Yes The power of the Secretary to remove local government officials is anchored on both the Constitution and a statutory grant from the legislative branch. The constitutional basis is provided by Articles VII (17) and X (4) of the 1987 Constitution which vest in the President the power of control over all executive departments, bureaus and offices and the power of general supervision over local governments. It is a constitutional doctrine that the acts of the department head are presumptively the acts of the President unless expressly rejected by him. Furthermore, it cannot be said that BP337 was

2. 3. 4.

There is nothing improper in suspending an officer before the charges against him are heard and before he is given an opportunity to prove his innocence. Preventive suspension is allowed so that the respondent may not hamper the normal course of the investigation through the use of his influence and authority over possible witnesses.Since the mayor believed that his preventive suspension was unjustified and politically motivated, he should have sought relief first from the Secretary of DILG, not from the courts. Mayor Melgar's direct recourse to the courts without exhausting administrative remedies was premature. The RTC had no jurisdiction over Special Civil Action No. R-5003 and gravely abused its discretion in refusing to dismiss the case. As a general rule, the office or body that is invested with the power of removal or suspension should be the sole judge of the necessity and sufficiency of the cause. However, in this case, since the 60-day preventive

125 D2010

Local Government
repealed by the effectivity of the present Constitution as both the 1973 and 1987 Constitution grants to the legislature the power and authority to enact a local government code, which provides for the manner of removal of local government officials. The power of the DILG secretary to remove local elective government officials is found in Secs. 60 and 61 of BP 337. As to Aguinaldos argument of the want of authority of the Secretary to appoint Vargas as Governor, Section 48 (1) of B.P. Blg. 337 shows otherwise. Equally without merit is petitioner's claim that before he could be suspended or removed from office, proof beyond reasonable doubt is required because he is charged with a penal offense of disloyalty to the Republic which is defined and penalized under Article 137 of the RPC. Petitioner is not being prosecuted criminally, but administratively where the quantum of proof required is only substantial evidence. Aguinaldos re-election to the position of Governor of Cagayan has rendered the administrative case pending before Us moot and academic. It appears that after the canvassing of votes, petitioner garnered the most number of votes among the candidates for governor of Cagayan province. The rule is that a public official cannot be removed for administrative misconduct committed during a prior term, since his re-election to office operates as a condonation of the officer's previous misconduct to the extent of cutting off the right to remove him therefor. The foregoing rule, however, finds no application to criminal cases pending against petitioner for acts he may have committed during the failed coup. turn over the office to the incumbent vice mayor but he refused to accept the service of the order. Thereafter, Reyes filed a certificate of candidacy with the Comelec but a petition for disqualification was filed against him. Thus, the Comelec canceled Reyess certificate of candidacy. However, the Municipal Board of Canvassers of Bongabong unaware of the disqualification of Reyes by the Comelec, proclaimed him the duly-elected mayor. The Comelec en banc affirmed. Reyes argues that his election on May 8, 1995 is a bar to his disqualification. Garcia, who obtained the highest number of votes next to Reyes intervened, contending that because Reyes was disqualified, he was entitled to be proclaimed mayor. The Comelec en banc denied Garcias prayer. WON the decision of the Sangguniang Panlalawigan is not yet final because he has not been served a copy thereof. Held: No The failure of the Sangguniang Panlalawigan to deliver a copy of its decision was due to the refusal of petitioner and his counsel to receive the decision. Repeated attempts had been made to serve the decision on Reyes personally and by registered mail, but Reyes refused to receive the decision. If a judgment or decision is not delivered to a party for reasons attributable to him, service is deemed completed and the judgment or decision will be considered validly served as long as it can be shown that the attempt to deliver it to him would be valid were it not for his or his counsel's refusal to receive it. Reyess refusal to receive the decision may, therefore, be construed as a waiver on his part to have a copy of the decision. Petitioner was given sufficient notice of the decision. Rather than resist the service, he should have received the decision and taken an appeal to the Office of the President in accordance with R.A. No. 7160 Section 67. But petitioner did not do so. Accordingly, the decision became final 30 days after the first service upon petitioner. Thus, when the elections were held the decision of the Sangguniang Panlalawigan had already become final and executory. The filing of a petition for certiorari with the RTC did not prevent the administrative decision from attaining finality. An original action of certiorari is an independent action and does not interrupt the course of the principal action nor the running of the reglementary period involved in the proceeding. Consequently, to arrest the course of the principal action during the pendency of the certiorari proceedings, there

Reyes v. Comelec
Reyes was the incumbent mayor of the municipality of Bongabong, Oriental Mindoro. An administrative complaint was filed against him with the Sangguniang Panlalawigan by Dr. Manalo. It was alleged that Reyes exacted and collected P50,000,00 from each market stall holder in the Bongabong Public Market. Also, that certain checks issued to him by the National Reconciliation and Development Program of the DILG were never received by the Municipal Treasurer nor reflected in the books of accounts of the same officer; and that he took 27 heads of cattle from beneficiaries of a cattle dispersal program. The Sangguniang Panlalawigan found petitioner guilty of the charges and ordered his removal from office. Reyes filed a petition for certiorari, prohibition and injunction with the RTC of Oriental Mindoro. Later, the Presiding Officer of the Sangguniang Panlalawigan issued an order for Reyes to vacate the position of mayor and to

D2010 126

UP College of Law
must be a restraining order or a writ of preliminary injunction from the appellate court directed to the lower court. In the case at bar, although a temporary restraining order was issued by the Regional Trial Court, no preliminary injunction was subsequently issued. The temporary restraining order issued expired after 20 days. From that moment on, there was no more legal barrier to the service of the decision upon petitioner. WON petitioners reelection rendered the administrative charges against him moot and academic Held: No This case is different from Aguinaldo v. Santos. Here, although Reyes brought an action to question the decision in the administrative case, the TRO issued in the action he brought lapsed with the result that the decision was served on petitioner and became final. Thus, because petitioner failed to appeal to the Office of the President, he was validly removed from office and, pursuant to Section 40(b) of the LGC, he was disqualified from running for reelection. It is noteworthy that at the time the Aguinaldo cases were decided there was no provision similar to Section 40(b) which disqualifies any person from running for any elective position on the ground that he has been removed as a result of an administrative case. R.A. No. 7160 could not be given retroactive effect. Furthermore, the Aguinaldo decision has not yet attained finality. As indicated earlier, the decision of the then Secretary of Local Government was questioned by the petitioner in this Court and that to date, the petition remains unresolved. Garcia's plea that the votes cast for Reyes be invalidated is without merit. The candidate who obtains the second highest number of votes may not be proclaimed winner in case the winning candidate is disqualified. To simplistically assume that the second placer would have received the other votes would be to substitute our judgment for the mind of the voter. The votes cast for Reyes are presumed to have been cast in the belief that Reyes was qualified and for that reason cannot be treated as stray, void, or meaningless. The subsequent finding that he is disqualified cannot retroact to the date of the elections so as to invalidate the votes cast for him. Councilors. The respondents were charged with having violated R.A No. 3019, as amended; Articles 170 and 171 RPC; and R.A. No. 6713. Councilors Dionson and Bercede averred that respondent officials, acting in conspiracy, had caused the alteration and/or falsification of Ordinance No. 018/92 by increasing the allocated appropriation therein without authority from the Sangguniang Panlungsod of Mandaue City. Aside from opposing the motion for preventive suspension, respondent officials prayed for the dismissal of the complaint on the ground that the Ombudsman Hagad supposedly was bereft of jurisdiction to try, hear and decide the administrative case filed against them since, under Section 63 LGC, the power to investigate and impose administrative sanctions against said local officials, as well as to effect their preventive suspension, had now been vested with the Office of the President. The Office of the Deputy Ombudsman denied the motion to dismiss and recommended the preventive suspension of respondent officials, except City Budget Officer Guido, until the administrative case would have been finally resolved by the Ombudsman. A petition for prohibition, with prayer for a writ of preliminary injunction and temporary restraining order was filed by respondent officials with the RTC. The RTC issued a restraining order directed at the Ombudsman, enjoining him from enforcing the preventive suspension. WON the Ombudsman has jurisdiction over the present case Held: Yes The general investigatory power of the Ombudsman is decreed by Section 13(1,) Article X1, of the 1987 Constitution, while his statutory mandate to act on administrative complaints is contained in Section 19 of R.A. No. 6770. Section 21 of the same statute names the officials who could be subject to the disciplinary authority of the Ombudsman. Taken in conjunction with Section 24 of R.A. No. 6770, the Office of the Ombudsman correspondingly has the authority to decree preventive suspension on any public officer or employee under investigation by it. The argument of the respondents that the disciplinary authority of the Ombudsman over local officials has been removed by the subsequent enactment of the Local Government Code of 1991 is without merit. Although Section 63 of the Local Government Code provides that preventive suspension can only be imposed by: ". . . the President if the respondent is an elective official of a province, a highly urbanized or an independent

Hagad v. Gozo-Dadole
Criminal and administrative complaints were filed in the Office of the Deputy Ombudsman against Mayor Ouano, Vice-Mayor Caete and Sangguniang Panlungsod Member Mayol of Mandaue City, by Mandaue City

127 D2010

Local Government
component city; . . ." There is nothing in the LGC to indicate that it has repealed, whether expressly or impliedly, the pertinent provisions of the Ombudsman Act. The two statutes on the specific matter in question are not so inconsistent, let alone irreconcilable, as to compel us to only uphold one and strike down the other. Well settled is the rule that repeals of laws by implication are not favored, and that courts must generally assume their congruent application. The two laws must be absolutely incompatible, and a clear finding thereof must surface, before the inference of implied repeal may be drawn. The rule is expressed in the maxim, interpretare et concordare leqibus esf optimus interpretendi: every statute must be so interpreted and brought into accord with other laws as to form a uniform system of jurisprudence. All doubts must be resolved against any implied repeal, and all efforts should be exerted in order to harmonize and give effect to all laws on the subject. The authority to conduct administrative investigation and to impose preventive suspension over elective provincial or city officials was at that time entrusted to the Minister of Local Government until it became concurrent with the Ombudsman upon the enactment of R.A No. 6770 (Sec. 21 & 24), to the extent of the common grant, the LGC did not effect a change from what already prevailed, the modification being only in the substitution of the Secretary of Local Government by the Office of the President. Respondent local officials contend that the 6-month preventive suspension without pay under Section 24 of the Ombudsman Act is much too repugnant to the 60day preventive suspension provided by Section 63 LGC to even now maintain its application. This contention is without merit. The two provisions govern differently and there is justification for the imposition of the 6 month preventive suspension.
Ombudsman 6 month preventive suspension RA 7160 60 day preventive suspension, at any time after the issues have been joined or Elective officials only after the issues are joined. Grounds for suspension: 1. preventive oppression or grave misconduct or neglect in the performance of duty, or The charges should warrant removal from the service, or The respondents continued stay in office would prejudice the case filed against him of The evidence of guilt is strong The gravity of the offense so warrants The continuance in office of the respondent could influence the witnesses or pose a threat to the safety and integrity of the records and other evidence

2. 3. 4.

b.

c.

Salalima v. Guingona (supra)


Administratrive Order No. 153 was signed by the President and respondent Teofisto Guingona, which approved the findings of an Ad Hoc Committee holding Salalima et al liable in four (4) consolidated administrative cases. Petitioners were elective officials of the Province of Albay, and were handed out penalties in the following manner: 1) 5 months suspension for Albay Governor Salalima and 4 months suspension for the other respondents for abuse of authority, due to the passage of an illegal ordinance which deprived the barangays of Tiwi and Daraga of their share in delinquency payments made by Napocor to the government; 2) 6 months suspension for Salalima and Azana, 4 months for all the other respondents for abuse of authority under Section 60 of the LGC, when they hired private lawyers (Cortes & Reyna Law Firm) to represent them in their case in the Supreme Court and disbursed public money to do so; 3) 4 months suspension each for oppression and abuse of authority, when they assumed jurisdiction and hastily and arbitrarily meted out suspensions to Tiwi Mayor Corral pending the administrative cases she had filed against the respondents; and 4) 5 months suspension to Governor Salalima for abuse of authority and gross negligence for failing to impose and collect damages from RYU Construction Corp when the latter incurred in delay. Petitioners challenge AO 153 on the grounds that: 1) the AO effectively suspends petitioners for periods ranging from twelve to twenty months; 2) the Office of the President committed grave abuse of discretion in suspending petitioners for administrative offenses allegedly committed during prior terms. Held: 1) Petitioners contend that the challenged administrative order deprived them of their respective offices without procedural and substantive due process. Their suspensions ranging from twelve months to twenty months or for the entire duration of their unexpired

All public officials (Elective appointive) under investigation.

Grounds for preventive suspension: (S. 24, RA 9770) 1. The evidence of guilt should be strong, AND a. The charge against the officer or employee should involve dishonestly,

There is a reasonable ground to believe that the respondent has committed the act or acts complained

D2010 128

UP College of Law
term, which was then only seven months, constituted permanent disenfranchisement or removal from office in clear violation of Section 60 of R.A. No. 7160 which mandates that an elective local official may be removed from office by order of the court. However, Section 66(b) of R. A. No. 7160 expressly provides that the penalty of suspension shall not exceed the unexpired term of the respondent or a period of six (6) months for every administrative offense, nor shall said penalty be a bar to the candidacy of the respondent so suspended as long as he meets the qualifications for the office. Administrative Offense means every act or conduct or omission which amounts to, or constitutes any of the grounds for disciplinary action. The Office of the President committed no grave abuse of discretion in imposing the penalty of suspension, although the aggregate thereof exceeded six months and the unexpired portion of the petitioners' term of office. The fact remains that the suspension imposed for each administrative offense did not exceed six months and there was an express provision that the successive service of the suspension should not exceed the unexpired portion of the term of office of the petitioners. 2) Governor Salalima could no longer be held liable in connection with the negotiated contract RYU Construction, nor could the petitioners be held administratively liable for the execution in November 1989 of the retainer contract with Atty. Jesus Cornago and the Cortes and Reyna Law Firm. This is so because public officials cannot be subject to disciplinary action for administrative misconduct committed during a prior term. His reelection to office operates a condonation of the officer's previous misconduct to the extent of cutting off the right to remove him therefor. This doctrine of forgiveness or condonation cannot, however, apply to criminal acts which the reelected official may have committed during his previous term. 3) The grant of the power to remove elective local officials by the Oversight Committee to the disciplining authority in drafting the Implementing Rules for the LGC is ultra vires; such power is vested only with the court. having been dismissed. On the third instance when Basco was again elected Councilor, petitioner Grego filed with the COMELEC a petition praying for Basco's disqualification, for the suspension of his proclamation, and for the declaration of seventh placer Romualdo S. Maranan as the sixth duly elected Councilor of Manila's Second District. While the case was ongoing, the Manila City Board of Canvassers proclaimed Basco as a duly elected councilor for the Second District of Manila, placing sixth among several candidates who vied for the seats. Basco immediately took his oath of office before the Honorable Ma. Ruby Bithao-Camarista, Presiding Judge, Metropolitan Trial Court, Branch I, Manila. HELD: Section 40 (b) of the LGC disqualifies those removed from office as a result of an administrative case from running for any elective position. However, it does NOT apply retroactively to those removed from office before it took effect on January 1, 1992. Well-settled is the principle that while the Legislature has the power to pass retroactive laws which do not impair the obligation of contracts, or affect injuriously vested rights, it is equally true that statutes are not to be construed as intended to have a retroactive effect so as to affect pending proceedings, unless such intent is expressly declared or clearly and necessarily implied from the language of the enactment. There is no provision in the statute which would clearly indicate that the same operates retroactively. Lex prospicit, non respicit. As such, the issue of whether or not Bascos election to office in the 1988, 1992 and 1995 elections wipe away and condone the administrative penalty against him is beside the point since he is deemed NOT subject to disqualification under Sec 40 (b) of the LGC. Also, Basco is deemed NOT to have circumvented the prohibition in the Tordesillas decision since under the former Civil Service Decree, (the law applicable at the time of the decision) reinstatement referred only to an appointive position. Moreover, there is no reason why the Manila City BOC should not have proclaimed Basco as the sixth winning City Councilor. Absent any determination of irregularity in the election returns, as well as an order enjoining the canvassing and proclamation of the winner, it is a mandatory and ministerial duty of the Board of Canvassers concerned to count the votes based on such returns and declare the result. Lastly, Romualdo S. Maranan, the seventh placer, may NOT be legally declared a winning candidate since Basco was not disqualified.

Grego v. Comelec
October 31, 1981 (before the effectivity of the LGC), Basco was removed from his position as Deputy Sheriff (with prejudice to reinstatement to any position in the national or local government and its agencies and instrumentalities or GOCCs, in the words of the Court) upon a finding of serious misconduct in an administrative complaint lodged by Nena Tordesillas. Basco then ran as a candidate for Councilor on two consecutive occasions and won, with subsequent challenges to his election

Joson v. Executive Secretary Torres

129 D2010

Local Government
Members of the Sangguniang Panlalawigan of Nueva Ecija filed a letter-complaint with the Office of the President charging petitioner Governor Joson with grave misconduct and abuse of authority, praying for his suspension and removal from office. Governor Joson had allegedly barged into the Hall during a scheduled session of the SP and angrily kicked the door and chairs in the Hall and uttered threatening words at respondents while men with firearms encircled the area. Acting on the complaint, President Ramos ordered Secretary of Internal and Local Government Robert Barbers take appropriate preemptive and investigative actions, but to break not the peace. Upon recommendation of Secretary Barbers, Executive Secretary Ruben Torres issued an order, by authority of the President, placing petitioner under preventive suspension for sixty (60) days pending investigation of the charges against him. Joson filed a petition for certiorari and prohibition with the Court of Appeals challenging the order of preventive suspension and the order of default, which was dismissed. Petitioner alleges that subsequent to the institution of this petition, the SILG rendered a resolution on the case finding him guilty of the offenses charged, whose finding was based on the position papers and affidavits of witnesses submitted by the parties. The Executive Secretary, by authority of the President, then adopted the findings and recommendation of the DILG Secretary and imposed a six-month suspension. Joson now questions the CA decision affirming his preventive suspension and the implementation of the SILG Resoultion without formal investigation. HELD: An administrative complaint against an elective official must be verified and filed with the proper government office. A complaint against an elective provincial or city official must be filed with the Office of the President, one against an elective municipal official must be filed with the Sangguniang Panlalawigan, while that of a barangay official must be filed before the Sangguniang Panlungsod or Sangguniang Bayan. Joson is an elective provincial official, thus the complaint against him was properly filed with the Office of the President. According to petitioner, the complaint was not verified by private respondents. However, the defect was not fatal. The requirement was deemed waived by the President himself when he acted on the complaint. Petitioner also claims undue delegation of the disciplining authority to the DILG. Jurisdiction over administrative disciplinary actions against elective local officials is lodged in two authorities: the Disciplining Authority (the President or the Executive Secretary) and the Investigating Authority (the DILG, as per A. O. No. 23). What is delegated is the power to investigate, not the power to discipline. The DILG did not err when it recommended preventive suspension, which may be imposed by the Disciplining Authority at any time (a) after the issues are joined; (b) when the evidence of guilt is strong; and (c) given the gravity of the offense, there is great probability that the respondent, who continues to hold office, could influence the witnesses or pose a threat to the safety and integrity of the records and other evidence. However, the rejection of petitioner's right to a formal investigation denied him procedural due process. The records show that petitioner filed a motion for formal investigation. An erring elective local official has rights akin to the constitutional rights of an accused. These rights are essentially part of procedural due process. The local elective official has the (1) right to appear and defend himself in person or by counsel; (2) the right to confront and cross-examine the witnesses against him; and (3) the right to compulsory attendance of witness and the production of documentary evidence. Petitioner's right to a formal investigation was not satisfied when the complaint against him was decided on the basis of position papers. The procedure of requiring position papers in lieu of a hearing in administrative cases is expressly allowed with respect to appointive officials but not to those elected. An elective official, elected by popular vote, is directly responsible to the community that elected him. Suspension and removal are thus imposed only after the elective official is accorded his rights and the evidence against him strongly dictates their imposition.

Conducto v. Monzon
Judge Iluminado Monzon was charged with ignorance of the law for deliberately refusing to suspend a barangay chairman who was charged with unlawful appointment before his sala. Barangay chairman Benjamin Maghirang was charged with violation of Section 394 of the Local Government Code and Article 244 of the Revised Penal Code for appointing his sister-in-law as barangay secretary. The Office of the City Prosecutor dismissed the complaint, stating that the appointment was made before the effectivity of the Local Government Code of 1991. Complainant was later able to secure an Opinion from the DILG Director Jacob Montesa, which declared that the appointment issued by Maghirang to his sisterin-law violated the Local Government Code in effect prior to that of 1991. This prompted the Office of the City Prosecutor to file an information with the Municipal Trial Court of San Pablo. Respondent judge issued a warrant for Maghirangs arrest. A motion for suspension was filed pursuant to Sec. 13 of RA 3019 or the Anti Graft and Corrupt Practices Act (which provides that any

D2010 130

UP College of Law
incumbent public officer or official under criminal prosecution under Title 7, Book II of the RPC shall be suspended). Respondent judge denied the motion on the ground that offenses committed during a prior term shall not be cause for suspension during the present term. In denying a motion for reconsideration of the same, Monzon stated that preventive suspension only applies if there is an administrative case against the official filed at the same time as the criminal charge. HELD: There is misplaced reliance by the judge upon the case of Pascual vs Provincial Board of Nueva Ecija. The doctrine of forgiveness or condonation finds no application in criminal liability. It was subsequently held in Ingco vs. Sanchez that the reelection of a public officer does not wipe away any criminal liability incurred by him in a previous term. Section 13 of RA 3019 makes it mandatory upon the Court to suspend any public officer against whom a valid information is filed for a violation of Title 7, Book II of the RPC or any offense involving fraud upon government or public funds or property. Respondent judge is fined P5,000 for ignorance of the law and given a warning against committing similar acts in the future. alter, amend, or contravene a provision of law such as the LGC. Such power to remove elective local officials from service is lodged exclusively with the courts.

Sangguniang Barangay of Don Mariano v. Punong Barangay Martines


Severino Martinez was administratively charged with Dishonesty and Graft and Corruption by petitioner through the filing of a verified complaint before the Sangguniang Bayan as the disciplining authority over elective barangay officials pursuant to Section 64 of RA 7160. The complaint was later amended for Dishonesty, Misconduct in Office and Violation of the Anti-Graft and Corrupt Practices Act. The Sangguniang Bayan rendered its Decision which imposed upon Martinez the penalty of removal from office. The decision was conveyed to the mayor of Bayombong, Nueva Ecija. The mayor, however, issued a Memo wherein he stated that the Sangguniang Bayan has no power to order Martinez removal from office. However, the decision remains valid until reversed. WON the Sanggunian may remove Martinez, an elective local official, from office. Held: No. Section 60 of the Local Government Code conferred upon the courts the power to remove elective local officials from office. During the deliberations of the Senate on the Local Government Code,[16] the legislative intent to confine to the courts, i.e., RTCs, the Sandiganbayan and the appellate courts, jurisdiction over cases involving the removal of elective local officials was evident. In Salalima v. Guingona, Jr., the Court en banc categorically ruled that the Office of the President is without any power to remove elected officials, since the power is exclusively vested in the proper courts as expressly provided for in the last paragraph of Section 60 of the LGC. It further invalidated Article 125, Rule XIX of IRR. The Court nullified the rule since the Oversight Committee that prepared the Rules and Regulations of the Local Government Code exceeded its authority when it granted to the disciplining authority the power to remove elective officials, a power which the law itself granted only to the proper courts. Thus, it is clear that under the law, the Sangguniang Bayan is not vested with the power to remove Martinez. Petitioner contends that administrative cases involving elective barangay officials may be filed with, heard and

Pablico v. Villapando
An administrative complaint was filed with the Sangguniang Panlalawigan of Palawan against then Mayor of San Vicente, Palawan Alejandro Villapando for abuse of authority and culpable violation of the Constitution because he entered into a consultancy agreement with Orlando Tiape, a defeated mayoralty candidate. Complainants argue that this amounted to appointment to a government position within the prohibited one-year period under Article IX-B, Sec. 6 of the 1987 Constitution. The Sangguniang Panlalawigan found respondent guilty and imposed on him the penalty of dismissal from service, and was affirmed by the Office of the President. Vice-mayor Pablico took his oath as municipal mayor in place of Villapando. The Court of Appeals declared the decisions of the SP and OP void, and ordered Pablico to vacate the Office of the Mayor of San Vicente, Palawan. HELD: The last paragraph of Sec. 60 of the Local Government Code clearly provides that the dismissal from service of an erring elective local official may only be decreed by a court of law. Although Article 124(b), Rule XIX of the Rules and Regulations Implementing the Local Government Code adds that such removal may be had by the disciplining authority (pertaining to the Sangguniang Panlalawigan), no rule or regulation may

131 D2010

Local Government
decided by the Sangguniang Panlungsod or Sangguniang Bayan concerned, which can, thereafter, impose a penalty of removal from office. It further claims that the courts are merely tasked with issuing the order of removal, after the Sangguniang Panlungsod or Sangguniang Bayan finds that a penalty of removal is warranted. The aforementioned position put forward by the petitioner would run counter to the rationale for making the removal of elective officials an exclusive judicial prerogative. In Pablico v. Villapando, the court declared that:The law on suspension or removal of elective public officials must be strictly construed and applied, and the authority in whom such power of suspension or removal is vested must exercise it with utmost good faith, for what is involved is not just an ordinary public official but one chosen by the people through the exercise of their constitutional right of suffrage. Their will must not be put to naught by the caprice or partisanship of the disciplining authority. Where the disciplining authority is given only the power to suspend and not the power to remove, it should not be permitted to manipulate the law by usurping the power to remove. (Emphasis supplied.) The rule which confers to the proper courts the power to remove an elective local official from office is intended as a check against any capriciousness or partisan activity by the disciplining authority. As the law stands, Section 61 of the Local Government Code provides for the procedure for the filing of an administrative case against an erring elective barangay official before the Sangguniang Panlungsod or Sangguniang Bayan. However, the Sangguniang Panlungsod or Sangguniang Bayan cannot order the removal of an erring elective barangay official from office, as the courts are exclusively vested with this power under Section 60 of the Local Government Code. Thus, if the acts allegedly committed by the barangay official are of a grave nature and, if found guilty, would merit the penalty of removal from office, the case should be filed with the regional trial court. Once the court assumes jurisdiction, it retains jurisdiction over the case even if it would be subsequently apparent during the trial that a penalty less than removal from office is appropriate. On the other hand, the most extreme penalty that the Sangguniang Panlungsod or Sangguniang Bayan may impose on the erring elective barangay official is suspension; if it deems that the removal of the official from service is warranted, then it can resolve that the proper charges be filed in court. The doctrine of separation of powers is not absolute in its application; rather, it should be applied in accordance with the principle of checks and balances. The removal from office of elective officials must not be tainted with partisan politics and used to defeat the will of the voting public. The local government units are not deprived of the right to discipline local elective officials; rather, they are prevented from imposing the extreme penalty of dismissal.

Cases of Sexual Harassment versus elective local government officials and local government employees, heads of departments Civil Service Administrative Rule on Sexual Harassment and RA No 7877
See attachments

Book I, Title 2, Chapter 4, LGC (Sec 60 68)


See above B. Disciplinary Actions over Local Appointive Officials

Sec 84 89, LGC


Administrative discipline (Sec 84) Investigation and adjudication of administrative complaints against appointive local officials and employees as well as their suspension and removal shall be in accordance with the civil service law and rules and other pertinent laws. The results of such administrative investigations shall be reported to the Civil Service Commission. Preventive suspension of appointive local officials and employees (Sec 85) Who may impose: The local chief executive

When to impose: Pending investigation after filing of administrative charges against the subordinate official or employee which involves: o Dishonesty o Oppression

D2010 132

UP College of Law
o o o Grave misconduct Neglect in the performance of duty If there is reason to believe that the respondent is guilty of the charges which would warrant his removal from the service 5. 6. Appeal
4

Reprimand Other disciplinary actions

Duration of Preventive Suspension: Not exceeding sixty (60) days

APPEALABLE If the penalty imposed is heavier than suspension of thirty (30) days

NOT APPEALABLE If the penalty imposed is suspension without pay for not more than thirty (30) days

What happens after preventive suspension: The suspended official shall be automatically reinstated in office without prejudice to the continuation of the administrative proceedings against him until its termination. NOTE: If the delay in the proceedings of the case of the case is due to the fault, neglect, or request of the respondent, the time of the delay shall not be counted in computing the period of suspension herein provided. Administrative investigation (Sec 86) Who may perform: Any person or committee duly authorized by the local chief executive Where to appeal: Civil Service Commission (judgment must be rendered within 30 days from receipt of appeal) Execution pending appeal (Sec 88) An appeal shall not prevent the execution of a decision of removal or suspension of a respondent-appellant. In case the respondent-appellant is EXONERATED, he shall be reinstated to his position with all the rights and privileges appurtenant thereto from the time he had been deprived thereof. Prohibited business and pecuniary interest (Sec 89) 1. How performed: The person or committee duly authorized shall conduct hearings on the cases brought against appointive local officials and employees. The investigating body shall submit their findings and recommendations to the local chief executive concerned within fifteen (15) days from conclusion of the hearings. Duration of Administrative Proceedings (Rendition of Judgment): 90 days from the time the respondent is formally notified of the charges. Engage in any business transaction with the local government unit in which he is: a. an official, or b. employee, o c. over which he has the power of supervision, or with any of its d. authorized boards, e. officials, f. agents, or g. attorney,

whereby money is to be paid, or property or anything of value is to be transferred, directly or indirectly, out of the resources of the LGU to such person or firm; 2. 3. Hold such interests in any cockpit or other games licensed by an LGU; Purchase any real estate or other property forfeited in favor of such LGU a. for unpaid taxes or assessment, or

Disciplinary jurisdiction (Sec 87) Imposable penalties: 1. 2. 3. 4. Removal from service Demotion in rank Suspension for not more than one (1) year without pay Fine in an amount not exceeding six (6) months salary

The provision is unclear whether the imposition of demotion, reprimands etc. are also final and not appealable. The provision speaks only of suspension without pay for not more than thirty days which is not appeable.

133 D2010

Local Government
by virtue of a legal process at the instance of the said local official; Be a surety for any person contracting or doing business with the LGU for which a surety is required; and Possess or use any public property of the LGU for private purposes. b. instant case, Coloyan, who filed the appeal, cannot be considered an aggrieved party because he is not the respondent in the administrative case below. Moreover, The remedy of appeal may be availed of only in a case where the respondent is found guilty of the charges filed against him. But when the respondent is exonerated of said charges, as in this case, there is no occasion for appeal. The CSC decision is set aside and the decision of the Quezon City Mayor is reinstated (Complaint against Mendez dismissed).

4.

5.

Other prohibited businesses and interests as provided under R.A. 6713 shall also be applicable.

Mendez v. CSC
Then Acting Register of Deeds of Quezon City Vicente N. Coloyan filed an administrative complaint against the petitioner, a legal research assistant in the Quezon City Office of the City Attorney, for Gross Misconduct and Dishonesty, allegedly for having torn off a portion of Transfer Certificate of Title No. 209287 from the registry book of Quezon City and for having pocketed it. After three months of investigation, then Quezon City Mayor Adelina Rodriguez dismissed the said complaint against the petitioner for insufficiency of evidence. Coloyan appealed to the Merit Systems Protection Board (MSPB) reversed the decision of the Mayor and dismissed Mendez from the service. The CSC affirmed the MSPB decision. Mendez filed a motion for reconsideration, assailing the reversal of the city mayor's decision by the MSPB and the CSC on the ground that Coloyan is not an aggrieved party or "party adversely affected by the decision" allowed by law to file an appeal. Moreover, the petitioner claimed that his exoneration by the city mayor is unappealable pursuant to Section 37, paragraph (b) of P.D. 807. HELD: P.D. 807, otherwise known as The Philippine Civil Service Law, does not contemplate a review of decisions exonerating officers or employees from administrative charges. Section 37 paragraph (a) thereof, provides: "The Commission shall decide upon appeal all administrative disciplinary cases involving the imposition of a penalty of suspension for more than thirty days, or fine in an amount exceeding thirty days' salary, demotion in rank or salary or transfer, removal or dismissal from office " Said provision must be read together with Section 39 paragraph (a) of P.D. 805 which contemplates: "Appeals, where allowable, shall be made by the party adversely affected by the decision . . ." The phrase "party adversely affected by the decision" refers to the government employee against whom the administrative case is filed for the purpose of disciplinary action which may take the form of suspension, demotion in rank or salary, transfer, removal or dismissal from office. In the

Macalingag and Carlos v. Chang


Pursuant to an administrative charge against him for dishonesty, neglect of duty, and act prejudicial to the best interest of the service, an Order of Preventive suspension was issued against Roberto Chang, then the acting municipal treasurer of Makati. Said Order was signed by Lorinda Carlos, the Executive director of the Bureau of Local Government, and Victor Macalincag, Undersecretary of Finance, who was then the acting Secretary. Chang filed a complaint for prohibition with preliminary injunction with the lower court. The trial court found in that in order for preventive suspension to take effect, there are two steps involved: 1) service of the copy the order on the respondent, and 2) designation of his replacement. The order of preventive suspension was held to have been incomplete and without effect since an acting municipal treasurer had yet to be appointed to replace Chang. HELD: Preventive suspension is governed by Sec. 41 of P.D. 807 or the Civil Service Law, which does not require a replacement to be designated for the Order to take effect. BP 337 (the LGC in effect at the time) provides for the automatic assumption of the assistant municipal treasurer or next in rank officer in case of suspension of the municipal treasurer. There can be no question that the Order of Preventive Suspension became effective upon respondent Changs receipt thereof. Chang argues that EO 392, which gave rise to the creation of the Metropolitan Manila Authority, vested in the President of the Philippines the power to appoint the municipal treasurer, and thus only the President may suspend or remove him. However, Section 8 of EO 392 provides that the appointments made by the President of the Philippines shall be subject to the Civil Service Law, rules and regulations. Moreover, the Office of the Municipal Treasurer unquestionably falls under the Department of Finance. Hence, the Secretary of Finance is the proper disciplining authority to issue the preventive suspension order. Even assuming that the power to appoint includes the power to discipline (as argued by Chang), Acting

D2010 134

UP College of Law
Secretary Macalingag, as Secretary of Finance, is the alter ego of the President. It is therefore within his authority to preventively suspend Chang. strong; (2) [w]hen the respondent is a recidivist x x x; and (3) *w+hen the respondent is notoriously undesirable. Technical rules of procedure and evidence are not strictly applied; due process in the administrative context cannot be fully equated with that in the strict judicial sense. The power to discipline is specifically granted by Section 47 of the Administrative Code of 1987 to heads of departments, agencies and instrumentalities, provinces and cities. On the other hand, the power to commence administrative proceedings against a subordinate officer or employee is granted by Section 34 of the Omnibus Rules Implementing Book V of the said Administrative Code to the secretary of a department, the head of office of equivalent rank, the head of a local government unit, the chief of an agency, the regional director or a person with a sworn written complaint. Further, the city treasurer may institute, motu propio, disciplinary proceedings against a subordinate officer or employee. Local Administrative Regulations (LAR) No. 2-85, which was issued by the Ministry of Finance on March 27, 1985, authorized the minister (now secretary) of finance, the regional director, and head of a local treasury or an assessment office to start administrative disciplinary action against officers or employees subordinate to them. In the case at bar, the city treasurer is the proper disciplining authority referred to in Section 47 of the Administrative Code of 1987. The term agency refers to any of the various units of the government including a department, a bureau, an office, an instrumentality, a government-owned or controlled corporation, or a local government or a distinct unit therein. Respondent Pajaro, as the city treasurer, was the head of the Office of the Treasurer; while petitioner, a senior revenue collector, was an officer under him. Thus, the city treasurer is the proper disciplining authority who could investigate petitioner and issue a preventive suspension order against him. Likewise, the old Local Government Code does not vest in city mayors the sole power to discipline and to institute criminal or administrative actions against any officers or employees under their jurisdiction. In fact, there is no provision under the present Local Government Code expressly rescinding the authority of the Department of Finance to exercise disciplinary authority over its employees. By the same token, there is nothing that prohibits the city treasurer from filing a complaint against petitioner. Due process has not been violated. In an administrative proceeding, the essence of due process is simply the

Garcia v. Pajaro and the City of Dagupan


Sebastian Garcia is an employee at the City Treasurers Office, Dagupan City. He was ordered suspended by City Treasurer Juanito Pajaro and directed the withholding of his salary because of the Formal Charge filed against him. However, Pajaro continued reporting for work because he did not honor the suspension order as the City Treasurer acted as the complainant and that there was no complaint against him from the Office of the City Mayor. Juanito Pajaro, the City Treasurer of Dagupan City, claimed that Garcia has been rating unsatisfactory in his performance for several semesters, which is the reason why he was formally charged. Garcia was preventively suspended for ninety days since the charge is a major offense. An investigation was scheduled but Garcia failed to appear and testify. Garcia also did not answer the subpoena. So, Pajaro proceeded with an ex parte investigation. The Bureau of Local Government Finance favorably approved the suspension. This was affirmed by the Regional Director. Affirming the RTC Decision, the CA held that private respondent was vested with legal power and authority to institute disciplinary action against subordinate officers and employees. The appellate court further held that the requisites of administrative due process had been fully observed by Pajaro while investigating petitioner. But despite being informed of the charges against him and being given the opportunity to be heard in a formal investigation, petitioner chose not to answer those charges. HELD: Under the old and the present Local Government Codes, appointive officers and employees of local government units are covered by the Civil Service Law; and such rules, regulations and other issuances duly promulgated pursuant thereto, unless otherwise specified. Moreover, the investigation and the adjudication of administrative complaints against appointive local officials and employees, as well as their suspension and removal, shall be in accordance with the Civil Service Law and rules and other pertinent laws. The Administrative Code of 1987, -- specifically Book V on the civil service -- is the primary law governing appointive officials and employees in the government. They may be removed or dismissed summarily (1) [w]hen the charge is serious and the evidence of guilt is

135 D2010

Local Government
opportunity to explain ones side. Such process requires notice and an opportunity to be heard before judgment is rendered. One may be heard, not solely by verbal presentation in an oral argument, but also -- and perhaps even many times more creditably and practicably -through pleadings. So long as the parties are given the opportunity to explain their side, the requirements of due process are satisfactorily complied with. This constitutional mandate is deemed satisfied if a person is granted an opportunity to seek reconsideration of an action or a ruling. In the case at bar, the administrative proceedings were conducted in accordance with the procedure set out in the 1987 Administrative Code and other pertinent laws. First, petitioner was furnished a copy of the May 30, 1990 formal charge against him. Second, Pajaro requested the approval of the Order of Preventive Suspension in his June 1, 1990 letter addressed to the Bureau of Local Government Finance regional director, who approved the Order in the First Indorsement dated June 4, 1990. Third, a subpoena dated July 31, 1990 was issued to petitioner ordering him to testify during an investigation on August 15, 1990. However, he admittedly refused to attend the investigation; thus, it was conducted ex parte. Fourth, the Department of Finance affirmed Respondent Pajaros findings in its August 1, 1991 Decision.

D2010 136

UP College of Law

Recall
Sec 69-75, LGC
See above For the three-term limit for elective local government officials to apply, two conditions or requisites must concur, to wit: (1) that the official concerned has been elected for three (3) consecutive terms in the same local government post, and (2) that he has fully served three (3) consecutive terms. Here, Morales was elected for the term July 1, 1998 to June 30, 2001. He assumed the position. He served as mayor until June 30, 2001. He was mayor for the entire period notwithstanding the Decision of the RTC in the electoral protest case filed by petitioner Dee ousting him (respondent) as mayor. To reiterate, as held in Ong v. Alegre, such circumstance does not constitute an interruption in serving the full term. Section 8, Article X of the Constitution can not be more clear and explicit. Respondent Morales is now serving his fourth term. He has been mayor of Mabalacat continuously without any break since July 1, 1995. In just over a month, by June 30, 2007, he will have been mayor of Mabalacat for twelve (12) continuous years. This Court reiterates that the framers of the Constitution specifically included an exception to the peoples freedom to choose those who will govern them in order to avoid the evil of a single person accumulating excessive power over a particular territorial jurisdiction as a result of a prolonged stay in the same office. Morales maintains that he served his second term (1998 to 2001) only as a caretaker of the office or as a de facto officer. Section 8, Article X of the Constitution is violated and its purpose defeated when an official serves in the same position for three consecutive terms. Whether as caretaker or de facto officer, he exercises the powers and enjoys the prerequisites of the office which enables him to stay on indefinitely. Morales should be promptly ousted from the position of mayor of Mabalacat. Having found respondent Morales ineligible, his Certificate of Candidacy dated December 30, 2003 should be cancelled. In the light of the foregoing, Morales cannot be considered a candidate in the May 2004 elections. Not being a candidate, the votes cast for him SHOULD NOT BE COUNTED and must be considered stray votes. Since respondent Morales is DISQUALIFIED from continuing to serve as mayor of

RA 9244
See attachments This amends Sec 70 71 of the LGC

Rivera v. Comelec
In the May 2004 Synchronized National and Local Elections, Marino Morales ran as candidate for mayor of Mabalacat. On January 5, 2004, he filed his Certificate of Candidacy. On January 10, petitioners filed before the COMELEC a petition to cancel Morales certificate of candidacy on the ground that the was elected and had served three previous consecutive terms as mayor of Mabalacat contrary to RA 43(b) of RA 7160.Morales admitted that he was elected mayor of Mabalacat for the term commencing July 1, 1995 to June 30, 1998 (first term) and July 1, 2001 to June 30, 2004 (third term), but he served the second term from July 1, 1998 to June 30, 2001 only as a caretaker of the office or as a de facto officer because he was not validly elected as his proclamation as mayor was declared void by the RTC and thereafter, he was preventively suspended by the ombudsman. The COMELEC ruled that Morales was disqualified to run for public office. Morales MR was however granted. The COMELEC ruled that his proclamation before was void and that the discharge of the duties is that of a de facto mayor. In the other case filed by Anthony Dee: After Morales was proclaimed as the duly elected mayor, Anthony Dee filed a petition for quo warranto before the RTC. Dee reiterated the previous arguments of petitioners. The RTC dismissed Dees petition for quo warranto on the ground that Morales did not serve the three-term limit since he was not the duly elected mayor of Mabalacat, but Dee in the May 1998 elections for the term 1998 to 2001. Comelec affirmed. HELD: Morales is disqualified from running as mayor. In Ong v. Alegre the Court held, thus:

137 D2010

Local Government
Mabalacat, the instant petition for quo warranto has become moot. In Labo v. Comelec, this Court has ruled that a second place candidate cannot be proclaimed as a substitute winner. As a consequence of petitioners ineligibility, a permanent vacancy in the contested office has occurred. This should now be filled by the vice-mayor in accordance with Section 44 of the Local Government Code. and determining the outcome of such notice. Despite his urgent prayer for the issuance of a TRO, Evardone filed the petition for prohibition only on 10 July 1990. The Court issued a TRO on 12 July 1990 but the signing of the petition for recall took place just the same on the scheduled date through no fault of the COMELEC and Apelado. The signing process was undertaken by the constituents of the Municipality of Sulat and its Election Registrar in good faith and without knowledge of the TRO earlier issued by this Court. As attested by Election Registrar Sumbilla, about 2,050 of the 6,090 registered voters of Sulat, Eastern Samar or about 34% signed the petition for recall. The right to recall is complementary to the right to elect or appoint. It is included in the right of suffrage. It is based on the theory that the electorate must maintain a direct and elastic control over public functionaries. It is also predicated upon the idea that a public office is "burdened" with public interests and that the representatives of the people holding public offices are simply agents or servants of the people with definite powers and specific duties to perform and to follow if they wish to remain in their respective offices. Whether or not the electorate of Sulat has lost confidence in the incumbent mayor is a political question. It belongs to the realm of politics where only the people are the judge. Loss of confidence is the formal withdrawal by an electorate of their trust in a person's ability to discharge his office previously bestowed on him by the same electorate. The constituents have made a judgment and their will to recall Evardone has already been ascertained and must be afforded the highest respect. Thus, the signing process held last 14 July 1990 for the recall of Mayor Felipe P. Evardone of said municipality is valid and has legal effect. However, recall at this time is no longer possible because of the limitation provided in Sec. 55 (2) of B.P. Blg, 337. The Constitution has mandated a synchronized national and local election prior to 30 June 1992, or more specifically, as provided for in Article XVIII, Sec. 5 on the second Monday of May, 1992. Thus, to hold an election on recall approximately seven (7) months before the regular local election will be violative of the above provisions of the applicable Local Government Code

Evardone v. Comelec
Felipe Evardone the mayor of Sulat, Eastern Samar, having been elected to the position during the 1988 local elections. He assumed office immediately after proclamation. In 1990, Alexander R. Apelado, Victozino E. Aclan and Noel A. Nival filed a petition for the recall of Evardone with the Office of the Local Election Registrar, Municipality of Sulat. The Comelec issued a Resolution approving the the recommendation of Election Registrar Vedasto Sumbilla to hold the signing of petition for recall against Evardone. Evardone filed a petition for prohibition with urgent prayer of restraining order and/or writ of preliminary injunction. Later, in an en banc resolution, the Comelec nullified the signing process for being violative of the TRO of the court. Hence, this present petition. HELD: Article XVIII, Section 3 of the 1987 Constitution express provides that all existing laws not inconsistent with the 1987 Constitution shall remain operative, until amended, repealed or revoked. Republic Act No. 7160 providing for the Local Government Code of 1991, approved by the President on 10 October 1991, specifically repeals B.P. Blg. 337 as provided in Sec. 534, Title Four of said Act. But the Local Government Code of 1991 will take effect only on 1 January 1992 and therefore the old Local Government Code (B.P. Blg. 337) is still the law applicable to the present case. Prior to the enactment of the new Local Government Code, the effectiveness of B.P. Blg. 337 was expressly recognized in the proceedings of the 1986 Constitutional Commission. We therefore rule that Resolution No. 2272 promulgated by the COMELEC is valid and constitutional. Consequently, the the COMELEC had the authority to approve the petition for recall and set the date for the signing of said petition. In the present case, the records show that Evardone knew of the Notice of Recall filed by Apelado, on or about 21 February 1990 as evidenced by the Registry Return Receipt; yet, he was not vigilant in following up

Garcia v. Comelec
Enrique Garcia was elected governor of the province of Bataan. Some mayors, vice-mayors and members of the Sangguniang Bayan of the twelve (12) municipalities of the province constituted themselves into a Preparatory Recall Assembly to initiate the recall election of Garcia. The mayor of Mariveles, Honorable Oscar, de los Reyes,

D2010 138

UP College of Law
and the mayor of Dinalupihan, the Honorable Lucila Payumo, were chosen as Presiding Officer and Secretary of the Assembly, respectively. Thereafter, the ViceMayor of Limay, the Honorable Ruben Roque, was recognized and he moved that a resolution be passed for the recall of the petitioner on the ground of "loss of confidence." The motion was "unanimously seconded." Petitioners filed with the COMELEC a petition to deny due course to the Resolution for failure to comply with the requirements under the LGC, which dismissed the petition and scheduled the recall election. Petitioners filed a petition for certiorari and prohibition with the SC on the ground that section 70 of R.A. 7160 allowing recall through the initiative of the PRAC is unconstitutional because: (1) the people have the sole and exclusive right to decide whether or not to initiate proceedings, and (2) that the initiation of a recall through the PRA had de facto recalled Garcia from office and it effectively shortens and ends the term of the incumbent local officials and (3 )it violated the right of elected local public officials belonging to the political minority to equal protection of law. They also argued that the proceedings followed by the PRAC in passing Resolution No. I suffered from numerous defects, the most fatal of which was the deliberate failure to send notices of the meeting to sixtyfive (65) members of the assembly. The Court granted petition on the narrow ground that sending of selective notices to members of the PRAC violated the due process protection of the Constitution and fatally flawed the enactment of Resolution No. 1. Requirement of notice is mandatory and its non-observance is fatal to the validity of the resolution to recall Garcia as Governor. Petitioners then filed again Urgent Petition reiterating their contention that section 70 of RA 7160 is unconstitutional. HELD: Section 70 of R.A. 7160 allowing recall through the initiative of the PRAC is constitutional. As to the first contention, the Court stated that the legislative records reveal there were two (2) principal reasons why this alternative mode of initiating the recall process thru an assembly was adopted, viz: (a) to diminish the difficulty of initiating recall thru the direct action of the people; and (b) to cut down on its expenses. Our lawmakers took note of the undesirable fact that the mechanism initiating recall by direct action of the electorate was utilized only once in the City of Angeles, Pampanga, but even this lone attempt to recall the city mayor failed. This initiatory process by direct action of the people was too cumbersome, too expensive and almost impossible to implement. Consequently, our legislators added in the a second mode of initiating the recall of local officials thru a preparatory recall assembly. To be sure, there is nothing in the Constitution that will remotely suggest that the people have the "sole and exclusive right to decide on whether to initiate a recall proceeding." The Constitution did not provide for any mode, let alone a single mode, of initiating recall elections. Neither did it prohibit the adoption of multiple modes of initiating recall elections. The mandate given by section 3 of Article X of the Constitution is for Congress to "enact a local government code which shall provide for a more responsive and accountable local government structure through a system of decentralization with effective mechanisms of recall, initiative, and referendum . . ." By this constitutional mandate, Congress was clearly given the power to choose the effective mechanisms of recall as its discernment dictates. Using its constitutionally granted discretion, Congress deemed it wise to enact an alternative mode of initiating recall elections to supplement the former mode of initiation by direct action of the people. Congress has made its choice as called for by the Constitution and it is not the prerogative of this Court to supplant this judgment. The choice may be erroneous but even then, the remedy against a bad law is to seek its amendment or repeal by the legislative. By the principle of separation of powers, it is the legislative that determines the necessity, adequacy, wisdom and expediency of any law. As to the second contention, the Court held that Petitioners have misconstrued the nature of the initiatory process of recall by the PRAC. They have embraced the view that initiation by the PRAC is not initiation by the people. This is a misimpression for initiation by the PRAC is also initiation by the people, albeit done indirectly through their representatives. It is not constitutionally impermissible for the people to act through their elected representatives. Nothing less than the paramount task of drafting our Constitution is delegated by the people to their representatives, elected either to act as a constitutional convention or as a congressional constituent assembly. More far out is petitioners' stance that a PRA resolution of recall is the recall itself. It cannot be seriously doubted that a PRA resolution of recall merely, starts the process. It is part of the process but is not the whole process. This ought to be self evident for a PRA resolution of recall that is not submitted to the COMELEC for validation will not recall its subject official. Likewise, a PRA resolution of recall that is rejected by the people in the election called for the purpose bears no effect whatsoever. The initiatory resolution merely sets the stage for the official concerned to appear before the tribunal of the people so he can justify why he should be allowed to continue in office. Before the people render their sovereign

139 D2010

Local Government
judgment, the official concerned remains in office but his right to continue in office is subject to question. This is clear in section 72 of the Local Government Code which states that "the recall of an elective local official shall be effective only upon the election and proclamation of a successor in the person of the candidate receiving the highest number of votes cast during the election on recall." As to the third contention, the fear is expressed that the members of the PRAC may inject political color in their decision as they may initiate recall proceedings only against their political opponents especially those belonging to the minority. A careful reading of the law, however, will ineluctably show that it does not give an asymmetrical treatment to locally elected officials belonging to the political minority. First to be considered is the politically neutral composition of the preparatory recall assembly, all mayors, vice-mayors and sangguniang members of the municipalities and component cities are made members of the preparatory recall assembly at the provincial level. Its membership is not apportioned to political parties. No significance is given to the political affiliation of its members. Secondly, the preparatory recall assembly, at the provincial level includes all the elected officials in the province concerned. Considering their number, the greater probability is that no one political party can control its majority. Thirdly, sec. 69 of the Code provides that the only ground to recall a locally elected public official is loss of confidence of the people. The members of the PRAC are in the PRAC not in representation of their political parties but as representatives of the people. By necessary implication, loss of confidence cannot be premised on mere differences in political party affiliation. There is only one ground for the recall of local government officials: loss of confidence. This means that the people may petition or the Preparatory Recall Assembly may resolve to recall any local elective officials without specifying any particular ground except loss of confidence. There is no need for them to bring up any charge of abuse or corruption against the local elective officials who are the subject of any recall petition. provided by law. To prevent the holding of the recall election, petitioner filed before the RTC petition for injunction. After conducting a summary hearing, the trial court lifted the restraining order, dismissed the petition and required petitioner and his counsel to explain why they should not be cited for contempt for misrepresenting that the barangay recall election was without COMELEC approval. The Comelec again re-scheduled the recall election, hence the instant petition for certiorari with urgent prayer for injunction the issue being WON the recall election to be held on January 13, 1996 is barred by the SK election to be held on May 1996. HELD: The issue on recall has become moot and academic. But the Court held that it would be more in keeping with the intent of the recall provision of the Code to construe regular local election as one referring to an election where the office held by the local elective official sought to be recalled will be contested and be filled by the electorate. Petitioner's argument is simple and to the point. Citing Section 74 (b) of Republic Act No. 7160, otherwise known as the Local Government Code, which states that "no recall shall take place within one (1) year from the date of the official's assumption to office or one (1) year immediately preceding a regular local election", petitioner insists that the scheduled January 13, 1996 recall election is now barred as the Sangguniang Kabataan (SK) election was set by Republic Act No. 7808 on the first Monday of May 1996, and every three years thereafter. The evident intent of Section 74 is to subject an elective local official to recall election once during his term of office. Paragraph (b) construed together with paragraph (a) merely designates the period when such elective local official may be subject of a recall election, that is, during the second year of his term of office. Thus, subscribing to petitioner's interpretation of the phrase regular local election to include the SK election will unduly circumscribe the novel provision of the Local Government Code on recall, a mode of removal of public officers by initiation of the people before the end of his term. And if the SK election which is set by R.A No. 7808 to be held every three years from May 1996 were to be deemed within the purview of the phrase "regular local election", as erroneously insisted by petitioner, then no recall election can be conducted rendering inutile the recall provision of the LGC. In the interpretation of a statute, the Court should start with the assumption that the legislature intended to enact an effective law, and the legislature is not presumed to have done a vain thing

Paras v. Comelec
Danilo E. Paras is the incumbent Punong Barangay of Pula, Cabanatuan City. A petition for his recall as Punong Barangay was filed by the registered voters of the barangay. The COMELEC scheduled the petition signing on October 14, 1995, and set the recall election on November 13,1995. At least 29.30% of the registered voters signed the petition, above the 25% requirement

D2010 140

UP College of Law
in the enactment of a statute. An interpretation should, if possible, be avoided under which a statute or provision being construed is defeated, or as otherwise expressed, nullified, destroyed, emasculated, repealed, explained away, or rendered insignificant, meaningless, inoperative or nugatory. It is likewise a basic precept in statutory construction that a statute should be interpreted in harmony with the Constitution. Thus, the interpretation of Section 74 of the Local Government Code, specifically paragraph (b) thereof, should not be in conflict with the Constitutional mandate of Section 3 of Article X of the Constitution to "enact a local government code which shall provide for a more responsive and accountable local government structure instituted through a system of decentralization with effective mechanism of recall, initiative, and referendum . . . ." Finally, recall election is potentially disruptive of the normal working of the local government unit necessitating additional expenses, hence the prohibition against the conduct of recall election one year immediately preceding the regular local election. The proscription is due to the proximity of the next regular election for the office of the local elective official concerned. The electorate could choose the official's replacement in the said election who certainly has a longer tenure in office than a successor elected through a recall election. It would, therefore, be more in keeping with the intent of the recall provision of the Code to construe regular local election as one referring to an election where the office held by the local elective official sought to be recalled will be contested and be filled by the electorate. Nevertheless, recall at this time is no longer possible because of the limitation stated under Section 74 (b) of the Code considering that the next regular election involving the barangay office concerned is barely seven (7) months away, the same having been scheduled on May 1997. the institution and proceedings of the recall, putting to fore the propriety of the service of notices to the members of the Preparatory Recall Assembly, and the proceedings held, resulting in the issuance of the questioned Resolution. HELD: The Court ruled that notices were properly sent to the members of the PRA and that the proceedings held by the PRA are valid. The Commission regards the sending of notice one thing, and the completion of service thereof another, for indeed, the requirement of notice can only be fully satisfied, if there was not only service, but also completion of service thereof. Personal services were acknowledged by receipts signed, if not by the addressee himself, then, as indicated thereon, by his or her spouse, nearest relative or a person of sufficient discretion in the member 's residence or office. Service by registered mail was evinced by the return card duly signed by the addressee or by persons acting for him. There were instances when notices were served but were refused, this fact noted in the acknowledgment receipt by the server and his witnesses. The circumstances being thus, it was held that there was complete service of the notices as contemplated in Section 8, Rule 13 of the Rules of Court. That it was Alex David, President of the LIGA ng mga Barangay who sent the notices is of no moment. As member of the PRA, he can legally exercise the prerogatives attached to his membership in the Preparatory Recall Assembly, sending notices to the other members of its scheduled convening. It is evident from the foregoing and, therefore, the Commission so holds that the requirements of notice had been fully complied with. Needless to state, the issue of propriety of the notices sent to the PRA members is factual in nature, and the determination of the same is therefore a function of the COMELEC. In the absence of patent error, or serious inconsistencies in the findings, the Court should not disturb the same. Petitioner's insistence, that the initiation of the recall proceedings was infirm since it was convened by the Liga ng mga Barangays, is misplaced. Petitioner observes that "respondent Liga is an organization of all barangays. It is not an organization of barangay captains and kagawads. The barangays are represented in the Liga by the barangay captains as provided under Section 492 LGC. It also provides that the Kagawad may represent the barangay in the absence of the barangay chairman." The Liga ng mga Barangay is undoubtedly an entity distinct from the Preparatory Recall Assembly. It just so happens that the personalities representing the barangays in the Liga are the very members of the Preparatory Recall

Malonzo v. Comelec
Petitioner was duly elected as Mayor, winning over former Mayor Macario Asistio, Jr. Barely one year into his term, 1,057 Punong Barangays and Sangguniang Barangay members and SK chairmen, constituting a majority of the members of the Preparatory Recall Assembly of the City of Caloocan, met, and upon deliberation and election, voted for the approval of Preparatory Recall Assembly Resolution No. 01-96, expressing loss of confidence in Mayor Malonzo, and calling for the initiation of recall proceedings against him. The Comelec declared the recall proceedings to be in order. Mayor Malonzo filed a petition for certiorari with a prayer for TRO assailing the Comelecs resolution. The Petition, in the main, raises the issue of the validity of

141 D2010

Local Government
Assembly, the majority of whom met on July 7, 1996, and voted in favor of the resolution calling for the recall of Mayor Malonzo, after deliberation reported in the record, in accordance with the existing law. Thus, the Punong Barangays and Sangguniang Barangay members convened and voted as members of the Preparatory Recall Assembly of Caloocan, and not as members of the Liga ng mga Barangay. The recall proceedings, therefore, cannot be denied merit on this ground. Any doubt as to the propriety of the proceedings held during the recall assembly should be laid to rest. As the COMELEC pertinently observes: The Minutes of the session of the Preparatory Assembly indicated that there was a session held. Attendees constitute the majority of all the members of the Preparatory Assembly, as we shall later on establish. Rules of procedure, simple they may be were formulated. Deliberations were conducted on the main issue, which was that of petitioner's recall. The members were given the opportunity to articulate on their resolve about the matter. More importantly, their sentiments were expressed through their votes signified by their signatures and thumbmarks affixed to the Resolution. No proof was adduced by Petitioner to substantiate his claim that the signatures appearing thereon represented a cause other than that of adopting the resolution. The charges of graft and corruption, violence and irregularities, before and during the session of the preparatory recall assembly are largely uncorroborated, and cannot override the substantiated findings of the respondent COMELEC. took place within the one-year prohibited period; (4) the election case, filed by Wenceslao Trinidad in this Court, seeking the annulment of the proclamation of petitioner Claudio as mayor of Pasay City, should first be decided before recall proceedings against petitioner could be filed; and (5) the recall resolution failed to obtain the majority of all the members of the PRA, considering that 10 were actually double entries, were not duly accredited members of the barangays, 40 Sangguniang Kabataan officials had withdrawn their support, and 60 barangay chairs executed affidavits of retraction. The COMELEC granted the petition and dismissed the opposition. It ruled that the 1,073 members who attended the May 29, 1999 meeting were more than necessary to constitute the PRA, considering that its records showed the total membership of the PRA was 1,790, while the statistics of the Department of Interior and Local Government (DILG) showed that the total membership of the PRA was 1,876. In either case, since only a majority is required to constitute the PRA, clearly, a majority had been obtained in support of the recall resolution. Hence, this petition. WON the word "Recall" in Paragraph (b) of 74 of the Local Government Code Includes the Convening of the Preparatory Recall Assembly and the Filing by it of a Recall Resolution Held: Yes. Recall is a process which begins with the convening of the preparatory, recall assembly or the gathering of the signatures at least 25% of the registered voters of a local government unit, and then proceeds to the filing of a recall resolution or petition with the COMELEC, the verification of such resolution or petition, the fixing of the date of the recall election, and the holding of the election on the scheduled date. However, as used in paragraph (b) of 74, "recall" refers to the election itself by means of which voters decide whether they should retain their local official or elect his replacement. Several reasons can be cited in support of this conclusion. First, 74 deals with restrictions on the power of recall. It is in fact entitled "Limitations on Recall." On the other hand, 69 provides that "the power of recall ...shall be exercised by the registered voters of a local government unit to which the local elective official belongs." Since the power vested on the electorate is not the power to initiate recall proceedings but the power to elect an official into office, the limitations in 74 cannot be deemed to apply to the entire recall proceedings. In other words, the term "recall" in paragraph (b) refers

Claudio v. Comelec
Jovito Claudio was the duly elected mayor of Pasay City in the May 11, 1998 elections. On May 19, 1999, several barangay chairs formed an ad hoc committee for the purpose of convening the PRA. Richard Advincula was designated chair. The members of the PRA adopted Resolution No. 01, S-1999, initiating Claudios recall. The petition for recall was filed in the Office of the City Mayor. The COMELEC also posted the petition on the bulletin boards of certain public places. Oppositions to the petition were filed by Jovito Claudio, Rev. Ronald Langub, and Roberto L. Angeles, alleging procedural and substantive defects in the petition, to wit: (1) the signatures affixed to the resolution were actually meant to show attendance at the PRA meeting; (2) most of the signatories were only representatives of the parties concerned who were sent there merely to observe the proceedings; (3) the convening of the PRA

D2010 142

UP College of Law
only to the recall election, excluding the convening of the PRA and the filing of a petition for recall with the COMELEC, or the gathering of the signatures of at least 25 % of the voters for a petition for recall. Thus, there may be several PRAs held or petitions for recall filed with the COMELEC - there is no legal limit on the number of times such processes may be resorted to. These are merely preliminary steps for the purpose of initiating a recall. The limitations in 74 apply only to the exercise of the power of recall which is vested in the registered voters. It is this - and not merely, the preliminary steps required to be taken to initiate a recall - which paragraph (b) of 74 seeks to limit by providing that no recall shall take place within one year from the date of assumption of office of an elective local official. The second reason why the term "recall" in paragraph (b) refers to recall election is to be found in the purpose of the limitation itself. There are two limitations in paragraph (b) on the holding of recalls: (1) that no recall shall take place within one year from the date of assumption of office of the official concerned, and (2) that no recall shall take place within one year immediately preceding a regular local election. The purpose of the first limitation is to provide a reasonable basis for judging the performance of an "The only logical reason which we can ascribe for requiring the electors to wait one year before petitioning for a recall election is to prevent premature action on their part in voting to remove a newly elected official before having had sufficient time to evaluate the soundness of his policies and decisions." The one-year limitation was reckoned as of the filing of a petition for recall because the Municipal Code involved in that case expressly provided that "no removal petition shall be filed against any officer or until he has actually held office for at least twelve months." But however the period of prohibition is determined, the principle announced is that the purpose of the limitation is to provide a reasonable basis for evaluating the performance of an elective local official. Hence, in this case, as long as the election is held outside the one-year period, the preliminary proceedings to initiate a recall can be held even before the end of the first year in office of a local official. Third, to construe the term "recall" in paragraph (b) as including the convening of the PRA for the purpose of discussing the performance in office of elective local officials would be to unduly restrict the constitutional right of speech and of assembly of its members. The people cannot just be asked on the day of the election to decide on the performance of their officials. The crystallization and formation of an informed public opinion takes time. To hold, therefore, that the first limitation in paragraph (b) includes the holding of assemblies for the exchange of ideas and opinions among citizens is to unduly curtail one of the most cherished rights in a free society. Indeed, it is wrong to assume that such assemblies will always eventuate in a recall election. To the contrary, they may result in the expression of confidence in the incumbent. To sum up, the term "recall" in paragraph (b) refers to the recall election and not to the preliminary proceedings to initiate recall 1. 1. Because 74 speaks of limitations on "recall" which, according to 69, is a power which shall be exercised by the registered voters of a local government unit. Since the voters do not exercise such right except in an election, it is clear that the initiation of recall proceedings is not prohibited within the one-year period provided in paragraph (b); 2. Because the purpose of the first limitation in paragraph (b) is to provide voters a sufficient basis for judging an elective local official, and final judging is not done until the day of the election; and 3. Because to construe the limitation in paragraph (b) as including the initiation of recall proceedings would unduly curtail freedom of speech and of assembly guaranteed in the Constitution. As the recall election in Pasay City is set on April 15, 2000, more than one year after petitioner assumed office as mayor of that city, we hold that there is no bar to its holding on that date. WON the Phrase "Regular Local Election" in the Same Paragraph (b) of 74 of the LGC includes the election period or campaign period? Held: No. The law is unambiguous in providing that "[n]o recall shall take place within . . . one (1) year immediately preceding a regular local election." Had Congress intended this limitation to refer to the campaign period, which period is defined in the Omnibus Election Code, it could have expressly said so. Petitioner's interpretation would severely limit the period during which a recall election may be held. Actually, because no recall election may be held until one year after the assumption of office of an elective local official, presumably on June 30 following his election, the free period is only the period

143 D2010

Local Government
from July 1 of the following year to about the middle of May of the succeeding year. This is a period of only nine months and 15 days, more or less. To construe the second limitation in paragraph (b) as including the campaign period would reduce this period to eight months. Such an interpretation must be rejected, because it would devitalize the right of recall which is designed to make local government units" more responsive and accountable." Indeed, there is a distinction between election period and campaign period. Under the Omnibus Election Code, unless otherwise fixed by the COMELEC, the election period commences ninety (90) days before the day of the election and ends thirty (30) days thereafter. Thus, to follow petitioner's interpretation that the second limitation in paragraph (b) includes the "election period" would emasculate even more a vital right of the people. WON the Recall RESOLUTION was Signed by a Majority of the PRA and Duly Verified Held: Yes. Petitioner contends that a majority of the signatures of the members of the PRA was not obtained because 74 members did not really sign the recall resolution. According to petitioner, the 74 merely signed their names on pages 94-104 of the resolution to signify their attendance and not their concurrence. Petitioner claims that this is shown by the word "Attendance" written by hand at the top of the page on which the signatures of the 74 begin. This contention has no basis. To be sure, this claim is being raised for the first time in this case. It was not raised before the COMELEC. Although the word "Attendance" appears at the top of the page, it is apparent that it was written by mistake because it was crossed out by two parallel lines drawn across it. Apparently, it was mistaken for the attendance sheet which is a separate document. It is absurd to believe that the 74 members of the PRA who signed the recall resolution signified their attendance at the meeting twice. It is more probable to believe that they signed pages 94-104 to signify their concurrence in the recall resolution of which the pages in question are part. The other point raised by petitioner is that the recall petition filed in the COMELEC was not duly verified, because Atty. Nelson Ng, who notarized it, is not commissioned as notary public for Pasay City but for Makati City. As in the case of the first claim, this issue was not raised before the COMELEC itself. It cannot, therefore, be raised now.

D2010 144

UP College of Law

Human Resources and Development


Book I, Title 3, LGC (Sec 76 97)
For Sec 84 89, see above Limitation on appointment (Sec 79) No person shall be appointed in the career service of the local government if he is related within the fourth civil degree of consanguinity or affinity to the appointing or recommending authority. Public notice of vacancy; Personnel Selection Board (Sec 80) Requirement in filling up a vacant career position: Posting of notice of the vacancy in at least three (3) conspicuous public places in the LGU concerned for a period of not less than fifteen (15) days. Personnel Selection Board Functions: 1. To assist the local chief executive in the judicious and objective selection of personnel for employment and for promotion; To assist the local chief executive in the formulation of policies that would contribute to employee welfare.

Title III Human Resources and Development Organization structure and staffing pattern (Sec 76) In designing and implementing the organizational structure and staffing pattern by the LGU the following factors shall be considered: 1. 2. Service Requirements of the LGU; Financial Capability of the LGU Subject to the guidelines prescribed by the Civil Service Commission

Responsibility for human resources and development (Sec 77) The local chief executive of every LGU shall be responsible for human resources and development of the said LGU. The local chief executive shall take all personnel actions in accordance with the Constitution, pertinent laws, rules and regulations including such policies, guidelines and standards as may be established by the Civil Service Commission. The local chief executive may employ emergency or casual employees or laborers paid on a daily wage, piecework, or per job order basis for local projects authorized by the Sangguniang concerned, without the need for approval or attestation by the Civil Service Commission BUT the period of employment of these workers SHALL NOT exceed six (6) months. Civil Service Law, rules and regulations, and other related issuances (Sec 78) General rule: All matters pertaining to human resources and development in LGUs shall be governed by the Civil Service Law and such rules and regulations and other issuances promulgated pursuant thereto. Exception: When specified otherwise by the Local Government Code.

2.

Composition: Headed by the Local Chief Executive; Members shall be determined by resolution of the sanggunian concerned; A representative of the Civil Service Commission as an ex-officio member; The personnel officer of the LGU also as an exofficio member.

Compensation of local officials and employees (Sec 81) General rule: It shall be determined by the sanggunian of their respective LGU. Limitations of the rule: 1. The increase in compensation of elective local officials shall take effect only after the terms of office of those approving such increase shall have expired;

145 D2010

Local Government
2. The increase in the compensation of the appointive officials and employees shall take effect as provided in the ordinance authorizing the such increase; The increases as provided shall not exceed the limitations on budgetary allocations for personal services provided under Title 5, Book 2 of LGC; The compensation as provided may be based upon the pertinent provisions of R.A. 6758. session of the sanggunian concerned and duly entered in its records. Exceptions: 1. 2. The above rule does not apply to sanggunian members subject to recall election; The same does not apply in cases where existing laws prescribe the manner of acting upon such resignations.

3.

4.

Exception: The punong barangay, the sangguniang barangay members, the SK chairman, the barangay treasurer, and the barangay secretary shall be entitled to such compensation, allowances, emoluments, and such other privileges as provided under Title 1, Book 3 of LGC. (see Sec. 393, LGC) Elective local officials shall be entitled to the same leave privileges as those enjoyed by appointive local officials, including the cumulation and commutation thereof. Resignation of elective local officials (Sec 82) Resignation by elective local officials shall be deemed effective only upon acceptance of the proper authorities as shown below: Officials tendering resignation Governors, vice-governors, and mayors and vicemayors of highly urbanized cities and independent component cities Municipal mayors, municipal vice-mayors, city mayors and city vicemayors of component cities Sanggunian member Barangay officials Proper Authority to accept the resignation President of the Philippines

Grievance procedure (Sec 83) The local chief executive, of every LGU shall establish a procedure to inquire into, act upon, resolve or settle complaints and grievances presented by local government employees. Practice of profession (Sec 90) Governors, city and municipal mayors are prohibited from practicing their profession or engaging in any business other than the exercise of their function as local chief executives. Sanggunian members may practice their professions, engage in any occupation, or teach in schools. Exception: During session hours. Exception to the exception: Doctors of medicine may practice their profession even during official hours of work ONLY on occasion of emergency PROVIDED; they do not derive any monetary compensation therefrom. Special rules for Sanggunian members who are also lawyers: 1. They shall not appear as counsel before any court in any civil case wherein an LGU or any office, agency, or instrumentality of the government is the adverse party; They shall not appear as counsel in any criminal case wherein an officer or employee of the national or local government is accused of an offense committed in relation to his office; They shall not collect any fee for their appearance in administrative proceedings involving the local government unit of which he is an official; They shall not use property and personnel of the Government except when they are defending the interest of the Government.

Governor Sanggunian to which he/she belongs City or municipal mayor

2.

Copies of the resignation letters of elective local officials, together with the action taken by the proper authorities accepting the resignation shall be furnished the DILG. The resignation shall be deemed accepted if not acted upon by the authority concerned within fifteen (15) working days from receipt thereof. Irrevocable resignations by sanggunian members shall be deemed accepted upon presentation before an open

3.

4.

Statement of assets and liabilities (Sec 91)

D2010 146

UP College of Law
Officials and employees of LGUs shall file their: 1. 2. 3. 4. Sworn statements of assets, liabilities and net worth Lists of relatives within the fourth civil degree of consanguinity or affinity in government service; Financial and business interests; Personal data sheets as required by law. No elective or appointive local official shall hold any other office or employment in the Government or any subdivision, agency or instrumentality including government owned or controlled corporations or their subsidiaries. Exception: When otherwise allowed by law or by the primary functions of his position (Ex officio positions) No candidate who lost in any election shall, within one (1) year after such election, be appointed to any office in the Government or any government owned or controlled corporations or their subsidiaries. Exception: Losers in the barangay elections. The oath or affirmation shall be filed with the office of the local chief executive concerned. A copy of the oath or affirmation of office of all elective and appointive local officials and employees shall be preserved in the individual personal records file under the custody of the personnel office, division, or section of the LGU concerned. Partisan political activity (Sec 93) No local official or career civil service employee shall: 1. Engage directly or indirectly in any partisan political activity or take part in any election, initiative, referendum, plebiscite, or recall EXCEPT to vote; Use his official authority or influence to cause the performance of any political activity by any person or body. Additional or double compensation (Sec 95) No elective or appointive local official or employee shall receive additional, double, or indirect compensation. Exception: When specifically allowed by law. No elective or appointive local official or employee shall accept any present, emoluments, office, or title of any kind from any foreign government. Exception: When Congress gives consent. Pension and gratuities shall not be considered as additional, double, or indirect compensation. Permission to leave station (sec 96) Provincial, city, municipal, and barangay appointive officials going on OFFICIAL TRAVEL shall apply and secure written permission from their respective local chief executives before departure. The application (for permission to leave station) shall specify the reason for the travel. The permission shall be given or withheld based on: 1. 2. 3. considerations of pubic interest; financial capability of the local government unit concerned; and urgency of the travel.

Oath of office (Sec 92) All elective and appointive local officials and employees shall, upon assumption to office, subscribe to an oath or affirmation of office in the prescribed form.

2.

Local officials or career civil service employees MAY express his views on current issues, or mention the names of certain candidates for public office whom he supports. Elective local officials may take part in partisan political and electoral activities BUT it shall be unlawful for them to solicit contributions from their subordinates or subject these subordinates to any of the prohibited acts under the Omnibus Election Code. Appointment of elective and appointive local officials; candidates who lost in an election (Sec 94) No elective of appointive local official shall be eligible for appointment or designation in any capacity to any public office or position during his tenure.

Should the local chief executive concerned fail to act upon the application within four (4) working days from receipt thereof, it shall be deemed APPROVED. Mayors of component cities and municipalities shall secure the permission of the governor concerned for any travel outside the province.

147 D2010

Local Government
Local government officials TRAVELING ABROAD shall notify their respective sanggunian PROVIDED that when the period of travel extends to MORE THAN THREE (3) MONTHS, during periods of emergency or crisis or when the travel involves the use of public funds, permission from the Office of the President shall be secured. Field officers of national agencies or offices assigned in provinces, cities, municipalities shall not leave their official stations without giving PRIOR WRITTEN NOTICE to the local chief executive concerned. The written notice shall state the duration of the travel and the name of the officer whom he shall designate to act for and in his behalf during his absence. Annual report (Sec 97) On or before March 31 of every year, every local chief executive shall submit an annual report to the sanggunian concerned on the socio-economic, political and peace and order conditions, and other matters concerning the local government unit, which shall cover the immediately preceding calendar year. A copy of the report shall be forwarded to the DILG. Component cities and municipalities shall likewise provide the sangguniang panlalawigan copies of their respective annual reports. I. Practice of Profession by Mayors, Governors and other elective officials appeared as counsel in several criminal and civil cases in the city, without prior authority of the DLG Regional Director, in violation of DLG Memorandum Circular No. 80-38. Javellana filed a Motion to Dismiss the administrative case against him on the ground mainly that DLG Memorandum Circular Nos. 80-38 and 90-81 are unconstitutional because the Supreme Court has the sole and exclusive authority to regulate the practice of law. The motion to dismiss was denied. A few months later, the LGC was enacted which provides: "SEC. 90. Practice of Profession. "(b) Sanggunian members may practice their professions, engage in any occupation, or teach in schools except during session hours: Provided, That sanggunian members who are also members of the Bar shall not: "(1) Appear as counsel before any court in any civil case wherein a local government unit or any office, agency, or instrumentality of the government is the adverse party; "(2) Appear as counsel in any criminal case wherein an officer or employee of the national or local government is accused of an offense committed in relation to his office: "(3) Collect any fee for their appearance in administrative proceedings involving the local government unit of which he is an official; and "(4) Use property and personnel of the Government except when the sanggunian member concerned is defending the interest of the Government. Javellana thereupon filed this petition for certiorari praying that DLG Memorandum Circulars Nos. 80-38 and 90-81 and Section 90 of RA 7160 be declared unconstitutional and null and void. HELD: The Memorandum Circulars and Section 90 of RA 7160 are constitutional. In the first place, complaints against public officers and employees relating or incidental to the performance of their duties are necessarily impressed with public interest for by express constitutional mandate, a public office is a public trust. The complaint for illegal dismissal filed by Javiero and Catapang against City Engineer Divinagracia is in effect a complaint against the City Government of Bago City, their real employer, of which petitioner Javellana is a councilman. Hence, judgment against City Engineer

Sec 90, LGC


See above

Javellana v. DILG
Attorney Erwin B. Javellana was an elected City Council or of Bago City, Negros Occidental. City Engineer Ernesto C. Divinagracia filed Administrative Case against Javellana. Divinagracia's complaint alleged that Javellana has continuously engaged in the practice of law without securing authority for that purpose from the Regional Director, Department of Local Government, as required by DLG Memorandum Circular No. 80-38 in relation to DLG Memorandum Circular No. 74-58 of the same department: that on July 8, 1989, Javellana, as counsel for Antonio Javiero and Rolando Catapang, filed a case against City Engineer Ernesto C. Divinagracia of Bago City for "Illegal Dismissal and Reinstatement with Damages" putting him in public ridicule: that Javellana also

D2010 148

UP College of Law
Divinagracia, would actually be a judgment against the City Government. By serving as counsel for the complaining employees and assisting them to prosecute their claims against City Engineer Divinagracia, the petitioner violated Memorandum Circular No. 74-58 (in relation to Section 7[b-2] of RA 6713) prohibiting a government official from engaging in the private practice of his profession, if such practice would represent interests adverse to the government. Petitioner's contention that Section 90 of the Local Government Code of 1991 and DLG Memorandum Circular No. 90-81 violate Article VIII. Section 5 of the Constitution is completely off tangent. Neither the statute nor the circular trenches upon the Supreme Court's power and authority to prescribe rules on the practice of law. The Local Government Code and DLG Memorandum Circular No. 90-81 simply prescribe rules of conduct for public officials to avoid conflicts of interest between the discharge of their public duties and the private practice of their profession, in those instances where the law allows it. Section 90 of the Local Government Code does not discriminate against lawyers and doctors. It applies to all provincial and municipal officials in the professions or engaged in any occupation. Section 90 explicitly provides that Sanggunian Members "may practice their professions, engage in any occupation, or teach in schools except during session hours." If there are some prohibitions that apply particularly to lawyers, it is because of all the professions, the practice of law is more likely than others to relate to, or affect, the area of public service. II. Civil Service Law, Rules and Regulations; Limitations on Appointments; Resignation of Elective Local Officials; Preventive Suspension of Local Officials and Employees; Administrative Investigation; Disciplinary Jurisdiction; Additional or Double Compensation

149 D2010

Local Government

Local Boards and Councils


Book I, Titles 4-7 (Sec 98 116)
Title IV Local School Boards 4. There shall be established in every province, city, or municipality a provincial, city or municipal school board respectively. Composition i. Provincial School Board 1. 2. 1. Provincial Governor Provincial Division Schools Superintendent Chairman of Education Committee in the Sangguniang Panlalawigan Provincial Treasurer SK Federation Representative in the Sangguniang Panlalawigan President of the Provincial Federation of PTA Representative of the teachers organization in the province Representative of the non-academic personnel of public schools in the province 5. Representative in the Sangguniang Panlungsod President of the City Federation of PTA Representative of the teachers organization in the city Representative of the non-academic personnel of public schools in the city

Co - Chairmen

Members

iii.

Municipal School Board 1. 2. 1. Municipal Mayor District Supervisor Chairman of Education Committee in the Sangguniang Bayan Municipal Treasurer SK Federation Representative in the Sangguniang Bayan President of the Municipal Federation of PTA Representative of the teachers organization in the municipality Representative of the non-academic personnel of public schools in the municipality

Co Chairmen Members

2. 3.

4.

2. 3.

5.

4.

6.

5.

ii.

City School Board 1. 2. City Mayor City Division Schools Superintendent Chairman of Education Committee in the Sangguniang Panlungsod City Treasurer SK Federation

Co Chairmen

Members

1.

In the event that a province or a city has two (2) or more school superintendents, and in the event that a municipality has two (2) or more district supervisors, the co-chairmen of the local school board shall be determined as follows: 1. The DepEd shall designate the co-chairman for the provincial or city schools board; and

2. 3.

D2010 150

UP College of Law
2. The Schools Division Superintendent shall designate the district supervisor who shall serve as co-chairman as in the municipal school board The division schools superintendent, city schools superintendent or district supervisor, as the case may be, shall prepare the budget of the school board concerned. Such budget shall be supported by programs, projects, and activities of the school board for the ensuing fiscal year. The affirmative vote of the majority of all the members shall be necessary to approve the budget. Priorities in the school board budget 1. Construction, repair, and maintenance of school buildings and other facilities of public elementary and secondary schools; Establishment and maintenance of extension classes where necessary; and Sports activities at the division, district, municipal, and barangay levels.

The performance of the duties and responsibilities of the abovementioned officials in their respective school boards shall no be delegated. Functions of a local school board 1. To determine in accordance with the criteria set by the DepEd, the annual supplementary budgetary needs for the operation and maintenance of public schools within the province, city, or municipality, as the case may be, and the supplementary local costs of meeting such needs, which shall be reflected in the form of an annual school board budget corresponding to its share of the proceeds of the special levy on real property constituting the Special Education Fund and such other sources of revenue as this Code and other laws or ordinances may provide; To authorize the provincial, city or municipal treasurer, as the case may be, to disburse funds from the Special Education Fund pursuant to the budget prepared and in accordance with existing rules and regulations; To serve as an advisory committee to the sanggunian concerned on educational matters such as, but not limited to, the necessity for and the uses of local appropriations for educational purposes; and To recommend changes in the names of public schools within the territorial jurisdiction of the local government unit for enactment by the sanggunian concerned.

2. 3.

Title V Local Health Boards There shall be established a local health boards in every province, city, or municipality. Composition: i. Provincial Health Board Provincial Governor Provincial health officer 1. Chairman of the committee on health of the sangguniang panlalawigan 2. Representative from Private sector or NGO involved in health services 3. Representative from the DOH in the province

2.

3.

Chairman Vice Chairman

4.

Members

The DepEd shall consult the local school board on the appointment of division superintendents, district supervisors, school principals, and other officials. Meetings and quorum ii. The local school board shall meet at least once a month or as often as may be necessary. Any of the co-chairmen may call a meeting. A majority of all its members shall constitute a quorum. However, when both co-chairmen are present in a meeting, the local chief executive concerned, as a matter of protocol, shall be given preference to preside over the meeting. Chairman Vice Chairman

City Health Board City Mayor City Health Officer 1. Chairman of the committee on health of the Sangguniang panlalawigan 2. Representative from Private sector or NGO involved in health

Members

151 D2010

Local Government
services Representative from DOH in the province A majority of the members of the board shall constitute a quorum but the chairman or the vice-chairman must be present during meetings where budgetary proposals are being prepared or considered. The affirmative vote of all the majority of the members shall be necessary to approve such in proposals. Compensation and remuneration General Rule: The chairman, vice-chairman and members of the local health board shall perform their duties without compensation or remuneration. Exception: Members of the local health board who are not government officials or employees shall be entitled to necessary traveling expenses and allowances chargeable against the funds of the local health board concerned, subject to the existing accounting and auditing rules and regulations. Direct national supervision and control by the Secretary of Health (Sec 105) Functions of a local health board 1. To propose to the sanggunian concerned, in accordance with standards and criteria set by the DOH, annual budgetary allocations for the operation and maintenance of health facilities and services within the municipality, city, or province, as the case may be; To serve as an advisory committee to the sanggunian concerned on health matters such as, but not limited to, the necessity for, and application of, local appropriations for public health purposes; and To create committees, consistent with the technical standards and administrative standards of the DOH, which shall advise local health agencies on matters such as but not limited to: a. personnel selection and promotion b. bids and awards c. grievance and complaints d. personnel discipline e. budget review f. operations review g. other similar functions. General Rule: The local health matters in a particular LGU shall be directly controlled and supervised by the local health board. Exception: In cases of epidemics, pestilence, and other widespread public health dangers, the Secretary of Health may, upon the direction of the President and in consultation with the LGU concerned, temporarily assume direct supervision and control over health operations in any LGU for the duration of the emergency, but in no case exceeding six (6) months. With the concurrence of the LGU concerned, the period for such direct supervision and control may be further extended. Title VI Local Development Councils Section 106. Local Development Councils. Each local government unit shall have a comprehensive multisectoral development plan to be initiated by its development council and approved by its sanggunian. For this purpose, the development council at the provincial, city, municipal or barangay level, shall assist the corresponding sanggunian in setting the direction of economic and social development, and coordinating development efforts within its territorial jurisdiction. Comments: The Code has established a local development council for every province, city, municipality and barangay. Its primary duty is to initiate a comprehensive multisectoral development plan for the

3.

iii.

Municipal Health Board Municipal Mayor Municipal health officer 1. Chairman of the committee on health of the sangguniang panlalawigan 2. Representative from Private sector or NGO involved in health services 3. Representative from the DOH in the province

Chairman Vice chairman

Members

2.

3.

Meetings and quorum The board shall meet at least once a month or as may be necessary.

D2010 152

UP College of Law
LGU concerned, which is submitted to the proper sanggunian for its approval. By initiating the development plan for the LGU concerned, the local development council sets the direction of economic and social development and coordinates development efforts within the said local government unit. It is not specifically directed by the Code that development planning should start from below, the local people, and not imposed from the top, the central government. Section 107. Composition of Local Development Councils. The composition of the local development council shall be as follows: (a)The barangay development council shall be headed by the punong barangay and shall be composed of the following members: 1. 2. Members of the Sangguniang Barangay; Representatives of nongovernmental organizations operating in the barangay, who shall constitute not less than one fourth (1/4) of the members of the fully organized council; A representative of the congressman. shall constitute not less than one-fourth (1/4) of the members of the fully organized council. (d) The local development councils may call upon any local official concerned or any official of national agencies or offices in the local government unit to assist in the formulation of their respective development plans and public investment programs. Comments: The local chief executives chair the local development councils: (a) the governor for the Provincial Development Council, (b) the mayor for the City or Municipal Development Council and (c) the punong barangay for the Barangay Development Council. NGO Members The Code gives the NGOs a sizable number of representatives (not less than of the total number of council members) in the LDCs in recognition of the vital role that the private sector plays in the development of the different provinces, cities, municipalities and barangays. As members of the LDCs, NGOs can play a substantial role in defining the thrusts of local development. Power to Summon Assistance (b)The City or Municipal Development Council shall be headed by the mayor and shall be composed of the following members: 1. 2. All punong barangays in the city or municipality; The chairman of the committee on appropriations of the Sangguniang Panlungsod or Sangguniang Bayan concerned; The congressman or his representative; and Representatives of nongovernmental organizations operating in the city or municipality, as the case may be, who shall constitute not less than one-fourth (1/4) of the members of the fully organized council. The local development councils have the power to summon any official of an LGU concerned or of the national government agency or office in the said LGU to assist them in the formulation of their respective development plans and public investment programs. Section 108. Representation of Nongovernmental Organizations. Within a period of sixty (6) days from the start of organization of local development councils, the nongovernmental organizations shall choose from among themselves their representatives to said councils. The local sanggunian concerned shall accredit nongovernmental organizations subject to such criteria as may be provided by law. Comments: Period to Choose NGO Representatives This section directs that within 60 days from the organization of the LDC, the NGOs shall choose their representatives to the council from among themselves. It bears repeating that NGO representatives are chosen by them. They are not to be appointed by the mayor, the governor nor any other politician.

3.

3. 4.

(c) The Provincial Development Council shall be headed by the governor and shall be composed of the following members: 1. 2. All mayors of component cities and municipalities; The chairman of the committee on appropriations of the Sangguniang Panlalawigan; The congressman or his representative; and Representatives of nongovernmental organizations operating in the province, who

3. 4.

153 D2010

Local Government
Accreditation of NGOs This section deals with the accreditation of the NGOs by the local sanggunian concerned according to such criteria as may be provided by law. Till this date, there is no such law yet. The IRR, however, indicate some criteria for the accreditation of NGOs. The criteria laid down by the IRR are helpful guides for accreditation. Section 109. Functions of Local Development Councils. (a)The Provincial, City and Municipal Development Councils shall exercise the following functions: 1. 2. 3. 4. Formulate long-term, medium-term, and annual socioeconomic development plans and policies; Formulate the medium-term and annual public investment programs; Appraise and prioritize socioeconomic development programs and projects; Formulate local investment incentives to promote the inflow and direction of private investment capital; Coordinate, monitor and evaluate the implementation of development programs and projects; and Perform such other functions as may be provided by law or competent authority. Section 110. Meetings and Quorum. The local development council shall meet at least once every six (6) months or as often as may be necessary. Section 111. Executive Committee. (a) Each local development council shall create an executive committee to represent it and act in its behalf when it is not in session. The composition of the executive committee shall be as follows: 1. The executive committee of the Provincial Development Council shall be composed of the governor as chairman, the representative of component city and municipal mayors to be chosen from among themselves, the chairman of the committee on appropriations of the Sangguniang Panlalawigan, the president of the provincial league of barangays, and a representative of non-governmental organizations that are represented in the council, as members; The executive committee of the City or Municipal Development Council shall be composed of the mayor as chairman, the chairman of the committee on appropriations of the Sangguniang Panlalawigan, the president of the city or municipal league of barangays, and a representative of nongovernmental organizations that are represented in the council, as members; and The executive committee of the barangay development council shall be composed of the punong barangay as chairman, a representative of the Sangguniang Barangay to be chosen from among its members, and a representative of nongovernmental organizations that are represented in the council, as members.

2.

5.

6.

(b) The barangay development council shall exercise the following functions: 1. 2. 3. 4. Mobilize peoples participation in local development efforts; Prepare barangay development plans based on local requirements; Monitor and evaluate the implementation of national or local programs and projects; and Perform such other functions as may be provided by law or competent authority. 3.

Comments: The Barangay Development Council does essentially the same things for the barangay, although there is one thing that it is explicitly empowered to do which the other development councils have not been expressly authorized to do to mobilize peoples participation in local development efforts. It does not mean, however, that other LDCs cannot mobilize popular participation in local development. In fact, they should do so because without popular involvement and support, there will be no substantial development in their community.

(b) The executive committee shall exercise the following powers and functions: 1. 2. 3. Ensure that the decision of the council are faithfully carried out and implemented; Act on matters requiring immediate attention or action by the council; Formulate policies, plans and programs based on the general principles laid down by the council; and Act on other matters that may be authorized by the council.

4.

Comments:

D2010 154

UP College of Law
Excom Functions The main function of the executive committee of a local development council is to represent it and act in its behalf when the council is not meeting. Section 112. Sectoral or Functional Committee. The local development councils may form sectoral or functional committees to assist them in the performance of their functions. Comments: Sectoral/Functional Committees. The Code allows the LDCs to create sectoral or functional committees to assist them. A sectoral committee may be composed of members who come from a particular sector of society, for example, an urban poor committee or a committee of fisherfolk. Functional committees may encompass definite duties like a committee on ways and means or a committee on beautification. Section 113. Secretariat. There is hereby constituted for each local development council a secretariat which shall be responsible for providing technical support, documentation of proceedings, preparation of reports and such other assistance as may be required in the discharge of its functions. The local development council may avail of the services of any nongovernmental organization or educational or research institution for this purpose. The secretariats of the Provincial, City and Municipal Development Councils shall be headed by their respective planning and development coordinators. The secretariat of the barangay development council shall be headed by the barangay secretary who shall be assisted by the city or municipal planning and development coordinator concerned. Comments: LDC Secretariat. The Code does not define the membership of the secretariat of the LDCs but it must be headed by the provincial, city or municipal Development Coordinators in the case of a province, city or municipality and by the barangay secretary in the case of a barangay. The barangay secretary as head of the Barangay Secretariat shall be assisted by the city or municipal planning and development coordinator concerned. The reason is that very few, if any, barangay secretaries would have the expertise to discharge the duties of the head of the barangay secretariat on socioeconomic planning. Section 114. Relation of Local Development Councils to the Sanggunian and the Regional Development Council. (a) The policies, programs and projects proposed by local development councils shall be submitted to the sanggunian concerned for appropriate action. The local development plans approved by their respective sanggunian may be integrated with the development plans of the next higher level of local development council. (b) The approved development plans of provinces, highly urbanized cities and independent component cities shall be submitted to the Regional Development Council, which shall be integrated into the regional development plan for submission to the National Economic and Development authority, in accordance with existing laws. Comments: Sanggunian Approval Needed. Plans, programs and projects prepared by LDCs do not automatically acquire the force of law. They must be submitted to the sanggunian concerned, which enacts the corresponding ordinance to make them enforceable within the territory of the LGU concerned. The Sanggunian may or may not adopt the said plans, programs and projects. Integration with Higher Level LDC Plans. If adopted by the Sanggunian, the said plans, programs and projects may be integrated with the development plans of the next higher LDC. For example, if the development plans of a Municipal Development Council is adopted by the municipal Sanggunian, the plans may be made a part of the provincial development plan by the Provincial Development Council. Submission to Regional Development Council. Development plans approved by the Sanggunian of a province, a highly urbanized city or an independent component city shall be submitted to the Regional Development Council which shall integrate them into the regional development plan for submission to the NEDA in accordance with existing laws. The regional development Council has no authority to disapprove a provincial, city or municipal development plan but it may make its implementation difficult by recommending its disapproval for funding. Section 115. Budget Information. The Department of Budget and Management shall furnish the various local development councils information on financial resources and budgetary allocations applicable to their respective jurisdictions to guide them in their planning functions. Comments:

155 D2010

Local Government
It is not a requirement for the Department of Budget and Management to furnish LDCs with information on financial resources of and budgetary allocations to the LGUs to guide them in the discharge of their functions. Title VII Local Peace and Order Council Section 116. Organization. There is hereby established in every province, city, and municipality a local peace and order council, pursuant to Executive Order Numbered Three hundred nine (EO No. 309), as amended, Series of 1988. The local peace and order councils shall have the same composition and functions as those prescribed by the said executive board. Comments: Composition of Peace and Order Council Executive Order No. 309 as amended, Series of 1988, defines the membership of the local peace and order council. Provincial Peace and Order Council Composition The peace and order council of the province is composed of the following: (a) the governor as chair; (b) the representative of the Sangguniang Panlalawigan, chosen by its members; (c) the Social Welfare and Development Officer; (d) the Information Officer; (e) the Health Officer; and (f) the representatives of the central government office or agency in the province who are appointed by their respective heads, such as (i) the PNP director of the province; (ii) the Commission on Human Rights in the province, (iii) the commanding general or officer of the armed forces, if any, in the province; (iv) the NBI provincial office; (v) the National Security Council, if any, in the province; (vi) the provincial prosecutors office; (vii) the DILK, if any, in the province; (viii) the executive director of the Dangerous Drugs Board; and (ix) 3 representatives of the NGOs and Pos in the province, representing the academic, civic and religious organizations, who are appointed by the governor. City/Municipal Peace and Order Council Composition The peace and order council of the city or municipality is composed of the following: (a) the mayor as chair; (b) the Sangguniang Panglungsod or Sangguniang Bayan representative, chosen by the sanggunian from among its members; (c) the SWDO; (d) the Information Officer; (e) the Health Officer; and (f) the representatives of the central government office or agency in the city or municipality who are appointed by their respective heads, such as (i) the chief of police; (ii) the Commission on Human Rights, if any, in the city or municipality; (iii) the commanding general or officer of the armed forces, if any, in the city or municipality; (iv) the NBI city or provincial office; (v) the National Security Council; (vi) the city or municipal prosecutors office or in their absence, the city or municipal attorney; (vii) the DILG, if any; (viii) the executive director of the Dangerous Drugs Board; and (ix) 3 representatives of the NGOs and Pos, representing the academic, civic and religious organizations in the city or municipality, who are appointed by the mayor upon consultation with the members of the Council. Local Peace and Order Council Functions The provincial, city and municipal peace and order councils have the following duties and functions: (a) Formulate plans and recommend such measures to improve or enhance peace and order and public safety in their respective areas; (b) Monitor the implementation of peace and order programs and projects at the provincial, city or municipal levels, and the operation of Civilian Volunteer SelfDefense Organizations and such other counterinsurgency programs and activities; (c) Make periodic assessments of the prevailing peace and order situation in their respective areas and submit a report thereon with recommendations to the chair of the national peace and order council; and (d) Perform all other functions assigned by law to the peace and order council. Regional Peace and Order Council In between the national and the local peace and order councils, there is another council called the Regional Peace and Order Council, which is composed of their counterpart members of the local peace and order councils. The Regional Peace and Order Council performs essentially the same functions as those of the local peace and order councils.

Osea v. Malaya
Petitioner filed a protest case with the Civil Service Commission alleging that she was appointed as Officerin-Charge, Assistant Schools Division Superintendent of Camarines Sur, by the then Secretary of DECS, upon the

D2010 156

UP College of Law
endorsement of the Provincial School Board of Camarines Sur. However, despite this, President Fidel Ramos, appointed respondent to the position of Schools Division Superintendent of Camarines Sur. Petitioner claims that the appointment of respondent was made without prior consultation with the Provincial School Board, in violation of Section 99 of the Local Government Code as well as her vested right as the Schools Division Superintendent of Camarines Sur. HELD: Section 99 of the LGC applies to appointments made by the DECS because at the time of the enactment of the LGC, schools division superintendents were appointed by the DECS to specific division or location. However, in 1994, the Career Executive Service Board issued a Memorandum Circular placing the positions of schools division superintendent and assistant schools division superintendent within the career executive service. Consequently, the power to appoint persons to career executive service positions was transferred from the DECS to the President. In addition, under the circumstances, the designation of respondent as Schools Division Superintendent of Camarines Sur was not a case of appointment but rather in the nature of reassignment. Therefore, Section 99 of the LGC, which requires prior consultation with the local school board does not apply. Appointment should be distinguished from reassignment. An appointment may be defined as the selection, by the authority vested with the power, of an individual who is to exercise the functions of a given office. When completed, usually with its confirmation, the appointment results in security of tenure for the person chosen unless he is replaceable at pleasure because of the nature of his office. On the other hand, a reassignment is merely a movement of an employee from one organizational unit to another in the same department or agency which does not involve a reduction in rank, status or salary and does not require the issuance of an appointment. In the same vein, a designation connotes merely the imposition of additional duties on an incumbent official Petitioner's designation as Officer-in-Charge, Assistant Schools Division Superintendent, was expressly made subject to further advice from the DECS. Thus, her designation was temporary. In fact, there was a need to recommend her to the President for appointment in a permanent capacity. Inasmuch as she occupied her position only temporarily, petitioner can be transferred or reassigned to other positions without violating her right to security of tenure. Indeed, petitioner has no vested right to the position of Schools Division Superintendent of Camarines Sur.

157 D2010

Local Government

Local Taxation and Fiscal Matters


Book II Title 3, LGC
See above

Pimentel v. Aguirre (supra)


See above

Shares of LGUs in national taxes

Local Government Units

THE BARANGAY
Sec 384-439
CHAPTER I - Role and creation of the Barangay Section 384. Role of the Barangay. As the basic political unit, the barangay serves as the primary planning and implementing unit of government policies, plans, programs, projects and activities in the community, and as a forum wherein the collective views of the people may be expressed, crystallized and considered, and where disputes may be amicably settled. Comments: The barangay discharges three seminal functions: (a) as a basic political unit the barangay is the smallest political entity used for governance in the country. (b) as a primary planning and implementing unit the barangay is mandated to plan development projects in its territory and to deliver some basic services of the government to its people. (c) as a forum the barangay gets soundings of the views of the people on various topics. It also provides a venue for the settlement of disputes amicably. Section 385. Manner of Creation. - A barangay may be created, divided, merged, abolished, or its boundary substantially altered, by law or by an ordinance of the sangguniang panlalawigan or sangguniang panlungsod, subject to approval by a majority of the votes cast in a plebiscite to be conducted by the Comelec in the local government unit or units directly affected within such period of time as may be determined by the law or ordinance creating said barangay. In the case of the creation of barangays by the sangguniang panlalawigan, the recommendation of the sangguniang bayan concerned shall be necessary. Section 386. Requisites for Creation. (a) A barangay maybe created out of a contiguous territory which has apopulation of at least two thousand (2,000) inhabitants ascertified by the National Statistics Office except in cities and municipalities within Metro Manila and other metropolitan political subdivisions or in highly urbanized cities where such territory shall have a certified population of at least five thousand (5,000) inhabitants: Provided, That the creation thereof shall not reduce the population of the original barangay or barangays to less than the minimum requirement prescribed herein. To enhance the delivery of basic services in the indigenous cultural communities, barangays may be created in such communities by an Act of Congress, notwithstanding the above requirement. (b) The territorial jurisdiction of the new barangay shall be properly identified by metes and bounds or by more

D2010 158

UP College of Law
or less permanent natural boundaries. The territory need not be contiguous if it comprises two (2) or more islands. (c) The governor or city mayor may prepare a consolidation plan for barangays, based on the criteria prescribed in this Section, within his territorial jurisdiction. The plan shall be submitted to the sangguniang panlalawigan or sangguniang panlungsod concerned for appropriate action. In the case of municipalities within the Metropolitan Manila area and other metropolitan political subdivisions, the barangay consolidation plan shall be prepared and approved by the sangguniang bayan concerned. Comments: Unlike provinces, cities or municipalities which need a definite territorial size to be created, all that the Code requires in terms of area for newly created barangays is that it be contiguous. The Code takes into account the fact that when barangays were created during the Marcos years, their territories were not defined by specific metes and bounds and therefore came in different sizes, some comprising only a block or two in the city and others covering areas larger than some municipalities. Now, the Code directs that the territory of the new barangay shall be identified by metes and bounds or by more or less permanent boundaries. The requirement of contiguity of barangay areas is not mandatory when the barangay comprises two or more islands. Population Requirement In general, at least 2,000 inhabitants are needed to qualify a barangay for creation. But when the barangay being created is within highly urbanized cities or in cities and municipalities in the Metropolitan Manila Area or other metropolitan political subdivisions, the population requirement is 5,000. The population required must be certified by the NSO. It is also required that the population of the new barangay does not reduce the population of the original barangay from which it is being created to levels below the numbers now required by the Code. Nonetheless, barangays may be created by law within areas occupied by indigenous cultural communities even if their population may be below the required numbers stipulated in the Code. The reason for this exemption is to enhance the delivery of basic services. Consolidation of Barangays With an appropriate plan for consolidation of barangays prepared by the governor or city mayor, the local sanggunian may consolidate barangays within its territory based upon the criteria set forth in this section. Consolidation may be the only way to solve the problem of hundreds of barangays throughout the country whose territorial jurisdictions are ill-defined and which in the urban centers may comprise only a block or two. In the Metropolitan Manila Area, the consolidation plan for barangays shall be approved by the Sangguniang Bayan of the municipalities and Sangguniang Panlungsod of the cities. CHAPTER II - Barangay officials and offices Section 387. Chief Officials and Offices. (a) There shall be in each barangay a punong barangay, seven (7) sangguniang barangay members, the sangguniang kabataan chairman, a barangay secretary, and a barangay treasurer. (b) There shall also be in every barangay a lupong tagapamayapa. The sangguniang barangay may form community brigades and create such other positions or offices as may be deemed necessary to carry out the purposes of the barangay government in accordance with the needs of public service, subject to the budgetary limitations on personal services prescribed under Title Five, Book II of this Code. Comments: Aside from the principal officials for the barangay, namely the punong barangay and the seven Sangguniang Barangay members, the Sangguniang Kabataan chair, the barangay secretary and the barangay treasurer, there are other important officials in the barangay, namely the members of the Lupong Tagapamayapa and the Community Brigades such as the Tanod Brigade and the Disaster Brigade. Section 388. Persons in Authority. - For purposes of the Revised Penal Code, the punong barangay, sangguniang barangay members, and members of the lupong tagapamayapa in each barangay shall be deemed as persons in authority in their jurisdictions, while other barangay officials and members who may be designated by law or ordinance and charged with the maintenance of public order, protection and security of life and property, or the maintenance of a desirable and balanced environment, and any barangay member who

159 D2010

Local Government
comes to the aid of persons in authority, shall be deemed agents of persons in authority. Comments: Punong barangays, members of Sangguniang Barangays and Lupong Tagapamayapa are considered persons in authority under this section. For purposes of the Code, the definition of a person in authority in the Revised Penal Code is relevant. Article 152 of the Revised Penal Code states that any person directly vested with jurisdiction, whether as an individual or as a member of some court or governmental corporation, board or commission, shall be deemed a person in authority. A barangay captain and a barangay chairman shall also be deemed a person in authority. The article also defines an agent of a person in authority as: Any person who, by direct provision of law or by election or by appointment by competent authority, is charged with the maintenance of public order and the protection and security of life and property, such as a barrio councilman, barrio policeman and barangay leader and any person who comes to the aid of persons in authority Definition Modified That definition is now modified. In addition to the punong barangay, the members of the Sangguniang Barangay and the Lupong Tagapamayapa are now considered not merely as agents of but as persons in authority in the Code. But other barangay officials and members who may be designated by law or ordinance and charged with the maintenance of public order, protection and security of life and property, or the maintenance of a desirable and balanced environment, and any barangay member who comes to the aid of persons in authority continue to be deemed agents of persons in authority. Section 388 of the Code provides that for purposes of the Revised Penal Code, the punong barangay, the Sangguniang Barangay members and the members of the Lupong Tagapamayapa in each barangay shall be deemed as persons in authority in their jurisdictions This law expands the definition of a person in authority under the Revised Penal Code, wherein among the barangay officials, only the barangay captain or chairman, now called the punong barangay, is expressly considered a person in authority, as provided in Article 152 thereof. Thus, in addition to the punong barangay, the members of the Sangguniang Barangay or kagawads and members of the Lupong Tagapamayapa are now considered not merely agents of, but as persons, in authority. Protecting Environment as Agents of Persons in Authority Barangay officials and members designated by law or ordinance to maintain a desirable and balanced environment or who come to the aid of persons in authority who protect the environment are considered agents of persons in authority. Consequences of Being Persons in Authority/Agents of Persons in Authority As persons in authority, they are entitled to respect and may request assistance from barangay residents in the performance of their duties. Also, because they are considered persons in authority, if they are physically harmed, the person responsible may be charged for higher degree felonies than would otherwise be the case. For example, if a barangay resident is attacked physically but without any intent to kill and he suffers some injuries, the attacker may be charged for physical injuries. But if the victim is a barangay official, the charge may be for the more serious offense of assault upon a person in authority or upon an agent of a person in authority for which a higher penalty is imposable. Power to Arrest and Detain A barangay captain (now called punong barangay) is a peace officer in the barrio (barangay) and is considered under the law as a person in authority. As such, he may make arrests and detain persons within legal limits but if the detention is without legal grounds, the punong barangay may be charged for arbitrary detention as defined in Article 124 of the Revised Penal Code. CHAPTER III - The Punong Barangay Section 389. Chief Executive: Powers, Duties and Functions. (a)The punong barangay, as the chief executive of the barangay government, shall exercise such powers and perform such duties and functions, as provided by this Code and other laws. (b)For efficient, effective and economical governance, the purpose of which is the general welfare of the barangay and its inhabitants pursuant to Section 16 of this Code, the punong barangay shall:

D2010 160

UP College of Law
1. 2. Enforce all laws and ordinances which are applicable within the barangay; Negotiate, enter into, and sign contracts for and in behalf of the barangay, upon authorization of the sangguniang barangay; Maintain public order in the barangay and, in pursuance thereof, assist the city or municipal mayor and the sanggunian members in the performance of their duties and functions; Call and preside over the sessions of the sangguniang barangay and the barangay assembly, and vote only to break a tie; Upon approval by a majority of all the members of the sangguniang barangay, appoint or replace the barangay treasurer, the barangay secretary, and other appointive barangay officials; Organize and lead an emergency group whenever the same may be necessary for the maintenance of peace and order or on occasions of emergency or calamity within the barangay; In coordination with the barangay development council, prepare the annual executive and supplemental budgets of the barangay; Approve vouchers relating to the disbursement of barangay funds; Enforce laws and regulations relating to pollution control and protection of the environment; Administer the operation of the Katarungang Pambarangay in accordance with the provisions of this Code; Exercise general supervision over the activities of the sangguniang kabataan; Ensure the delivery of basic services as mandated under Section 17 of this Code; Conduct an annual palarong barangay which shall feature traditional sports and disciplines included in national and international games, in coordination with the Department of Education, Culture and Sports; Promote the general welfare of the barangay; and Exercise such other powers and perform such other duties and functions as may be prescribed by law or ordinance. Barangay Chief Executive To a lesser extent, the punong barangay is the equivalent of the governor of a province or the mayor of a city or municipality as chief executive for his barangay. Power of Appointment The appointment of barangay officials by the punong barangay is subject to confirmation by the majority of all the members of the Sangguniang Barangay. Some Powers of Punong Barangay Among the more important powers attached to his office by the Code are the (a) enforcement of laws relative to pollution control and protection of the environment; (b) administration of barangay justice or Katarungang Pambarangay; and (c) holding of annual palarong barangay in coordination with the Department of Education, Culture and Sports. Enforcement of Anti-Pollution Laws. The punong barangay has the power to protect the environment and to enforce laws against pollution Palarong Barangay. The punong barangay has the power to conduct the annual Palarong Barangay. The idea is to encourage the youth of the land, including those living in the remotest areas, to participate in sports activities. It is hoped that through the palarong barangay, they may develop into national, if not international, class athletes. Right to Carry Firearms. The punong barangay is entitled to possess and carry a firearm within his barangay while discharging his duties.

3.

4.

5.

6.

7.

8. 9.

10.

11. 12. 13.

14. 15.

CHAPTER IV - The Sangguniang Barangay Section 390. Composition. The Sangguniang Barangay, the legislative body of the barangay, shall be composed of the punong barangay as presiding officer, and the seven (7) regular Sangguniang Barangay members elected at large and Sangguniang Kabataan chairman, as members. Comments:

(c) In the performance of his peace and order functions, the punong barangay shall be entitled to possess and carry the necessary firearm within his territorial jurisdiction, subject to appropriate rules and regulations. Comments:

161 D2010

Local Government
The punong barangay is a part of the Sangguniang Barangay of which he is the presiding officer. The Supreme Court has stressed that *a+ petition or protest contesting the election of barangay officer should be decided by the municipal or metropolitan trial court within 15 days from filing thereof. xxx Election cases, unlike ordinary actions, involve public interest. Time is of the essence in its disposition since the uncertainty as to who is the real choice of the people for the position must soonest be dispelled. It is neither fair nor just that one of whose right to the office is in doubt should remain in that office for an uncertain period. Section 391. Powers, Duties and Functions. (a) The sangguniang barangay, as the legislative body of the barangay, shall: 1. Enact ordinances as may be necessary to discharge the responsibilities conferred upon it by law or ordinance and to promote the general welfare of the inhabitants therein; Enact tax and revenue ordinances, subject to the limitations imposed in this Code; Enact annual and supplemental budgets in accordance with the provisions of this Code; Provide for the construction and maintenance of barangay facilities and other public works projects chargeable to the general fund of the barangay or such other funds actually available for the purpose; Submit to the sangguniang panlungsod or sangguniang bayan such suggestions or recommendations as it may see fit for the improvement of the barangay or for the welfare of the inhabitants thereof; Assist in the establishment, organization, and promotion of cooperative enterprises that will improve the economic condition and well-being of the residents; Regulate the use of multi-purpose halls, multipurpose pavements, grain or copra dryers, patios and other post-harvest facilities, barangay waterworks, barangay markets, parking areas or other similar facilities constructed with government funds within the jurisdiction of the barangay and charge reasonable fees for the use thereof; Solicit or accept monies, materials and voluntary labor for specific public works and cooperative enterprises of the barangay from residents, land owners, producers and merchants in the barangay; monies from grants-in-aid, subsidies, contributions, and revenues made available to the barangays from national, provincial, city or municipal funds; and monies from other private agencies and individuals: Provided, however, That monies or properties donated by private agencies and individuals for specific purposes shall accrue to the barangay as trust fund; 9. Solicit or accept, in any or all the foregoing public works and cooperative enterprises, such cooperation as is made available by national, provincial, city, or municipal agencies established by law to render financial, technical, and advisory assistance to barangays and to barangay residents: Provided, however, That in soliciting or accepting such cooperation, the sangguniang barangay need not pledge any sum of money for expenditure in excess of amounts currently in the barangay treasury or encumbered for other purposes; 10. Provide compensation, reasonable allowances or per diems as well as travel expenses for sangguniang barangay members and other barangay officials, subject to the budgetary limitations prescribed under Title Five, Book II of this Code: Provided, however, That no increase in the com- pensation or honoraria of the sangguniang barangay members shall take effect until after the expiration of the full term of all members of the sangguniang barangay approving such increase; 11. Hold fund-raising activities for barangay projects without the need of securing permits from any national or local office or agency. The proceeds from such activities shall be tax-exempt and shall accrue to the general fund of the barangay: Provided, That in the appropriation thereof, the specific purpose for which such fund-raising activity has been held shall be first satisfied: Provided, further, That no fund-raising activities shall be held within a period of sixty (60) days immediately preceding and after a national or local election, recall, referendum, or plebiscite: Provided, finally, That said fund-raising activities shall comply with national policy standards and regulations on morals, health, and safety of the persons participating therein. The sangguniang barangay, through the punong barangay, shall render a public accounting of the funds raised at the completion of the project for which the fund-raising activity was under- taken;

2. 3. 4.

5.

6.

7.

8.

D2010 162

UP College of Law
12. Authorize the punong barangay to enter into contracts in behalf of the barangay, subject to the provisions of this Code; 13. Authorize the barangay treasurer to make direct purchases in an amount not exceeding One thousand pesos (P1,000.00) at any one time for the ordinary and essential administrative needs of the barangay; 14. Prescribe fines in amounts not exceeding One thousand pesos (P1,000.00) for violation of barangay ordinances; 15. Provide for the administrative needs of the lupong tagapamayapa and the pangkat ng tagapagkasundo; 16. Provide for the organization of community brigades, barangay tanod, or community service units as may be necessary; 17. Organize regular lectures, programs, or fora on community problems such as sanitation, nutrition, literacy, and drug abuse, and convene assemblies to encourage citizen participation in government; 18. Adopt measures to prevent and control the proliferation of squatters and mendicants in the barangay; 19. Provide for the proper development and welfare of children in the barangay by promoting and supporting activities for the protection and total development of children, particularly those below seven (7) years of age; 20. Adopt measures towards the prevention and eradication of drug abuse, child abuse, and juvenile delinquency; 21. Initiate the establishment of a barangay high school, whenever feasible, in accordance with law; 22. Provide for the establishment of a non-formal education center in the barangay whenever feasible, in coordination with the Department of Education, Culture and Sports, ; 23. Provide for the delivery of basic services; and 24. Exercise such other powers and perform such other duties and functions as may be prescribed by law or ordinance. Comments: Wider powers are now enjoyed by the Sangguniang Barangay than was previously the case. Among the more noteworthy of its powers are: (a) to enact tax and other revenue measures authorized by the Code; (b) to regulate and charge fees for the use of barangay facilities, including parking areas, markets, copra dryers, multipurpose halls and the like; (c) to assist in the establishment of cooperatives to improve the economic well-being of the barangay residents; (d) to provide compensation, allowances, per diems and travel expenses for barangay officials subject to the limitations provided for in this Code; (e) to authorize direct purchases by the barangay treasurer of not more than P1,000 worth of items at any one time that are ordinarily and essentially needed by the barangay; (f) to prescribe fines of not more than P1,000 for violations of barangay ordinances; (g) to adopt measures to combat drug abuse, child abuse and juvenile delinquency; (h) to provide for the establishment of non-formal education centers; and (i) to provide for the delivery of basic services. Section 392. Other Duties of Sangguniang Barangay Members. In addition to their duties as members of the Sangguniang Barangay, Sangguniang Barangay members may: (a) Assist the punong barangay in the discharge of his duties and functions; (b) Act as peace officers in the maintenance of public order and safety; and (c) Perform such other duties and functions as the punong barangay may delegate. Section 393. Benefits of Barangay Officials. (a) Barangay officials, including barangay tanods and members of the lupong tagapamayapa, shall receive honoraria, allowances, and such other emoluments as may be authorized by law or barangay, municipal or city ordinance in accordance with the provisions of this Code, but in no case shall it be less than One thousand pesos (P=1,000.00) per month for the punong barangay and Six hundred pesos (P=600.00) per month for the sangguniang barangay members, barangay treasurer, and barangay secretary: Provided, however, That the annual appropriations for personal services shall be subject to the budgetary limitations prescribed under Title Five, Book II of this Code; (b) The punong barangay, the sangguniang barangay members, the barangay treasurer, and the barangay secretary shall also: 1. Be entitled to Christmas bonus of at least One thousand pesos (P=1,000.00) each, the funds for which shall be taken from the general fund of the barangay or from such

163 D2010

Local Government
other funds appropriated by the national government for the purpose; Be entitled, during their incumbency, to insurance coverage which shall include, but shall not be limited to temporary and permanent disability, double indemnity, accident insurance, death and burial benefits, in accordance with Republic Act Numbered Sixty-nine hundred forty-two (R.A. No. 6942), entitled "An Act Increasing the Insurance Benefits of Local Government Officials and Providing Funds Therefor"; Be entitled to free medical care including subsistence, medicines, and medical attendance in any government hospital or institution: Provided, That such hospital care shall include surgery or surgical expenses, medicines, X-rays, laboratory fees, and other hospital expenses; In case of extreme urgency where there is no available government hospital or institution, the barangay official concerned may submit himself for immediate medical attendance to the nearest private clinic, hospital or institution and the expenses not exceeding Five thousand pesos (P=5,000.00) that may be incurred therein shall be chargeable against the funds of the barangay concerned; Be exempted during their incumbency from paying tuition and matriculation fees for their legitimate dependent children attending state colleges or universities. He may likewise avail of such educational benefits in a state college or university located within the province or city to which the barangay belongs; and Be entitled to appropriate civil service eligibility on the basis of the number of years of service to the barangay, pursuant to the rules and regulations issued by the Civil Service Commission. insurance or other benefits during their incumbency, chargeable to the barangay or the city or municipal government to which the barangay belongs. Comments: Barangays may now grant honoraria, allowances and other emoluments to their barangay officials, barangay tanods and members of the Lupong Tagapamayapa. The minimum is P1,000 per month for the punong barangay and P600 for the Sangguniang Barangay members, Barangay Treasurers and Barangay Secretaries subject, however, to the requirement that not more than 55% of the total annual income actually realized by the barangay from local sources during the next preceding fiscal year shall be set aside for personal services. Increase of honoraria or compensation awarded by the Sangguniang Barangay for their elective members cannot take effect until after their term is over as provided for under Section 391(10). Among the more noteworthy benefits to which barangay officials are entitled are the following: (a) a Christmas bonus of at least P1,000 each; (b) insurance coverage; (c) free medical care in government hospitals, (d) free education for their legitimate dependent children in state colleges or universities and for themselves in state colleges or universities located in the province or city where their barangays belong; (e) appropriate civil service eligibility on the basis of their length of service to their barangays; and (f) after their tenure of office, preference in appointments to government-owned or controlled corporations. The insurance for barangay officials includes disability benefits, double indemnity, accident coverage, death and burial benefits pursuant to the law that increases the insurance benefits of local government officials (R.A. 6942). The free medical care to which these officials are entitled includes surgery or surgical expenses, medicines, x-rays, laboratory fees and other hospital expenses in government hospitals. There is no limit as to the number of legitimate dependent children of barangay officials who are entitled to free tertiary education in state colleges or universities. Barangay tanods are also entitled to insurance coverage and other benefits chargeable to the barangay concerned or to the city or municipality to which the barangay belongs.

2.

3.

4.

5.

(c) Elective barangay officials shall have preference in appointments to any government position or in any government-owned or -controlled corporations, including their subsidiaries, after their tenure of office, subject to the requisite qualifications and the provisions of the immediately preceding paragraph. (d) All duly appointed members of the barangay tanod brigades, or their equivalent, which shall number not more than twenty (20) in each barangay, shall be granted

D2010 164

UP College of Law
CHAPTER V - Appointive barangay officials Section 394. Barangay Secretary: Appointment, Qualifications, Powers and Duties. (a)The barangay secretary shall be appointed by the punong barangay with the concurrence of the majority of all the sangguniang barangay members. The appointment of the barangay secretary shall not be subject to attestation by the Civil Service Commission. (b) The barangay secretary shall be of legal age, a qualified voter and an actual resident of the barangay concerned. (c) No person shall be appointed barangay secretary if he is a sangguniang barangay member, a government employee, or a relative of the punong barangay within the fourth civil degree of consanguinity or affinity. (d) The barangay secretary shall: 1. Keep custody of all records of the sangguniang barangay and the barangay assembly meetings; Prepare and keep the minutes of all meetings of the sangguniang barangay and the barangay assembly; Prepare a list of members of the barangay assembly, and have the same posted in conspicuous places within the barangay; Assist in the preparation of all necessary forms for the conduct of barangay elections, initiatives, referenda or plebiscites, in coordination with the Comelec; Assist the municipal civil registrar in the registration of births, deaths, and marriages; Keep an updated record of all inhabitants of the barangay containing the following items of information: name, address, place and date of birth, sex, civil status, citizenship, occupation, and such other items of information as may be prescribed by law or ordinances; Submit a report on the actual number of barangay residents as often as may be required by the sangguniang barangay; and Exercise such other powers and perform such other duties and functions as may be prescribed by law or ordinance. Mandatory Barangay Appointive Officials The barangay secretary and the barangay treasurer are mandatory officers who are to be appointed by the Punong Barangay subject to the approval of the majority of all the members of the Sangguniang Barangay concerned. There are other mandatory barangay officials like the Lupong Tagapamayapa and the Pangkat ng Tagapagkasundo who are mentioned specifically by the Code. The barangay may create other positions which may be filled by appointment by the punong barangay subject to approval of a majority of the members of the Sangguniang Barangay. Civil Registrar Duties of Barangay Secretary One of the more important duties of the barangay secretary is to assist the municipal civil registrar in the registration of births, deaths and marriages. Another is to keep an updated record of all inhabitants of the barangay containing their names, addresses, places of birth, sexes, civil statuses, citizenships, occupations and other items of information as may be prescribed by law or ordinance. Section 395. Barangay Treasurer: Appointment, Qualifications, Powers and Duties. (a) The barangay treasurer shall be appointed by the punong barangay with the concurrence of the majority of all the sangguniang barangay members. The appointment of the barangay treasurer shall not be subject to attestation by the Civil Service Commission. (b) The barangay treasurer shall be of legal age, a qualified voter, and an actual resident of the barangay concerned. (c) No person shall be appointed barangay treasurer if he is a sangguniang barangay member, a government employee, or a relative of the punong barangay within the fourth civil degree of consanguinity or affinity. (d) The barangay treasurer shall be bonded in accordance with existing laws in an amount to be determined by the sangguniang barangay but not exceeding Ten thousand pesos (P=10,000.00), premiums for which shall be paid by the barangay. (e) The barangay treasurer shall: 1. Keep custody of barangay funds and properties;

2.

3.

4.

5.

6.

7.

8.

Comments:

165 D2010

Local Government
2. Collect and issue official receipts for taxes, fees, contributions, monies, materials, and all other resources accruing to the barangay treasury and deposit the same in the account of the barangay as provided under Title Five, Book II of this Code; Disburse funds in accordance with the financial procedures provided in this Code; Submit to the punong barangay a statement covering the actual and estimates of income and expenditures for the preceding and ensuing calendar years, respectively, subject to the provisions of Title Five, Book II of this Code; Render a written accounting report of all barangay funds and property under his custody at the end of each calendar year, and ensure that such report shall be made available to the members of the barangay assembly and other government agencies concerned; Certify as to the availability of funds whenever necessary; Plan and attend to the rural postal circuit within his jurisdiction; and Exercise such other powers and perform such other duties and functions as may be prescribed by law or ordinance. CHAPTER VI - Barangay Assembly Section 397. Composition; Meetings. (a) There shall be a barangay assembly composed of all persons who are actual residents of the barangay for at least six (6) months, fifteen (15) years of age or over, citizens of the Philippines, and duly registered in the list of barangay assembly members. (b) The barangay assembly shall meet at least twice a year to hear and discuss the semestral report of the sangguniang barangay concerning its activities and finances as well as problems affecting the barangay. Its meetings shall be held upon call of the punong barangay or of at least four (4) members of the sangguniang barangay, or upon written petition of at least five percent (5%) of the assembly members. (c) No meeting of the barangay assembly shall take place unless a written notice is given one (1) week prior to the meeting except on matters involving public safety or security, in which case notice within a reasonable time shall be sufficient. The punong barangay, or in his absence, the sangguniang barangay member acting as punong barangay, or any assembly member selected during the meeting, shall act as presiding officer in all the meetings of the assembly. The barangay secretary, or in his absence, any member designated by the presiding officer to act as secretary, shall discharge the duties of secretary of the barangay assembly. Comments: Barangay Assembly Composition Actual residents of a barangay for at least 6 months who are citizens of the Republic, at least 15 years of age and are registered in the list of barangay assembly members compose the Barangay Assembly. Barangay Assembly Meetings Mandatory meetings of the Barangay Assembly are at least twice a year. Meetings may be called by the punong barangay or by at least four members of the Sangguniang Barangay or upon petition of at least 5% of the assembly members. Written notice is required to be sent at least one week before the meeting is held except when matters involving public safety or security are the main agenda of the meeting, in which case, notice for a shorter period is allowed.

3. 4.

5.

6. 7. 8.

Comments: Barangay treasurers are appointed by the punong barangay subject to the approval of the majority of the members of the Sangguniang Barangay. The Code prohibits the appointment of a person as barangay treasurer if he is a member of the Sangguniang Barangay, a government employee or a relative of the punong barangay within the fourth civil degree of consanguinity or affinity. It is absolute only in the case of a relative of the punong barangay within the fourth civil degree of consanguinity or affinity. It is not absolute in the case of a member of the Sangguniang Barangay or a government employee who can resign as such and then accept appointment as barangay treasurer. The barangay treasurer shall be bonded in an amount not exceeding P10,000. Section 396. Other Appointive Officials. The qualifications, duties and functions of all other barangay officials appointed by the punong barangay shall be governed by the provisions of this Code and other laws or by barangay ordinances.

D2010 166

UP College of Law
The punong barangay presides at the Barangay Assembly meetings. If he is absent, the acting punong barangay takes over or any assembly member selected during the meeting may act as president officer. Section 398. Powers of the Barangay Assembly. The barangay assembly shall: (a) Initiate legislative processes by recommending to the sangguniang barangay the adoption of measures for the welfare of the barangay and the city or municipality concerned; (b) Decide on the adoption of initiative as a legal process whereby the registered voters of the barangay may directly propose, enact, or amend any ordinance; and (c) Hear and pass upon the semestral report of the sangguniang barangay concerning its activities and finances. Comments: Assemblies as Forums. Barangay Assemblies provide the forum for the discussion of barangay development plans or petitions for the adoption of initiative to enact or amend a barangay ordinance. CHAPTER VII Katarungang Pambarangay Section 399. Lupong Tagapamayapa What (a) There is hereby created in each barangay a lupong tagapamayapa, composed of the Punong Barangay, as chairman ten (10) to twenty (20) members. The lupon shall be constituted every three (3) years in the manner provided herein. How (c) A notice to constitute the lupon which shall include the names of proposed members who have expressed their willingness to serve shall be prepared by the punong barangay within the first fifteen (15) days from the start of his term of office Notice shall be posted in three (3) conspicuous places in the barangay continuously for a period of not less than three (3) weeks

Appointment, when discretionary (d) Taking into consideration any opposition to the proposed appointment or any recommendations for appointment as may have been made within the period of posting, the Punong Barangay shall within ten (10) days thereafter appoint as members those whom he determines to be suitable therefor Appointments shall be in writing, signed by the punong barangay, and attested to by the barangay secretary.

(e) The list of appointed members shall be posted in three (3) conspicuous places in the barangay for the entire duration of their term of office; and (f) In barangays where majority of the inhabitants are members of indigenous cultural communities, local systems of settling disputes through their councils of datus or elders shall be recognized without prejudice to the applicable provisions of this Code. Section 400. Oath and Term of Office. 1. Upon appointment, each lupon member shall take an oath of office before the punong barangay. He shall hold office until a new lupon is constituted on the third year following his appointment unless sooner terminated by resignation, transfer of residence or place of work, or withdrawal of appointment by the punong barangay with the concurrence of the majority of all the members of the lupon.

Who (b) Any person actually residing or working in the barangay not otherwise expressly disqualified by law and possessing integrity, impartiality, independence of mind, sense of fairness, and reputation for probity, may be appointed a member of the lupon.

2.

Section 401. Vacancies. Punong barangay shall immediately appoint a qualified

167 D2010

Local Government
person who shall hold office only for the unexpired portion of the term. Section 402. Functions of the Lupon. (a) Exercise administrative supervision over the conciliation panels provided herein; (b) Meet regularly once a month to provide a forum for exchange of ideas among its members and the public on matters relevant to the amicable settlement of disputes, to enable various conciliation panel members to share with one another their observations and experiences in effecting speedy resolution of disputes; and prepares minutes of the pangkat proceedings submits a copy duly attested to by the chairman to the lupon secretary and to the proper city or municipal court issue and cause to be served notices to the parties concerned.

The lupon secretary shall issue certified true copies of any public record in his custody that is not by law otherwise declared confidential. Section 405. Vacancies in the Pangkat. Chosen by the parties to the dispute from among the other lupon members. Should the parties fail to agree on a common choice, the vacancy shall be filled by lot to be drawn by the lupon chairman. Section 406. Character of Office and Service of Lupon Members. (a) The lupon members deemd as persons in authority (as defined in the RPC) while in the performance of their official duties or on the occasion thereof. (b) Lupon and pangkat members serve without compensation without prejudice to incentives.. The DILG shall provide for a system of granting economic or other incentives to the lupon or pangkat members who adequately demonstrate the ability to judiciously and expeditiously resolve cases referred to them. While in the performance of their duties, the lupon or pangkat members, whether in public or private employment, shall be deemed to be on official time, and shall not suffer from any diminution in compensation or allowance from said employment by reason thereof. Section 407. Legal Advice on Matters Involving Questions of Law. The provincial, city legal officer or prosecutor or the municipal legal officer shall render legal advice on matters involving questions of law to the punong barangay or any lupon or pangkat member whenever necessary in the exercise of his functions in the administration of the katarungang pambarangay.

(c) Exercise such other powers and perform such other duties and functions as may be prescribed by law or ordinance. Section 403. Secretary of the Lupon. The barangay secretary concurrently serves as the secretary of the lupon. Records the results of mediation proceedings before the punong barangay Submits a report thereon to the proper city or municipal courts. Receives and keeps the records of proceedings submitted to him by the various conciliation panels.

Section 404. Pangkat ng Tagapagkasundo. (Conciliation Panel) (a) Constituted for each dispute brought before the lupon consisting of three (3) members chosen by the parties to the dispute from the list of members of the lupon. Should the parties fail to agree on the pangkat membership, the same shall be determined by lots drawn by the lupon chairman.

(b) The three (3) members constituting the pangkat shall elect from among themselves the chairman and the secretary. Pangkat Secretary Functions

Section 408. Subject Matter for Amicable Settlement; Exception Thereto. The lupon of each barangay shall have authority to bring together the parties actually residing in the same city or

D2010 168

UP College of Law
municipality for amicable settlement of all disputes except: (a) Where one party is the government, or any subdivision or instrumentality thereof; (b) Where one party is a public officer or employee, and the dispute relates to the performance of his official functions; (c) Offenses punishable by imprisonment exceeding one (1) year or a fine exceeding Five thousand pesos (P5,000.00); (d) Offenses where there is no private offended party; (e) Where the dispute involves real properties located in different cities or municipalities unless the parties thereto agree to submit their differences to amicable settlement by an appropriate lupon; (f) Disputes involving parties who actually reside in barangays of different cities or municipalities, except where such barangay units adjoin each other and the parties thereto agree to submit their differences to amicable settlement by an appropriate lupon; (g) Such other classes of disputes which the President may determine in the interest of Justice or upon the recommendation of the Secretary of Justice. The court in which non-criminal cases not falling within the authority of the lupon under this Code are filed may, at any time before trial motu propio refer the case to the lupon concerned for amicable settlement. Section 409. Venue. (a) Disputes between persons actually residing in the same barangay shall be brought for amicable settlement before the lupon of said barangay. (b) Those involving actual residents of different barangays within the same city or municipality shall be brought in the barangay where the respondent or any of the respondents actually resides, at the election of the complaint. (c) All disputes involving real property or any interest therein shall be brought in the barangay where the real property or the larger portion thereof is situated. (d) Those arising at the workplace where the contending parties are employed or at the institution where such parties are enrolled for study, shall be brought in the barangay where such workplace or institution is located. OBJECTIONS to venue shall be raised in the mediation proceedings before the punong barangay; otherwise, the same shall be deemed WAIVED. Any legal question which may confront the punong barangay in resolving objections to venue may be submitted to the Secretary of Justice, or his duly designated representative, whose ruling thereon shall be binding.

Section 410. Procedure for Amicable Settlement. (a) Who may initiate proceeding Upon payment of the appropriate filing fee any individual who has a cause of action against another individual involving any matter within the authority of the lupon may complain, orally or in writing, to the lupon chairman of the barangay.

(b) Mediation by lupon chairman Upon receipt of the complaint, the lupon chairman shall within the next working day summon the respondent(s), with notice to the complainant(s) for them and their witnesses to appear before him for a mediation of their conflicting interests. If he fails in his mediation effort within fifteen (15) days from the first meeting of the parties before him he shall forthwith set a date for the constitution of the pangkat.

(c) Suspension of prescriptive period of offenses While dispute is under mediation, conciliation, or arbitration, the prescriptive periods for offenses and cause of action INTERRUPTED upon filing the complaint with the punong barangay. Period RESUMES upon receipt by the complainant of the complainant or the certificate of repudiation or of the certification to file action issued by the lupon or pangkat secretary: Interruption shall not exceed sixty (60) days from the filing of the complaint with the punong barangay.

169 D2010

Local Government
(d) Issuance of summons; hearing; grounds for disqualification The pangkat shall convene not later than three (3) days from its constitution on the day and hour set by the lupon chairman to hear both parties and their witnesses, simplify issues, and explore all possibilities for amicable settlement. For this purpose, the pangkat may issue summons for the personal appearance of parties and witnesses before it. If party moves to disqualify any member of the pangkat by reason of relationship, bias, interest, or any other similar grounds discovered AFTER the constitution of the pangkat, the matter shall be resolved by the affirmative vote of the majority of the pangkat whose decision shall be final. Should disqualification be decided upon, the resulting vacancy shall be filled as herein provided for. or instituted directly in court or any other government office for adjudication unless there has been a confrontation between the parties before the lupon chairman or the pangkat and that no conciliation or settlement has been reached as certified by the lupon secretary or pangkat secretary as attested to by the lupon or pangkat chairman or unless the settlement has been repudiated by the parties thereto.

(b) Where Parties May Go Directly to Court. The parties may go directly to court in the following instances: 1. 2. Where the accused is under detention; Where a person has otherwise been deprived of personal liberty calling for habeas corpus proceedings; Where actions are coupled with provisional remedies such as preliminary injunction, attachment, delivery of personal property and support pendente lite; and Where the action may otherwise be barred by the statute of limitations.

3.

(e) Period to arrive at a settlement Within fifteen (15) days from the day the pangkat convenes in accordance with this section. This period is extendible at the discretion of the pangkat, which shall not exceed fifteen (15) days, except in clearly meritorious cases. Section 411. Form of settlement. All amicable settlements shall be in writing in a language or dialect known to the parties signed by them and attested to by the lupon chairman or the pangkat chairman, as the case may be. When the parties to the dispute do not use the same language or dialect, the settlement shall be written in the language known to them. 4.

(c) Conciliation among members of indigenous cultural communities. Customs and traditions of indigenous cultural communities SHALL be applied in settling disputes between members of the cultural communities. Section 413. Arbitration. (a) The parties may, at any stage of the proceedings, agree in writing that they shall abide by the arbitration award of the lupon chairman or the pangkat. Such agreement to arbitrate may be repudiated within five (5) days from the date thereof for the same grounds and in accordance with the procedure hereinafter prescribed. Arbitration award shall be made after the lapse of the period for repudiation and within ten (10) days thereafter.

Section 412. Conciliation. (a) Pre-condition to Filing of Complaint in Court. No complaint, petition, action, or proceeding involving any matter within the authority of the lupon shall be filed

(b) The arbitration award shall be in writing in a language or dialect known to the parties. When the parties to the dispute do not use the same language or dialect, the

D2010 170

UP College of Law
award shall be written in the language or dialect known to them. Section 414. Proceedings Open to the Public; Exception. All proceedings for settlement shall be public and informal. Provided, however, That the lupon chairman or the pangkat chairman, as the case may be, may motu proprio or upon request of a party, exclude the public from the proceedings in the interest of privacy, decency, or public morals. Section 415. Appearance of Parties in Person. Parties must appear in person without the assistance of counsel or representative, except for minors and incompetents who may be assisted by their next-of-kin who are not lawyers. Section 416. Effect of Amicable Settlement and Arbitration Award. The amicable settlement and arbitration award shall have the force and effect of a final judgment of a court upon the expiration of ten (10) days from the date thereof unless repudiation of the settlement has been made or a petition to nullify the award has been filed before the proper city or municipal court. This provision shall not apply to court cases settled by the lupon, in which case the compromise or the pangkat chairman shall be submitted to the court and upon approval thereof, have the force and effect of a judgment of said court. where the consent is vitiated by fraud, violence, or intimidation. Such repudiation shall be sufficient basis for the issuance of the certification for filing a complaint.

Section 419. Transmittal of Settlement and Arbitration. Award to the Court. - The secretary of the lupon shall transmit the settlement or the arbitration award to the appropriate city or municipal court within five (5) days from the date of the award or from the lapse of the ten-day period repudiating the settlement and shall furnish copies thereof to each of the parties to the settlement and the lupon chairman.

Section 420. Power to Administer Oaths. The punong barangay, as chairman of the lupong tagapamayapa, and the members of the pangkat are hereby authorized to administer oaths in connection with any matter relating to all proceedings in the implementation of the katarungang pambarangay. Section 421. Administration; Rules and Regulations. The city or municipal mayor, as the case may be, shall see to the efficient and effective implementation and administration of the katarungang pambarangay. The Secretary of Justice shall promulgate the rules and regulations necessary to implement this Chapter. Section 422. Appropriations. Such amount as may be necessary for the effective implementation of the katarungang pambarangay shall be provided for in the annual budget of the city or municipality concerned. CHAPTER VIII - Sangguniang Kabataan Section 423. Creation and Election. (a) There shall be in every barangay a sangguniang kabataan to be composed of a chairman seven (7) members a secretary a treasurer

Section 417. Execution. The amicable settlement or arbitration award may be enforced by execution by the lupon within six (6) months from the date of the settlement. After the lapse of such time, the settlement may be enforced by action in the appropriate city or municipal court. Section 418. Repudiation. Any party to the dispute may, within ten (10) days from the date of the settlement, repudiate the same by filing with the lupon chairman a statement to that effect sworn to before him

(b) A sangguniang kabataan official who, during his term of office, shall have passed the age of twenty-one (21) years shall be allowed to serve the remaining portion of

171 D2010

Local Government
the term for which he was elected. Section 424. Katipunan ng Kabataan. Composed of all citizens of the Philippines actually residing in the barangay for at least six (6) months who are fifteen (15) but not more than twenty-one (21) years of age duly registered in the list of the sangguniang kabataan or in the official barangay list in the custody of the barangay secretary. the implementation of youth development projects and programs at the national level; (h) Exercise such other powers and perform such other duties and functions as the sangguniang barangay may determine or delegate; and (i) Exercise such other powers and perform such other duties and functions as may be prescribed by law or ordinance. Section 427. Meetings of the Sangguniang Kabataan. Meet regularly once a month on the date, time, and place to be fixed by the said sanggunian. Special meetings may be called by the sangguniang kabataan chairman or any three (3) of its members by giving written notice to all members of the date, time, place and agenda of the meeting at least one (1) day in advance. Notices of regular or special meetings shall be furnished the punong barangay and the sangguniang barangay.

Section 425. Meetings of the Katipunan ng Kabataan. At least once every three (3) months or at the call of the chairman of the sangguniang kabataan or upon written petition of at least one-twentieth (1/20) of its member to decide on important issues affecting the youth of the barangay.

Section 426. Powers and Functions of the Sangguniang Kabataan. The sangguniang kabataan shall: (a) Promulgate resolutions necessary to carry out the objectives of the youth in the barangay in accordance with the applicable provisions of this Code; (b) Initiate programs designed to enhance the social, political, economic, cultural, intellectual, moral, spiritual, and physical development of the members; (c) Hold fund-raising activities, the proceeds of which shall be tax-exempt and shall accrue to the general fund of the sangguniang kabataan: Provided, however, That in the appropriation thereof, the specific purpose for which such activity has been held shall be first satisfied; (d) Create such bodies or committees as it may deem necessary to effectively carry out its programs and activities; (e) Submit annual and end-of-term reports to the sangguniang barangay on their projects and activities for the survival and development of the youth in the barangay; (f) Consult and coordinate with all youth organizations in the barangay for policy formulation and program implementation; (g) Coordinate with the appropriate national agency for

A majority of the members of the sangguniang kabataan shall constitute a quorum. Section 428. Qualifications. An elective official of the sangguniang kabataan must be a citizen of the Philippines a qualified voter of the katipunan ng kabataan a resident of the barangay for at least one (1) year immediately prior to election at least fifteen (15) years but not more than twenty- one (21) years of age on the day of his election able to read and write Filipino, English, or the local dialect must not have been convicted of any crime involving moral turpitude.

Section 429. Term of Office. Three (3) years, unless sooner removed for cause as provided by law, permanently incapacitated, die or resign from office. Section 430. Sangguniang Kabataan Chairman. The registered voters of the katipunan ng kabataan shall elect the chairman of the sangguniang kabataan who shall automatically serve as an ex officio member of the

D2010 172

UP College of Law
sangguniang barangay upon his assumption to office. As such, he shall exercise the same powers, discharge the same duties and functions, and enjoy the same privileges as the regular sangguniang barangay members, and shall be the chairman of the committee on youth and sports development in the said sanggunian. Section 431. Powers and Duties of the Sangguniang Kabataan Chairman. (a) Call and preside over all meetings of the katipunan ng kabataan and the sangguniang kabataan; (b) Implement policies, programs, and projects within his jurisdiction in coordination with the sangguniang barangay; (c) Exercise general supervision over the affairs and activities of the sangguniang kabataan and the official conduct of its members, and such other officers of the sangguniang kabataan within his jurisdiction; (d) With the concurrence of the sangguniang kabataan, appoint from among the members of the sangguniang kabataan, the secretary and treasurer and such other officers as may be deemed necessary; and (e) Exercise such other powers and perform such other duties and functions as may be prescribed by law or ordinance. Section 432. Sangguniang Kabataan Secretary. (a) Keep all records of the katipunan ng kabataan and sangguniang kabataan; (b) Prepare and keep the minutes of all meetings of the katipunan ng kabataan and sangguniang kabataan; (c) Prepare all forms necessary for the conduct of registrations, elections, initiatives, referenda, or plebiscites, in coordination with the barangay secretary and the COMELEC; and (d) Perform such other duties and discharge such other functions as the chairman of the sangguniang kabataan may prescribe or direct. Section 433. Sangguniang Kabataan Treasurer. (a) Take custody of all sangguniang kabataan property and funds not otherwise deposited with the city or municipal treasurer; (b) Collect and receive contributions, monies, materials, and all other sources intended for the sangguniang kabataan and katipunan ng kabataan; (c) Disburse funds in accordance with an approved budget of the sangguniang kabataan; (d) Certify to the availability of funds whenever necessary; (e) Submit to the sangguniang kabataan and to the sangguniang barangay certified and detailed statements of actual income and expenditures at the end of every month; and (f) Perform such other duties and discharge such other functions as the chairman of the sangguniang kabataan may direct. Section 434. Privileges of Sangguniang Kabataan Officials. Same privileges enjoyed by other sangguniang barangay officials under this Code subject to such requirements and limitations provided herein. During their incumbency, sangguniang kabataan officials exempt from payment of tuition and matriculation fees while enrolled in public tertiary schools, including state colleges and universities. The national government shall reimburse said college or university the amount of the tuition and matriculation fees: Provided, That, to qualify for the privilege, the said officials shall enroll in a state college or university within or nearest their area of jurisdiction.

Section 435. Succession and Filling of Vacancies. (a) In case a sangguniang kabataan chairman refuses to assume office fails to qualify is convicted of a felony voluntarily resigns dies is permanently incapacitated is removed from office or has been absent without leave for more than three (3) consecutive months

the sangguniang kabataan member who obtained the next highest number of votes in the election immediately preceding shall assume the office of the chairman for the unexpired portion of the term discharge the powers and duties

173 D2010

Local Government
enjoy the rights and privileges appurtenant to the office. 2. The panlalawigang pederasyon shall be composed of presidents of the panlungsod and pambayang pederasyon; The pangmetropolitang pederasyon shall be composed of presidents of the panlungsod and pambayan pederasyon;

In case the said member refuses to assume the position or fails to qualify, the sangguniang member obtaining the next highest number of votes shall assume the position of the chairman for the unexpired portion of the term. (b) Where two (2) or more sangguniang kabataan members obtained the same next highest number of votes, the other sangguniang kabataan members shall conduct an election to choose the successor to the chairman from among the said members. (c) After the vacancy shall have been filled, the sangguniang kabataan chairman shall call a special election to complete the membership of said sanggunian. Such sangguniang kabataan member shall hold office for the unexpired portion of the term of the vacant seat. (d) In case of suspension of the sangguniang kabataan chairman, the successor, as determined in subsections (a) and (b) of this Section shall assume the position during the period of such suspension. CHAPTER IX - Pederasyon ng mga Sangguniang Kabataan Section 436. Pederasyon ng mga Kabataan. (a) There shall be an organization of all the pederasyon ng mga sangguniang kabataan to be known as follows: 1. 2. 3. 4. in municipalities pambayang pederasyon ng mga sangguniang kabataan; in cities, panlungsod na pederasyon ng mga sangguniang kabataan; in provinces, panlalawigang pederasyon ng mga kabataan; in special metropolitan political subdivisions, pangmetropolitan pederasyon ng mga sangguniang kabataan; and on the national level pambansang pederasyon ng mga sangguniang kabataan.

3.

(c) The elected presidents of the pederasyon at the provincial, highly urbanized city, and metropolitan political subdivision levels shall constitute the pambansang katipunan ng mga sangguniang kabataan. Section 437. Constitution and By-Laws. The term of office, manner of election, removal and suspension of the officers of the pederasyon ng mga sangguniang kabataan at all levels shall be governed by the constitution and by-laws of the pederasyon in conformity with the provisions of this Code and national policies on youth. Section 438. Membership in the Sanggunian. (a) A sangguniang kabataan chairman shall, upon certification of his election by the COMELEC and during his tenure of office is elected as pederasyon president, serve as an ex-officio member of the sangguniang panlalawigan, sangguniang panlungsod, and sangguniang bayan, as the case may be, without need of further appointment. (b) The vice-president of the pederasyon whose president has been elected as president of a higher pederasyon shall serve as ex-officio member of the sanggunian concerned without need of further appointment. (c) The pederasyon president or vice-president, as the case may be, shall be the chairman of the committee on youth and sports development of the sanggunian concerned. CHAPTER X - Linggo ng Kabataan Section 439. Observance of Linggo ng Kabataan. (a) Every barangay, municipality, city and province shall, in coordination with the pederasyon ng mga sangguniang kabataan at all levels, conduct an annual activity to be known as the Linggo ng Kabataan on such date as shall be determined by the Office of the President. (b) The observance of the Linggo ng Kabataan shall include the election of the counterparts of all local elective and appointive officials, as well as heads of national offices or agencies stationed or assigned in the territorial jurisdiction of the local government unit, among in-school and community youth residing in the

5.

(b) The pederasyon ng mga sangguniang kabataan shall, at all levels, elect from among themselves the president, vice- president and such other officers as may be necessary and shall be organized in the following manner: 1. The panlungsod and pambayang pederasyon shall be composed of the sangguniang kabataan chairmen of barangays in the city or municipality, respectively;

D2010 174

UP College of Law
local government unit concerned from ages thirteen (13) to seventeen (17). During said week, they shall hold office as boy and girl officials and shall perform such duties and conduct such activities as may be provided in the ordinance enacted pursuant to this Chapter. d. Placing the woman or her child in fear of imminent physical harm; e. Attempting to compel or compelling the woman or her child to engage in conduct which the woman or her child has the right to desist from or desist from conduct which the woman or her child has the right to engage in, or attempting to restrict or restricting the woman's or her child's freedom of movement or conduct by force or threat of force, physical or other harm or threat of physical or other harm, or intimidation directed against the woman or child. This shall include, but not limited to, the following acts committed with the purpose or effect of controlling or restricting the woman's or her child's movement or conduct: i. Threatening to deprive or actually depriving the woman or her child of custody to her/his family; ii. Depriving or threatening to deprive the woman or her children of financial support legally due her or her family, or deliberately providing the woman's children insufficient financial support; iii. Depriving or threatening to deprive the woman or her child of a legal right; iv. Preventing the woman in engaging in any legitimate profession, occupation, business or activity or controlling the victim's own mon4ey or properties, or solely controlling the conjugal or common money, or properties; f. Inflicting or threatening to inflict physical harm on oneself for the purpose of controlling her actions or decisions; g. Causing or attempting to cause the woman or her child to engage in any sexual activity which does not constitute rape, by force or threat of force, physical harm, or through intimidation directed against the woman or her child or her/his immediate family;

A. Katarungang Pambarangay

Sec 399 422, LGC


See above

The Laws on VAW in the Philippines (expert paper of Mam Guanzon)


See attachments

Sec 14, Sec 5, RA 9262


SECTION 14. Barangay Protection Orders (BPOs); Who May Issue and How. - Barangay Protection Orders (BPOs) refer to the protection order issued by the Punong Barangay ordering the perpetrator to desist from committing acts under Section 5 (a) and (b) of this Act. A Punong Barangay who receives applications for a BPO shall issue the protection order to the applicant on the date of filing after ex parte determination of the basis of the application. If the Punong Barangay is unavailable to act on the application for a BPO, the application shall be acted upon by any available Barangay Kagawad. If the BPO is issued by a Barangay Kagawad the order must be accompanied by an attestation by the Barangay Kagawad that the Punong Barangay was unavailable at the time for the issuance of the BPO. BPOs shall be effective for fifteen (15) days. Immediately after the issuance of an ex parte BPO, the Punong Barangay or Barangay Kagawad shall personally serve a copy of the same on the respondent, or direct any barangay official to effect is personal service. The parties may be accompanied by a non-lawyer advocate in any proceeding before the Punong Barangay. SECTION 5. Acts of Violence Against Women and Their Children.- The crime of violence against women and their children is committed through any of the following acts: a. Causing physical harm to the woman or her child; b. Threatening to cause the woman or her child physical harm; c. Attempting to cause the woman or her child physical harm;

175 D2010

Local Government
h. Engaging in purposeful, knowing, or reckless conduct, personally or through another, that alarms or causes substantial emotional or psychological distress to the woman or her child. This shall include, but not be limited to, the following acts: i. Stalking or following the woman or her child in public or private places; ii. Peering in the window or lingering outside the residence of the woman or her child; iii. Entering or remaining in the dwelling or on the property of the woman or her child against her/his will; iv. Destroying the property and personal belongings or inflicting harm to animals or pets of the woman or her child; and v. Engaging in any form of harassment or violence; Causing mental or emotional anguish, public ridicule or humiliation to the woman or her child, including, but not limited to, repeated verbal and emotional abuse, and denial of financial support or custody of minor children of access to the woman's child/children. MeTC decided in favor of Aquino on the ground of noncompliance with the barangay conciliation process. RTC Affirmed this decision. However, the Court of Appeals REVERSED and reasoned that the failure of Aure to undergo barangay conciliation is not a jurisdictional flaw and it will not affect the sufficiency of Aures Complaint since Aquino failed to seasonably raise such issue in her Answer. HELD: The primordial objective of barangay conciliations is to reduce the number of court litigations and prevent the deterioration of the quality of justice which has been brought by the indiscriminate filing of cases in the courts. To ensure this, the law requires the parties to undergo a conciliation process as a precondition to filing a complaint in court subject to certain exceptions which are inapplicable to this case. This has been declared compulsory in nature. However, the conciliation process is not a jurisdictional requirement, so that non-compliance therewith cannot affect the jurisdiction which the court has otherwise acquired over the subject matter or over the person of the defendant; in other words, the same would not prevent a court of competent jurisdiction from exercising its power of adjudication over the case before it, where the defendants, as in this case, failed to object to such exercise of jurisdiction in their answer and even during the entire proceedings a quo. In the case at bar, Aquino cannot be allowed to attack the jurisdiction of the MeTC after having submitted herself voluntarily thereto. An examination of Aquinos Answer before the MeTC shows that there is utter lack of any objection on her part to any deficiency in the complaint which could oust the MeTC of its jurisdcition. The fact that Aquino raised such objection during the pre-trial and in her Position Paper is of no moment, for the issue of non-recourse to barangay mediation proceedings should be impleaded in her Answer. Thus, although Aquinos defense is meritorious, procedurally, such defense is no longer available for failure to plead the same in the Answer as required by the omnibus motion rule. Neither could the MeTC dismiss the case motu proprio. The 1997 Rules of Civil Procedure provide only three instances when the court may motu proprio dismiss the claim. It is clear that a court may not motu proprio dismiss a case on the ground of failure to comply with the requirement for barangay conciliation, this ground not being among those mentioned for the dismissal by the trial court of a case on its own initiative.

i.

Aquino v. Aure
Aure and E.S. Aure Lending Investors, Inc. (Aure Lending) filed a Complaint for ejectment against Aquino before the MeTC, alleging that they acquired the subject property from the spouses Aquino by virtue of a Deed of Sale. However, after the spouses Aquino received substantial consideration for the sale of the subject property, they refused to vacate the same. In her Answer, Aquino countered that Aure Lending do not have any legal right over the property, as per their Memorandum of Agreement, Aure shall secure a loan from a bank or financial institution in his own name using the subject property as collateral and turn over the proceeds thereof to the spouses Aquino. However, even after Aure successfully secured a loan, the spouses Aquino did not receive the proceeds thereon or benefited therefrom.

D2010 176

UP College of Law Morata v. Go


Respondents Victor Go and Flora D. Go filed in the CFI of Cebu a complaint against petitioners Morata for recovery of a sum of money plus damages. On the basis of the allegation in the complaint that the parties-litigants are all residents of Cebu City, petitioners filed a motion to dismiss, citing as ground the failure of the complaint to allege prior availment by the plaintiffs of the barangay conciliation process required by P.D. 1508, as well as the absence of a certification by the Lupon or Pangkat Secretary that no conciliation or settlement had been reached by the parties. HELD: Except in the instances enumerated in sections 2 and 6 of the law, the Lupon has the authority to settle amicably all types of disputes involving parties who actually reside in the same city or municipality. The law makes no distinction whatsoever with respect to the classes of civil disputes that should be compromised at the barangay level. In fact, in defining the Lupon's authority, Section 2 of said law employed the universal and comprehensive term "all", to which usage the court should neither add nor subtract in consonance with the rudimentary precept in statutory construction that "where the law does not distinguish, the court should not distinguish. The conciliation process at the barangay level is designed to discourage indiscriminate filing of cases in court in order to decongest its clogged dockets and, in the process, enhance the quality of justice dispensed by it. Thus, to say that the authority of the Lupon is limited to cases exclusively cognizable by the inferior courts is to lose sight of this objective. Moreover, if it is the intention of the law to restrict its coverage only to cases cognizable by the inferior courts, then it would not have provided in Section 3 thereof the rule on Venue, which looks to the location of the real property in the determination of venue, for it should be noted that, traditionally and historically, jurisdiction over cases involving real property or any interest therein, except forcible entry and detainer cases, has always been vested in the courts of first instance [now regional trial court]. Sections 11, 12 and 14, relied upon by respondent judge, deal with the nullification or execution of the settlement or arbitration awards obtained at the barangay level. These sections conferred upon the city and municipal courts the jurisdiction to pass upon and resolve petitions or actions for nullification or enforcement of settlement/arbitration awards issued by the Lupon, regardless of the amount involved or the nature of the original dispute. But there is nothing in the context of said sections to justify the thesis that the mandated conciliation process in other types of cases applies exclusively to said inferior courts. Lastly, the circular issued by then Chief Justice embodying the directive "to desist from receiving complaints, petitions, actions and proceedings in cases falling within the authority of said Lupons," has been addressed not only to judges of city and municipal courts, but also to all the judges of the courts of first instance, circuit criminal courts, juvenile and domestic courts and courts of agrarian relations, now known as regional trial courts. This clearly shows that conciliation process at the barangay level, prescribed by P.D. 1508 as a pre-condition for filing a complaint in court, is compulsory not only for cases falling under the exclusive competence of the metropolitan and municipal trial courts, but for actions cognizable by the regional trial courts as well.

Uy v. Contreras
An argument arose between the petitioner and respondent when the former sought to withdraw from premises of the latter certain movable properties that the petitioner failed to remove despite the expiry of their sublease agreement. This led to a scuffle between the parties respective employees, which allegedly resulted in injuries inflicted on the private respondents. The private respondents then filed a complaint with the barangay captain of Valenzuela, Makati, however, during their scheduled confrontation before the barangay captain, only the petitioner appeared. The prosecutor then filed two informations for slight physical injuries against the petitioner with the MTC of Makati. Petitioner alleged in a motion to dismiss the prematurity of the filing of the criminal cases for failure to undergo conciliation proceedings. On the other hand, private respondents contend that a denial of motion to dismiss is proper because prior referral of the dispute to the lupon is not applicable since she and petitioner are not residents of barangays in the same city or municipality or of adjoining barangays in different cities or municipalities and that referral to the lupon is not likewise required if the case may otherwise be barred by the statute of limitations. Moreover, even assuming arguendo that prior referral to the lupon applies to the case of private respondent, the latter had, nevertheless, substantially

177 D2010

Local Government
complied with the requirement with the subsequent certification of the barangay to file the action. HELD: While P.D. No. 1508 has been repealed by the L GC of 1991, the jurisprudence built thereon regarding prior referral to the lupon as a pre-condition to the filing of an action in court remains applicable because its provisions on prior referral were substantially reproduced in the Code. In view of the respondents' failure to appear at the scheduled mediation, no complaint for slight physical injuries could be validly filed with the MTC of Makati at any time before such date. The filing then of criminal cases was premature. In addition, Section 6 of P.D. No. 1508 (more properly, Section 412(b)(4) of the LGC) which states that the parties may go directly to court where the action is about to prescribe, cannot justify the dismissal of the case. This is because pursuant to paragraph (c), Section 410 of the Code, the prescriptive period was automatically suspended for a maximum period of sixty days. Moreover, having brought the dispute before the lupon of barangay Valenzuela, Makati, the private respondents are estopped from disavowing the authority of the body which they themselves had sought. Their act of trifling with the authority of the lupon by unjustifiably failing to attend the scheduled mediation hearings and instead filing the complaint right away with the trial court cannot be countenanced for to do so would wreak havoc on the barangay conciliation system. Neither is the argument that petitioner "had already waived the right to a reconciliation proceedings before the barangay, persuasive. The petitioner did not waive the reconciliation proceedings before the lupon of Valenzuela, Makati; she submitted to it and attended the scheduled conciliation and invoked the pre-condition of referral to the lupon in her counter-affidavit. Lastly, nor could the Court accept the contention of the respondent that the parties could not agree on a compromise and that they had to request the barangay captain to issue a certification to file action. The request was nearly one and a half months after criminal cases were filed with the court a quo. Evidently, this was done to support their contention that, in any event, there was substantial compliance with the requirement of referral to the lupon. It must be stressed that the private respondents, after failing to appear at the initial confrontation and long after the criminal cases were filed, had no right to demand the issuance of a certification to file action.

Wingarts v. Mejia
These administrative complaints were an offshoot of criminal cases decided by the respondent judge. The respondent judge is charged with incompetence, ignorance of the law and abuse of authority for taking cognizance of a criminal case for grave threats and issuing a warrant of arrest against the accused despite the lack of prior barangay conciliation. The respondent judge explained that he took cognizance of the criminal case in the belief that there had been substantial compliance of the requirements of the Katarungang Pambarangay Law since a certification of the barangay captain regarding a confrontation of the partiesd, the fact that no amicable settlement was reached by them, and that he was endorsing the filing of the case in court, had been duly submitted to respondent judge. HELD: Under the LGC of 1991, offenses punishable by imprisonment not exceeding 1 year or a fine not exceeding Php5,000 require prior barangay conciliation. The crime of grave threats punishable under Art. 282 of the Revised Penal Code fall within the purview of this section. Furthermore, Sec. 412 (a) also requires the same mandate. Therefore, respondent judge should have remanded the case to the lupon instead of taking cognizance thereof and prematurely issuing a warrant of arrest against the accused. The respondent judge is liable for incompetence and ignorance of the law for taking cognizance of this case especially since judges are directed from improvidently receiving and acting on complaints in cases falling within the authority of the Lupon. It is a well-settled rule that proceedings before the lupon are a precondition to the filing of any action or proceeding in court or other government office. Such an initiatory pleading, if filed without compliance with the precondition, may be dismissed on the motion of any interested party on the ground that it fails to state a cause of action. Although there is no clear proof of malice or bad faith, respondent judge should have exercised the requisite prudence, which he owes to the public and his profession, especially in a case where personal liberty of the accused is involved.

Corpuz v. CA
Carlito Corpuz filed an action for unlawful detainer against private respondent Juanito Alvarado with the

D2010 178

UP College of Law
MTC of Manila, for recovery of possession of the room being occupied by the latter, which Corpuz children allegedly needed for their own use. Finding the defenses of Alvarado to be without merit, the MTC ordered Alvarado to vacate the room. Alvarado raises the issue in the instant petition that the ejectment suit was not referred to the Lupon Tagapayapa as required by P.D. No. 1508 HELD: Alvarados defense was only stated in a single short sentence in his answer. In Dui vs. CA, the Court held that failure of a party to specifically allege the fact that there was no compliance with the barangay conciliation procedure constitutes waiver of that defense. A perusal of Alvarados answer reveals that no reason or explanation was given to support his allegation, which is deemed a mere general averment. In any event, the proceeding outlined in P.D. 1508 is not a jurisdictional requirement and non-compliance therewith cannot affect the jurisdiction which the lower court has already acquired over the subject matter and the parties therein. of the parties had taken place before it is substantiated by the Minutes submitted by complainant. Evidently, complainant failed to complete the barangay conciliation proceedings. In addition, the Complaint before the barangay was dated February 16, 1996. Records show that the hearing was scheduled for February 26, 1996 and was reset for February 29, 1996. And yet, the Certification to File Action was issued on March 1, 1996, less than fifteen days after the first scheduled hearing before the barangay chairman in contravention of Section 410 (b) of the LGC, requiring a mediation effort within 15 days from the first meeting. Evidently, the barangay failed to exert enough effort required by law to conciliate between the parties and to settle the case before it. Hence, respondent judge was correct in remanding the case to it for completion of the mandated proceedings. We cannot fault him for seeking to promote the objectives of barangay conciliation and for taking to heart the provisions of Supreme Court Circular No. 14-93. His referral of the case back to the barangay cannot be equated with gross ignorance of the law. Neither does it constitute grave abuse of discretion or obvious partiality. Despite this, however, respondent judge failed to comply with the requirements of the Rules on Summary Procedure when, acting on the complainants motion to consider the proceedings already held before the barangay as substantial compliance with the requirements of law, he chose to continue with the proceedings of the case, and failed to render a judgment within 30 days from the failure of the respondents to answer. This undue delay constitutes a less serious charge of gross inefficiency and warrants the imposition of administrative sanction.

Bonifacio Law Office v. Judge Bellosillo


In a letter-complaint, Atty. Salomon, Jr. charged Judge Bellosillo with ignorance of the law, grave abuse of discretion and obvious partiality and assailed the order of the said judge, which referred an ejectment case back to the barangay for conciliation proceedings despite the fact that it was alleged in the verified complaint, that the matter had already been referred to the barangay and that a copy of the Certification to File Motion was attached. In its Answer, the judge denied the charges and averred that there was premature issuance of the Certificate to File Action considering that there is no proof to show that the Pangkat was duly constituted before the said certificate was issued. Moreover, the belated submission by complainant of the Minutes of Proceedings before the Barangay Chairman, which was inaccurate and difficult to decipher reveals the noncompliance of complainant with the requirement of the law. HELD: The records reveal that the Certification to File Action was improperly and prematurely issued as it clearly shows that no personal confrontation before a duly constituted Pangkat ng Tagapagkasundo took place. This supports the respondents position that the Pangkat was not constituted, and that no face to face conciliation

Mendoza v. Judge Afable


Mendoza alleged that on February 18, 1998, he filed with the Office of the Barangay Chairman a complaint for slight physical injuries against Palada, however, despite the hearings conducted thereon, the parties failed to reach an amicable settlement. Therefore, on May 4, 1998 complainant Mendova filed a complaint for slight physical injuries before the MTC. However, the case was dismissed by Judge Afable on the ground of prescription (the complaint alleging a light offense, which prescribes in two months). In this administrative case, the complainant alleged that, in dismissing the case, Judge Afable showed his

179 D2010

Local Government
ignorance of the law when he did not apply the provisions of Section 410(c) of the LGC, which suspends the prescriptive period of offences upon the filing of the complaint with the Punong Barangay. In his Answer, Judge Afable merely admitted his error and claimed a mere mental lapse on his part. HELD: It is axiomatic that an administrative complaint is not the appropriate remedy for every irregular or erroneous order or decision issued by a judge where a judicial remedy is available, such as a motion for reconsideration, or an appeal. For, obviously, if subsequent developments prove the judges challenged act to be correct, there would be no occasion to proceed against him at all. Besides, to hold a judge administratively accountable for every erroneous ruling or decision he renders, assuming he has erred, would be nothing short of harassment and would make his position doubly unbearable. To hold otherwise would be to render judicial office untenable, for no one called upon to try the facts or interpret the law in the process of administering justice can be infallible in his judgment. It is only where the error is so gross, deliberate and malicious, or incurred with evident bad faith that administrative sanctions may be imposed against the erring judge. In the present case, the complainant did not bother at all to file a motion for reconsideration of respondent judges decision dismissing the criminal case. No reason was advanced by complainant why he failed to do so. Thus, following our settled pronouncements cited above, his instant administrative complaint is premature. In addition, records fail to show when complainant received the Barangay Certification to File Action. The undated certification he submitted merely states that the case was set for hearing before the barangay on March 16, 22 and 29, 1998, but the parties failed to reach an amicable settlement. When he filed on May 4, 1998 the criminal case for slight physical injuries with respondent's court, until the dismissal of the case on November 3, 1998, he still failed to present proof of his receipt of the Barangay Certification to File Action. Clearly, he cannot now fault respondent judge for dismissing the case on the ground of prescription. While respondent admitted his mistake, the same may not be considered ignorance of the law. If at all, it can only be an error of judgment. Finally, we noted that the complaint does not allege any bad faith or malice on the part of respondent judge when he dismissed the criminal case. B. Sangguniang Kabataan

Sec 423 439, LGC


See above

Monteclaros v. Comelec
SK, previously known as the Kabataang Barangay is a youth organization established by PD 684. The KB was composed of residents less than 18 years old, with no minimum age specified. The LGC renamed the KB to SK and limited membership to those aged 15-21 years. It is tasked to initiate programs to enhance the social, political, economic cultural, intellectual, moral, spiritual, and physical development of the youth. It is composed of a chairperson and 7 members to be elected by the Katipunan ng Kabataan, in turn composed of citizens actually residing in the barangay for at least 6 months who meet the age requirement. RA 7808 scheduled the SK elections on the 1st Monday of 1996 and every 3 years thereafter. The next elections are supposed to be held on May 6, 2002. A bill by the Bicameral Committee however reset the SK and barangay elections to July 15, 2002. The said bill also lowered the membership age to at least 15 but not more than 18 years old. Montesclaros et al. filed a petition for certiorari, prohibition and mandamus with prayer for TRO or preliminary injunction to: 1) prevent the postponement of the SK elections originally scheduled for May 6, 2002, 2) to prevent respondents from passing laws that reduce the age requirement membership in the SK, and 3) to compel respondents to allow those who have turned 21 on May 2002 to participate in any rescheduled SK elections. They allege that those aged 18-21 will be unduly dismembered, unfairly discriminated, unnecessarily disenfranchised, unjustly associated and obnoxiously disqualified from the SK. They also allege grave abuse of discretion on the COMELECs part. The bill was approved by Congress and signed into law as RA 9164 on March 19, 2002. HELD: The petition is bereft of merit. There is no actual controversy as to the date of the elections that requires judicial intervention. Neither was there grave abuse of discretion in the postponement of the elections. The prayer to prevent Congress from enacting laws that lower the age requirement cannot be granted because a proposed bill is not subject to judicial review. The court can only exercise judicial review after and not before a law is enacted, lest it resort to rendering advisory opinions. Absent a clear violation of constitutional rights, Courts cannot exercise review over the internal

D2010 180

UP College of Law
processes of Congress. The Court has no power to dictate the object or subject of bills that Congress should enact into law. Public office is a public trust. It is not a property right. No one has a vested right to any public office, much less the expectancy of holding a public office. While the state encourages the youths involvement in public affairs, the policy refers to those who belong to the class of people defined as the youth. Congress has the power to define who are the youth qualified to join the SK. Every law is subject to amendment or repeal and those who no longer qualify for the SK because they are past the age limit cannot insist on being part of the youth. Under PD 9164, Congress merely restored the maximum age requirement of 18 years in PD684, SKs original charter. RA 9164 enjoys the presumption of constitutionality.

THE MUNICIPALITY, CITY AND PROVINCE


MUNICIPALITY a. Composition Group of barangays [440] CITY Group of more urbanized and developed barangays [448] PROVINCE Cluster of municipalities, or municipalities and component cities [459] As a political and corporate of government serves as a dynamic mechanism for developmental processes and effective governance of LGUs within its territorial jurisdiction [459] Same as the municipality [460]

b. Role As a general-purpose government for the Same as the municipality[448] coordination and delivery of basic, regular and direct services and effective governance of the inhabitants within its territorial jurisdiction [440] c. How created, divided, merged, abolished or its boundary substantially altered Only by an Act of Congress and subject to the Same as the municipality [449] approval by a majority of the votes cast in a plebiscite conducted by the COMELEC in the LGU/s directly affected The plebiscite shall be held 120 days from the date of its effectivity except as may otherwise be provided in the Act. [441] d. Requisites for creation Ave. annual income of at least Ave. annual income of at least P2,500,000.00 for the last 2 consecutive P20,000,000.00 for the last 2 consecutive years based on the 1991 constant prices, as years based on the 1991 constant prices, as certified by the DOF certified by the DOF - includes the income accruing to the - includes the income accruing to the gen. fund gen. fund - excludes special funds, transfers and - excludes special funds, transfers and non-recurring income non-recurring income Population of at least 25,000 inhabitants as Either of: certified by the NSO Population of at least 150,000 inhabitants A contiguous territory of at least 50 sq. kms. as certified by the NSO as certified by the LMB A contiguous territory of at least 100 sq. kms. as certified by the LMB should be properly identified by metes and bounds should be properly identified by metes and bounds does not apply if there is 1 or more islands does not apply if there is 1 or more islands need not be contiguous if thereare 2 need not be contiguous if there are 2 or more islands or more islands Municipalities existing as of the date of the Provided, that, the creation thereof shall effectivity of the LGC shall continue to exist not reduce the land area, population, and and operate as such. Existing municipal income of the original unit/s at the time of districts organized prusuant to presidential said creation to less than the min. reqts. issuances or EOs and which have their prescribed. [450] respective set of elective municipal officials holding office at the time of the effectivity of For HIGHLY URBANIZED CITIES: the LGC shall be considered as regular Ave. annual income of at least

Ave. annual income of at least P20,000,000.00 for the last 2 consecutive years based on the 1991 constant prices, as certified by the DOF - includes the income accruing to the gen. fund - excludes special funds, transfers and non-recurring income Either of: Population of at least 250,000 inhabitants as certified by the NSO A contiguous territory of at least 2,000 sq. kms. as certified by the LMB should be properly identified by metes and bounds does not apply if there is 1 or more islands need not be contiguous if there are 2 or more islands Provided, that, the creation thereof shall not reduce the land area, population, and income of the original unit/s at the time of said creation to less than the min. reqts. prescribed. [461] Existing sub-provinces are converted into regular provinces upon approval by a

181 D2010

Local Government
municipalities. [442] P50,000,000.00 for the last 2 consecutive years based on the 1991 constant prices, as certified by the DOF Population of at least 200,000 inhabitants as certified by the NSO [452] * It shall be the duty of the President to declare a city as highly urbanized within 30 days after it shall have met the min. reqts. upon properapplication therefor and ratification in a plebiscite by qualified voters therein. [453] e. Officials mayor vice-mayor sangguniang bayan members secretary to the sangguniang bayan treasurer assessor accountant budget officer planning and devt. coordinator engineer/building official health officer civil registrar Optional: administrator legal officer agriculturist environment and natural resources officer social welfare and devt. officer architect information officer The sangguniang bayan may: 1. maintain existing offices not mentioned above 2. create such other offices as may be necessary to carry out the purposes of the provincial government 3. consolidate the functions of any office with those of another in the interest of efficiency and economy Unless, otherwise provided herein, heads of departments and offices shall be appointed by the municipal mayor with the concurrence of the majority of all the sangguniang bayan members, subject to civil service law, rules and regulations. The sangguniang bayan shall act on the appointment within 15 days from the date of its submission; otherwise, the same shall be confirmed. [443] mayor vice-mayor sangguniang panlungsod members secretary to the sangguniang panlungsod treasurer assessor accountant budget officer planning and devt. coordinator engineer health officer civil registrar administrator legal officer social welfare and devt. officer gen. services officer veterinarian governor vice-governor sangguniang panlalawigan members secretary to the sangguniang panlalawigan treasurer assessor accountant budget officer planning and devt. coordinator engineer health officer administrator legal officer social welfare and devt. officer gen. services officer agriculturist veterinarian majority of the votes cast in a plebiscite to be held in the said sub-provinces and the original provinces directly affected. The plebiscite shall be conducted by the COMELEC simultaneously with the national elections following the effectivity of the LGC. [462]

Optional: architect information officer agriculturist population officer - cities which have existing population offices shall continue to maintain such ofices for 5 years from the date of effectivity of the LGC, after which said office shall become optional environment and natural resources officer cooperatives officer population officer The sangguniang panlungsod may: 1. maintain existing offices not mentioned above 2. create such other offices as may be necessary to carry out the purposes of the provincial government 3. consolidate the functions of any office with those of another in the interest of efficiency and economy Unless, otherwise provided herein, heads of departments and offices shall be appointed by the city mayor with the concurrence of the majority of all the sangguniang panlungsod members, subject to civil service law, rules and regulations. The sangguniang panlungsod shall act on the appointment within 15 days from the date of its submission; otherwise, the same shall be confirmed. [454]

Optional: population officer - cities which have existing population offices shall continue to maintain such ofices for 5 years from the date of effectivity of the LGC, after which said office shall become optional natural resources and envrionment officer cooperative officer architect information officer The sangguniang panlalawigan may: 1. maintain existing offices not mentioned above 2. create such other offices as may be necessary to carry out the purposes of the provincial government 3. consolidate the functions of any office with those of another in the interest of efficiency and economy Unless, otherwise provided herein, heads of departments and offices shall be appointed by the governor with the concurrence of the majority of all the sangguniang panlalawigan members, subject to civil service law, rules and regulations. The sangguniang panlalawigan shall act on the appointment within 15 days from the date of its submission; otherwise, the same shall be confirmed. [463]

D2010 182

UP College of Law
f. Composition of the sanggunian Presiding officer: municipal vice-mayor Members: the regular sanggunian members the president of the municipal chapter of the liga ng mga barangay the president of the pambayang pederasyon ng mga SK sectoral representatives 1. 1 from the women 2. 1 from the agricultural or industrial workers 3. 1 from other sectors, including the urban poor, indigenous cultural communities, or disabled persons [446]

Presiding officer: city vice-mayor Members: the regular sanggunian members,the president fothe city chapter of the liga ng mga barangay the president of the panlungsod na pederasyon ng mga SK the sectoral representatives 1. 1 from the women 2. 1 from the agricultural or industrial workers 3. 1 from other sectors, including the urban poor, indigenous cultural communities, or disabled persons [457]

Presiding officer: provincial vice-governor Members: the regular sanggunian members the president of the provincial chapter of the liga ng mga barangay the president of the panlalawigang pederasyon ng mga SK the president of the provincial federation of sangguniang members of municipalities and component cities sectoral representatives 1. 1 from the women 2. 1 from the agricultural or industrial workers 3. 1 from other sectors, including the urban poor, indigenous cultural communities, or disabled persons [467]

g. Salary grades as prescribed under RA 6758 Municipal mayor: Salary Grade 27 [444(d)] Muncipal vice-mayor: Salary Grade 25 [445(b)] Members of the sangguniang bayan: Salary Grade 24 Members of the sanggunian of municipalities in Metropolitan Manila Area and other metropolitan political subdivisions: Salary Grade 25 [447(b)]

City mayor: Salary Grade 30 [455(d)] City vice-mayor of a highly urbanized city: Salary Grade 28 City vice-mayor of a component city: Salary Grade 26 [456(b)] Members of the sangguniang panlungsod: Salary Grade 25 Members of the sanggunian of highlyurbanized cities: Salary Grade 27 [458(b)]

Provincial governor: Salary Grade 30 [465(c)] Vice-governor:Salary Grade 28 [466(b)] Members of the sangguniang panlalawigan: Salary Grade 27 [468(b)]

Classes of cities: a. Component 1. Dependent component cities 2. Independent component cities - those component cities whose charters prohibit their voters from voting for provincial elective officials. Independent component cities shall be independent of the province b. Highly-urbanized - Voters of highly urbanized cities shall remain excluded from voting for elective provincial officials. Duties and Functions of the Municipal Mayor, City Mayor, Governor (Sec 444, 455, 465) a. b. Such duties and functions as provided by the LGC and other laws Efficient, effective and economical governance for the general welfare (see Sec.16) of the (municipality/city/province) and its inhabitants Hold office in the (municipal/city/provincial) hall during his incumbency

c.

Under the efficient, effective and economical governance:

A. Exercise general supervision and control over all programs, projects, services, and activities of the (municipal/city/provincial) government, and in this connection shall: 1. Determine the guidelines of (municipal/city/provincial) policies and be responsible to the sanggunian for the program of the government; 2.1. Direct the formulation of the (municipal/city/provincial) development plan, with the assistance of the (municipal/city/provincial) development council 2.2. Implement the (municipal/city/provincial) development plan upon approval thereof by the sanggunian 3. At the opening of the regular session of the sanggunian for every calendar year, and as may be deemed necessary, present the program of government and propose policies and projects for the consideration of the sangguian as the general welfare of the inhabitants and the needs of the (municipal/city/provincial) government may require; 4.1. Initiate and propose legislative measures to the sanggunian

183 D2010

Local Government
4.2. Provide such information and data needed or requested by said sanggunian in the performance of its legislative functions, from time to time as the situation may require Appoint all officials and employees whose salaries and wages are wholly or mainly paid out of (municipal/city/provincial) funds and whose appointments are not otherwise provided for in the LGC, as well as those he may be authorized by law to appoint; Represent the (municipality/city/province) in all its business transactions upon authorization by the sanggunian Sign on behalf of the (municipality/city/province) all bonds, contracts, and obligations, and such other documents made pursuant to law or ordinance upon authorization by the sanggunian Carry out such emergency measures as may be necessary during and in the aftermath of manmade and natural disasters and calamities; Determine, according to law or ordinance, the time, manner and place of payment of salaries or wages of the officials and employees of the (municipality/city/province) Allocate and assign office space to (municipal/city/provincial) and other officials and employees who, by law or ordinance, are entitled to such space in the municipal hall and other buildings owned or leased by the (municipal/city/provincial) government; Ensure that all executive officials and employees of the municipality faithfully discharge their duties and functions as provided by law and the LGC; Cause to be instituted administrative or judicial proceedings against any official or employee of the (municipality/city/province) who may have committed an offense in the performance of his official duties; Examine the books, records and other documents of all offices, officials, agents or employees of the (municipality/city/province) Require all national officials and employees stationed in or assigned to the (municipality/city/province) to make available to him such books, records, and other documents in their custody, in aid of his executive powers and authority, except those classified by law as confidential Furnish copies of EOs issued by him within 72 hours after their issuance a. if municipal mayor, to the provincial governor b. if mayor of a municipality of Metropolitan Manila Area and that of any metropolitan political subdivision, to the metropolitan authority council chairman and to the Office of the President c. if component city mayor, to the provincial governor d. if highly-urbanized city mayor, to the Office of the President e. if mayor of a city in the Metropolitan Manila Area and other metropolitan political subdivision, to the metropolitan authority council chairman f. if governor, to the Office of the President 13. Visit component (barangays/barangays/cities and municipalities) of the (municipality/city/province) at least once in every 6 months to deepen his understanding of problems and conditions therein, listen and give appropriate counsel to local officials and inhabitants, inform the officials and inhabitants of the component (barangays/barangays/cities and municipalities) general laws and ordinances which especially concern them, and otherwise conduct visits and inspections to the end that the governance of the (municipality/city/province) will improve the quality of life of the inhabitants; 14. Act on leave applications of officials and employees appointed by him and the commutation of the monetary value of leave credits according to law; 15. Authorize official trips outside of the (municipality/city/province) of (municipal/city/provincial) officials and employees for a period not exceeding 30 days; 16.1. Call upon any national office or employee stationed in or assigned to the (municipality/city/province) to advise him on matters affecting the (municipality/city/province) and to make recommendations thereon, or to coordinate in the formulation and implementation of plans, programs, projects, 16.2. Initiate an administrative or judicial action against a national government official or employee who may have committed an offense in the performance of his official duties while stationed in or assigned to the LGU concerned, when appropriate

5.

6.1.

6.2.

7.

8.

9.

10.1.

10.2.

11.1.

11.2.

12.

D2010 184

UP College of Law
17. Authorize payment of medical care, necessary transportation, subsistence, hospital or medical fees of (municipal/city/provincial) officials and employees who are injured while in the performance of their official duties and functions, subject to the availability of funds; Solemnize marriages, any provision of law to the contrary notwithstanding; Conduct a palarong (bayan/panlungsod/panlalawigan) in coordination with the DECS, as an annual activity which shall feature traditional sports and disciplines included in national and international games; and Submit the following reports: (same receivers as in NO. 12) an annual report containing a summary of all matters pertaining to the management, administration and development of the (municipality/city/province) and all information and data relative to its political, social and economic conditions; and supplemental reports when unexpected events and situations arise at any time during the year, particularly when man-made or natural disasters or calamities affect the general welfare of the municipality, province, region or country. 4. 5. Be entitled to carry the necessary firearm within his territorial jurisdiction; Act as the deputized representative of the National Police Commission, formulate the peace and order plan of the (municipality/city/province) and upon its approval, implement the same and exercise general and operational control and supervision over the local police forces in the (municipality/city/province) in accordance with RA 6975 (DILG Act of 1990) Call upon the appropriate law enforcement agencies to suppress disorder, riot, lawless violence, rebellion or sedition or to apprehend violators of the law when public interest so requires and the (municipal/city/provincial) police forces are inadequate to cope with the situation or the violators;

18. 19.

6.

20.

B. Enforce all laws and ordinances relative to the governance of the (municipality/city/province) and the exercise of its corporate powers (See Sec. 22), implement all approved policies, programs, projects, services and activities of the (municipality/city/province) and, in addition shall: 1. Ensure that the acts of the component (barangays/barangays/cities and municipalities) of the (muncipality/city/province) and of its officials and employees are within the scope of their prescribed powers, functions, duties and responsibilites; 2. Call conventions, conferences, seminars or meetings of any elective and appointive officials of the (municipality/city/province), including provincial officials and national officials and employees stationed in or assigned to the (municipality/city/province) at such time and place and on such subject as he may deem important for the promotion of the general welfare of the LGU and its inhabitants; 3. Issue such EOs as are necessary for the proper enforcement and execution of laws and ordinances;

C. Initiate and maximize the generation of resources and revenues, and apply the same to the implementation of development plans, programs objectives and priorities (see Sec. 18), particularly those resources and revenues programmed for agro-industrial development and country-wide growth and progress, and relative thereto shall: 1. Require each head of an office or department to prepare and submit an estimate of appropriations for the ensuing calendar year, in accordance with the budget preparation process 2. Prepare and submit to the sanggunian for approval the executive and supplemental budgets of the (municipality/city/province) for the ensuing calendar year 3. Ensure that all taxes and other revenues of the (municipality/city/province) are collected, and that (municipal/city/provincial) funds are applied to the payment of expenses and settlement of obligations of the (municipality/city/province), in accordance with law or ordinance; 4. Issue licenses and permits and suspend or revoke the same for any violation of the conditions upon which said licenses or permits had been issued, pursuant to law or ordinance; 5. Issue permits, without need of approval therefor from any national agency, for the holding of activities for any charitable or welfare purpose, excluding prohibited games of chance or shows contrary to law, public policy and public owners; (does not apply to governors)

185 D2010

Local Government
6. Require owners of illegally constructed houses, buildings or other structures to obtain the necessary permit, subect to such fines and penalties as may be imposed by law or ordinance, to make necessary changes in the construction of the same when said construction violates any law or ordinance, or to order the demolition or removal of said house, building or structure within the period prescribed by law or ordinance; (does not apply to governors) Adopt adequate measures to safeguard and conserve land, mineral, marine, forest, and other resources for the (municipality/city/province in coordination with the mayors of component cities and municipalities); Provide efficient and effective property and supply management in the (municipality/city/province); Protect funds, credits, rights, and other properties of the (municipality/city/province); Institute or cause to be instituted administrative or judicial proceedings for the violation of ordinances in the collection of taxes, fees or charges, and for the recovery of funds and property; Cause the (municipality/city/province) to be defended against all suits to ensure that its interests, resources and rights shall be adequately protected; d. rendered by national offices for the province and its component cities and municipalities Power, duties of the Municipal Vice-Mayor, City ViceMayor, Vice Governor (Sec 445, 456, 466) 1. Be the presiding officer of the sanggunian and sign all warrants drawn on the (municipal/city/provincial) treasury for all expenditures appropriated for the operation of the sanggunian 2. Subject to civil service law, rules and regulations, appoint all officials and employees of the sanggunian, except those whose manner of appointment is specifically provided in the LGC 3. Assume the office of the (municipal mayor/city mayor/governor) for the unexpired term of the latter in the event of permanent vacancy 4. Exercise the powers and perform the duties and functions of the (municipal mayor/city mayor/governor) in cases of temporary vacancy 5. Exercise such powers and perform such other duties and functions as may be prescribed by law or ordinance Powers, duties and functions of the Sangguniang (Bayan, Panglungsod, Panlalawigan) (Sec 444, 455, 465) A. Approve ordinances and pass resolutions necessary for an efficient and effective (municipal/city/provincial) government, and in this connection: 1. Review all ordinances approved by the (sangguniang barangay/sangguniang barangay/sanggunians of component cities and municipalities) and EOs issued by the (punong barangay/punong barangay/mayors of said component units) to determine whether these are within the scope of the prescribed powers of the sanggunian and of the (punong barangay/punong barangay/mayor) 2. Maintain peace and order by enacting measures to prevent and suppress lawlessness, disorder, riot, violence, rebellion, or sedition and impose penalties for the violation of said ordinances 3. Appprove ordinances imposing a fine not exceeding (P2,500/P5,000/P5,000) or an imprisonment for a period not exceeding (6 months/1 year/1 year), or both, in the discretion of the court, for the violation of a (municipal/city/provincial) ordinance 4. Adopt measures to protect the inhabitants of the (municipality/city/province) from the

7.1.

7.2.

7.3. 8.1.

8.2.

D. Ensure the delivery of basic services and the provision of adequate facilities (see Sec. 17 (B) (2)), and in addition thereto, shall: 1. Ensure that the construction and repair of roads and highways funded by the National Government shall be, as far as practicable, carried out in a spatially contiguous manner and in coordination with the construction and repair of the roads and bridges of the (municipality/city and the province/province and its component cities and municipalities); 2. Coordinate the implementation of technical services, including public works and infrastructure programs in the municipality a. rendered by national and provincial offices in the case of municipalities b. rendered by national offices in the case of highly urbanized and independent component cities c. rendered by national and provincial offices in the case of component cities

D2010 186

UP College of Law
harmful effects of man-made or natural disasters and calamities and to provide relief services and assistance for victims during and in the aftermath of said disasters or calamities and their return to productive livelihood following said events Enact ordinances intended to prevent, suppress and impose appropriate penalties for habitual drunkenness in public places, vagrancy, mendicancy, prostitution, establishment and maintenance of houses of ill repute, gambling and other prohibited games of chance, fraudulent devices and ways to obtain money or property, drug addiction, maintenance of drug dens, drug pushing, juvenile delinquency, the printing, distribution or exhibition of obscene or pornographic materials or publications, and such other activities inimical to the welfare and morals of the inhabitants of the (municipality/city/province) Protect the environment and impose appropriate penalties for acts which endanger the environment, such as dynamite fishing and other forms of destructive fishing, illegal logging and smuggling of logs, smuggling of natural resources products and of endangered species of flora and fauna, slash and burn farming, and such other activities which result in pollution, acceleration of eutrophication of rivers and laks, or of ecological imbalance Subject to the provisions of the LGC and pertinent laws, determine the powers and duties of officials and employees of the (municipality/city/province) Determine the positions and salaries, wages, allowances and other emoluments and benefits of officials and employees paid wholly or mainly from (municipal/city/provincial) funds and provide for expenditures necessary for the proper conduct of programs, projects, services, and activities of the (municipal/city/provincial) government Authorize the payment of compensation to a qualified person not in the government service who fills up to a temporary vacancy or grant honorarium to any qualified official or employee designated to fill a temporary vacancy in a concurrent capacity at the rate authorized by law Provide a mechanism and the appropriate funds therefor, to ensure the safety and protection of all (municipal/city/provincial) government property, public documents, or records such as those relating to property inventory, land ownership, records of births, marriages, deaths, assessments, taxation, accounts, business permits, and such other records and documents of public interest in the offices and departments of the (municipal/city/provincial) government When the finances of the (municipal/city/provincial) government allow, provide for additional allowances and other benefits to judges, prosecutors, public elementary and high school teachers, and other national government officials stationed in or assigned to the (municipality/city/province) Provide for legal assistance to barangay officials who, in the performance of their official duties or on the occasion thereof, have to initiate judicial proceedings or defend themselves against legal action (does not apply to the sangguniang panlalawigan) Provide for group insurance or additional insurance coverage for barangay officials, including members of barangay tanod brigades and other service units, with public or private insurance companies, when the finances of the (municipal/city) government allow said coverage (does not apply to the sangguniang panlalawigan)

5.

11.

12.

6.

13.

7.

8.

9.

10.

B. Generate and maximize the use of resources and revenues for the development plans, program objectives and priorities of the (municipality/city (see Sec. 18) with particular attention to agro-industrial development and countryside growth and progress, and relative thereto, shall: 1. Approve the annual and supplemental budgets of the (municipal/city/provincial) government and appropriate funds for specific programs, projects, services and activities of the (municipality/city/province), or for other purposes not contrary to law, in order to promote the general welfare of the (municipality/city/province) and its inhabitants 2. Subject to the provisions of Book II of the LGC and applicable laws and upon the majority vote of all the members of the sanggunian, enact ordinances levying taxes, fees and charges, prescribing the rates thereof for general and specific purposes, and granting tax exemptions, incentives or reliefs 3. Subject to the provisions of Book II of the LGC and upon the majority vote of all the members

187 D2010

Local Government
of the sanggunian, authorize the (municipal mayor/city mayor/provincial governor) to negotiate and contract loans and other forms of indebtedness Subject to the provisions of Book II of the LGC and applicable laws and upon the majority vote of all the members of the sanggunian, enact ordinances authorizing the floating of bonds or other instruments of indebtedness, for the purpose of raising funds to finance development projects Appropriate funds for the construction and maintenance of the rental of buildings for the use of the (municipality/city/province); and, upon the majority vote of all the members of the sanggunian, authorize the (municipal mayor/city mayor/provincial governor) to lease to private parties such public buildings held in a proprietary capacity, subject to existing laws, rules and regulations Prescribe reasonable limits and restraints on the use of property within the jurisdiction of the (municipality/city/province) a. For the sangguniang bayan and sangguniang panlungsod: adopt a comprehensive land use plan for the (municipality/city), provided, that the formulation, adoption, or modification of said plan shall be in coordination with the approved provincial comprehensive land use plan b. For the sangguniang panlalawigan: review the comprehensive land use plans and zoning ordinances of component cities and municipalities and adopt a comprehensive provincial land use plan, subject to existing laws Reclassify land within the jurisdiction of the (municipality/city), subject to the pertinent provisions of the LGC (does not apply to the sangguniang panlalawigan) Enact integrated zoning ordinances in consonance with the approved comprehensive land use plan, subject to existing laws, rules and regulations; establish fire limits or zones, particularly in populous centers; and regulate the construction, repair or modification of buildings within said fire limits or zones in accordance with the provisions of the Fire Code (does not apply to the sangguniang panlalawigan) Subject to national law, process and approve subdivision plans for residential, commercial, or industrial purposes, and other development purposes, and collect processing fees and other charges, the proceeds of which shall accrue entirely to the (municipality/city): Provided, however, that where approval by a national agency or office is required, said approval shall not be withheld for more than 30 days from receipt of the application. Failure to act on the application within the period stated above shall be deemed as approval (does not apply to the sangguniang panlalawigan) Subject to the provisions of Book II of the LGC, grant the exclusive privilege of constructing fish corrals or fish pens, or the taking or catching of bangus fry, prawn fry or kawag-kawag, or fry of any species or fish within the municipal waters (does not apply to the sangguniang panlalawigan) With the concurrence of at least 2/3 of all the members of the SANGGUNIAN, grant tax exemptions, incentives or reliefs to entities engaged in community growth-inducing industries (does not apply to the sangguniang panlalawigan) * Under 192, LGC, LGUs may, through ordinances duly approved, grant tax exemptions, incentives or reliefs under such terms and conditions as they may deem necessary. Grant loans or provide grants to other LGUs or to national, provincial and municipal charitable, benevolent or educational institutions: Provided, that said institutions are operated and maintained within the (municipality/city) (does not apply to the sangguniang panlalawigan) Regulate the numbering of residential, commercial and other buildings (does not apply to the sangguniang panlalawigan) Regulate the inspection, weighing and measuring of articles of commerce (does not apply to the sangguniang panlalawigan) For the sangguniang panlalawigan: adopt measures to enhance the full implementation of the national agrarian reform program in coordination with the DAR

4.

5.

11.

12.

6.

7.

13.

8.

14.

9.

15.

16.

10.

C. Subject to the provisions of Book II of the LGC, grant franchises, enact ordinances authorizing the issuance of

D2010 188

UP College of Law
permits or licenses, or enact ordinances levying taxes, fees and charges upon such conditions and for such purposes intended to promote the general welfare of the inhabitants of the (municipality/city, and pursuant to his legislative authority shall: 1. Fix and impose reasonable fees and charges for all services rendered by the (municipal/city/provincial) government to private persons or entities 2. a. For the Sangguniang Bayan: regulate any business, occupation, or practice of profession or calling which does not require government examination within the municipality and the conditions under which the license for said business or practice of profession may be issued or revoked b. For the Sangguniang Panlungsod: regulate or fix license fees for any business or practice of profession within the city and the conditions under which the license for said business or practice of profession may be revoked and enact ordinances levying taxes thereon c. For the Sangguniang Panlalawigan: regulate or fix the license fees for such activities as provided under the LGC 3. Prescribe the terms and conditions under which public utilities owned by the (municipality/city) shall be operated by the municipal/city government or leased to private persons or entities, preferably cooperatives 4. Regulate the display of and fix the license fees for signs, signboards, or billboards at the place/s where the profession or business advertised thereby is, in whole or in part, conducted 5. Any law to the contrary notwithstanding, authorize and license the establishment, operation, and maintenance of cockpits, and regulate cockfighting and commercial breeding of gamecocks: Provided, that existing rights should not be prejudiced 6. Subject to the guidelines prescribed by the DOTC, regulate the operation of tricycles and grant franchises for the operation thereof within the territorial jurisdiction of the (municipality/city) 7. Upon approval by a majority vote of all the members of the sanggunian, grant a franchise to any person, partnership, corporation, or cooperative to establish, construct, operate and maintain ferries, wharves, markets or slaughterhouses, or such other similar activities within the (municipality/city) as may be allowed by applicable laws: Provided, that, cooperatives shall be given preference in the grant of such a franchise (Nos. 3-7 do not apply to the sangguniang panlalawigan) D. Regulate activities relative to the use of land, buildings and structures within the (municipality/city) in order to promote the general welfare and for said purpose shall: 1. Declare, prevent ora abate any nuisance 2. Require that buildings and the premises thereof and any land within the (municipality/city be kept and maintained in a sanitary condition; impose penalties for any violation thereof, or upon failure to comply with said requirements, have the work done and require the owner, administrator or tenant concerned to pay the expenses of the same; or require the filling up of any land or premises to a grade necessary for proper sanitation 3. Regulate the disposal of clinical and other wastes from hospitals, clinics and other similar establishments 4. Regulate the establishment, operation and maintenance of cafes, restaurants, beerhouses, hotels, motels, inns, penison houses, lodging houses, and other similar establishments, inlcuding tourist guides and transports 5. Regulate the sale, giving away or dispensing of any intoxicating malt, vino, mixed or fermented liquors at any retail outlet 6. Regulate the establishment and provide for the inspection of steam boilers or any heating device in buildings and the storage of inflammable and highly combustible materials within the municipality 7. Regulate the establishment, operation, and maintenance of entertainment or amusement facilities, including theatrical performances, circuses, billiard pools, public dancing schools, public dance halls, sauna baths, massage parlors, and other places of entertainment or amusement; regulate such other events or activities for amusement or entertainment, particularly those which tend to disturb the community or annoy the inhabitants, or require the suspension or suppression of the same; or prohibit certain forms of amusement or entertainment in order to protect the social and moral welfare of the community 8. Provide for the impounding of stray animals; regulate the keeping of animals in homes or as

189 D2010

Local Government
part of a business, and the slaughter, sale or disposition of the same; and adopt measures to prevent and penalize cruelty to animals Regulate the establishment, operation and maintenance of funeral parlors, and the burial or cremation of the dead, subject to existing laws, rules and regulations supply water for the inhabitants; regulate the construction, maintenance, repair and use of hydrants, pumps, cisterns and reservoirs; protect the purity and quantity of the water supply of the (municipality/city and, for this purpose, extend the coverage of appropriate ordinances over all territory within the drainage area of said water supply and within 100 meters of the reservoir, conduit, canal, aqueduct, pumping station, or watershed used in connection with the water service; and regulate the consumption, use or wastage of water Regulate the drilling and excavation of the ground for laying of water, gas, sewer, and other pipes and the construction, repair and maintenance of public drains, sewers, cesspools, tunels and similar structures; regulate the placing of poles and the use of crosswalks, curbs, and gutters; adopt measures to ensure public safety agaisnt open canals, manholes, live wires and other similar hazards to life and property; and regulate the construction and use of private water closets, privies and other similar structures in buildings and homes Regulate the placing, stringing, attaching, installing, repair and construction of all gas mains, electric, telegraph and telephone wires, conduits, meters and other apparatus; and, provide for the correction, condemnation or removal of the same when found to be dangerous, defective or otherwise hazardous to the welfare of the inhabitants Subject to the availability of funds and to existing laws, rules and regulations, establish and provide for the operation of vocational and technical schools and similar post-secondary institutions and, with the approval of the DECS, fix and collect reasonable fees and other school charges on said institutions, subject to existing laws on tuition fees Establish a scholarship fund for poor but deserving students residing within the muncipality in schools located within its jurisdiction Approve measures and adopt quarantine regulations to prevent the introduction and spread of diseases Provide for an efficient and effective system of solid waste and garbage collection disposal and prohibit littering and the placing or throwing of garbage, refuse and other filth and wastes Provide for the care of paupers, the aged, the sick, persons of unsound mind, disabled

9.

E. Approve ordinances which shall ensure the efficient and effective delivery of the basic services and facilities (see Sec. 17), and in addition: For the sangguniang bayan and sangguniang panlungsod: 1. Provide for the establishment, maintenance, protection, and conservation of communal forests and watersheds, tree parks, greenbelts, mangroves, and other similar forest development projects 2. Establish markets, slaughterhouses or animal corrals and authorize the operation thereof, and regulate the construction and operation of private markets, talipapas or other similar buildings and structures 3. Authorize the establishment, maintenance and operation of ferries, wharves, and other structures, and marine seashore or offshore activities intended to accelerate productivity 4. Regulate the preparation and sale of meat, poultry, fish, vegetables, fruits, fresh dairy products, and other foodstuffs for public consumption 5. Regulate the use of streets, avenues, alleys, sidewalks, bridges, parks and other public places and approve the construction, improvement, repair and maintenance of the same; establish bus and vehicle stops and terminals or regulate the use of the same by privately-owned vehicles which serve the public; regulate garages and the operation of conveyances for hire; designate stands to be occupied b public vehicles when not in use; regulate the putting up of signs, signposts, awnings and awning posts on the streets; and provide for the lighting, cleaning and sprinkling of streets and public places 6. Regulate traffic on all streets and bridges, prohibit the putting up of encroachments or obstacles thereon, and, when necessary in the interest of public welfare authorize the removal of encroachments and illegal constructions in public places 7. Subject to existing laws, provide for the establishment, operation, maintenance, and repair of an efficient waterworks system to

8.

9.

10.

11.

12.

13.

14.

D2010 190

UP College of Law
persons, abandoned minors, juvenile delinquents, drug dependents, abused children and other needy and disadvantaged persons, partcularly children and youth below 18 years of age and, subject to the availability of funds, establish and provide for the operation of centers and facilities for said needy and disadvantaged persons Establish and provide for the maintenance and improvement of jails and detention centers, institute sound jail management programs, and appropriate funds for the subsistence of detainees and convicted prisoners in the municipality/city Establish a municipal/city council whose purpose is the promotion of culture and the arts, coordinate with government agencies and NGOs and subject to the availability of funds, appropriate funds for the support and development of the same Establish a municipal/city council for the elderly which shall formulate policies and adopt measures mutually beneficial to the elderly and to the community; provide incentives for nongovernmental agencies and entities and, subject to the availability of funds, appropriate funds to support programs and projects for the benefits of the elderly 5. Approve measures and adopt quarantine regulations to prevent the introduction and spread of diseases within its territorial jurisdiction; Provide for the care of paupers, the aged, the sick, persons of unsound mind, abandoned minors, abused children, disabled persons, juvenile delinquents, drug dependents, and other needy and disadvantaged persons, particularly children and youth below 18 years of age; subject to availability of funds, establish and support the operation of centers and facilities for said needy and disadvantaged persons, and facilitate efforts to promote the welfare of families below the poverty threshold, the disadvantaged, and the exploited; Establish and provide for the maintenance and improvement of jails and detention centers, institute a sound jail management program, and appropriate funds for the subsistence of detainees and convicted prisoners in the province Establish a provincial council whose purpose is the promotion of culture and the arts, coordinate with government agencies and nongovernmental organizations and, subject to the availability of funds, appropriate funds for the support and development of the same; Establish a provincial council for the elderly which shall formulate policies and adopt measures mutually beneficial to the elderly and to the province; and subject to the availability of funds, appropriate funds to support programs and projects for the elderly; and provide incentives for nongovernmental agencies and entities to support the programs and projects of the elderly;

6.

15.

16.

7.

17.

8.

9. For the sangguniang panlalawigan: 1. Adopt measures and safeguards against pollution and for the preservation of the natural ecosystem in the province, in consonance with approved standards on human settlements and environmental sanitation 2. Subject to applicable laws, facilitate or provide for the establishment and maintenance of waterworkds system or district waterworks for supplying water to inhabitants of component cities and municipalities 3. Subject to the availability offunds and to existing laws, rules and regulations, provide for the establishment and operation of vocational and technical schools and similar postsecondary institutions; and, with the approval of the DECS and subject to existing laws on tuition fees, fix reasonable tuition fees and other school charges in educational institutions supported by the provincial government; 4. Establish a scholarship fund or the poor but deserving students in schools located within its jurisdiction or for students residing within the province;

F. Exercise such other powers and perform such other duties and functions as may be prescribed by law or ordinance A. The Municipality

Scope of power to tax municipalities; share of provinces in municipal taxes (Sec 440 447)
See above

Munez v. Arlino

191 D2010

Local Government
Mayor Asuero Irisari of Loreto, Agusan del Sur summoned Apolinario S. Munez for a conference on a land dispute with one Tirso Amado. Munez failed to appear and Mayor Irisari issued a warrant of arrest against him. It was served on the former although no investigation later ensued. Munez filed a criminal complaint against Mayor Irisari for grave misconduct and usurpation of judicial function with the Office of the Ombudsman and an administrative complaint for misconduct in office and abuse of authority with the Sangguniang Panlalawigan of Agusan del Sur. The investigating officer of the Office of the Ombudsman filed a case for usurpation of judicial function against Irisari which the latter moved to quash, alleging that Sec 143 of the former LGC authorized mayors to issue warrants of arrest. Judge Arino denied the MTQ on the ground that the power of Mayors to issue warrants ceased to exist with the effectivity of the 1987 Constitution. The Sangguniang Panlalawigan (SP) on the other hand adjudged Irisari to be guilty of the administrative complaint and suspended him for 8 months without pay. The DILG reversed the SP and said that while the warrant issued by the Mayor was one of arrest, it was actually just a summons or an invitation. Judge Arino dismissed the case based on the decision by the DILG saying that decisions by administrative agencies, when not tainted by unfairness and arbitrariness should be respected. Acting on the said decision, Munez sent two letters to the Presidential AntiCrime Commission charging Judge Arino with knowingly rendering an unjust judgment. HELD: The Court agrees with the Ombudsman that the Judge may have acted in good faith but must be administratively liable. It is not true that what was issued wasnt a warrant of arrest. The mayor even justified his order by citing Sec. 143 of the former LGC, which provided that in cases where the mayor may conduct preliminary investigation, the mayor shall, upon probable cause after examination of the witnesses, have the authority to order the arrest of the accused. The said provision was however repealed by Art II, Sec 2 of the 1987 Constitution, which expressly provides that only a judge can personally issue warrants. It was also held in Ponsica vs. Ognalaga that the grant of the said power on the Mayor had been abrogated, rendered functus officio by the Constitution. That there was no pending criminal case did not make the order any less an order of arrest. There was only a land dispute and no criminal case against Munez. The Mayor thus performed a judicial function that even a judge couldnt have done. Judge Arino reversed himself citing the DILG decision when the case wasnt before him on review from the decision of the administrative agency. There was no reason to apply substantiality of evidence. Before him was a criminal case and he shouldve considered solely the facts alleged in the information. The acts alleged in the information constitute the crime of usurpation of judicial authority, satisfying the elements of the same. At the very least, he showed poor judgment and gross ignorance of basic legal principles. What the judge did was to rely on the opinion of the DILG, disregarding his own previous ruling, and showing a lack of capacity for independent judgment in the process.

Greater Balanga Devt Corp v. Municipality of Balanga, Bataan


The case involves Lot 261 B-6-A-3 with a land area of 8,467 sq meters located behind the public market in Balanga, Bataan, It is registered in the name of Greater Balanga Devt. Corp, owned and controlled by the Camacho family. The lot in question was part of Lot 261B, formerly registered in the name of Aurora Camacho. It was subdivided into certain lots- some were sold, other donated. Five buyers of the lot filed Civil Case 3803 against Camacho for partition and delivery of titles. Petitioner applied for and was granted a business permit by the Office of the Mayor of Balanga but failed to mention the existence of Civil Case 3803. The permit granted the privilege of a real estate dealer/privatelyowned market operator. The Sangguniang Bayan (SB) however passed Resolution No.12, s-88 annulling the Mayors permit issued to petitioner, mentioning that the civil dispute as to the ownership of the lot caused anxiety, uncertainty and restiveness among the stallholders and traders in the lot, and advising the Mayor to revoke the permit to operate a public market. The Mayor took the advice and revoked the permit by way of EO No.1, s-88. Petitioner filed instant petition with prayer for preliminary mandatory and prohibitory injunction or restraining order aimed at the reinstatement of the Mayors permit and the curtailment of the municipalitys collection of market and entrance fees from the occupants of the lot. Petitioner corporation alleges that: 1) it didnt violate any law, thus theres no reason for revocation of permit 2) the respondents failed to observe due process in the revocation 3) the collection of market fees is illegal. The Municipality asserts on the other hand that the Mayor as local chief executive has the power to issue, deny or revoke permits. They claim that the revocation was due to the violation by the corporation of Section 3A-o6(b) of the Balanga Revenue Code when it: 1) made a false statement in the application form, failing to disclose that

D2010 192

UP College of Law
the lot was subject of adverse claims for which Civil Case 3803 was filed, 2) failed to apply for two separate permits for the 2 lines of business (real estate and public market). HELD: The authority of the Mayor to revoke permits is premised on a violation by the grantee of any of the conditions for its grant. The application for the Mayors permit at bench requires the applicant to state the type of business, profession, occupation, privileges applied for. Revocation cannot be justified under the mentioned provision for there must be 1) proof of willful misrepresentation and 2) deliberate intent to make a false statement. Good faith is always presumed. Neither was the petitioners applying for two businesses in one permit a ground for revocation. Section 3A-06(b) does not expressly require two permits for two businesses but only that separate fees be paid for each. The Resolution by the SB stated that the land was earmarked for the expansion of the Balanga Public Market. The SB doesnt actually maintain a public market on the area. Until expropriation proceedings are instituted in court, the owner cannot be deprived of its land. The SB has the duty to regulate any business subject to municipal license fees and provide conditions for revocation but anxiety, uncertainty, restiveness among stallholders who are doing business on property not owned by the Municipal government is not among those conditions. Also, the manner by which revocation was done violated the petitioners right to due process the alleged violation of the Revenue Code was not mentioned in the revocation order, neither was the petitioner informed of his specific violation. Moreover, the respondent Municipality isnt the owner of Lot 261 B6-A-3 and thus cannot collect market fees and market entrance fees, which only an owner can do. B. The City and health certificates of its staff, causing stoppage of work in Bistros business operations. Lim also refused to accept Bistros application for a business permit and its staffs applications for work permits. Bistro says that Lims refusal to issue permits is against the doctrine laid down in Dela Cruz vs. Paras that Municipal corporations cannot prohibit the operation of nightclubs. They may be regulated but not prevented from carrying on their business. The trial court issued a TRO against Lim and after receiving evidence from the parties, issued a prohibitory mandatory injunction against the same, ordering him to cease and desist from impeding the business operations of Bistro while the case awaits resolution on the merits. Both orders were ignored by Lim, insisting that the power of a mayor to inspect commercial establishments is implicit in the statutory power to issue, suspend or revoke business licenses, provided in Sec 11 (1), Art II of the Revised Charter of the City of Manila and in Sec 455 of the LGC. Lim permanently closed down the operations of Bistro pursuant to newly-enacted Manila City Ordinance No7783. Lim argues that the passage of the ordinance has made the case filed by Bistro moot and academic. HELD: The authority of mayors to issue business licenses and permits is beyond question as the law, specifically the Revised Charter of Manila expressly provides for such authority. The power to issue includes the corollary power to suspend and revoke licenses premised on violation of the conditions upon which they were granted. True, the mayor has the power to investigate whether the conditions are complied with but he has no power to order a police raid on such establishments. In doing so, Lim acted beyond his authority and in patent violation of Ordinance 7716, which prohibits police raid inspections of business establishments. In refusing to issue a business permit, Lim didnt specify any violation committed by Bistro. Neither was Bistro given the proper notice and the opportunity to be heard, infringing its right to due process of law. There is no provision in any law authorizing the mayor to close down establishments without notice and hearing and if there is, such will be void. While Lims campaign against prostitution is commendable, his acts were arbitrary and the trial court was correct in restraining him. The regulatory powers of municipal corporations must be exercised in accordance with the rights of people to due process and equal protection of the law.

Scope of Power to tax of cities (Sec 448 458)


See above

Lim v. CA
Bistro Pigalle Inc. is the owner of the New Bangkok Club and Exotic Garden Restaurant in Malate Manila. Bistro filed a case against Mayor Lim after the policemen, under the Mayors instructions, inspected and investigated Bistros license, along with the work permits C. The Province

193 D2010

Local Government Sec 459 490, LGC


For Sec 459 468, see above

Appointive Local Officials Common to Municipalities, Cities and Province (Sections 469-490)
Position 1. Secretary to the Sanggunian Necessity mandatory position Qualifications Qualifications a. Citizen of the Philippines b. Resident of the LGU concerned c. Of good moral character d. A holder of a college degree preferably in law, commerce or public administration from a recognized college or university, and e. A first grade civil service eligible or its equivalent. (Sec 469[b]) Duties a. Attend meetings of the sanggunian and keep a journal of its proceedings; b. Keep the seal of the LGU and affix the same with his signature to all ordinances, resolutions, and other official acts of the sanggunian and present the same to the presiding officer for his signature; c. Forward to the governor or mayor for approval, copies of ordinances enacted by the sanggunian and duly certified by the presiding officer; d. Forward to the sanggunian panlungsod or bayan or the sangguniang panlungsod of component cities or sangguniang bayan, copies of duly approved ordinances; e. Furnish certified copies of records of public character in his custody; f. Record in a book kept for the purpose, all ordinances and resolutions enacted or adopted by the sanggunian, with the dates of passage and publication thereof; g. Keep his office and all non-confidential records therein open to the public during the usual business hours; h. Translate into the dialect used by the majority of the inhabitants all ordinances and resolutions immediately after their approval, and cause the publication of the same; i. Take custody of the local archives and, where applicable, the local library and annually account for the same; and j. Exercise such other powers and perform such other duties and functions as may be prescribed by law or ordinance relative to his position. (Sec. 469[c]) a. Perform the duties provided for under Book II of the Code b. Advise the governor or mayor, sanggunian, and other local government and national officials regarding disposition of local government funds, and other matters relative to public finance; c. Take custody of and exercise proper management of the funds of LGU; d. Take charge of the disbursement of all local government funds and such other funds the custody of which may be entrusted to him; e. Inspect private commercial and industrial establishments in relation to the implementation of tax ordinances; f. Maintain and update the tax information system of the LGU; g. In the case of the provincial treasurer, exercise technical supervision over all treasury offices of component cities and municipalities; and h. Exercise such other powers and perform such other duties and functions as may be prescribed by law or ordinance. (Sec 470[d]) a. Assist the treasurer and perform such duties as the latter may assign to him. b. Administer oaths concerning notices and notifications to those delinquent in the payment of the real property tax and concerning official matters relating to the accounts of the treasurer or arising in the offices of the treasurer and the assessor. (Sec 471[c])

2. Treasurer

mandatory position Appointed by the Secretary of Finance from a list of at least 3 ranking, eligible recommendees of the governor or mayor, as the case may be (Sec 470 [a])

a. Citizen of the Philippines b. A resident of the LGU concerned c. Of good moral character d. A holder of a college degree preferably in commerce, public administration or law from a recognized college or university, and e. A first grade civil service eligible or its equivalent. f. Acquired experience in treasury or accounting service for at least 5 years in the case of the city or provincial treasurer, and 3 years in the case of the municipal treasurer. (Sec 470[c])

3. Treasurer

Assistant

optional position Appointed by the Secretary of Finance from a list of at least 3 ranking, eligible recommendees of the governor or mayor (Sec 471[a])

a. A citizen of the Philippines, b. Resident of the LGU concerned, c. Of good moral character, d. A holder of a college degree preferably in commerce, public administration, or law from a recognized college or university, e. A first grade civil service eligible or its equivalent, f. Acquired at least 5 years experience in the treasury or accounting service in

D2010 194

UP College of Law
the case of the city or provincial assistant treasurer, and 3 years in the case of the municipal assistant treasurer. (Sec 471[b]) a. Citizen of the Philippines, b. A resident of the LGU concerned, c. Of good moral character, d. A holder of a college degree preferably in civil or mechanical engineering, commerce, or any other related course from a recognized college or university, e. A first grade civil service eligible or its equivalent. f. Acquired experience in real property assessment work or in any related field for at least 5years in the case of the city or provincial assessor, and 3 years in the case of the municipal assessor. (Sec 472[a])

4. Assessor

mandatory position

5. Assistant Assessor

optional position

6. Accountant

mandatory position

a. Citizen of the Philippines, b. Resident of the LGU concerned, c. Of good moral character, d. A holder of a college degree preferably in civil or mechanical engineering, commerce, or any related course from a recognized college or university e. A first grade civil service eligible or its equivalent. f. Acquired experience in assessment or in any related field for at least 3years in the case of the city or provincial assistant assessor, and 1 year in the case of the city or provincial assistant assessor. (Sec 473[a]) a. Citizen of the Philippines, b. A resident of the LGU concerned, c. Of good moral character, d. A certified public accountant. e. Acquired experience in the treasury or accounting service for at least 5 years in the case of the provincial or city accountant, and 3 years in the

a. Take charge of the assessor's office, b. Perform the duties provided for under Book II of the Code, c. Ensure that all laws and policies governing the appraisal and assessment of real properties for taxation purposes are properly executed; d. Initiate, review, and recommend changes in policies and objectives, plans and programs, techniques, procedures and practices in the valuation and assessment of real properties for taxation purposes; e. Establish a systematic method of real property assessment; f. Install and maintain a real property identification and accounting system, g. Prepare, install and maintain a system of tax mapping; h. Conduct frequent physical surveys to verify and determine whether all real properties within the province are properly listed in the assessment rolls; i. Exercise the functions of appraisal and assessment primarily for taxation purposes of all real properties in the LGU; j. Prepare a schedule of the fair market value for the different classes of real properties; k. Issue certified copies of assessment records of real property and all other records relative to its assessment; l. Submit every semester a report of all assessments, cancellations and modifications of assessments to the local chief executive and the sanggunian; m. In the case of the assessor of a component city or municipality attend sessions of the local board of assessment appeals whenever his assessment is the subject of the appeal; n. In the case of the provincial assessor, exercise technical supervision and visitorial functions over all component city and municipal assessors, coordinate with component city or municipal assessors in the conduct of tax mapping operations and all other assessment activities, and provide all forms of assistance; (Sec 472[b]) o. Exercise such other powers and perform such other duties and functions as may be prescribed by law or ordinance. (Sec 472[c]) a. Assist the assessor and perform such other duties as the latter may assign to him b. Administer oaths on all declarations of real property for purposes of assessment. (Sec 473[b])

a. Take charge of both the accounting and internal audit services of the LGU; b. Install and maintain an internal audit system in the local LGU; c. Prepare and submit financial statements to the governor or mayor and to the sanggunian; d. Apprise the sanggunian and other local government officials on the financial condition and operations of the LGU;

195 D2010

Local Government
case of the municipal accountant. (Sec 474[a]) f. Incumbent chief accountant in the office of the treasurer shall be given preference in the appointment to the position of accountant.(Sec 474[c]) e. Certify to the availability of budgetary allotment to which expenditures and obligations may be properly charged; f. Review supporting documents before preparation of vouchers to determine completeness of requirements; g. Prepare statements of cash advances, liquidation, salaries, allowances, reimbursements and remittances; h. Prepare statements of journal vouchers and liquidation of the same and other adjustments; i. Post individual disbursements to the subsidiary ledger and index cards; Maintain individual ledgers for officials and employees pertaining to payrolls and deductions; j. Record and post in index cards details of purchased furniture, fixtures, and equipment, including disposal thereof, if any; k. Account for all issued requests for obligations and maintain and keep all records and reports related thereto; l. Prepare journals and the analysis of obligations and maintain and keep all records and reports related thereto; and m. Exercise such other powers and perform such other duties and functions as may be provided by law or ordinance. (Sec 474[b]) a. Take charge of the budget office; b. Prepare forms, orders, and circulars embodying instructions on budgetary and appropriation matters for the signature of the governor or mayor, c. Review and consolidate the budget proposals of different departments and offices; d. Assist the governor or mayor in the preparation of the budget and during budget hearings; e. Study and evaluate budgetary implications of proposed legislation and submit comments and recommendations thereon; f. Submit periodic budgetary reports to the DBM; g. Coordinate with the treasurer, accountant, and the planning and development coordinator for the purpose of budgeting; h. Assist the sanggunian concerned in reviewing the approved budgets; i. Coordinate with the planning and development coordinator in the formulation of the local government unit development plan; (Sec 475 [b]) j. Exercise such other powers and perform such other duties and functions as may be prescribed by law or ordinance. (Sec 475[c]) a. Take charge of the planning and development office b. Formulate integrated economic, social, physical, and other development plans and policies for consideration of the local government development council; c. Conduct continuing studies, researches, and training programs necessary to evolve plans and programs for implementation; d. Integrate and coordinate all sectoral plans and studies undertaken by the different functional groups or agencies; e. Monitor and evaluate the implementation of the different development programs, projects, and activities in the local government unit concerned in accordance with the approved development plan; f. Prepare comprehensive plans and other development planning documents for the consideration of the local development council; g. Analyze the income and expenditure patterns, and formulate and recommend fiscal plans and policies for consideration of the finance committee of LGU; h. Promote people participation in development planning within the LGU; i. Exercise supervision and control over the secretariat of the local development council; and (Sec 476[b])

7. Budget Officer

mandatory position

a. Citizen of the Philippines, b. Resident of the LGU concerned, c. Of good moral character, d. A holder of a college degree preferably in accounting, economics, public administration or any related course from a recognized college or university, e. A first grade civil service eligible or its equivalent. f. Acquired experience in government budgeting or in any related field for at least 5 years in the case of the provincial or city budget officer, and at least 3 years in the case of the municipal budget officer. (Sec 475[a])

8. Planning Development Coordinator

and

mandatory position

a. Citizen of the Philippines b. A resident of the LGU concerned c. Of good moral character d. A holder of a college degree preferably in urban planning, development studies, economics, public administration, or any related course from a recognized college or university e. A first grade civil service eligible or its equivalent f. Acquired experience in development planning or in any related field for at least 5 years in the case of the provincial or city planning and development coordinator, and 3 years in the case of the municipal planning and development coordinator. (Sec 476[a])

D2010 196

UP College of Law
9. Engineer mandatory position a. Citizen of the Philippines, b. A resident of the LGU concerned, c. Of good moral character, d. A licensed civil engineer e. Acquired experience in the practice of his profession for at least 5 years in the case of the provincial or city engineer, and 3 years in the case of the municipal engineer. (Sec 477[a]) a. Act as the local building official (Sec 477[a]) b. Initiate, review and recommend changes in policies and objectives, plans and programs, techniques, procedures and practices in infrastructure development and public works in general; c. Advise the governor or mayor on infrastructure, public works, and other engineering matters; d. Administer, coordinate, supervise, and control the construction, maintenance, improvement, and repair of roads, bridges, and other engineering and public works projects; e. Provide engineering services to the LGU, including investigation and survey, engineering designs, feasibility studies, and project management; f. In the case of the provincial engineer, exercise technical supervision over all engineering offices of component cities and municipalities; and (Sec 477[b]) g. Exercise such other powers and perform such other duties and functions as may be prescribed by law or ordinance. (Sec477[c]) a. Take charge of the office on health services, supervise the personnel and staff of said office, formulate program implementation guidelines and rules and regulations for the operation of the said b. Formulate measures for the consideration of the sanggunian and provide technical assistance and support to the governor or mayor in carrying out activities to ensure the delivery of basic services and provision of adequate health facilities; c. Develop plans and strategies, implement the same, particularly those which have to do with health programs and projects which the governor or mayor, is empowered to implement and which the sanggunian is empowered to provide for; d. Formulate and implement policies, plans, programs and projects to promote the health of the people; e. Advise the governor or mayor and the sanggunian on matters pertaining to health; f. Execute and enforce all laws, ordinances and regulations relating to public health; g. Recommend to the sanggunian, through the local health board, the passage of such ordinances as he may deem necessary for the preservation of public health; h. Recommend the prosecution of any violation of sanitary laws, ordinances or regulations; i. Direct the sanitary inspection of all business establishments selling food items or providing accommodations such as hotels, motels, lodging houses, pension houses, and the like, in accordance with the Sanitation Code; j. Conduct health information campaigns and render health intelligence services; k. Coordinate with other government agencies and nongovernmental organizations involved in the promotion and delivery of health services; l. In the case of the provincial health officer, exercise general supervision over health officers of component cities and municipalities; and m. Be in the frontline of health services delivery, particularly during and in the aftermath of man-made and natural disasters and calamities; and (Sec478[b]) n. Exercise such other powers and perform such other duties and functions as may be prescribed by law or ordinance. (Sec478[c]) a. Responsible for the civil registration program pursuant to the Civil Registry Law, the Civil Code, and other pertinent laws, rules and regulations (Sec479[b]) b. Take charge of the office of the civil registry

10. Health Officer

mandatory position

a. Citizen of the Philippines, b. A resident of the LGU concerned, c. Of good moral character, and d. A licensed medical practitioner. e. Acquired experience in the practice of his profession for at least 5 years in the case of the provincial or city health officer, and 3 years in the case of the municipal health officer. (Sec478[a])

11. Civil Registrar

mandatory: city and municipal

a. Citizen of the Philippines, b. A resident of the LGU concerned, c. Of good moral character, d. A holder of a college degree from a

197 D2010

Local Government
recognized college or university, e. A first grade civil service eligible or its equivalent. f. Acquired experience in civil registry work for at least 5 years in the case of the city civil registrar and 3 years in the case of the municipal civil registrar. (Sec479[a]) c. Develop plans and strategies and implement the same, particularly those which have to do with civil registry programs and projects which the mayor is empowered to implement and which the sanggunian is empowered to provide for d. Accept all registrable documents and judicial decrees affecting the civil status of persons; e. File, keep and preserve in a secure place the books required by law; f. Transcribe and enter immediately upon receipt all registrable documents and judicial decrees affecting the civil status of persons in the civil registry books; g. Transmit to the Office of the Civil Registrar- General duplicate copies of registered documents required by law; h. Issue certified transcripts or copies of any certificate or registered documents upon payment of the prescribed fees to the treasurer; i. Receive applications for the issuance of a marriage license and issue the license upon payment of the authorized fee to the treasurer; g. Coordinate with the NSO in conducting educational campaigns for vital registration and assist in the preparation of demographic and other statistics (Sec479[c]) h. Exercise such other powers and perform such other duties and functions as may be prescribed by law or ordinance. (Sec479[d]) a. Take charge of the office of the administrator b. Develop plans and strategies and implement the same particularly those which have to do with the management and administration-related programs and projects which the governor or mayor is empowered to implement and which the sanggunian is empowered to provide for; c. Assist in the coordination of the work of all the officials of the LGU, under the supervision, direction, and control of the governor or mayor, and convene the chiefs of offices and other officials of the local government unit; d. Establish and maintain a sound personnel program for the LGU designed to promote career development and uphold the merit principle in the local government service; e. Conduct a continuing organizational development of the LGU with the end in view of instituting effective administrative reforms; f. Be in the frontline of the delivery of administrative support services, particularly those related to the situations during and in the aftermath of man-made and natural disasters and calamities; g. Recommend to the sanggunian and advise the governor and mayor on all other matters relative to the management and administration of the LGU (Sec480[b]) h. Exercise such other powers and perform such other duties and functions as may be prescribed by law or ordinance. (Sec480[c]) a. Take charge of the office of legal services b. Formulate measures for the consideration of the sanggunian and provide legal assistance and support to the governor or mayor, in carrying out the delivery of basic services and provisions of adequate facilities c. Develop plans and strategies and implement the same, particularly those which have to do with programs and projects related to legal services which the governor or mayor is empowered to implement and which the sanggunian is empowered to provide d. Represent the local government unit in all civil actions and special proceedings wherein the LGU or any official thereof, in his official capacity, is a party; e. Draft ordinances, contracts, bonds, leases and other instruments, involving any interest of the LGU; and provide

12. Administrator

mandatory: provincial and city, optional: municipal

a. Citizen of the Philippines, b. A resident of the LGU concerned, c. Of good moral character, d. A holder of a college degree preferably in public administration, law, or any other related course from a recognized college or university, e. A first grade civil service eligible or its equivalent f. Acquired experience in management and administration work for at least 5 years in the case of the provincial or city administrator, and 3 years in the case of the municipal administrator. g. Term is coterminous with that of his appointing authority (Sec480[a])

13. Legal Officers

mandatory: provincial and city, optional: municipal

a. Citizen of the Philippines, b. A resident of the LGU, c. Of good moral character, d. A member of the Philippine Bar e. Practiced his profession for at least 5 years in the case of the provincial and city legal officer, and 3years in the case of the municipal legal officer f. Term of the legal officer shall be coterminous with that of his appointing authority.(Sec481[a])

D2010 198

UP College of Law
comments and recommendations on any instruments already drawn; f. Render his opinion in writing on any question of law when requested to do so by the governor, mayor, or sanggunian; g. Investigate or cause to be investigated any local official or employee for administrative neglect or misconduct in office, and recommend appropriate action; h. Investigate or cause to be investigated any person, firm or corporation holding any franchise or exercising any public privilege for failure to comply with any term or condition in the grant of such franchise or privilege, and recommending appropriate action to the governor, mayor or sanggunian, as the case may be; i. Initiate and prosecute in the interest of the LGU any civil action on any bond, lease or other contract upon any breach or violation thereof; j. Review and submit recommendations on ordinances approved and executive orders issued by component units; k. Recommend measures to the sanggunian and advise the governor or mayor as the case may be on all other matters related to upholding the rule of law ; l. Be in the frontline of protecting human rights and prosecuting any violations thereof, particularly those which occur during and in the aftermath of man-made or natural disasters or calamities; and (Sec481[b]) m. Exercise such other powers and perform such other duties and functions as may be prescribed by law or ordinance. (Sec481[c]) a. Take charge of the office for agricultural service b. Formulate measures and provide technical assistance and support in carrying out said measures to ensure the delivery of basic services and provision of adequate facilities relative to agricultural services as provided for under Section 17 c. Develop plans and strategies and implement the same, particularly those which have to do with agricultural programs and projects which the governor or mayor is empowered to implement and which the sanggunian us empowered to provide for d. Ensure that maximum assistance and access to resources in the production, processing and marketing of agricultural and aqua-cultural and marine products are extended to farmers, fishermen and local entrepreneurs; e. Conduct or cause to be conducted location-specific agricultural researches and assist in making available the appropriate technology arising out of and disseminating information on basic research on crops, preventive and control of plant diseases and pests, and other agricultural matters f. Assist in the establishment and extension services of demonstration farms or aqua-culture and marine products; g. Enforce rules and regulations relating to agriculture and aquaculture; h. Coordinate with government agencies and NGOs which promote agricultural productivity through appropriate technology compatible with environmental integrity; i. Be in the frontline of delivery of basic agricultural services, particularly those needed for the survival of the inhabitants during and in the aftermath of man-made and natural disasters; j. Recommend and advise on all matters related to agriculture and aqua-culture which will improve the livelihood and living conditions of the inhabitants; (Sec482[b]) k. Exercise such other powers and perform such other duties and functions as may be prescribed by law or ordinance. (Sec482[c]) a. Take charge of the office on social welfare and development services

14. Agriculturist

mandatory: provincial; optional: city and municipal

a. Citizen of the Philippines, b. A resident of the LGU concerned, c. Of good moral character, d. A holder of a college degree in agriculture or any related course from a recognized college or university, e. A first grade civil service eligible or its equivalent. f. Practiced his profession in agriculture or acquired experience in a related field for at least 5 years in the case of the provincial and city agriculturist, and 3years in the case of the municipal agriculturist. (Sec482[a])

15. Social Welfare and Development

mandatory: provincial and city;

a. Citizen of the Philippines, b. A resident of the LGU concerned,

199 D2010

Local Government
Officer optional: municipal c. Of good moral character, d. A duly licensed social worker or a holder of a college degree preferably in sociology or any other related course from a recognized college or university, e. A first grade civil service eligible or its equivalent f. Acquired experience in the practice of social work for at least 5 years in the case of the provincial or city social welfare and development officer, and 3 years in the case of the municipal social welfare and development officer. (Sec483[a]) b. Formulate measures and provide technical assistance and support in carrying out measures to ensure the delivery of basic services and provision of adequate facilities relative to social welfare and development services as provided for under Section 17 c. Develop plans and strategies and mplement the same particularly those which have to do with social welfare programs and projects which the governor or mayor is empowered to implement and which the sanggunian is empowered to provide for d. Identify the basic needs of the needy, the disadvantaged and the impoverished and develop and implement appropriate measures to alleviate their problems and improve their living conditions; e. Provide relief and appropriate crisis intervention for victims of abuse and exploitation and recommend appropriate measures to deter further abuse and exploitation; f. Assist the governor or mayor in implementing the barangay level program for the total development and protection of children up to six (6) years of age; g. Facilitate the implementation of welfare programs for the disabled, elderly, and victims of drug addiction, the rehabilitation of prisoners and parolees, the prevention of juvenile delinquency and such other activities which would eliminate or minimize the ill-effects of poverty; h. Initiate and support youth welfare programs that will enhance the role of the youth in nation-building; i. Coordinate with government agencies and NGOs which have for their purpose the promotion and the protection of all needy, disadvantaged, underprivileged or impoverished groups or individuals, particularly those identified to be vulnerable and high-risk to exploitation, abuse and neglect; j. Be in the frontline of service delivery, particularly those which have to do with immediate relief during and assistance in the aftermath of man-made and natural disaster and natural calamities; k. Recommend to the sanggunian and advise the governor or mayor on all other matters related to social welfare and development services which will improve the livelihood and living conditions of the inhabitants; (Sec483[b]) l. Exercise such other powers and perform such other duties and functions as may be prescribed by law or ordinance. (Sec483[c]) a. Take charge of the office on environment and natural resources b. Formulate measures and provide technical assistance and support in carrying out measures to ensure the delivery of basic services and provision of adequate facilities relative to environment and natural resources services as provided for under Sec 17; c. Develop plans and strategies and implement the same, particularly those which have to do with environment and natural resources programs and projects which the governor or mayor is empowered to implement and which the sanggunian is empowered to provide for; d. Establish, maintain, protect and preserve communal forests, watersheds, tree parks, mangroves, greenbelts and similar forest projects and commercial forest; e. Provide extension services to beneficiaries of forest development projects and technical, financial and infrastructure assistance; f. Manage and maintain seed banks and produce seedlings for forests and tree parks; g. Provide extension services to beneficiaries of forest development projects and render assistance for natural resources-related conservation and utilization activities

16. Environment and Natural Resources Officer

optional position

a. A citizen of the Philippines, b. Resident of the local government unit concerned, c. Of good moral character, d. Holder of a college degree preferably in environment, forestry, agriculture or any related course from a recognized college or university, e. A first grade civil service eligible or its equivalent f. Acquired experience in environmental and natural resources management, conservation, and utilization, of at least 5 years in the case of the provincial or city environment and natural resources officer, and 3 years in the case of the municipal environment and natural resources officer. (Sec484[a])

D2010 200

UP College of Law
consistent with ecological balance; h. Promote the small-scale mining and utilization of mineral resources, particularly mining of gold; i. Coordinate with government agencies and NGOs in the implementation of measures to prevent and control land, air and water pollution with the assistance of the DENR; j. Be in the frontline of the delivery of services concerning the environment and natural resources, particularly in the renewal and rehabilitation of the environment during and in the aftermath of man-made and natural calamities and disasters; k. Recommend to the sanggunian and advise the governor or mayor on all matters relative to the protection, conservation, maximum utilization, application of appropriate technology and other matters related to the environment and natural resources; and (Sec484[b]) l. Exercise such other powers and perform such other duties and functions as may be prescribed by law or ordinance. (Sec484[c]) a. Take charge of the office on architectural planning and design b. Formulate measures for the consideration of the sanggunian and provide technical assistance and support to the governor or mayor in carrying out measures to ensure the delivery of basic services and provision of adequate facilities relative to architectural planning and design as provided for under Section 17; c. Develop plans and strategies and implement the same, particularly those which have to do with architectural planning and design programs and projects which the governor or mayor is empowered to implement and which the sanggunian is empowered to provide for under this Code; d. Prepare and recommend for consideration of the sanggunian the architectural plan and design for the local government unit or a part thereof, including the renewal of slums and blighted areas, land reclamation activities, the greening of land, and appropriate planning of marine and foreshore areas; e. Review and recommend for appropriate action of the sanggunian, governor or mayor the architectural plans and design submitted by governmental and non-governmental entities or individuals, particularly those for undeveloped, underdeveloped, and poorly-designed areas; f. Coordinate with government and NGOs and individuals involved in the aesthetics and the maximum utilization of the land and water within the jurisdiction of the LGU, compatible with environmental integrity and ecological balance. g. Be in the frontline of the delivery of services involving architectural planning and design, particularly those related to the redesigning of spatial distribution of basic facilities and physical structures during and in the aftermath of man-made and natural calamities and disasters; h. Recommend to the sanggunian and advise the governor or mayor on all other matters relative to the architectural planning and design as it relates to the total socioeconomic development of the local government unit; and (Sec 485[b]) i. Exercise such other powers and perform such other duties and functions as may be prescribed by law or ordinance. (Sec485[c]) a. Take charge of the office on public information b. Formulate measures and provide technical assistance and support in providing the information and research data required for the delivery of basic services and provision of adequate facilities so that the public becomes aware of said services and may fully avail of the same; c. Develop plans and strategies and implement the same, particularly those which have to do with public information

17. Architect

optional position

a. Citizen of the Philippines, b. A resident of the LGU concerned, c. Of good moral character, d. A duly licensed architect. e. Practiced his profession for at least 5 years in the case of the provincial or city architect, and 3 years in the case of the municipal architect. (Sec485[a])

18. Officer

Information

optional position

a. Citizen of the Philippines, b. A resident of the LGU concerned, c. Of good moral character, d. A holder of a college degree preferably in journalism, mass communication or any related course from a recognized college or university,

201 D2010

Local Government
e. A first grade civil service eligible or its equivalent. f. Have experience in writing articles and research papers, or in writing for print, television or broadcast media of at least 3 years in the case of the provincial or city information officer, and at least 1 year in the case of municipal information officer. g. Term of the information officer is co-terminous with his appointing authority. (Sec486[a]) and research data to support programs and projects which the governor or mayor is empowered to implement and which the sanggunian is empowered to provide for; d. Provide relevant, adequate, and timely information to the LGU and its residents; e. Furnish information and data on LGUs to government agencies or offices as may be required by law or ordinance; and NGOs to be furnished to said agencies and organizations; f. Maintain effective liaison with the various sectors of the community on matters and issues that affect the livelihood and the quality of life of the inhabitants and encourage support for programs of the local and national government; g. Be in the frontline in providing information during and in the aftermath of manmade and natural calamities and disasters, with special attention to the victims thereof, to help minimize injuries and casualties during and after the emergency, and to accelerate relief and rehabilitation; h. Recommend and advise on all other matters relative to public information and research data as it relates to the total socioeconomic development of the LGU; (Sec486[b]) i. Exercise such other powers and perform such other duties and functions as may be prescribed by law or ordinance. (Sec486[c]) a. Take charge of the office for the development of cooperatives b. Formulate measures and provide technical assistance and support in carrying out measures to ensure the delivery of basic services and provision of facilities through the development of cooperatives, and in providing access to such services and facilities; c. Develop plans and strategies and implement the same, particularly those which have to do with the integration of cooperatives principles and methods in programs and projects which the governor or mayor is empowered to implement and which the sanggunian is empowered to provide for; d. Assist in the organization of cooperatives; e. Provide technical and other forms of assistance to existing cooperatives to enhance their viability as an economic enterprise and social organization; f. Assist cooperatives in establishing linkages with government agencies and NGOs involved in the promotion and integration of the concept of cooperatives in the livelihood of the people and other community activities; g. Be in the frontline of cooperatives organization, rehabilitation or viability-enhancement, particularly during and in the aftermath of man-made and natural calamities and disasters, to aid in their survival and, if necessary subsequent rehabilitation; h. Recommend and advise on all other matters relative to cooperatives development and viability- enhancement which will improve the livelihood and quality of life of the inhabitants; (Sec487[b]) i. Exercise such other powers and perform such other duties and functions as may be prescribed by law or ordinance. (Sec487[c]) a. Take charge of the office on population development b. Formulate measures and provide technical assistance and support in carrying out measures to ensure the delivery of basic services and provision of adequate facilities relative to the integration of the population development principles and in providing access to said services and facilities; c. Develop plans and strategies and implement the same, particularly those which have to do with the integration of population development principles and methods in programs and projects which the governor or mayor is empowered to implement and which the sanggunian is empowered to provide

19. Officer

Cooperative

optional: provincial and city

a. Citizen of the Philippines, b. A resident of the LGU concerned, c. Of good moral character, d. Holder of a college degree preferably in business administration with special training in cooperatives or any related course from a recognized college or university, e. First grade civil service eligible or its equivalent f. Have experience in cooperatives organization and management of at least 5 years in the case of the provincial or city cooperatives officer, and 3 years in the case of municipal cooperatives officer. (Sec487[a])

20. Officer

Population

optional position

a. Citizen of the Philippines, b. A resident of the LGU concerned, c. Of good moral character, d. A holder of a college degree with specialized training in population development from a recognized college or university, e. A first grade civil service eligible or its equivalent. f. Have experience in the implementation of programs on

D2010 202

UP College of Law
population development or responsible parenthood for at least 5 years in the case of the provincial or city population officer and 3 years in the case of the municipal population officer. (Sec488[a]) for; d. Assist the governor or mayor in the implementation of the Constitutional provisions relative to population development and the promotion of responsible parenthood; e. Establish and maintain an updated data bank for program operations, development planning and an educational program to ensure the people's participation in and understanding of population development; f. Implement appropriate training programs responsive to the cultural heritage of the inhabitants; (Sec488[b]) g. Exercise such other powers and perform such other duties and functions as may be prescribed by law or ordinance. (Sec488[c]) a. Take charge of the office for veterinary services; b. Formulate measures and provide technical assistance and support in carrying out measures to ensure the delivery of basic services and provision of adequate facilities pursuant to Section 17; c. Develop plans and strategies and implement the same particularly those which have to do with the veterinary-related activities which the governor or mayor is empowered to implement and which the sanggunian is empowered to provide for d. Advise the governor or the mayor on all matters pertaining to the slaughter of animals for human consumption and the regulation of slaughterhouses; e. Regulate the keeping of domestic animals; f. Regulate and inspect poultry, milk and dairy products for public consumption; g. Enforce all laws and regulations for the prevention of cruelty to animals; h. Take the necessary measures to eradicate, prevent or cure all forms of animal diseases; i. Be in the frontline of veterinary related activities, such as in the outbreak of highly-contagious and deadly diseases, and in situations resulting in the depletion of animals for work and human consumption, particularly those arising from and in the aftermath of man-made and natural calamities and disasters; j. Recommend and advise on all other matters relative to veterinary services which will increase the number and improve the quality of livestock, poultry and other domestic animals used for work or human consumption; (Sec489[b]) k. Exercise such other powers and perform such other duties and functions as may be prescribed by law or ordinance. (Sec489[c]) a. Take charge of the office on general services; b. Formulate measures and provide technical assistance and support in carrying out measures to ensure the delivery of basic services and provision of adequate facilities pursuant to Section 17 and which require general services expertise and technical support services; c. Develop plans and strategies and implement the same, particularly those which have to do with the general services supportive of the welfare of the inhabitants which the governor or mayor is empowered to implement and which the sanggunian is empowered to provide for; d. Take custody of and be accountable for all properties, real or personal, owned by the LGU and those granted to it in the form of donation, reparation, assistance and counterpart of joint projects; e. Assign building or land space to local officials or other public officials, who by law, are entitled to such space; f. Recommend the reasonable rental rates for local government properties, whether real or personal, which will be leased to public or private entities g. Recommend reasonable rental rates of private properties

21. Veterinarian

mandatory: provincial and city

a. Citizen of the Philippines, b. A resident of the local government concerned, c. Of good moral character, d. A licensed doctor of veterinary medicine, e. Have practiced his profession for at least 3 years in the case of provincial or city veterinarian and at least 1 year in the case of the municipal veterinarian. (Sec489[a])

22. General Services Officer

mandatory: provincial and city

a. Citizen of the Philippines, b. A resident of the local government unit concerned, c. Of good moral character, d. A holder of a college degree on public administration, business administration and management from a recognized college or university, e. A first grade civil service eligible or its equivalent. f. Have acquired experience in general services, including management of supply, property, solid waste disposal, and general sanitation, of at least 5 years in the case of the provincial or city general services officer, and at least 3 years in the case of the municipal general services officer. (Sec490[a])

203 D2010

Local Government
which may be leased for the official use of the local government unit; h. Maintain and supervise janitorial, security, landscaping and other related services in all local government public buildings and other real property i. Collate and disseminate information regarding prices, shipping and other costs of supplies and other items commonly used by the LGU; j. Perform archival and record management with respect to records of offices and departments of the LGU k. Perform all other functions pertaining to supply and property management performed by the local government treasurer; and enforce policies on records creation, maintenance, and disposal; l. Be in the frontline of general services related activities, such as the possible or imminent destruction or damage to records, supplies, properties, and structures and the orderly and sanitary clearing up of waste materials or debris, particularly during and in the aftermath of man-made and natural calamities and disasters; m. Recommend and advise on all other matters relative to general services; (Sec490[b]) n. Exercise such other powers and perform such other duties and functions as may be prescribed by law or ordinance. (Sec490[c])

LOCAL FISCAL ADMINISTRATION


Sec 303 306, LGC
SEC. 303. Remedies and Sanctions. - Local government units shall appropriate in their respective annual budgets such amounts as are sufficient to pay the loans and other indebtedness incurred or redeem or retire bonds, debentures, securities, notes and other obligations issued under this Title: Provided, That failure to provide the appropriations herein required shall render their annual budgets inoperative. SEC. 304. Scope. - This Title shall govern the conduct and management of financial affairs, transactions, and operations of provinces, cities, municipalities, and barangays. SEC. 305. Fundamental Principles. - The financial affairs, transactions, and operations of local government units shall be governed by the following fundamental principles: a. No money shall be paid out of the local treasury except in pursuance of an appropriations ordinance or law; Local government funds and monies shall be spent solely for public purposes; Local revenue is generated only from sources expressly authorized by law or ordinance, and d. collection thereof shall at all times be acknowledged properly; All monies officially received by a local government officer in any capacity or on any occasion shall be accounted for as local funds, unless otherwise provided by law; Trust funds in the local treasury shall not be paid out except in fulfillment of the purpose for which the trust was created or the funds received; Every officer of the local government unit whose duties permit or require the possession or custody of local funds shall be properly bonded, and such officer shall be accountable and responsible for said funds and for the safekeeping thereof in conformity with the provisions of law; Local governments shall formulate sound financial plans, and the local budgets shall be based on functions, activities, and projects, in terms of expected results; development plans, goals, and strategies in order to optimize the utilization of resources and to avoid duplication in the use of fiscal and physical resources; Local budget plans and goals shall, as far as practicable, be harmonized with national development plans, goals, and strategies in

e.

f.

g.

b. c.

h.

D2010 204

UP College of Law
order to optimize the utilization of resources and to avoid duplication in the use of fiscal and physical resources; i. Local budgets shall operationalize approved local development plans; j. Local government units shall ensure that their respective budgets incorporate the requirements of their component units and provide for equitable allocation of resources among these component units; k. National planning shall be based on local planning to ensure that the needs and aspirations of the people as articulated by the local government units in their respective local development plans are considered in the formulation of budgets of national line agencies or offices; l. Fiscal responsibility shall be shared by all those exercising authority over the financial affairs, transactions, and operations of the local government units; and m. The local government unit shall endeavor to have a balanced budget in each fiscal year of operation. SEC. 306. Definitions. - When used in this Title, the term a. "Annual Budget" refers to a financial plan embodying the estimates of income and expenditures for one (1) fiscal year; "Appropriation" refers to an authorization made by ordinance, directing the payment of goods and services from local government funds under specified conditions or for specific purposes; "Budget Document" refers to the instrument used by the local chief executive to present a comprehensive financial plan to the sanggunian concerned; "Capital Outlays" refers to appropriations for the purchase of goods and services, the benefits of which extend beyond the fiscal year and which add to the assets of the local government unit concerned, including investments in public utilities such as public markets and slaughterhouses; "Continuing Appropriation" refers to an appropriation available to support obligations for a specified purpose or projects, such as those for the construction of physical structures or for the acquisition of real property or equipment, even when these obligations are incurred beyond the budget year; f. "Current Operating Expenditures" refers to appropriations for the purchase of goods and services for the conduct of normal local government operations within the fiscal year, including goods and services that will be used or consumed during the budget year; g. "Expected Results" refers to the services, products, or benefits that will accrue to the public, estimated in terms of performance measures or physical targets; h. "Fund" refers to a sum of money, or other assets convertible to cash, set aside for the purpose of carrying out specific activities or attaining certain objectives in accordance with special regulations, restrictions, or limitations, and constitutes an independent fiscal and accounting entity; i. "Income" refers to all revenues and receipts collected or received forming the gross accretions of funds of the local government unit; j. "Obligations" refers to an amount committed to be paid by the local government unit for any lawful act made by an accountable officer for and in behalf of the local unit concerned; k. "Personal Services" refers to appropriations for the payment of salaries, wages and other compensation of permanent, temporary, contractual, and casual employees of the local government unit; l. "Receipts" refers to income realized from operations and activities of the local government or are received by it in the exercise of its corporate functions, consisting of charges for services rendered, conveniences furnished, or the price of a commodity sold, as well as loans, contributions or aids from other entities, except provisional advances for budgetary purposes; and m. "Revenue" refers to income derived from the regular system of taxation enforced under authority of law or ordinance, and, as such, accrue more or less regularly every year.

b.

c.

d.

e.

Rivera v. Malolos
Petitioner Rivera won the bidding for supply of road construction materials in the Municipality of Malolos, Bulacan. A contract was signed between the municipal mayor and the petitioner, with a stipulation that the latter will supply 2700 cubic meters of crushed adobe stones and 1400 cu. m. of gravel in consideration of P19,235.oo. A year after the contract was entered into,

205 D2010

Local Government
the petitioner still hasnt been paid. He requested that the balance be included in the municipalitys appropriations for the next fiscal year. The petitioner sued for the payment of the unpaid amount but the Auditor General denied the claim on the ff. grounds: 1) that no appropriation was made before the execution of the contract, as mandated by Revised Administrative Code Section 607 and 2) that the deliveries could no longer be verified by the Provincial Auditor of Bulacan or his representative. Petitioner Rivera argues that: 1) the annual allotment from the Motor Vehicle Law could be used to pay the indebtedness, 2) according to the Revised Administrative Code, municipalities are endowed with the faculties of a municipal corporation, therefore competent to contract and be contracted with. HELD: The Revised Administrative Code imposes the ff. requirements before a contract involving P2000 or more may be authorized: a) There must be an appropriation b) The Municipal Treasurer must certify to the officer entering into the contract that the funds have been duly appropriated for the purpose and that the proposed amount is available for expenditure. The Motor Vehicle Law invoked by petitioner merely allocates 10% of the money collected and deposited in a special trust account of the National treasury to the road and bridge funds of the different municipalities in proportion to their population. This alone does not satisfy the requisite appropriation and authority to disburse part of the proceeds from the Motor Vehicle Law to pay the petitioner. The Auditor General had no alternative but to comply with the law and as the contract entered into by the Malolos Mayor was contrary to it, the Auditor General was correct in denying the petitioners claim.

APPLICATION OF THE LGC TO LGUS IN THE AUTONOMOUS REGION


Sec 526, LGC
SEC. 526. Application of this Code to Local Government Units in the Autonomous Regions. This Code shall apply to all provinces, cities, municipalities and barangays in the autonomous regions until such time as the regional government concerned shall have enacted its own local government code.

THE ARMM
Sec 1, 15, 16, 17, 18, 20, 21, Art X, 1987 Constitution
Section 1. The territorial and political subdivisions of the Republic of the Philippines are the provinces, cities, municipalities, and barangays. There shall be autonomous regions in Muslim Mindanao and the Cordilleras as hereinafter provided. Section 15. There shall be created autonomous regions in Muslim Mindanao and in the Cordilleras consisting of provinces, cities, municipalities, and geographical areas sharing common and distinctive historical and cultural heritage, economic and social structures, and other relevant characteristics within the framework of this Constitution and the national sovereignty as well as territorial integrity of the Republic of the Philippines. Section 16. The President shall exercise general supervision over autonomous regions to ensure that the laws are faithfully executed. Section 17. All powers, functions, and responsibilities not granted by this Constitution or by law to the autonomous regions shall be vested in the National Government. Section 18. The Congress shall enact an organic act for each autonomous region with the assistance and participation of the regional consultative commission composed of representatives appointed by the President from a list of nominees from multisectoral bodies. The organic act shall define the basic structure of government from the region consisting of the executive department and legislative assembly, both of which shall be reflective and representative of the constituent political units. The organic acts shall likewise provide for special courts with personal, family, and property law jurisdiction consistent with the provisions of this Constitution and national laws. The creation of the autonomous region shall be effective when approved by majority of the votes cast by the constituent units in a plebiscite called for the purpose,

D2010 206

UP College of Law
provided that only provinces, cities, and geographic areas voting favorably in such plebiscite shall be included in the autonomous region. Section 20. Within its territorial jurisdiction and subject to the provisions of this Constitution and national laws, the organic act of autonomous regions shall provide for legislative powers over: (1) Administrative organization; (2) Creation of sources of revenues; (3) Ancestral domain and natural resources; (4) Personal, family, and property relations; (5) Regional urban and rural planning development; (6) Economic, social, and tourism development; (7) Educational policies; (8) Preservation and development of the cultural heritage; and (9) Such other matters as may be authorized by law for the promotion of the general welfare of the people of the region. Section 21. The preservation of peace and order within the regions shall be the responsibility of the local police agencies which shall be organized, maintained, supervised, and utilized in accordance with applicable laws. The defense and security of the regions shall be the responsibility of the National Government. Engineering District in the First District of Lanao del Surin effect reestablishing the National Governments jurisdiction over infrastructure programs in the area. Petitioners assail the constitutionality of R.A. 8999 and D.O. 119, saying that the two run counter to the constitutional autonomy of the ARMM. HELD: R.A 8999 is antagonistic and cannot be reconciled with the ARMM Organic Acts 6734 and 9054. The idea behind the Constitutional provisions for the autonomous regions is to allow separate development of peoples with distinct cultures and traditions. It strives to free Philippine society of the strain caused by the assimilationist approach. Decentralization is a prerequisite to autonomy. It comes in two formsdeconcentration and devolution. The former is a decentralization of administration, the latter the decentralization of power. The framers of the Constitution intended for the autonomy of the ARMM to be in the nature of the latter- a meaningful and authentic regional autonomy. Ratified through a plebiscite, the ARMM Organic Act cannot be amended without a plebiscite, which R.A. 8999 clearly lacked. It is the intention of the ARMM Organic Acts to cede some, if not most of the powers of the national government to the autonomous government. The enforcement of RA 8999 runs afoul of these organic acts. It bears stressing that national laws are subject to the Constitution, one of the policies of which is to ensure autonomy of autonomous regions, subject only to general supervision by the President. Meanwhile, the office created under D.O. 119 is a duplication of the DPWH-ARMM First Engineering District in Lanao del Sur formed under the auspices of E.O. 426. The Department order in effect takes back powers, which had already been devolved to the ARG. The DPWH order, cannot rise higher than its source the Executive. It is also worthy to note that E.O 124 on which D.O. 119 is based is merely a general law organizing the DPWH while E.O. 426 is a special law transferring control and supervision of DPWH offices within the ARMM to the ARG. The latter should prevail. Even without applying the principle of lex specialis derogat generali, the enactment of R.A. 9054 in 2001 which repealed laws, orders and issuances inconstent with it rendered D.O. 119 functus officio.

RA 6734
See attachments

Disomancop v. Datumanong
Pursuant to Article 10, Sec 15 of the 1987 Constitution, RA 6734 entitled An Act Providing for An Organic Act for the Autonomous Region of Muslim Mindanao was enacted. In a plebiscite, the provinces of Lanao del Sur, Maguindanao, Sulu and Tawi-Tawi voted to become part of the ARMM. To implement RA 6734, then President Cory Aquino issued E.O 426, placing the control and supervision of the Offices of the DPWH within the ARMM under the jurisdiction of the Autonomous Regional Government (ARG). Nine years later, them DPWH Secretary issued D.O 119 creating a DPWH Marawi SubDistrict Engineering Office, which shall have jurisdiction over all national infrastructure projects and over all facilities under the DPWH within Marawi City and Lanao del Sur. R.A. 9054 was later passed, detailing and expanding the specific powers of the ARG. In 2001 however, RA 8999 was enacted, establishing an

Abbas v. Comelec
Petitioners argue that R.A. 6734 or the Act Providing for an Organic Act for the Autonomous Region in Muslim Mindanao is unconstitutional. According to them, R.A.

207 D2010

Local Government
6734 unconditionally creates an autonomous region in Mindanao, regardless of how many provinces vote for autonomy, contrary to Article X, Section 5 of the Constitution, which makes the creation of such a region dependent on the outcome of the plebiscite. One of the petitioners also insist that the RA is unconstitutional because it includes non-Muslim areas, which do not share the same characteristics with the other provinces. Seemingly contradicting himself, the petitioner also claims that since some non-Muslim areas have been included, other non-Muslim areas in Mindanao must also be covered in order for them to similarly enjoy the benefits of autonomy. The RAs failure to include all such areas is allegedly a violation of the equal protection clause of the Constitution. Petitioners likewise assert that Article XIX, Section 13 of RA 6734 grants the President power to merge regions a power not conferred by the Constitution. Lastly, they contend that the creation of an Oversight Committee is unconstitutional in that it delays the creation of an autonomous region. HELD: R.A. 6734 substantially incorporates the requirements for the creation of an autonomous region embodied in the Constitution. Creation of an autonomous region takes effect only when approved by a majority of the votes cast in a plebiscite, and only those provinces and cities voting favorably shall be included. Majority means a simple majority of votes approving the Organic Act in individual constituent units and not a double majority of the votes in all constituent units put together, as well as the constituent units taken individually. Contrary to the petitioners claims, creation of autonomous region is not rendered absolute. It may be that even if an autonomous region is created, not all of the 13 provinces and 9 municipalities shall be included. This is because the plebiscite will be determinative of two points: 1) whether there will be an autonomous region in Muslim Mindanao, and 2) which provinces and cities shall comprise it. With respect to the inclusion of non-Muslim areas, the contention is not tenable. Ascertainment of the areas that share common attributes is within the legislatures discretion. It is beyond the review powers of the judiciary. The same explanation supports the rejection of the claim that other non-Muslim areas should be included in the autonomous region, lest the equal protection clause be violated. Equal protection permits of reasonable classification. Since the Congress based its classification on real and substantial distinctions, no violation was made. The power of the President to merge administrative regions was upheld, it pertaining merely to groupings of contiguous provinces for administrative purposes. Administrative regions are not territorial and political subdivisions like provinces, municipalities and barangays that require a plebiscite to be merged. The power to merge administrative regions although not expressly provided for in the Constitution is traditionally lodged with the President. The questioned provisions requiring an Oversight Committee to supervise the transfer of national offices to the regional government is precisely aimed to effect a smooth transition and cannot be considered an impediment or a cause of delay. Every law has in its favor the presumption of constitutionality and the petitioners failed to overcome that presumption.

Pandi v. CA
Take note of the dates because they are significant in determining the validity of appointments

On August 9, 1993, Dr. Jamila R. Macacaua, in her capacity as Regional Director and DOH-ARMM Secretary issued a Memorandum appointing Dr. Pandi as Officerin-Charge of the Integrated Provincial Health Office-Amai Pakpak General Hospital in Lanao del Sur (IPHO-APGH) and transferring the incumbent OIC, Dr. Sani to the DOHARMM Regional Office in Cotabato City. On September 15, 1993, Lanao del Sur Governor Mahid Mutilan appointed Dr. Saber also to the position of OIC of the IPHO-APGH. In other words, Saber and Pandi were appointed to the same position by different appointing officers. Sani on the other hand, contests her being moved to Cotabato and claims to be the holder of a permanent appointment as provincial health officer (PHO) of IPHO-PGH, the same post that Saber and Pandi were appointed to. On October 5, 1993 President Ramos issued EO 133, transferring powers and functions of the DOH in the region to the ARMM Regional Government (ARG), pursuant to which Macacaua reiterated her appointments. The parties are in dispute as to which appointments are valid. The case traces the enactment of various legislation, divided into five periods, to wit: 1) the time prior to effectivity of Organic Act of 1989, 2) the time after Organic Act 1989 but before the LGC of 1991, 3) after the LGC of 1991 but before the ARMM Code, 4) after the ARMM Code but before Organic Act of 2001, 5) after Organic Act of 2001. During the first period, the governing law was the DOH Charter (EO 119), in which the power to appoint was granted to the Minister of

D2010 208

UP College of Law
Health. Then LGC of 1984 classified the Provincial Health Officer as a national government official whose salary is paid out of national funds. The ARMM was created after the enactment of the Organic Act of 1989. The latter transferred certain agencies and offices of the national government to the Regional Government but the DOH was not among them. PHOs were still part of the national government until the Regional Government adopts its own Local Government Code. During the third period, the LGC of 1991 took effect, naming the provincial health officer as an official of the Provincial Government to be appointed by the Governor if his salary is paid out of provincial funds. One must however note that although LGC 1991 is a later law than OA 1989, the latter is not affected by the formers enactment because an organic act requires an approval through plebiscite to be amended. Thus, even with LGC 1991s passage, the appointment of the PHO is still with the Secretary of Health. It was only upon the effectivity of EO 133 in October 1993 that the power to appoint provincial health officers to any province was assigned to the ARMM Secretary of Health (Regional Secretary). In the fourth period, the ARMM Local Code came into being, stating that if the salary of a PHO comes from provincial funds, appointing power is with the Provincial Governor and if it comes from regional funds, then it is with the Regional Governor, upon recommendation from the Provincial Governor. In case of doubt, the ARMM Local Code is interpreted in favor of devolution- in favor of the provincial governors powers. The fifth period covers the passage of the Organic Act of 2001, through which the powers and functions of any other provincial governor under the LGC of 1991, including the power to appoint PHOs, are now enjoyed as a minimum by the Provincial Governor of the ARMM. HELD: When Governor Mahid Mutilan appointed Saber as OIC on September 1993, the provincial health officer was still a national government official paid out of national funds. The provincial health officer became a provincial government official only on March 3, 1994 after the effectivity of the ARMM Local Code. The governor had no power to make such designation at the time, hence the appointment of Saber as OIC is void. Reliance on the LGC is misplaced since it did not amend the Organic Act of 1989. The provision in the LGC which states that The appointment of a health officer shall be mandatory for provincial, city and municipal governments is merely a directive that those empowered to appoint local health officers are mandated to do so. With respect to the initial transfer of Sani by Macacaua from Lanao del Sur to Cotabato on August 1993, the Court holds that the act is void, since the power to appoint was still with the Secretary of Health at the time, not with the Regional Secretary. However, when Macacaua issued a second memorandum on November 6, 1993, reiterating the transfer of Sani, the prior error was cured and the transfer became valid since it was made after the issuance of EO 133 which expressly transferred supervision and control over all functions and activities of the Regional Department of Health to the Head of the Regional Department of Health. The same is true for the appointment of Pandi, which was similarly made by Macacaua on the same dates. The appointments made by Macacaua are valid while that made by Governor Mutilan is void. The Court reminds us however, that after the effectivity of the ARMM Local Code, the Regional Secretary had been stripped of authority to make such a designation. The said power is now with the Provincial Governor.

Bai Sema v. Comelec (supra)


Maguindanao forms part of the ARMM, created under Organic Act (RA 9054). The Ordinance appended to the Constitution apportioned two legislative districts to the Province of Maguindanao, the first consisting of Cotabato City and eight other municipalities. The ARMMs legislature, exercising its power to create provinces under Section 19 of R.A 9054, enacted Muslim Mindanao Autonomy Act No. 201 (MMA Act 201) creating the Province of Shariff Kabunsuan composed of the said eight municipalities in Maguindanaos first district. The Act however provided that despite the creation of the new province, the existing legislative district (8 municipalities + Cotabato) shall still remain. The voters of Maguindanao ratified Shariff Kabunsuans creation in a plebiscite. Cotabatos Sangguniang Panlunsod asked the COMELEC to clarify the status of Cotabato City in view of the creation of Shariff Kabunsuan. To this, COMELEC answered with a Resolution maintaining the status quo- with Cotabato and Shariff Kabunsuan constituting part of the first legislative district in Maguindanao. COMELEC later promulgated Resolution No. 7902 renaming Maguindanaos first district as Shariff Kabunsuan with Cotabato City. Bai Sema, a candidate for Congresswoman of Shariff Kabunsuan with Cotabato City, asserts that according to Article VI, Section 5(3) of the Constitution, and Ordinance 3 appended to the latter, Shariff Kabunsuan is entitled to one representative, and Cotabato with a population of only 163, 849 to another. She cites Felwa vs. Salas, which held that when a province is created by statute, the corresponding legislative district comes into existence

209 D2010

Local Government
neither by authority of that statute nor by apportionment but by operation of the Constitution, without reapportionment. In other words, she wants Cotabato and Shariff Kabunsuan to have separate representatives, since according to her, the creation of a new province necessitates the creation of a corresponding legislative district. HELD: Although the Congress is not given by the Constitution express powers to delegate the creation of local government units, such power may be gleaned from its plenary powers. While there is no conflict between the Constitution and Congress delegation of the power to create municipalities and barangays, it is an altogether different matter when it comes to the creation of cities and provinces. This is because of the situation created by Article VI, Section 5(3) of the Constitution which says, "Each city with a population of at least two hundred fifty thousand, or each province, shall have at least one representative" in the House of Representatives. A province cannot be created without a legislative district because it will violate the aforequoted provision of the Constitution as well as Section 3 of the Ordinance appended to the former. For Congress to delegate validly the power to create a province or city, it must also validly delegate the power to create a legislative district. The latter is however disallowed by the Section 5(1) Article VI of the Constitution, giving Congress the exclusive power to create or reapportion legislative districts. It would be anomalous for regional or local legislative bodies to create or reapportion legislative districts for a national legislature like Congress. The office of a legislative district representative to Congress is a national office, and its occupant, a Member of the House of Representatives, is a national official. It would be incongruous for a regional legislative body like the ARMM Regional Assembly to create a national office when its legislative powers extend only to its regional territory. The office of a district representative is maintained by national funds and the salary of its occupant is paid out of national funds. To allow the ARMM Regional Assembly to create a national office is to allow its legislative powers to operate outside the ARMM's territorial jurisdiction. This violates Section 20, Article X of the Constitution which expressly limits the coverage of the Regional Assembly's legislative powers "within its territorial jurisdiction." Since a province cannot be legally created without a legislative district, the creation of the Province of Shariff Kabunsuan is unconstitutional. The reliance by Sema on the Felwa case is misplaced. A district is created in two ways: a) indirectly, through the creation of a province, and b) directly, by creation of legislative districts. The court sustained the constitutionality of the creation of a new district in that case because it was made indirectly through a special law enacted by the Congress creating a province and also because the creation of legislative districts will not exceed the maximum number of representatives allowed by the Constitution. Semas theory will lead to the following disastrous consequences: (1) An inferior legislative body like the ARMM Regional Assembly can create 100 or more provinces and thus increase the membership of a superior legislative body, the House of Representatives, beyond the maximum limit of 250 fixed in the Constitution (unless a national law provides otherwise); (2) The proportional representation in the House of Representatives based on 1 rep/at least 250,000 residents will be negated because the ARMM Regional Assembly need not comply with the requirement in Section 461(a)(ii) of RA 7160 that a province or city must have a minimum population of 250,ooo; and (3) Representatives from the ARMM provinces can become the majority in the House of Representatives through the ARMM Regional Assembly's continuous creation of provinces or cities within the ARMM. The Congress and the framers of the Constitution did not intend such consequences. Organic acts of autonomous regions cannot prevail over the Constitution, Sec 20 of which provides that legislative powers of regional assemblies are limited within its territorial jurisdiction and subject to the provisions of the Constitution. Section 19, Article VI of R.A. 9054 is unconstitutional insofar as it grants to the ARMM Regional Assembly the power to create provinces and cities. MMA 201 is thus void and of no effect. Separate Opinion by Tinga: The only constitutional provision that concerns with the creation of provinces is Section 10, Article X, which reads: No province, city, municipality or barangay may be created, divided, merged, abolished, or its boundary substantially altered, except in accordance with the criteria established in the local government code and subject to approval by a majority of the votes cast in a plebiscite in the political units directly affected. Nothing in this provision specifically limits the power to create provinces, cities, municipalities or barangays to Congress alone. The provision does embody a significant limitation - that the creation of these political subdivisions must be in accordance with the criteria established in the local government code, a law which is enacted by Congress. It would thus be proper to say that the Constitution limits

D2010 210

UP College of Law
the ability to set forth the standards for the creation of a province exclusively to Congress. But to say that the Constitution confines to Congress alone the power to establish the criteria for creating provinces is vastly different from saying that the Constitution confines to Congress alone the power to create provinces. There is nothing in the Constitution that supports the latter proposition.

CAR
Admin Order 220
See attachments Since Ifugao is very small province, it would have too many government officials for so few people. The law also creates a Regional Planning and Development Board consisting of several members, with functions similar to that of a Provincial Coordinators. If it takes only one person in the provincial level to perform those functions while it takes an entire Board to perform substantially the same tasks in the regional level, it only means that a larger area is contemplated by the law to make up the autonomous region. Also, the huge allotment of P10M to the Regional Government for its initial organizational requirements is too much to fund a lone and small province.

Ordillo v. Comelec
The people of Benguet, Mountain Province, Ifugao, Abra, Kalinga-Apayao and Baguio City cast their votes in a plebiscite pursuant to R.A. 6766 entitled An Act Providing for an Organic Act for the Cordillera Autonomous Region. A total of 5,889 people voted for the creation of the region while an overwhelming majority of 148,676 votes rejected it. Consequently, COMELEC issued resolution No.2259 stating that since the Organic Act for the region has been approved by a majority of votes in Ifugao Province only, the latter alone will constitute the autonomous region. The President also issued Administrative Order No. 160 abolishing the Cordillera Executive Board and the Cordillera Regional Assembly in view of the Organic Acts ratification. Petitioner assails the Comelec Resolution 2259 and AO No. 160, saying that the province of Ifugao cannot solely constitute the Cordillera Autonomous Region. HELD: The petition is meritorious. Article X, Sec 15 of the Constitution is explicit in providing that provinces, cities, municipalities and geographical areas shall constitute the autonomous region- meaning more than one constituent unit. The term region used in its ordinary sense means two or more provinces. This is supported by the fact that the 13 regions we have in the country are groupings of contiguous provinces. Ifugao is a province in itself, one of the smallest in the country to boot making up only 11% of the total population of the areas mentioned in RA 6766. The law reiterates the provision in the Constitution by providing that The Regional Government shall exercise powers...for the proper governance of all provinces, cities, municipalities and barangays. It can be gleaned that Congress never intended a single province to constitute an autonomous region. Otherwise, we would be faced with an absurd situation of having two sets of officials- a set of provincial officials and a set of regional officials exercising executive and legislative powers over exactly the same small area.

Cordillera Broad Coalition v. COA


Executive Order No. 220, issued by the President in the exercise of her legislative powers under Art. XVIII, sec. 6 of the 1987 Constitution, created the Cordillera Administrative Region (CAR), which covers the provinces of Abra, Benguet, Ifugao, Kalinga-Apayao and Mountain Province and the City of Baguio [secs. 1 and 2]. It was created to accelerate economic and social growth in the region and to prepare for the establishment of the autonomous region in the Cordilleras [sec. 3]. Its main function is to coordinate the planning and implementation of programs and services in the region, particularly, to coordinate with the local government units as well as with the executive departments of the National Government in the supervision of field offices and in identifying, planning, monitoring, and accepting projects and activities in the region [sec. 5]. It shall also monitor the implementation of all ongoing national and local government projects in the region [sec. 20]. The CAR shall have a Cordillera Regional Assembly as a policyformulating body and a Cordillera Executive Board as an implementing arm [secs. 7, 8 and 10]. The CAR and the Assembly and Executive Board shall exist until such time as the autonomous regional government is established and organized [sec. 17]. In these cases, petitioners principally argue that by issuing E.O. No. 220 the President, in the exercise of her legislative powers prior to the convening of the first

211 D2010

Local Government
Congress under the 1987 Constitution, has virtually preempted Congress from its mandated task of enacting an organic act and created an autonomous region in the Cordilleras. During the pendency of this case, R.A. 6766 entitled "An Act Providing for an Organic Act for the Cordillera Autonomous Region," was enacted and signed into law. The Act recognizes the CAR and the offices and agencies created under E.O. No. 220 and its transitory nature is reinforced in Art. XXI of R.A. No. 6766, to wit: SEC. 3. The Cordillera Executive Board, the Cordillera Region Assembly as well as all offices and agencies created under Execute Order No. 220 shall cease to exist immediately upon the ratification of this Organic Act. All funds, properties and assets of the Cordillera Executive Board and the Cordillera Regional Assembly shall automatically be transferred to the Cordillera Autonomous Government. WON E.O.220 is unconstitutional because it pre-empts the Congress from enacting an organic act for the autonomous region in Cordillera. Held: No. A reading of E.O. No. 220 will easily reveal that what it actually envisions is the consolidation and coordination of the delivery of services of line departments and agencies of the National Government in the areas covered by the administrative region as a step preparatory to the grant of autonomy to the Cordilleras. It does not create the autonomous region contemplated in the Constitution. It merely provides for transitory measures in anticipation of the enactment of an organic act and the creation of an autonomous region. In short, it prepares the ground for autonomy. This does not necessarily conflict with the provisions of the Constitution on autonomous regions. The transitory nature of the CAR does not necessarily mean that it is, as petitioner Cordillera Broad Coalition asserts, "the interim autonomous region in the Cordilleras. The Constitution provides for a basic structure of government in the autonomous region composed of an elective executive and legislature and special courts with personal, family and property law jurisdiction [Art. X, sec. 18]. Using this as a guide, we find that E.O. No. 220 did not establish an autonomous regional government. It merely created a region, covering a specified area, for administrative purposes with the main objective of coordinating the planning and implementation of programs and services [secs. 2 and 5]. The bodies created by E.O. No. 220 do not supplant the existing local governmental structure, nor are they autonomous government agencies. They merely constitute the mechanism for an "umbrella" that brings together the existing local governments, the agencies of the National Government, the ethno-linguistic groups or tribes, and non-governmental organizations in a concerted effort to spur development in the Cordilleras. WON the CAR is a territorial and political subdivision. Held: No. E.O. 220 did not create a new territorial and political subdivision or merge existing ones into a larger subdivision. Firstly, the CAR is not a public corporation or a territorial and political subdivision. It does not have a separate juridical personality, unlike provinces, cities and municipalities. Neither is it vested with the powers that are normally granted to public corporations, e.g. the power to sue and be sued, the power to own and dispose of property, the power to create its own sources of revenue, etc. As stated earlier, the CAR was created primarily to coordinate the planning and implementation of programs and services in the covered areas. The CAR may be considered more than anything else as a regional coordinating agency of the National Government, similar to the regional development councils which the President may create under the Art. X, Sec. 14 of the Constitution. As we have said earlier, the CAR is a mere transitory coordinating agency that would prepare the stage for political autonomy for the Cordilleras. It fills in the resulting gap in the process of transforming a group of adjacent territorial and political subdivisions already enjoying local or administrative autonomy into an autonomous region vested with political autonomy.

MMDA AND LGUS


Sec 11, Art X, 1987 Constitution
Section 11. The Congress may, by law, create special metropolitan political subdivisions, subject to a plebiscite as set forth in Section 10 hereof. The component cities and municipalities shall retain their basic autonomy and shall be entitled to their own local executives and legislative assemblies. The jurisdiction of the

D2010 212

UP College of Law
metropolitan authority that will hereby be created shall be limited to basic services requiring coordination. agencies, peoples organizations, non-governmental organizations, and the private sector for the efficient and expeditious delivery of basic services in the vast metropolitan area. All its functions are administrative in nature and these are actually summed up in the charter itself, viz: "Sec. 2. Creation of the Metropolitan Manila Development Authority. -- x x x. The MMDA shall perform planning, monitoring and coordinative functions, and in the process exercise regulatory and supervisory authority over the delivery of metro-wide services within Metro Manila, without diminution of the autonomy of the local government units concerning purely local matters." Having no legislative power, the MMDA cannot enact ordinances. Thus, it becomes necessary for the City of Makati to enact an ordinance declaring the Neptune Street open to the public before the MMDA can implement such activity. WON the consolidated cases of Sangalang vs. IAC (as regards the SCs upholding of MMDAs alleged police power) are applicable in this case. Held: No. Firstly, the Sangalang cases involved zoning ordinances passed by the municipal council of Makati and the Metro Manila Commission (MMC the forerunner of MMDA). In the instant case, the basis for the proposed opening of Neptune Street is a mere notice sent by MMDA to the respondent, the former relying on its authority under its charter to rationalize the use of roads and/or thoroughfares for the safe and convenient movement of persons. Secondly, the MMDA is not the same entity as the MMC in Sangalang. Although the MMC is the forerunner of the present MMDA, an examination of P. D. No. 824, the charter of the MMC, shows that the latter possessed greater powers which were not bestowed on the present MMDA. Metropolitan Manila was first created in 1975 by P.D. 824. Its administration was placed under the Metro Manila Commission (MMC), which was specifically vested with legislative powers. The MMC was the central government of Metro Manila and fully possessed legislative and police powers. Whatever legislative powers the component cities and municipalities had were all subject to review and approval by the MMC.

RA 7924
See attachments

MMDA v. Bel Air Village Association Inc (supra)


MMDA attempted to demolish the wall separating Kalayan Avenue from the subdivision owned by the respondent. Thus, respondents filed a case for injunction against the MMDA. The CA ruled for the respondent and issued a permanent writ of injunction. Hence, this petition. MMDAs argument: that it has the authority to open the Neptune Street to public traffic because it is an agent of the state endowed with police power in the delivery of basic services in Metro Manila (in this case, traffic management). From the premise that it has police power, it is now urged that there is no need for the City of Makati to enact an ordinance opening Neptune street to the public. Moreover, it is alleged that the police power of MMDA was affirmed by this Court in the consolidated cases of Sangalang v. Intermediate Appellate Court. WON MMDA has police power and WON there is no need for the City of Makati to enact an ordinance opening the disputed street to the public. Held: No to both. MMDA has no police power and an ordinance enacted by the City of Makati is necessary for the opening of Neptune Street to the public. It will be noted that the powers of the MMDA are limited to the following acts: formulation, coordination, regulation, implementation, preparation, management, monitoring, setting of policies, installation of a system and administration. There is no syllable in R. A. No. 7924 that grants the MMDA police power, let alone legislative power. Even the Metro Manila Council has not been delegated any legislative power. Unlike the legislative bodies of the local government units, there is no provision in R. A. No. 7924 that empowers the MMDA or its Council to "enact ordinances, approve resolutions and appropriate funds for the general welfare" of the inhabitants of Metro Manila. The MMDA is, as termed in the charter itself, a "development authority." It is an agency created for the purpose of laying down policies and coordinating with the various national government

213 D2010

Local Government
In 1990, President Aquino issued E.O. No. 392 and constituted the Metropolitan Manila Authority (MMA). The powers and functions of the MMC were devolved to the MMA. It ought to be stressed, however, that not all powers and functions of the MMC were passed to the MMA. The MMAs power was limited to the "delivery of basic urban services requiring coordination in Metropolitan Manila." The MMAs governing body, the Metropolitan Manila Council, although composed of the mayors of the component cities and municipalities, was merely given the power of: (1) formulation of policies on the delivery of basic services requiring coordination and consolidation; and (2) promulgation of resolutions and other issuances, approval of a code of basic services and the exercise of its rule-making power. Under the 1987 Constitution, the local government units became primarily responsible for the governance of their respective political subdivisions. The MMAs jurisdiction was limited to addressing common problems involving basic services that transcended local boundaries. It did not have legislative power. Its power was merely to provide the local government units technical assistance in the preparation of local development plans. Any semblance of legislative power it had was confined to a "review [of] legislation proposed by the local legislative assemblies to ensure consistency among local governments and with the comprehensive development plan of Metro Manila," and to "advise the local governments accordingly." When R.A. No. 7924 took effect, Metropolitan Manila became a "special development and administrative region" and the MMDA a "special development authority" whose functions were "without prejudice to the autonomy of the affected local government units." The MMDA is not a political unit of government. The power delegated to the MMDA is that given to the Metro Manila Council to promulgate administrative rules and regulations in the implementation of the MMDAs functions. There is no grant of authority to enact ordinances and regulations for the general welfare of the inhabitants of the metropolis. It is good to note that the explanatory note to the bill which created MMDA stated that the proposed MMDA is a development authority which is a national agency, not a political government unit. It is beyond doubt that the MMDA is not a local government unit or a public corporation endowed with legislative power. It is not even a "special metropolitan political subdivision" as contemplated in Section 11, Article X of the Constitution. The creation of a "special metropolitan political subdivision" requires the approval by a majority of the votes cast in a plebiscite in the political units directly affected. R. A. No. 7924 was not submitted to the inhabitants of Metro Manila in a plebiscite. The Chairman of the MMDA is not an official elected by the people, but appointed by the President with the rank and privileges of a cabinet member. In fact, part of his function is to perform such other duties as may be assigned to him by the President, whereas in local government units, the President merely exercises supervisory authority. This emphasizes the administrative character of the MMDA. Clearly then, the MMC under P. D. No. 824 is not the same entity as the MMDA under R. A. No. 7924. Unlike the MMC, the MMDA has no power to enact ordinances for the welfare of the community. It is the local government units, acting through their respective legislative councils, that possess legislative power and police power. In the case at bar, the Sangguniang Panlungsod of Makati City did not pass any ordinance or resolution ordering the opening of Neptune Street, hence, its proposed opening by petitioner MMDA is illegal and the respondent Court of Appeals did not err in so ruling.

MMDA v. Garin
Dante O. Garin, a lawyer, was issued a traffic violation receipt (TVR) and his drivers license was confiscated for parking illegally along Gandara Street, Binondo, Manila, on 08/05/95. Due to the then MMDA Chairmans failure to heed Garins request that his drivers license be returned and that instead he be subjected to a case for traffic violation in court, Garin filed a cased for injunction in the RTC. Garins argument: In the absence of any implementing rules and regulations, Sec. 5(f) of R.A. No. 7924 grants the MMDA unbridled discretion to deprive erring motorists of their licenses, thereby violating the due process clause of the Constitution. Garin further contends that the provision violates the constitutional prohibition against undue delegation of legislative authority, allowing as it does the MMDA to fix and impose unspecified and therefore unlimited - fines and other penalties on erring motorists. MMDAs argument: The powers granted to it by Sec. 5(f) of Rep. Act No. 7924 are limited to the fixing, collection and imposition of fines and penalties for traffic violations, which powers are legislative and executive in nature; the judiciary retains the right to determine the

D2010 214

UP College of Law
validity of the penalty imposed. It further argues that the doctrine of separation of powers does not preclude admixture of the three powers of government in administrative agencies. Moreover, MMDA has already formulated its implementing rules for Sec. 5(f), which is Memorandum Circular No. YY-95-001 dated 04/15/95. The RTC ruled in favor of Garin. Thus, MMDA filed this petition in the SC. MMDA reiterates and reinforces its argument that a license to operate a motor vehicle is neither a contract nor a property right, but is a privilege subject to reasonable regulation under the police power in the interest of the public safety and welfare. It further argues that revocation or suspension of this privilege does not constitute a taking without due process as long as the licensee is given the right to appeal the revocation. Meanwhile, on 12 August 2004, the MMDA, through its Chairman Bayani Fernando, implemented Memorandum Circular No. 04, Series of 2004, outlining the procedures for the use of the Metropolitan Traffic Ticket (MTT) scheme. Under the circular, erring motorists are issued an MTT, which can be paid at any Metrobank branch. Traffic enforcers may no longer confiscate drivers licenses as a matter of course in cases of traffic violations. All motorists with unredeemed TVRs were given seven days from the date of implementation of the new system to pay their fines and redeem their license or vehicle plates. It would seem, therefore, that insofar as the absence of a prima facie case to enjoin the petitioner from confiscating drivers licenses is concerned, recent events have overtaken the Courts need to decide this case, which has been rendered moot and academic by the implementation of Memorandum Circular No. 04, Series of 2004. The petitioner, however, is not precluded from re-implementing Memorandum Circular No. TT-95-001, or any other scheme, for that matter, that would entail confiscating drivers licenses. For the proper implementation, therefore, of the petitioners future programs, the Supreme Court deemed it appropriate to make the following observations: 1) A license to operate a motor vehicle is a privilege that the state may withhold in the exercise of its police power; 2) The MMDA is not vested with police power; and 3) Sec. 5(f) grants the MMDA with the duty to enforce existing traffic rules and regulations (which means that MMDA may enforce, but cannot enact, ordinances). WON MMDA has police power and WON it has the authority to confiscate traffic violators drivers license without an enabling law enacted by Congress. Held: No Police power, as an inherent attribute of sovereignty, is the power vested by the Constitution in the legislature to make, ordain, and establish all manner of wholesome and reasonable laws, statutes and ordinances, either with penalties or without, not repugnant to the Constitution, as they shall judge to be for the good and welfare of the commonwealth, and for the subjects of the same. Having been lodged primarily in the National Legislature, it cannot be exercised by any group or body of individuals not possessing legislative power. The National Legislature, however, may delegate this power to the president and administrative boards as well as the lawmaking bodies of municipal corporations or local government units (LGUs). Once delegated, the agents can exercise only such legislative powers as are conferred on them by the national lawmaking body. In Metro Manila Development Authority v. Bel-Air Village Association, Inc., the SC categorically stated that Rep. Act No. 7924 does not grant the MMDA with police power, let alone legislative power, and that all its functions are administrative in nature. The MMDA is not a political unit of government. The power delegated to the MMDA is that given to the Metro Manila Council to promulgate administrative rules and regulations in the implementation of the MMDAs functions. There is no grant of authority to enact ordinances and regulations for the general welfare of the inhabitants of the metropolis. Therefore, insofar as Sec. 5(f) of Rep. Act No. 7924 is understood by the lower court and by the petitioner to grant the MMDA the power to confiscate and suspend or revoke drivers licenses without need of any other legislative enactment, such is an unauthorized exercise of police power. Sec. 5(f) grants the MMDA with the duty to enforce existing traffic rules and regulations. Thus, where there is a traffic law or regulation validly enacted by the legislature or those agencies to whom legislative powers have been delegated (the City of Manila in this case), the petitioner is not precluded and in fact is duty-bound to confiscate and suspend or revoke drivers licenses in the exercise of its mandate of transport and traffic management, as well as the administration and implementation of all traffic enforcement operations, traffic engineering services and traffic education programs. This is consistent with the ruling in Bel-Air that the MMDA is a development authority created for the

215 D2010

Local Government
purpose of laying down policies and coordinating with the various national government agencies, peoples organizations, non-governmental organizations and the private sector, which may enforce, but not enact, ordinances.

D2010 216

UP College of Law

Municipal Contracts
CORPORATE POWERS
Sec 22, LGC
See above all government agencies, including government-owned and controlled corporations (GOCCs) with original charters. An LWD is a GOCC with an original charter. Section 2(1), Article IX-D of the Constitution provides for COAs audit jurisdiction, as follows: SECTION 2. (1) The Commission on Audit shall have the power, authority and duty to examine, audit, and settle all accounts pertaining to the revenue and receipts of, and expenditures or uses of funds and property, owned or held in trust by, or pertaining to, the Government, or any of its subdivisions, agencies, or instrumentalities, including government-owned and controlled corporations with original charters, x x x The Constitution authorizes Congress to create government-owned or controlled corporations through special charters. Since private corporations cannot have special charters, it follows that Congress can create corporations with special charters only if such corporations are government-owned or controlled. Obviously, LWDs are not private corporations because they are not created under the Corporation Code. LWDs are not registered with the Securities and Exchange Commission. Section 14 of the Corporation Code states that *A+ll corporations organized under this code shall file with the Securities and Exchange Commission articles of incorporation x x x. LWDs have no articles of incorporation, no incorporators and no stockholders or members. There are no stockholders or members to elect the board directors of LWDs as in the case of all corporations registered with the Securities and Exchange Commission. The local mayor or the provincial governor appoints the directors of LWDs for a fixed term of office. This Court has ruled that LWDs are not created under the Corporation Code. LWDs exist by virtue of PD 198, which constitutes their special charter. Since under the Constitution only government-owned or controlled corporations may have special charters, LWDs can validly exist only if they are government-owned or controlled. To claim that LWDs are private corporations with a special charter is to admit that their existence is constitutionally infirm.

Feliciano v. COA
This is a petition for certiorari to annul the Commission on Audits (COA) Resolution dated 01/03/00 and the Decision dated 01/30/01 denying the Motion for Reconsideration. The COA denied petitioner Ranulfo C. Felicianos request for COA to cease all audit services, and to stop charging auditing fees, to Leyte Metropolitan Water District (LMWD). The COA also denied petitioners request for COA to refund all auditing fees previously paid by LMWD. Background: A Special Audit Team from COA Regional Office No. VIII audited the accounts of LMWD. Subsequently, LMWD received a letter from COA dated 19 July 1999 requesting payment of auditing fees. As General Manager of LMWD, petitioner sent a reply dated 12 October 1999 informing COAs Regional Director that the water district could not pay the auditing fees. Petitioner cited as basis for his action Sections 6 and 20 of Presidential Decree 198 (PD 198), as well as Section 18 of Republic Act No. 6758 (RA 6758). The Regional Director referred petitioners reply to the COA Chairman on 18 October 1999. On 19 October 1999, petitioner wrote COA through the Regional Director asking for refund of all auditing fees LMWD previously paid to COA. On 16 March 2000, petitioner received COA Chairman Celso D. Gangans Resolution dated 3 January 2000 denying his requests. Petitioner filed a motion for reconsideration on 31 March 2000, which COA denied on 30 January 2001. On 13 March 2001, petitioner filed this instant petition. WON a Local Water District (LWD) created under PD 198, as amended, is a government-owned or controlled corporation subject to the audit jurisdiction of COA. Held:Yes. The Constitution and existing laws mandate COA to audit

217 D2010

Local Government
Unlike private corporations, which derive their legal existence and power from the Corporation Code, LWDs derive their legal existence and power from PD 198. Sections 6 and 25 of PD 198 provide: Section 6. Formation of District. This Act is the source of authorization and power to form and maintain a district. For purposes of this Act, a district shall be considered as a quasi-public corporation performing public service and supplying public wants. As such, a district shall exercise the powers, rights and privileges given to private corporations under existing laws, in addition to the powers granted in, and subject to such restrictions imposed, under this Act. x x x Sec. 25. Authorization. The district may exercise all the powers which are expressly granted by this Title or which are necessarily implied from or incidental to the powers and purposes herein stated. For the purpose of carrying out the objectives of this Act, a district is hereby granted the power of eminent domain, the exercise thereof shall, however, be subject to review by the Administration. Clearly, LWDs exist as corporations only by virtue of PD 198, which expressly confers on LWDs corporate powers. Section 6 of PD 198 provides that LWDs shall exercise the powers, rights and privileges given to private corporations under existing laws. Without PD 198, LWDs would have no corporate powers. Thus, PD 198 constitutes the special enabling charter of LWDs. The ineluctable conclusion is that LWDs are governmentowned and controlled corporations with a special charter. The phrase government-owned and controlled corporations with original charters means GOCCs created under special laws and not under the general incorporation law. There is no difference between the term original charters and special charters. Petitioners contention that the Sangguniang Bayan resolution creates the LWDs assumes that the Sangguniang Bayan has the power to create corporations. This is a patently baseless assumption. The Local Government Code does not vest in the Sangguniang Bayan the power to create corporations. What the Local Government Code empowers the Sangguniang Bayan to do is to provide for the establishment of a waterworks system subject to existing laws. Thus, Section 447(5)(vii) of the Local Government Code provides: SECTION 447. Powers, Duties, Functions and Compensation. (a) The sangguniang bayan, as the legislative body of the municipality, shall enact ordinances, approve resolutions and appropriate funds for the general welfare of the municipality and its inhabitants pursuant to Section 16 of this Code and in the proper exercise of the corporate powers of the municipality as provided for under Section 22 of this Code, and shall: (vii) Subject to existing laws, provide for the establishment, operation, maintenance, and repair of an efficient waterworks system to supply water for the inhabitants; regulate the construction, maintenance, repair and use of hydrants, pumps, cisterns and reservoirs; protect the purity and quantity of the water supply of the municipality and, for this purpose, extend the coverage of appropriate ordinances over all territory within the drainage area of said water supply and within one hundred (100) meters of the reservoir, conduit, canal, aqueduct, pumping station, or watershed used in connection with the water service; and regulate the consumption, use or wastage of water; The Sangguniang Bayan may establish a waterworks system only in accordance with the provisions of PD 198. The Sangguniang Bayan has no power to create a corporate entity that will operate its waterworks system. However, the Sangguniang Bayan may avail of existing enabling laws, like PD 198, to form and incorporate a water district. Besides, even assuming for the sake of argument that the Sangguniang Bayan has the power to create corporations, the LWDs would remain government-owned or controlled corporations subject to COAs audit jurisdiction. The resolution of the Sangguniang Bayan would constitute an LWDs special charter, making the LWD a government-owned and controlled corporation with an original charter. In any event, the Court has already ruled in Baguio Water District v. Trajano that the Sangguniang Bayan resolution is not the special charter of LWDs, thus: While it is true that a resolution of a local sanggunian is still necessary for the final creation of a district, this Court is of the opinion that said resolution cannot be considered as its charter, the same being intended only to implement the provisions of said decree.

D2010 218

UP College of Law

AUTHORITY TO NEGOTIATE OR SECURE GRANTS AND INCUR INDEBTEDNESS


Sec 23, LGC
See above

BUILD-OPERATE-TRANSFER
Sec 302, LGC
Sec. 302. Financing, Construction, Maintenance, Operation, and Management of Infrastructure Projects by the Private Sector. (a) Local government units may enter into contracts with any duly pre-qualified individual contractor, for the financing, construction, operation, and maintenance of any financially viable infrastructure facilities, under the buildoperate-and-transfer agreement, subject to the applicable provisions of Republic Act Numbered Sixty-nine hundred fifty-seven (R.A. No. 6957) authorizing the financing, construction, operation and maintenance of infrastructure projects by the private sector and the rules and regulations issued thereunder and such terms and conditions provided in this Section. (b) Local government units shall include in their respective local development plans and public investment programs priority projects that may be financed, constructed, operated and maintained by the private sector under this Section. It shall be the duty of the local government unit concerned to disclose to the public all projects eligible for financing under this Section, including official notification of duly registered contractors and publication in newspapers of general or local circulation and in conspicuous and accessible public places. Local projects under the build-operate-andtransfer agreement shall be confirmed by the local development councils. (c) Projects implemented under this Section shall be subject to the following terms and conditions: (1) The provincial, city, or municipal engineer, as the case may be, upon formal request in writing by the local chief executive, shall prepare the plans and specifications for the proposed projects, which shall be submitted to the sanggunian for approval. (2) Upon approval by the sanggunian of the project plans and specifications, the provincial, city or municipal engineer shall, as the case may be cause to be published once every week for two (2) consecutive weeks in at least one (1) local newspaper which is circulated in the region, province, city or municipality in which the project is to be implemented, a notice inviting all duly qualified contractors to participate in a public bidding for the projects so approved. The conduct of public bidding and award of contracts for local government projects under this Section shall be in accordance with this Code and other applicable laws, rules and regulations. In the case of a build-operate-and-transfer agreement, the contract shall be awarded to the lowest complying bidder whose offer is deemed most advantageous to the local government and based on the present value of its proposed tolls, fees, rentals, and charges over a fixed term for the facility to be constructed, operated, and maintained according to the prescribed minimum design and performance standards, plans, and specifications. For this purpose the winning contractor shall be automatically granted by the local government unit concerned the franchise to operate and maintain the facility, including the collection of tolls, fees, rentals, and charges in accordance with subsection (c-1) hereof. In the case of a build-operate-and-transfer agreement, the contract shall be awarded to the lowest complying bidder based on the present value of its proposed schedule of amortization payments for the facility to be constructed according to the prescribed minimum design and performance standards, plans, and specifications.

219 D2010

Local Government
(3) Any contractor who shall undertake the prosecution of any project under this Section shall post the required bonds to protect the interest of the province, city, or municipality, in such amounts as may be fixed by the sanggunian concerned and the provincial, city or municipal engineer shall, as the case may be, not allow any contractor to initiate the prosecution of projects under this Section unless such contractor presents proof or evidence that he has posted the required bond. (4) The contractor shall be entitled to a reasonable return of its investment in accordance with its bid proposal as accepted by the local government unit concerned. In the case of a build-operate-and-transfer agreement, the repayment shall be made by authorizing the contractor to charge and collect reasonable tolls, fees, rentals, and charges for the use of the project facility not exceeding those proposed in the bid and incorporated in the contract: Provided, That the local government unit concerned shall, based on reasonableness and equity, approve the tolls, fees, rentals and charges: Provided, further, That the imposition and collection of tolls, fees, rentals and charges shall be for a fixed period as proposed in the bid and incorporated in the contract which shall in no case exceed fifty (50) years: Provided, finally, That during the lifetime of the contract, the contractor shall undertake the necessary maintenance and repair of the facility in accordance with standards prescribed in the bidding documents and in the contract. In the case of a build-operate-and-transfer agreement, the repayment shall be made through amortization payments in accordance with the schedule proposed in the bid and incorporated in the contract. In case of land reclamation or construction of industrial estates, the repayment plan may consist of the grant of a portion or percentage of the reclaimed land or the industrial estate constructed. (5) Every infrastructure project undertaken under this Section shall be constructed, operated, and maintained by the contractor under the technical supervision of the local government unit and in accordance with the plans, specifications, standards, and costs approved by it. (d) The provincial, city or municipal legal officer shall, as the case may be, review the contracts executed pursuant to this Section to determine their legality, validity, enforceability and correctness of form.

D2010 220

UP College of Law

Liability
LIABILITY ON CONTRACTS ULTRA VIRES ACT
San Diego v. Municipality of Naujan, Oriental Mindoro
Following a public bidding for the lease of the municipal waters of Respondent, Petitioner, being the highest bidder, was awarded a contract which granted to the lessee the exclusive privilege of erecting fish corrals along the Butas river up to the Nuajan Lake for a period of 5 years. About a year into the lease period, the council reduced the annual rental by 20% upon the petition of the lessee. Petitioner later asked for an extension because a typhoon destroyed most of his fish corrals. The municipal council adopted Resolution 222 which extended the lease for another five years on the condition that Plaintiff would waive the privilege to seek for the reduction of the annual rent. This resolution was approved by the Provincial Board and a new contract was drawn and approved through Resolution 229 by the municipal council whose term was then about to expire. The new municipal council, this time with a new set of members, adopted Resolutions 3 and 11 which revoked Resolutions 222 and 229 respectively. Petitioner argues that these resolutions violated his constitutional right against deprivation of property without due process. The respondent argues that Resolutions 222 and 229 are void. Held: Resolution 222, and hence also Resolution 229, are void. Sec. 2323 of the Revised Administrative Code requires public bidding for the exclusive privilege of fishery or the right to conduct a fish-breeding ground. There is no doubt that the original lease contract in this case was awarded to the highest bidder, but the reduction of the rental and the extension of the term of the lease appear to have been granted without previous public bidding. Statutes requiring public bidding apply to amendments of any contract already executed in compliance with the law where such amendments alter the original contract in some vital and essential particular. Resolution 3 is not an impairment of the obligation of contract, because the constitutional provision on impairment refers only to contract legally executed. Public biddings are held for the best protection of the public and to give the public the best possible advantages by means of open competition between the bidders." Contracts requiring public bidding affect public interest, and to change them without complying with that requirement would indeed be against public policy.

Rivera v. Municipality of Malolos


Petitioners bid the lowest in the public bidding for the supply of construction materials for road repair operations of the municipality. The acting municipal treasurer informed them that the contract was awarded to them and the Mayor signed it in behalf of the municipality. The contracted stipulated that petitioners were to deliver crushed adobe stones and gravel which they did in 1949 at the place designated by the Mayor. In 1950, petitioners wrote the municipal treasurer because the amount of P19, 339.56 that was due them was not yet paid. The municipal treasurer informed them that Municipal Council had agreed to put said amount as standing obligation of the municipality authorizing payment and authorizing the Municipal Treasurer to pay as soon as funds are available. Resolution No. 68 was later passed which ratified the public bidding held for the construction supplies. In 1951, petitioners filed an action with the CFI which was dismissed. They filed a case with the Presidential Complaints and Action Committee which forwarded it to the General Auditing Office. This office denied the claim for payment on grounds which the Petitioners argue are mere technicalities.

221 D2010

Local Government
Held: The law requires that before a contract involving the expenditure of P2,000 or more may be entered into or authorized, the municipal treasurer must certify to the officer entering into such contracts that funds have been duly appropriated for such purpose and that the amount necessary to cover the proposed contract-is available for expenditure on account thereof. The contracted entered into by the Petitioners is contrary to this provision and is wholly void. Moreover, the law provides that the provincial auditor or his representative must check up the deliveries made by a contractor pursuant to a contract lawfully and validly entered into. In the case at bar, there was no such check up and the Auditor General is not in duty bound to pass and allow in audit the sum claimed by the petitioner if he or his authorized representative did not check up the delivery of the crushed adobe stone and gravel. To say that the purpose and aim of this checking requirement is to forestall fraud and collusion is to state what is obvious. Petitioners claim that the Motor Vehicle Law constitutes sufficient appropriation is untenable. This law merely allocates 10 per cent of the money collected under its provisions to the road and bridge funds of the different municipalities in proportion to population as shown in the latest available census, for the repair, maintenance and construction of municipal roads. This alone is not sufficient appropriation and authority to disburse part of the 10 per cent collected under the Motor Vehicle Law for the purpose of paying the claim of the petitioner. Petitioners remedy can be found in Sec. 608 of the Revised Administrative Code.

Rivera v. Maclang
this is a continuation of the above case)

Petitioner filed a case against respondent Maclang (the Mayor) which the CFI dismissed because the Supreme Court has already declared that the contract is void and therefore cant produce any legal effects. Held: The present action is against defendant-appellee in his personal capacity on the strength of section 608 of the Revised Administrative Code, which provides as follows: SEC. 608. Void contract, Liability of officer. A purported contract entered into contrary to the requirements of the next preceding section hereof shall be wholly void, and the officer assuming to make such contract shall be liable to the Government or other contracting party for any consequent damage to the same extent as if the transaction had been wholly between private parties. The position of defendant-appellee, as the officer who signed the contract with appellant in violation of section 607, comes squarely under the provision just quoted. His liability is personal, as it the transaction had been entered into by him as a private party. We take it that the intention of the law in this respect is to ensure that public officers entering into transactions with private individuals calling for the expenditure of public funds observe a high degree of caution so that the government may not be the victim of ill-advised or improvident action by those assuming to represent it.

LIABILITY ON TORTS (QUASI-DELICT)


Sec 24, LGC
See above The father and, in case of his death or incapacity, the mother, are responsible for the damages caused by the minor children who live in their company. Guardians are liable for damages caused by the minors or incapacitated persons who are under their authority and live in their company. The owners and managers of an establishment or enterprise are likewise responsible for damages caused by their employees in the service of the branches in which the latter are employed or on the occasion of their functions.

Art 2180, 2189, 34, Civil Code


Art. 2180. The obligation imposed by article 2176 is demandable not only for one's own acts or omissions, but also for those of persons for whom one is responsible.

D2010 222

UP College of Law
Employers shall be liable for the damages caused by their employees and household helpers acting within the scope of their assigned tasks, even though the former are not engaged in any business or industry. The State is responsible in like manner when it acts through a special agent; but not when the damage has been caused by the official to whom the task done properly pertains, in which case what is provided in article 2176 shall be applicable. Lastly, teachers or heads of establishments of arts and trades shall be liable for damages caused by their pupils and students or apprentices, so long as they remain in their custody. The responsibility treated of in this article shall cease when the persons herein mentioned prove that they observed all the diligence of a good father of a family to prevent damage. (1903a) Art. 2189. Provinces, cities and municipalities shall be liable for damages for the death of, or injuries suffered by, any person by reason of the defective condition of roads, streets, bridges, public buildings, and other public works under their control or supervision. (n) Soon, more police arrived but they permitted Charles to wander about the crowd and continue to threaten Tracey. Finally, upon approaching Tracey once again, this time while she was lying on a stretcher, Charles Thurman was arrested and taken into custody. Held: Tracey Thurman sued the city for the violations of her rights under the U.S. Constitution. The City brought a motion to dismiss her claims arguing that the equal protection clause [no state shall deny any person the equal protection of the laws+ only prohibits intentional discrimination that is racially motivated. The Citys argument is clearly a misstatement of the law. The application of the equal protection clause is not limited to racial classifications or racially motivated discrimination. Classifications on the basis of gender will be held invalid under the equal protection clause unless they are substantially related to strike down classifications which are not rationally related to a legitimate governmental purpose. City officials and police officers are under an affirmative duty to preserve law and order, and to protect the personal safety of persons in the community. This duty applies equally to women whose personal safety is threatened by individuals with whom they have or have had a domestic relationship as well as to all other persons whose personal safety is threatened, including women not involved in domestic relationships. If officials have notice of the possibility of attacks on women in domestic relationships or other persons, they are under an affirmative duty to take reasonable measures to protect the personal safety of such persons in the community. Failure to perform this duty would constitute a denial of equal protection of the laws. The Citys motion to dismiss is denied

Thurman v. City of Torrington


Tracey Thurman repeatedly reported to the police that she and her son were being physically and emotionally abused by her husband Charles Thurman but the police continuously ignored her. There were instances that members of the police actually saw the abuse happening but did not intervene. Charles Thurman lived in Torrington and worked as a counterman and short order cook at Skies Diner. There he served many members of the Torrington Police Department, including some of the officers in this case. While at work, Charles Thurman boasted to the officers that he intended to get his wife and that he intended to kill her. The situation escalated to the point that after being issued a restraining order, Charles Thurman nevertheless went to Traceys home and demanded to be let in. Tracey called the police and went outside to plead with Charles not to hurt their son. Charles suddenly stabbed Tracey repeatedly in the chest, neck, and throat. 25 minutes later, a single police officer arrived and saw Charles still holding the bloody knife. In the presence of the police officer, Charles kicked Tracey in the head then went inside the house and came back holding their son whom he dropped on top of Tracey. Charles kicked Tracey in the head a second time.

Palafox v. Province of Ilocos Norte


Sabas Torralba was employed as a driver of the Provincial Government of Ilocos Norte and was detailed to the Office of the District Engineer. While driving his truck in the performance of his duties, he ran over and killed Proceto Palafox. Torralba was convicted of homicide through reckless imprudence. The heirs of Palafox instituted an action for damages against the provincial government. Issue: WON the Provincial Government of Ilocos Norte is liable. Held: No. To attach liability to the state, a declaration must be

223 D2010

Local Government
made that Torralba was a special agent within the scope of Article 1903 paragraph 5 of the Civil Code. But this principle applies only to the Insular Government of the Philippines as distinguished from provincial or municipal governments. The heirs of Palafox invoked the doctrine of respondeat superior which provides that the master shall answer for the negligent acts of its employees. In the Mendoza case, it was held that if the negligent employee was engaged in the performance of governmental duties as distinguished from proprietary or business functions, the government is not liable. In the present case, the construction or maintenance of roads in which the driver Torralba was engaged in at the time of the accident is admittedly governmental activities. The twofold character of the powers of a municipality, under our Municipal Code (Act No. 82) is so apparent and its private or corporate powers so numerous and important that we find no difficulty in reaching the conclusion that the general principles governing the liability of such entities to private individuals as enunciated in the United States are applicable to it. The distinction is also recognized by Dillon in his work on Municipal Corporations (5th ed.) sections 38 and 39. As is indicated in some of the above quoted cases, the municipality is not liable for the acts of its officers or agents in the performance of its governmental functions. Governmental affairs do not lose their governmental character by being delegated to the municipal government. Nor does the fact that such duties are performed by such officers of the municipality which, for convenience, the state allows the municipality to select, change their character. To preserve the peace, protect the morals and health of the community and so on is to administer government, whether it be done by the central government itself or is shifted to a local organization. And the state being immune for injuries suffered by private individuals in the administration of strictly governmental functions, like immunity is enjoyed by the municipality in the performance of the same duties, unless it is expressly made liable by statute. It should be clear that a municipality is not exempt from liability for the negligent performance of its corporate or proprietary or business functions. In the administration of its patrimonial property, it is to be regarded as a private corporation or individual 153113-13 so far as its liability to third persons on contract or in tort is concerned. Its contracts, valid entered into, may be enforced and damages may be collected from it for the torts of its officers or agents within the scope of their employment in precisely the same manner and to the same extent as those of private corporations or individuals. As to such matters the principle of respondeat superior applies. It is for these purposes that the municipality is made liable to suits in the courts. Here it is clear that the leasing of a municipal ferry to the highest bidder for a specified period of time is not a governmental but corporate function. Such a lease, when validly entered into, constitutes a contract with the lessee which the municipality is bound to respect. The matter is thus summed up by Dillon on Municipal Corporations (5th ed., sec. 1306): "Ordinances made by municipalities under charter or legislative authority, containing grants to water and light

Mendoza v. De Leon
This is an action for damages against the individual members of the municipal council of the municipality of Villasis, Pangasinan, for the revocation of the lease of an exclusive ferry privilege awarded to the plaintiff under the provisions of Act. No. 1634 of the Philippine Commission. After user of a little more than one year, the plaintiff was forcibly ejected under and in pursuance of a resolution adopted by the herein defendants, awarding a franchise for the same ferry to another person. Issue: WON the defendants are liable to the plaintiff for damages. Held: Yes. Municipalities of the Philippine Islands organized under the Municipal Code have both governmental and corporate or business functions. Of the first class are the adoption of regulation against fire and disease, preservation of the public peace, maintenance of municipal prisons, establishment of primary schools and post-offices, etc. Of the latter class are the establishment of municipal waterworks for the use of the inhabitants, the construction and maintenance of municipal slaughterhouses, markets, stables, bathing establishments, wharves, ferries, and fisheries. Act No. 1643 provides that the use of each fishery, fish-breeding ground, ferry, stable, market, and slaughterhouse belonging to any municipality or township shall be let to the highest bidder annually or for such longer period not exceeding five years as may have been previously approved by the provincial board of the province in which the municipality or township is located.

D2010 224

UP College of Law
companies and other public service corporations of the right to use the street pipes, mains, etc., upon the condition of the performance of service by the grantee, are, after acceptance and performance by the grantee, contracts protected by the prohibition of the Federal Constitution against the enactment of any State Law impairing the obligation of contracts." It seems clear, therefore, that under the provisions of the Municipal Code and Act No. 1634, above referred to, the plaintiff had a vested right to the exclusive operation of the ferry in question for the period of his lease. Were the municipality a party to this action, it would be patent that a judgment for damages against it for the rescission of the contract would be proper. This, be it said, is the usual method of exacting damages, either ex contractu or ex delicto arising from the exercise of corporate powers of municipalities. There is not a scintilla of evidence that there was any justifiable reason for forcibly evicting the plaintiff from the ferry which he had leased. On the contrary, the defendant councilors attempted to justify their action on the ground that the ferry which he was operating was not the one leased to him; this in spite of the fact the vice-president had personally placed him in possession of it more than a year before, and the fact that he had operated this ferry for over a year, evidently with the knowledge of the defendants. The evidence is so clear that the ferry of which the plaintiff was dispossessed was the one which he had leased that no reasonable man would entertain any doubt whatever upon the question. Hence, we cannot say that in rescinding the contract with the plaintiff, thereby making the municipality liable to an action for damages for no valid reason at all, the defendant councilors were honestly acting for the interests of the municipality. We are, therefore, of the opinion that the defendants are liable jointly and severally for the damages sustained by the plaintiff from the rescission of his contract of lease of the ferry privilege in question. dump truck of the Municipality of San Fernando, La Union and driven by Alfredo Bislig. Several passengers of the jeepney including Laureano Bania Sr. died as a result of the injuries they sustained and four (4) others suffered varying degrees of physical injuries. Private respondents (heirs of the deceased Laureano Bania Sr.) instituted a compliant for damages against the Estate of Macario Nieveras and Bernardo Balagot in the CFI of La Union. The aforesaid defendants filed a Third Party Complaint against the petitioner and the driver of a dump truck of petitioner. Thereafter, the private respondents amended the complaint wherein the petitioner and its regular employee, Alfredo Bislig were impleaded for the first time as defendants. The trial court rendered a decision for the plaintiffs, and defendants Municipality of San Fernando, La Union and Alfredo Bislig are ordered to pay them jointly and severally. Petitioner filed a motion for reconsideration and for a new trial without prejudice to another motion which was then pending but the MR was denied. WON respondent Judge Firme exceeded his jurisdiction when he ruled on the issue of liability of the Municipality of San Fernando, La Union. Held: Yes The doctrine of non-suability of the State is expressly provided for in Article XVI, Section 3 of the Constitution, to wit: "the State may not be sued without its consent." Stated in simple parlance, the general rule is that the State may not be sued except when it gives consent to be sued. Consent takes the form of express or implied consent. Express consent may be embodied in a general law or a special law. The standing consent of the State to be sued in case of money claims involving liability arising from contracts is found in Act No. 3083. A special law may be passed to enable a person to sue the government for an alleged quasi-delict. Consent is implied when the government enters into business contracts, thereby descending to the level of the other contracting party, and also when the State files a complaint, thus opening itself to a counterclaim. Municipal corporations, for example, like provinces and cities, are agencies of the State when they are engaged in governmental functions and therefore should enjoy the sovereign immunity from suit. Nevertheless, they are subject to suit even in the performance of such functions

Municipality of San Fernando, La Union v. Firme


Petitioner Municipality of San Fernando, La Union is a municipal corporation existing under and in accordance with the laws of the Republic of the Philippines. At about 7 o'clock in the morning of December 16, 1965, a collision occurred involving a passenger jeepney driven by Bernardo Balagot and owned by the Estate of Macario Nieveras, a gravel and sand truck driven by Jose Manandeg and owned by Tanquilino Velasquez and a

225 D2010

Local Government
because their charter provided that they can sue and be sued. It has already been remarked that municipal corporations are suable because their charters grant them the competence to sue and be sued. Nevertheless, they are generally not liable for torts committed by them in the discharge of governmental functions and can be held answerable only if it can be shown that they were acting in a proprietary capacity. In permitting such entities to be sued, the State merely gives the claimant the right to show that the defendant was not acting in its governmental capacity when the injury was committed or that the case comes under the exceptions recognized by law. Failing this, the claimant cannot recover. In the case at bar, the driver of the dump truck of the municipality insists that "he was on his way to the Naguilian river to get a load of sand and gravel for the repair of San Fernando's municipal streets. In the absence of any evidence to the contrary, the regularity of the performance of official duty is presumed pursuant to Section 3(m) of Rule 131 of the Revised Rules of Court. Hence, We rule that the driver of the dump truck was performing duties or tasks pertaining to his office. After a careful examination of existing laws and jurisprudence, We arrive at the conclusion that the municipality cannot be held liable for the torts committed by its regular employee, who was then engaged in the discharge of governmental functions. Hence, the death of the passenger tragic and deplorable though it may be imposed on the municipality no duty to pay monetary compensation. In fact, the septic tank was found to be almost empty and the victims were presumed to be the ones who did the re-emptying. Dr. Juan Abear of the City Health Office autopsied the bodies and in his reports, put the cause of death of all five victims as `asphyxia' caused by the diminution of oxygen supply in the body working below normal conditions. The lungs of the five victims burst, swelled in hemmorrhagic areas and this was due to their intake of toxic gas, which, in this case, was sulfide gas produced from the waste matter inside the septic tank. The trial court rendered a decision dismissing the case. Petitioners appealed to the then Intermediate Appellate Court (now Court of Appeals). The IAC reversed the appealed judgment and ordered the defendant to pay the plaintiffs. Both parties filed their separate motions for reconsideration. The Court of Appeals rendered an Amended Decision dismissing the case against the City of Davao. WON the City of Davao is guilty of negligence. Held: No Negligence has been defined as the failure to observe for the protection of the interests of another person that degree of care, precaution, and vigilance which the circumstances justly demand, whereby such other person suffers injury. Under the law, a person who by his omission causes damage to another, there being negligence, is obliged to pay for the damage done (Article 2176, New Civil Code). As to what would constitute a negligent act in a given situation, the case of Picart v. Smith (37 Phil. 809, 813) provides Us the answer, to wit: "The test by which to determine the existence or negligence in a particular case may be stated as follows: Did the defendant in doing the alleged negligent act use that reasonable care and caution which an ordinary person would have used in the same situation? If not, then he is guilty of negligence. "The question as to what would constitute the conduct of a prudent man in a given situation must of course be always determined in the light of human experience and in view of the facts involved in the particular case. The proper criterion for determining the existence of negligence in a given case is this: Conduct is said to be negligent when a prudent man in the position of the tortfeasor would have foreseen that an effect harmful to another was sufficiently probable to warrant his

Fernando v. CA and City of Davao


On November 7, 1975, Bibiano Morta, market master of the Agdao Public Market filed a requisition request with the Chief of Property of the City Treasurer's Office for the re-emptying of the septic tank in Agdao. An invitation to bid was issued to Aurelio Bertulano, Lito Catarsa, Feliciano Bascon, Federico Bolo and Antonio Suer, Jr. Bascon won the bid. On November 26, 1975 Bascon was notified and he signed the purchase order. However, before such date, specifically on November 22, 1975, bidder Bertulano with four other companions were found dead inside the septic tank. The bodies were removed by a fireman. The City Engineer's office investigated the case and learned that the five victims entered the septic tank without clearance from it nor with the knowledge and consent of the market master.

D2010 226

UP College of Law
foregoing the conduct or guarding against its consequences. To be entitled to damages for an injury resulting from the negligence of another, a claimant must establish the relation between the omission and the damage. He must drove under Article 2179 of the New Civil Code that the defendant's negligence was the immediate and proximate cause of his injury. The test is simple. Distinction must be made between the accident and the injury, between the event itself, without which there could have been no accident, and those acts of the victim not entering into it, independent of it, but contributing to his own proper hurt. Where he contributes to the principal occurrence, as one of its determining factors, he can not recover. Where, in conjunction with the occurrence, he contributes only to his own injury, he may recover the amount that the defendant responsible for the event should pay for such injury, less a sum deemed a suitable equivalent for his own imprudence. While it may be true that the public respondent has been remiss in its duty to re-empty the septic tank annually, such negligence was not a continuing one. Upon learning from the report of the market master about the need to clean the septic tank of the public toilet in Agdao Public Market, the public respondent immediately responded by issuing invitations to bid for such service. Thereafter, it awarded the bid to the lowest bidder, Mr. Feliciano Bascon. The public respondent, therefore, lost no time in taking up remedial measures to meet the situation. It is likewise an undisputed fact that despite the public respondent's failure to re-empty the septic tank since 1956, people in the market have been using the public toilet for their personal necessities but have remained unscathed. The absence of any accident was due to the public respondent's compliance with the sanitary and plumbing specifications in constructing the toilet and the septic tank. Hence, the toxic gas from the waste matter could not have leaked out because the septic tank was airtight. Toilets and septic tanks are not nuisances per se as defined in Article 694 of the New Civil Code which would necessitate warning signs for the protection of the public. While the construction of these public facilities demands utmost compliance with safety and sanitary requirements, the putting up of warning signs is not one of those requirements. It would appear that an accident such as toxic gas leakage from the septic tank is unlikely to happen unless one removes its covers. The accident in the case at bar occurred because the victims on their own and without authority from the public respondent opened the septic tank. Considering the nature of the task of emptying a septic tank especially one which has not been cleaned for years, an ordinarily prudent person should undoubtedly be aware of the attendant risks. The victims are no exception; more so with Mr. Bertulano, an old hand in this kind of service, who is presumed to know the hazards of the job. His failure, therefore, and that of his men to take precautionary measures for their safety was the proximate cause of the accident. The market master knew that work on the septic tank was still forthcoming. It must be remembered that the bidding had just been conducted. Although the winning bidder was already known, the award to him was still to be made by the Committee on Awards. Upon the other hand, the accident which befell the victims who are not in any way connected with the winning bidder happened before the award could be given. Considering that there was yet no award and order to commence work on the septic tank, the duty of the market master or his security guards to supervise the work could not have started. The surreptitious way in which the victims did their job without clearance from the market master or any of the security guards goes against their good faith. Even their relatives or family members did not know of their plan to clean the septic tank. There is a total absence of contractual relations between the victims and the City Government of Davao City that could give rise to any contractual obligation, much less, any liability on the part of Davao City.

Tuzon and Mapagu v. CA and Jurado


The petitioners are questioning the decision of the respondent court holding them liable in damages to the private respondent for refusing to issue to him a mayor's permit and license to operate his palay-threshing business. The case goes back to March 14, 1977, when the Sangguniang Bayan of Camalaniugan, Cagayan, unanimously adopted Resolution No. 9 which authorizes the municipal treasurer to enter into an agreement to all thresher operators, that will come to apply for a permit to thresh palay within the jurisdiction of the municipality to donate 1% of all the palay threshed by them to help finance the continuation of the construction of the Sports and Nutrition Center Building.

227 D2010

Local Government
To implement the above resolution, petitioner Lope C. Mapagu, then incumbent municipal treasurer, prepared a document for signature of all thresher/owner/operators applying for a mayor's permit. Private respondent Saturnino T. Jurado sent his agent to the municipal treasurer's office to pay the license fee of P285.00 for thresher operators. Mapagu refused to accept the payment and required him to first secure a mayor's permit. Mayor Domingo Tuzon said that Jurado should first comply with Resolution No. 9 and sign the agreement before the permit could be issued. Jurado ignored the requirement. Instead, he sent the P285.00 license fee by postal money order to the office of the municipal treasurer who, however, returned the said amount. Jurado filed with the Court of First Instance of Cagayan a special civil action for mandamus with actual and moral damages to compel the issuance of the mayor's permit and license. He filed another petition with the same court for declaratory judgment against the said resolution (and the implementing agreement) for being illegal either as a donation or as a tax measure. Named defendants were the same respondents and all the members of the Sangguniang Bayan of Camalaniugan. The trial court upheld the challenged measure. Jurado appealed to the Court of Appeals which affirmed the validity of Resolution No. 9 and the implementing agreement. Nevertheless, it found Tuzon and Mapagu liable for acting maliciously and in bad faith when they denied Jurado's application for the mayor's permit and license. WON petitioners are liable in damages to private respondent Jurado for having withheld from him the mayor's permit and license because of his refusal to comply with Resolution No. 9. Held: No The private respondent anchors his claim for damages on Article 27 of the New Civil Code, which reads: Art. 27. Any person suffering material or moral loss because a public servant or employee refuses or neglects, without just cause, to perform his official duty may file an action for damages and other relief against the latter, without prejudice to any disciplinary administrative action that may be taken. In the present case, it has not even been alleged that the Mayor Tuzon's refusal to act on the private respondent's application was an attempt to compel him to resort to bribery to obtain approval of his application. It cannot be said either that the mayor and the municipal treasurer were motivated by personal spite or were grossly negligent in refusing to issue the permit and license to Jurado. It is no less significant that no evidence has been offered to show that the petitioners singled out the private respondent for persecution. Neither does it appear that the petitioners stood to gain personally from refusing to issue to Jurado the mayor's permit and license he needed. The petitioners were not Jurado's business competitors nor has it been established that they intended to favor his competitors. On the contrary, the record discloses that the resolution was uniformly applied to all the threshers in the municipality without discrimination or preference. The Court is convinced that the petitioners acted within the scope of their authority and in consonance with their honest interpretation of the resolution in question. We agree that it was not for them to rule on its validity. In the absence of a judicial decision declaring it invalid, its legality would have to be presumed. As executive officials of the municipality, they had the duty to enforce it as long as it had not been repealed by the Sangguniang Bayan or annulled by the courts. The private respondent complains that as a result of the petitioners' acts, he was prevented from operating his business all this time and earning substantial profit therefrom, as he had in previous years. But as the petitioners correctly observed, he could have taken the prudent course of signing the agreement under protest and later challenging it in court to relieve him of the obligation to "donate." Pendente lite, he could have continued to operate his threshing business and thus avoided the lucro cesante that he now says was the consequence of the petitioners' wrongful act. He could have opted for the less obstinate but still dissentient action, without loss of face, or principle, or profit.

Torio v. Fontanilla
On October 21, 1958, the Municipal Council of Malasiqui, Pangasinan passed Resolution No. 159 whereby "it resolved to manage the 1959 Malasiqui town fiesta celebration on January 21, 22, and 23, 1959." Resolution No. 182 was also passed creating the "1959 Malasiqui Town Fiesta Executive Committee" which in turn organized a subcommittee on entertainment and stage, with Jose Macaraeg as Chairman. The council appropriated the amount of P100.00 for the construction

D2010 228

UP College of Law
of 2 stages, one for the "zarzuela" and another for the "cancionan". Jose Macaraeg supervised the construction of the stage. The "zarzuela" entitled "Midas Extravanganza" was donated by an association of Malasiqui employees of the Manila Railroad Company in Caloocan, Rizal. One of the members of the group was Vicente Fontanilla. Before the dramatic part of the play was reached, the stage collapsed and Vicente Fontanilla who was at the rear of the stage was pinned underneath. Fontanilla was taken to the San Carlos General Hospital where he died in the afternoon of the following day. The heirs of Vicente Fontanilla filed a complaint with the Court of First Instance of Manila to recover damages. Named party-defendants were the Municipality of Malasiqui, the Municipal Council of Malasiqui and all the individual members of the Municipal Council in 1959. Judge Gregorio T. Lantin dismissed the complaint. The Fontanillas appealed to the Court of Appeals which reversed the trial court's decision and ordered all the defendants-appellees to pay jointly and severally the heirs of Vicente Fontanilla. WON the celebration of a town fiesta an undertaking in the exercise of a municipality's governmental or public function or is it of a private or proprietary character. Held: Private or proprietary character The powers of a municipality are twofold in character: public, governmental, or political on the one hand, and corporate, private, or proprietary on the other. Governmental powers are those exercised by the corporation in administering the powers of the state and promoting the public welfare and they include the legislative, judicial, public, and political. Municipal powers on the other hand are exercised for the special benefit and advantage of the community and include those which are ministerial, private and corporate. This distinction of powers becomes important for purposes of determining the liability of the municipality for the acts of its agents which result in an injury to third persons. If the injury is caused in the course of the performance of a governmental function or duty no recovery, as a rule, can be had from the municipality unless there is an existing statute on the matter, nor from its officers, so long as they performed their duties honestly and in good faith or that they did not act wantonly and maliciously. With respect to proprietary functions, the settled rule is that a municipal corporation can be held liable to third persons ex contractu or ex delicto. Section 2282 of the Chapter on Municipal Law of the Revised Administrative Code provision simply gives authority to the municipality to celebrate a yearly fiesta but it does not impose upon it a duty to observe one. Holding a fiesta even if the purpose is to commemorate a religious or historical event of the town is in essence an act for the special benefit of the community and not for the general welfare of the public performed in pursuance of a policy of the state. The mere fact that the celebration, as claimed, was not to secure profit or gain but merely to provide entertainment to the town inhabitants is not a conclusive test. There can be no hard and fast rule for purposes of determining the true nature of an undertaking or function of a municipality; the surrounding circumstances of a particular case are to be considered and will be decisive. The basic element, however beneficial to the public the undertaking may be, is that it is governmental in essence, otherwise, the function becomes private or proprietary in character. Easily, no governmental or public policy of the state is involved in the celebration of a town fiesta. Under the doctrine of respondent superior, petitionermunicipality is to be held liable for damages for the death of Vicente Fontanilla if that was attributable to the negligence of the municipality's officers, employees, or agents. The Court of Appeals found and correctly held that there was negligence. It is incredible that any person in his right mind would remove the principal braces of the stage and leave the front portion of the stage practically unsupported as claimed by the defendants. Moreover, if that did happen, there was indeed negligence as there was lack of supervision over the use of the stage to prevent such an occurrence. At any rate, the guitarist who was pointed to as the person who removed the two bamboo braces denied having done so. The appellate court also found that the stage was not strong enough considering that only P100.00 was appropriate for the construction of two stages and while the floor of the "zarzuela" stage was of wooden planks, the posts and braces used were of bamboo material. Having failed to take the necessary steps to maintain the safety of the stage for the use of the participants in the stage presentation prepared in connection with the celebration of the town fiesta, particularly, in preventing non-participants or spectators from mounting and accumulating on the stage which was

229 D2010

Local Government
not constructed to meet the additional weight, the defendants-appellees were negligent and are liable for the death of Vicente Fontanilla. The "Midas Extravaganza" which was to be performed during the town fiesta was a "donation" offered by an association of Malasiqui employees of the Manila Railroad Co. in Caloocan, and that when the Municipality of Malasiqui accepted the donation of services and constructed precisely a "zarzuela stage" for the purpose, the participants in the stage show had the right to expect that the Municipality through its "Committee on entertainment and stage" would build or put up a stage or platform strong enough to sustain the weight or burden of the performance and take the necessary measures to insure the personal safety of the participants. Petitioner or appellant Municipality cannot evade responsibility and/or liability under the claim that it was Jose Macaraeg who constructed the stage. The municipality acting through its municipal council appointed Macaraeg as chairman of the sub-committee on entertainment and in charge of the construction of the "zarzuela" stage. Macaraeg acted merely as an agent of the Municipality. Under the doctrine of respondent superior, petitioner is responsible or liable for the negligence of its agent acting within his assigned tasks. The Municipality stands on the same footing as an ordinary private corporation with the municipal council acting as its board of directors. It is an elementary principle that a corporation has a personality, separate and distinct from its officers, directors, or persons composing it and the latter are not as a rule coresponsible in an action for damages for tort or negligence (culpa aquiliana) committed by the corporation's employees or agents unless there is a showing of bad faith or gross or wanton negligence on their part. The municipal councilors are not liable for the death of Vicente Fontanilla. The records do not show that said petitioners directly participated in the defective construction of the "zarzuela" stage or that they personally permitted spectators to go up the platform.

LIABILITY BY EXPRESS PROVISION OF LAW AND CONTRACTS


Art 34 and 2189, Civil Code
Art. 34. When a member of a city or municipal police force refuses or fails to render aid or protection to any person in case of danger to life or property, such peace officer shall be primarily liable for damages, and the city or municipality shall be subsidiarily responsible therefor. The civil action herein recognized shall be independent of any criminal proceedings, and a preponderance of evidence shall suffice to support such action. Art. 2189. Provinces, cities and municipalities shall be liable for damages for the death of, or injuries suffered by, any person by reason of the defective condition of roads, streets, bridges, public buildings, and other public works under their control or supervision. (n) his left eyelid. Several persons came to his assistance and pulled him out of the manhole. One of them brought Teotico to the Philippine General Hospital, where his injuries were treated, after which he was taken home. In addition to the lacerated wound in his left upper eyelid, Teotico suffered contusions on different parts of his body. These injuries and the allergic eruptions caused by anti-tetanus injections administered to him in the hospital required further medical treatment by a private practitioner. Teotico filed with the Court of First Instance of Manila a complaint which was subsequently amended for damages against the City of Manila, its mayor, city engineer, city health officer, city treasurer and chief of police. The Court of First Instance of Manila dismissed the complaint. On appeal taken by plaintiff, this decision was affirmed by the Court of Appeals, except insofar as the City of Manila is concerned, which was sentenced to pay damages to Teotico. WON Section 4 of Republic Act No. 409 (Charter of the City of Manila) or Article 2189 of the Civil Code is applicable to the present case.

City of Manila v. Teotico


On January 27, 1958, at about 8:00 p.m., Genaro N. Teotico fell inside an uncovered and unlighted catchbasin or manhole on P. Burgos Avenue as he was stepping down from the curb to board a jeepney . Due to the fall, his head hit the rim of the manhole breaking his eyeglasses and causing broken pieces thereof to pierce

D2010 230

UP College of Law
Held: Article 2189 of the Civil Code Section 4 of Republic Act No. 409 (Charter of the City of Manila) reads: "The city shall not be liable or held for damages or injuries to persons or property arising from the failure of the Mayor, the Municipal Board, or any other city officer, to enforce the provisions of this chapter, or any other law or ordinance, or from negligence of said Mayor, Municipal Board, or other officers while enforcing or attempting to enforce said provisions." While Article 2189 of the Civil Code of the Philippines provides: "Provinces, cities and municipalities shall be liable for damages for the death of, or injuries suffered by, any person by reason of the defective condition of roads, streets, bridges, public buildings, and other public works under their control or supervision." It is true that, insofar as its territorial application is concerned, Republic Act No. 409 is a special law and the Civil Code a general legislation; but, as regards the subject- matter of the provisions above quoted, Section 4 of Republic Act 409 establishes a general rule regulating the liability of the City of Manila for "damages or injury to persons or property arising from the failure of" city officers "to enforce the provisions of" said Act "or any other law or ordinance, or from negligence" of the city "Mayor, Municipal Board, or other officers while enforcing or attempting to enforce said provisions." Upon the other hand, Article 2189 of the Civil Code constitutes a particular prescription making "provinces, cities and municipalities . . . liable for damages for the death of, or injury suffered by, any person by reason" specifically "of the defective condition of roads, streets, bridges, public buildings, and other public works under their control or supervision." In other words, said section 4 refers to liability arising from negligence, in general, regardless of the object thereof, whereas Article 2189 governs liability due to "defective streets, "in particular. Since the present action is based upon the alleged defective condition of a road, said Article 2189 is decisive thereon. Under Article 2189 of the Civil Code, it is not necessary for the liability therein established to attach that the defective roads or streets belong to the province, city or municipality from which responsibility is exacted. What said article requires is that the province, city or municipality have either "control or supervision" over said street or road. Even if P. Burgos avenue were, therefore, a national highway, this circumstance would not necessarily detract from its "control or supervision" by the City of Manila, under Republic Act 409. The determination of whether or not P. Burgos Avenue is under the control or supervision of the City of Manila and whether the latter is guilty of negligence, in connection with the maintenance of said road, which were decided by the Court of Appeals in the affirmative, is one of fact, and the findings of said Court, thereon are not subject to review by the Supreme Court.

Abella v. Municipality of Naga


This is an appeal from a judgment of the Court of First Instance of Camarines Sur sentencing the municipality of Naga, now Naga City, to pay the plaintiff, now appellee, P300 damages resulting from the closing of a municipal street. The defendant municipality by resolution ordered the closing of that part of a municipal street which ran between the public market and the plaintiff's property, and used the closed thoroughfare to expand the market. As a consequence of this resolution, and immediately after the passage of the same, permanent, semipermanent, as well as temporary constructions were allowed by the defendant municipality of Naga along the sidewalk of plaintiff's property and abutting to said property, facing P. Prieto Street, and extending out in the middle of the same street, hence depriving the plaintiff's property of access to said street, and consequently retarding her reconstructions. WON appellant municipality of Naga is liable for damages to Abella. Held: Yes The municipality or city of Naga was not charged with any unlawful act, or with acting without authority, or with invasion of plaintiff's property rights; the basis of the lower court's decision is Section 2246 of the Revised Administrative Code which provides that no municipal road, street, etc. or any part thereof "shall be closed without indemnifying any person prejudiced thereby." That plaintiff was economically damaged is admitted in the stipulation of facts and it is not disputed that the indemnity assessed is within the bounds of the damages suffered. As a matter of fact, the damages awarded seem to be nominal judged by the description of the plaintiff's interests adversely affected by the conversion of P. Prieto Street into a market.

231 D2010

Local Government Statutory obligations: Magna Carta for Health Workers


See attachments

Credit financing: Sec 297 302

SATISFYING/EXECUTING JUDGMENT AGAINST MUNICIPAL CORPORATIONS


Tan Toco v. Municipal Council of Iloilo
Vda. de Tan Toco sued the municipal council of Iloilo for P42,966.40, the purchase price of two strips of land, which the municipality appropriated for street widening. Iloilo CFI decided in favor of Tan Toco. Judgment upheld by SC. Due to lack of funds, the municipality was unable to satisfy the judgment. Tan Toco had a writ of execution issue against the property of the municipality. The sheriff attached two auto trucks used for street sprinkling, one police patrol automobile, police stations, and lots and concrete structures therein used as markets. The provincial fiscal of Iloilo filed a motion with the CFI praying that the attachment on the said property be dissolved since the said attachment was null and void as being illegal and violative of the rights of the municipality. CFI granted the motion. WON the abovementioned property of the municipality may be attached. HELD: No. The Civil Code divides the property of provinces and municipalities into property for public use and patrimonial property. The principle governing property of the public domain of the State is applicable to property for public use of the municipalities. Hence, property for public use of the municipality is not within the commerce of man so long as it is used by the public and, consequently, said property is also inalienable and exempt from attachment and execution. Rationale for the rule: The character of the public use to which such kind of property is devoted. The necessity for government service justifies that the property of public use of the municipality be exempt from execution by then Mayor Pablo Cuneta) for the construction of a new city hall. Isip proceeded with the construction of the new city hall building as per duly approved plans and specifications and accomplished under various stages of construction the amount of work (including supplies and materials) equivalent to an estimated value of P1,713,096 of the total contract price of P4,914,500.80. Pasay City paid only P1,100,000, leaving a balance of P613,096. Isip filed an action for specific performance with damages against Pasay City before CFI Manila. The parties arrived at an amicable agreement which was submitted to the Municipal Board of Pasay City for its consideration. The Municipal Board then enacted Ordinance No. 1012 which approved the Compromise Agreement and also authorized and empowered then incumbent City Mayor Jovito Claudio to represent the city government. The court approved the compromise agreement. Isip then filed an urgent motion seeking a declaration of legality of the original contract and compromise agreement. Motion granted. An application for and notice of garnishment were made and effected upon the funds of the city government with the PNB. Pasay City filed an urgent motion to set aside the abovementioned order and to quash the writ of execution issued on the following grounds: (1) that the execution sought was then still premature, the period of 90 days stipulated not having elapsed as yet; (2) that the obligations of the parties under the Compromise Agreement were reciprocal and isip not having put up a new performance bond in the sufficient amount equivalent to 20% of the remaining cost of construction as per agreement, the Pasay City cannot be obliged to pay the sum due as yet; (3) that the Sheriff has no power or authority to levy or garnish on execution the general funds, especially more so, the trust funds of the Pasay City. Motion denied. WON Pasay City government funds in PNB may be garnished to effect the compromise agreement. HELD: Yes. A compromise agreement not contrary to law, public

Municipality of Makati v. CA
No citation

Pasay City Government v. CFI


V.D. Isip, Sons & Associates entered into a contract (Contract of Agreement) with Pasay City (represented

D2010 232

UP College of Law
order, public policy, morals or good customs is a valid contract which is the law between the parties themselves. A judgment on a compromise is a final and executory. It is immediately executory in the absence of a motion to set the same aside on the ground of fraud, mistake or duress. The general rule is that all government funds deposited with the PNB by any agency or instrumentality of the government, whether by way of general or special deposit, remain government funds and may not be subject to garnishment or levy (Commissioner of Public Highways v San Diego). However, an ordinance has already been enacted expressly appropriating the amount of P613,096.00 of payment to Isip. Hence, the case is covered by the exception to the general rule stated in the case of Republic v. Palacio: "Judgments against a State in cases where it has consented to be sued, generally operate merely to liquidate and establish plaintiff's claim in the absence of express provision; otherwise they cannot be enforced by processes of the law; and it is for the legislature to provide for the payment in such manner as sees fit." and execution. Even public revenues of municipal corporations destined for the expenses of the municipality are also exempt from the execution. The reason behind this exemption extended to properties for public use, and public municipal revenues is that they are held in trust for the people, intended and used for the accomplishment of the purposes for which municipal corporations are created, and that to subject said properties and public funds to execution would materially impede, even defeat and in some instances destroy said purpose (Vda. de Tan Toco v. Municipal Council of Iloilo). Property, however, which is patrimonial and which is held by municipality in its proprietary capacity is the private asset of the town and may be levied upon and sold under an ordinary execution. The same rule applies to municipal funds derived from patrimonial properties. The fishery or municipal waters of the town of Paoay, Ilocos Norte, which had been parceled out or divided into lots and later let out to private persons for fishing purposes at an annual rental are not subject to execution. They do not belong to the municipality. They may well be regarded as property of State. What the municipality of Paoay hold is merely what may be considered the usufruct or the right to use said municipal waters, which is not also subject to execution. But the revenue or income coming from the renting of these fishery lots is subject to execution. Unlike revenue derived from taxes, municipal licenses and market fees, revenue from rentals were granted by the Legislature merely to help or bolster up the economy of municipal government, and hence are not indispensable for the performance of governmental functions. They are also not definite or fixed; it depends upon the amounts which prospective bidders or lessees are willing to pay. This activity of municipalities in renting municipal waters for fishing purposes is a business for the reasons that the law itself allowed said municipalities to engage in it for profit. And it is but just that a town so engaged should pay and liquidate obligations contracted in connection with said fishing business, with the income derived therefrom.

Municipality of Paoay, Ilocos Norte v. Manaois


Manaois obtained a judgment against Paoay. CFI Pangasinan issued a writ of execution against the municipality. The Provincial Sheriff of Ilocos Norte levied upon and attached the following properties: (1) an amount of P1,712.01 in the Municipal Treasury of Paoay representing the rental paid by Tabije of a fishery lot belonging to the municipality, and; (2) about forty fishery lots leased to thirty-five different persons by the municipality. The municipality asked for the dissolution of the attachment since they are properties for public use. WON the properties can be levied Held: As to the fishery lots, NO. As to the revenues, YES. Properties for public use like trucks used for sprinkling the streets, police patrol wagons, police stations, public markets, together with the land on which they stand held by municipal corporations are not subject to levy

233 D2010

Local Government

PENAL PROVISIONS AND REPEALING CLAUSE


Sec. 511. Posting and Publication of Ordinances with Penal Sanctions. (a) Ordinances with penal sanctions shall be posted at prominent places in the provincial capitol, city, municipal or barangay hall, as the case may be, for a minimum period of three (3) consecutive weeks. Such ordinances shall also be published in a newspaper of general circulation, where available, within the territorial jurisdiction of the local government unit concerned, except in the case of barangay ordinances. Unless otherwise provided therein, said ordinances shall take effect on the day following its publication, or at the end of the period of posting, whichever occurs later. (b) Public officer or employee who violates an ordinance may be meted administrative disciplinary action, without prejudice to the filing of the appropriate civil or criminal action. (c) The secretary to the sanggunian concerned shall transmit official copies of such ordinances to the chief executive officer of the Official Gazette within seven (7) days following the approval of the said ordinance for publication purposes. The Official Gazette may publish ordinances with penal sanctions for archival and reference purposes. Sec. 512. Withholding of Benefits Accorded to Barangay Officials. Willful and malicious withholding of any of the benefits accorded to barangay officials under Section 393 hereof shall be punished with suspension or dismissal from office of the official or employee responsible therefor. Sec. 513. Failure to Post and Publish the Itemized Monthly Collections and Disbursements. Failure by the local treasurer or the local chief accountant to post the itemized monthly collections and disbursements of the local government unit concerned within ten (10) days following the end of every month and for at least two (2) consecutive weeks at prominent places in the main office building of the local government unit concerned, its plaza and main street, and to publish said itemization in a newspaper of general circulation, where available, in the territorial jurisdiction of such unit, shall be punished by a fine not exceeding Five hundred pesos (P500.00) or by imprisonment not exceeding one (1) month, or both such fine and imprisonment, at the discretion of the court. Sec. 514. Engaging in Prohibited Business Transactions or Possessing Illegal Pecuniary Interest. Any local official and any person or persons dealing with him who violate the prohibitions provided in Section 89 of Book I hereof, shall be punished with imprisonment for six (6) months and one day to six (6) years, or a fine of not less than Three thousand pesos (P3,000.00) nor more than Ten thousand pesos (P10,000.00), or both such imprisonment and fine, at the discretion of the court. Sec. 515. Refusal or Failure of Any Party or Witness to Appear before the Lupon or Pangkat. Refusal or willful failure of any party or witness to appear before the lupon or pangkat in compliance with a summons issued pursuant to the provisions on the Katarungang Pambarangay under Chapter 7, Title One of Book III of this Code may be punished by the city or municipal court as for indirect contempt of court upon application filed therewith by the lupon chairman, the pangkat chairman, or by any of the contending parties. Such refusal or willful failure to appear, shall be reflected in the records of the lupon secretary or in the minutes of the pangkat secretary and shall bar the complainant who fails to appear, from seeking judicial recourse for the same cause of action, and the respondent who refuses to appear from filing any counterclaim arising out of, or necessarily connected with the complaint. A pangkat member who serves as such shall be entitled to an honorarium, the amount of which is to be determined by the sanggunian concerned, subject to the provisions in this Code cited above. Sec. 516. Penalties for Violation of Tax Ordinances. The sanggunian of a local government unit is authorized to prescribe fines or other penalties for violation of tax ordinances but in no case shall such fines be less than One thousand pesos (P1,000.00) nor more than Five thousand pesos (P5,000.00), nor shall imprisonment be less than one (1) month nor more than six (6) months. Such fine or other penalty, or both, shall be imposed at the discretion of the court. The sangguniang barangay may prescribe a fine of not less than One hundred pesos (P100.00) nor more than One thousand pesos (P1,000.00). Sec. 517. Omission of Property from Assessment or Tax Rolls by Officers and Other Acts. Any officer charged with the duty of assessing real property who willfully fails to assess, or who intentionally omits from the assessment or tax roll any real property which he knows to be taxable, or who willfully or negligently underassesses any real property, or who intentionally

D2010 234

UP College of Law
violates or fail to perform any duty imposed upon him by law relating to the assessment of taxable real property shall, upon conviction, be punished by a fine of not less than One thousand pesos (P1,000.00) nor more than Five thousand pesos (P5,000.00), or by imprisonment of not less than one (1) month nor more than six (6) months, or both such fine and imprisonment, at the discretion of the court. The same penalty be imposed any officer charged with the duty of collecting the tax due on real property who willfully or negligently fails to collect the tax and institute the necessary proceedings for the collection of the same. Any other officer required by this Code to perform acts relating to the administration of the real property tax or to assist the assessor or treasurer in such administration, who willfully fails to discharge such duties shall, upon conviction be punished by a fine of not less than Five hundred pesos (P500.00) nor more than Five thousand pesos (P5,000.00) or imprisonment of not less than one (1) month nor more than six (6) months, or both such fine and imprisonment, at the discretion of the court. Sec. 518. Government Agents Delaying Assessment of Real Property and Assessment Appeals. Any government official who intentionally and deliberately delays the assessment of real property or the filing of any appeal against its assessment shall, upon conviction, be punished by a fine of not less than Five hundred pesos (P500.00) nor more than Five thousand pesos (P5,000.00), or by imprisonment of not less than one (1) month nor more than six (6) months, or both such fine and imprisonment, at the discretion of the court. Sec. 519. Failure to Dispose of Delinquent Real Property at Public Auction. The local treasurer concerned who fails to dispose of delinquent real property at public auction in compliance with the pertinent provisions of this Code, and any other local government official whose acts hinder the prompt disposition of delinquent real property at public auction shall, upon conviction, be subject to a fine of not less than One thousand pesos (P1,000.00) nor more than Five thousand pesos (P5,000.00), or imprisonment of not less than one (1) month nor more than six (6) months, or both such fine and imprisonment at the discretion of the court. Sec. 520. Prohibited Acts Related to the Award of Contracts Under the Provisions on Credit Financing. It shall be unlawful for any public official or employee in the provincial, city, or municipal government, or their relatives within the fourth civil degree of consanguinity or affinity, to enter into or have any pecuniary interest in any contract for the construction, acquisition, operation, or maintenance of any project awarded pursuant to the provisions of Title Four in Book II hereof, or for the procurement of any supplies, materials, or equipment of any kind to be used in the said project. Any person convicted for violation of the provisions of said Title shall be removed from office and shall be punished by imprisonment of not less than one (1) month, nor more than two (2) years, at the discretion of the court, without prejudice to prosecution under other laws.

235 D2010

Anda mungkin juga menyukai